You are on page 1of 240

Ths.

L Vn on
Ths. L Vn on

www.MATHVN.com

MC LC

Trang

PHN I I S
CHNG IV BT NG THC & BT PHNG TRNH ------------------------------------- 1
B BT PHNG TRNH ----------------------------------------------------------------------------- 1
I Bt phng trnh & H bt phng trnh bc nht mt n --------------------------------- 1
Dng ton 1. Gii phng bt trnh bc nht Hai phng trnh tng ng ------ 2
Dng ton 2. Bt phng trnh qui v bc nht H bt phng trnh ---------------- 4
Dng ton 3. Bt phng trnh bc nht mt n cha tham s -------------------------- 10
II Du ca tam thc bc hai & Bt phng trnh bc hai ------------------------------------ 15
Dng ton 1. Xt du & Gii bt phng trnh bc hai ----------------------------------- 15
Dng ton 2. Phng trnh & Bt phng trnh cha cn, tr tuyt i ---------------- 20
Dng ton 3. Bi ton cha tham s trong phng trnh & bt phng trnh --------- 35
CHNG V GC V CNG THC LNG GIC ---------------------------------------------- 47
A H THC LNG GIC C BN ------------------------------------------------------------- 47
B CUNG LIN KT ------------------------------------------------------------------------------------ 52
C CNG THC CNG CUNG ---------------------------------------------------------------------- 62
D CNG THC NHN ------------------------------------------------------------------------------- 69
E CNG THC BIN I --------------------------------------------------------------------------- 77

PHN II HNH HC
CHNG III PHNG PHP TA TRONG MT PHNG ------------------------------- 89
A TA VCT & TA IM ------------------------------------------------------------ 89
B PHNG TRNH NG THNG ------------------------------------------------------------ 97
Dng ton 1. Lp phng trnh ng thng & Bi ton lin quan -------------------------- 100
Dng ton 2. Cc bi ton dng tam gic S tng giao Khong cch Gc --------- 105
C PHNG TRNH NG TRN --------------------------------------------------------------- 133
D PHNG TRNH NG ELP ---------------------------------------------------------------- 177
E PHNG TRNH NG HYPERBOL ------------------------------------------------------- 197
F PHNG TRNH NG PARABOL --------------------------------------------------------- 211
G BA NG CONIC -------------------------------------------------------------------------------- 224
H NG DNG TA GII TON I S & GII TCH --------------------------------- 234

DeThiThuDaiHoc.com

cng hc tp mn Ton 10 Tp IIwww.MATHVN.com

Ths. L Vn on

B BT PHNG TRNH


I Bt phng trnh v h bt phng trnh bc nht mt n


 iu kin ca bt phng trnh
iu kin ca bt phng trnh l iu kin m n s phi tha mn cc biu thc hai v
ca bt phng trnh c ngha. C th, ta c ba trng hp:
+ Dng

iu kin c ngha:

+ Dng

iu kin c ngha:

+ Dng

iu kin c ngha:

 Hai bt phng trnh tng ng


Hai bt phng trnh c gi l tng ng vi nhau nu chng c cng mt tp nghim.
 Phng php gii bt phng trnh v h bt phng trnh bc nht mt n
a/ Gii bt phng trnh bc nht mt n
 Phng php:
Bc 1. t iu kin cho bt phng trnh c ngha (nu c)
Bc 2. Chuyn v v gii.
Bc 3. Giao nghim vi iu kin c tp nghim S.
b/ H bt phng trnh bc nht mt n
 Phng php:
Bc 1. t iu kin cho h bt phng trnh c ngha (nu c).
Bc 2. Gii tng bt phng trnh ca h ri ly giao cc tp nghim thu c.
Bc 3. Giao nghim vi iu kin c tp nghim S.
 Gii v bin lun bt phng trnh bc nht dng:
iu kin

Kt qu tp nghim

 Lu : Ta c th gii tng t cho cc trng hp:

"Cn c b thng minh"

DeThiThuDaiHoc.com

Page - 1 -

www.MATHVN.com
Chng 4.

Ths. L Vn on

Bt ng thc v Bt phng trnh

Dng 1. Gii phng trnh bc nht Hai phng trnh tng ng




BA
NG
BAI T
TP AAP DU
DUNG
Bi 1.

Tm iu kin c ngha ca cc phng trnh sau


1/

1
1
<1
.
x
x +1

3/

3x +

5/

3
x 2

x +1

(x 2)

x3

4/

x x3

< x +1.

6/

1+ x
2x2 1 .
x 3x + 2

9/

x +2
4
1+
.
2
2
1+x
(x 2)

10/ x +

x +1

(x 3)(x + 4)

3
6x

12/

16 2x .

8/

2x +x <2+ x .
x 1
x3

> 2

2x 3
x 1 x + 2

x4

3 4x +

1
x+6

Chng minh cc phng trnh sau v nghim


1/
3/
5/

7/

9/

11/

Page - 2 -

x
.
x3

x + x 4 <1+ x 4 .

x 1

Bi 3.

>2+

2x
.
x +1

2 x 1 + 3 x 1

7/

11/

Bi 2.

2/

x2 + x + 8 3 .
3 x + x 5 10 .

(x 3)

x 10 > x + 1

x2 x + 1 +

1
x x +1
2

x5 .

< 2.

4x 6 + 3 > x 4 + 2 .
4x 2 + 4x + 2 + x2 6x + 10 < 2 .

2/

x 6 + 3 x 4 .

4/

1 + x 2 2 + x2 > 1 .

5x

6/

x 10

x +2

<

4 x2

(x 4)(x + 5)

8/

x2 + 1 + x 4 x 2 + 1 < 2 4 x 6 + 1 .

10/

x2 + 1 +

4
x +1
2

< 4.

12/ x + 2 x 2 + x2 + 1 1 0 .

Xt s tng ng ca cc cp bt phng trnh sau


1/

4x + 1 > 0

2/

3x +

1
1
3+
.
x3
x3

&

4x 1 < 0 .

&

3x 3 0 .

DeThiThuDaiHoc.com
"All the flower of tomorrow are in the seeks of today"

cng hc tp mn Ton 10 Tp IIwww.MATHVN.com

3/

x 1 x.

&

(2x + 1)

4/

3x 5
>7.
x2 + 1

&

3x 5 > 7 x2 + 1 .

5/

2x 3

&

2x 3 < x 4 .

6/

x +3

7/

4x + 8 < 1 x .

&

(18 + x 2x )(4x + 8) < (18 + x 2x )(1 x) .

8/

3x + 1 < x + 3 .

&

(3x + 1)

9/

x+5
< 0.
x 1

&

(x + 5)(x 1) < 0 .

10/ x2 x .

&

x 1.

11/ x 4 x2 .

&

x2 1 .

1
< x4.
x5

1
1
<2
. &
x+7
x+7

x 1 x (2x + 1) .

&

x 1.

13/

1 x x.

&

1 x x2 .

14/

(x + 1)(x 2) x .

&

&

< (x + 3) .

1
1.
x

(2 x) (x + 1) > 2 (2 x) .

x + 3 < 2.

12/

15/
Bi 4.

Ths. L Vn on

x +1 x 2 x .
x +1> 2.

Gii cc bt phng trnh sau

3 3 (2x 7)
>
.
5
3

1/

2x +

3/

5 (x 1)
6

1 <

2 (x + 1)
3

2/

4/

2+

2x + 1
3
>x+ .
5
4
3 (x + 1)
8

< 3

x 1
.
4

5/

3x + 1 x 2 1 2x

<
.
2
3
4

6/

x +1 x +2
x

<2+ .
2
3
6

7/

10 3x
2x 7
+9>
2x .
2
4

8/

(x + 2)

9/

x+ x < 2 x +3

10/

)(

x 1 .

(x 1) + 4 .
2

)(

11/

(x 4) (x + 1) > 0 .

12/

(x + 2) (x 3) > 0 .

13/

x3 3x .

14/

x 1 < 3 + x 1 .

"Cn c b thng minh"

DeThiThuDaiHoc.com

1 x + 3 2 1 x 5 > 1 x 3.
2

Page - 3 -

www.MATHVN.com
Chng 4.

Ths. L Vn on

x 2

15/

x4

4
x4

16/

(10 x)

Bt ng thc v Bt phng trnh

x4

x4

0.

18/

x3
0.
1 2x

x 2 0.

20/

(4 x )

17/

(x 1)(x + 1)

19/

(x 3)

> 4.

5 x 0.

Dng 2. Bt phng trnh qui v bt phng trnh bc nht mt n


H bt phng trnh bc nht mt n

 Du ca nh thc bc nht
a/ S dng bng xt du (tri tri phi cng: vi h s a)

b/ S dng trc s
Nu

th :

Nu

th :

 Bt phng trnh tch s


Dng:

Trong :

l cc nh thc bc nht.

Phng php: Lp bng xt du

. T suy ra tp nghim ca

 Bt phng trnh cha n s mu


Dng:

Trong :

Phng php: Lp bng xt du

l cc nh thc bc nht.
. T suy ra tp nghim ca

 Lu : Khng nn qui ng v kh mu.


Page - 4 -

DeThiThuDaiHoc.com
"All the flower of tomorrow are in the seeks of today"

cng hc tp mn Ton 10 Tp IIwww.MATHVN.com

Ths. L Vn on

 Bt phng trnh cha n trong du gi tr tuyt i


Tng t nh gii phng trnh cha n trong du gi tr tuyt i, ta thng s dng nh
ngha hoc tnh cht ca gi tr tuyt i kh du gi tr tuyt i.
.

Dng 1.

: c ngha
.

Dng 2.

, ta chia bi ton thnh nhiu trng hp. Trong mi trng hp ta


 Lu : Vi

, ta lun c

xt du ca qui tc
.

BA
NG
BAI T
TP AAP DU
DUNG
Bi 5.

Lp bng xt du ca cc hm s sau
1/

f (x ) = x + 1 .

2/

f (x) = 2x + 1 .

3/

f (x ) = 2 x .

4/

f (x) = 2 2 + x .

5/

f (x) = 3 3x .

6/

f (x ) = m 2 + 1 x 1 .

7/

f (x) = 4m 1 m2 2m + 2 x .

8/

f (x) = 4m2 + 2m + 1 x 3m .

9/

f (x ) = m 3 + m 3 m 2 + 1 x .

11/ f (x) =
13/ f (x) =

5x 3

(x 3)(2x 1)

1
1
2
.
x 1 x 1

10/ f (x) = 3x (3x 1) .


12/ f (x) =

x (x + 1)
x2

14/ f (x) = (2x 5) .


2

15/ f (x) = (3 7x) .

16/ f (x) = (3x + 1) .

17/ f (x) = (2x 7) .

18/ f (x) = 3 x 2 .

19/ f (x) = (5x + 2) .

20/ f (x) = x (8 3x) .

21/ f (x) = (4x 1)(x 1) .

22/ f (x) = (3x + 7)(5 2x) .

23/ f (x) = (2x + 5)(3x + 7) .

24/ f (x) = x 3 (x 3) .

"Cn c b thng minh"

DeThiThuDaiHoc.com

Page - 5 -

www.MATHVN.com
Chng 4.

Ths. L Vn on

25/ f (x) = x (2 7x) .

26/ f (x) = (x + 1) (4 x) .

)(

27/ f (x) = x 3 1

2x .

28/ f (x) = (2 x) (2x + 5) .


3

30/ f (x) = (4 x)(x + 1)(5x 2) .

31/ f (x) = 3x (2x + 7)(9 3x) .

32/ f (x) = (1 3x) (x 1) .

33/ f (x) = (4 x) (5x 2) .

34/ f (x) = x2 (2x 3) .

Gii cc bt phng trnh sau


1/

(x + 1)(x 1)(3x 6) > 0 .

2/

3/

x 2 x 20 > 2 (x 11) .

4/

3x (2x + 7)(9 3x) 0 .

5/

2
> 0.
x3

6/

3
> 0.
2 3x

7/

1
2.
x 1

8/

x
1
.
x5 2

9/

x
0.
x x

10/

4x + 3
6.
2x 5

(2x 7)(4 5x) 0 .

11/

x 2
< 0.
x2 4

12/

1 x
< 0.
x

13/

5x 6
1.
x+6

14/

x+9
0.
x 1

16/

5 6x
1 .
4x + 1

18/

x3 x +5
>
.
x +1 x 2

15/

17/

x 1
2.
x3

(2x 5)(x + 2) > 0 .


4x + 3

19/

2x + 3 2
.
3x + 7 3

20/

7x 5
4.
8x + 3

21/

x 3 1 2x
<
.
x+5
x3

22/

3x 4
> 1.
x 2

23/

x2 + 2x
0.
x2 4

24/

x 2
0.
x2 4

25/

(4x + 3)
(2x 5)

26/

3 2x
> 0.
x2

Page - 6 -

29/ f (x) = (2x 4)(x + 1)(6 2x) .

Bi 6.

Bt ng thc v Bt phng trnh

0.

DeThiThuDaiHoc.com
"All the flower of tomorrow are in the seeks of today"

cng hc tp mn Ton 10 Tp IIwww.MATHVN.com

Ths. L Vn on

27/

2x 5
1 .
2x

28/

2x 5 3x + 2
<
.
3x + 2 2x 5

29/

4
3
<
.
3x + 1 2 x

30/

2x 2 + x
1 x
1 2x

33/

35/

x+9

32/

< 0.

(x 1)

(x + 9)

x2
31/
0.
3x 8

34/

(3x + 1)(x 3)

0.

5 2x

36/

0.

x 1

x2 + 6x + 9
> 0.
2x 2 x 1

(x 3)(x + 2) < 1 .
x2 1

37/

2
5

.
x 1 2x 1

38/ x + 1 >

39/

1
2
<
.
x 1 x x2

40/

1
2
3
+
>
.
x +1 x +2 x + 3

41/

(5 6x)
(4x + 1)

42/

1
3
<
.
x 2 3x 4

44/

x +2
x 2
>
.
3x + 1 2x 1

46/

x2 + 3x 1
x .
2x

48/

(x + 2) (x + 6) .
0
x

7
x

2
( )( )

50/

(x 1) (x + 2)
x (x 7)

0.

x 2 (2x 6)

43/

(1 x)(x + 4)

45/

(x 1)(x + 2)

47/

(x + 9)

0.

1 x

0.

< 0.

x 1

4
.
x +1

9
+
+1< 0.
49/
(x 2)(x 3) x 3
51/

x2 3x + 24
< 4.
x2 3x + 3

0.

52/ x 3 6x 2 + 11x 6 0 .

53/ x 3 + 8x 2 + 17x + 10 < 0 .

54/ x 3 + 6x 2 + 11x + 6 > 0 .

55/ 2x 3 5x 2 2x + 2 < 0 .

56/

(x

58/

5x 7
x
3x
<4
+ 2
4.
x5
5 x x 25

57/ 1 <
Bi 7.

3x 2 7x + 8
x +1
2

2.

2x 3 (3x 3) .
2

Gii cc h bt phng trnh sau

"Cn c b thng minh"

DeThiThuDaiHoc.com

Page - 7 -

Ths. L Vn on

Page - 8 -

1/

8x 5 > 15x 8

2
.

3
2 (2x 3) > 5x
4

3/

4
12x x + 1
3
2.
4x 3 2 x
<

3
2

5/

11 x 2x 5
2
.

x8
2
3x
1
+

) 2
(

www.MATHVN.com
Chng 4.

Bt ng thc v Bt phng trnh

2/

4x 5

<x+3
7
.
3x + 8
> 2x 5

4/

x
x + 4
2
3
.
2x 9 19 + x
<

2
3

6/

15x 2 > 2x + 1

3 .

3x 14
2 (x 4) <
2

7/

2x 3 3x + 1

<
4
5 .

5
x
3x + < 8
2
3

8/

5 3x
3x 1 3 (x 2)

1 >

8
2 .
4

4x 1 x 1 4 5x
>

3
18
12
9

9/

3x + 1 2x + 7

4x + 3 > 2x + 19

9x 12 4x + 15
10/
.

19 3x < 7 + 5x

5x + 7

3x
11/ 3
.
1 5x
<
3x
+
4

13

5x 3 2x + 1
2
6 .
12/
2
2
x
<
x
+
2
(
)

x + 3 4 + 2x
13/
.

5x 3 < 4x 1

x + 3

3x
14/ 7
.
1 5x
< 4x + 2

5x 2 < 4x + 5

15/ 2
.
2
x < (x + 2)

7x 5 < 0
16/
.
(2x + 3)(x 1) 0

(1 + x) < x 2 3x + 5
.
17/
x 3 6x 2 7x 5 < x 2 3
(
)

(x 2)(6 x) 0

18/ 4x 3
.

<x+3
2

(x 1)
19/ x 2 0 .

2x 4 > 0

x 2

>7
20/ x 3
.

2x

3
x
+
3

0
(
)(
)

2x + 3

21/ x 1
.
(x + 2)(2x 4)
0

x 1

x + 1
x4

>
22/ 1 2x 3 2x .
2
<1

x + 1

DeThiThuDaiHoc.com
"All the flower of tomorrow are in the seeks of today"

cng hc tp mn Ton 10 Tp IIwww.MATHVN.com

Bi 8.

(x + 1)(x + 4) < 0

23/ 2
.
1

>
2x + 1 x 3

x 1 2x 3

5 3x
24/
x3.
2
3x < x + 5

2
1

25/ 2x 1 3 x .

x < 1

5x 4 < 6
26/ 3
4 .

1 x x + 1

4x 2 1 0

27/ x (x 2) 0
.
2
2x 5x + 2 0

2
x 1
x
x
28/
>4+x .
x + 1
x + 2 2 x

<
1 x
x +1

Tm cc nghim nguyn ca cc h bt phng trnh sau

1/

Bi 9.

Ths. L Vn on

6x + 5 > 4x + 7

7
.
8x + 3
< 2x + 25

2/

15x 2 > 2x + 1

3 .

3x 14
2 (x 4) <
2

Gii cc bt phng trnh sau


1/

4 3x 8 .

2/

2x + 1 3 .

3/

2x 4 x + 12 .

4/

x 2 < x 1.

5/

x 3 < 3x + 15 .

6/

3x 2 > 7 .

7/

5x 12 < 3 .

8/

1 4x < 2x + 1 .

9/

2x 8 7 .

10/

3x + 15 3 .

11/

x 1 >

x +1
.
2

12/

x <

13/

x 2 <

x
.
2

14/

2x 5 x + 1 .

15/

2x + 1 x .

16/

x 2 > x + 1.

17/

2
> 1.
x4

18/

2x 1
>2.
x 1

19/

2
8
> .
x 13
9

20/

x < 2 x 4 +2.

21/

1
2

.
x +2
x 1

22/

"Cn c b thng minh"

DeThiThuDaiHoc.com

4
.
x

x 1
x +1

< 1.
Page - 9 -

www.MATHVN.com
Chng 4.

Ths. L Vn on

23/

x+2 x
x

25/

27/

< 2.

1 3x 2x 1
4x x + 1

24/

> 1.

2x + 2 + x + 2 < 3x 2 .

Bt ng thc v Bt phng trnh

x+3 +x
x +2

> 1.

26/

x 1 + 2x + 6 x 5 .

28/

x 1 + 2x 4 4 x < 2 .

Dng 3. Bt phng trnh bc nht mt n cha tham s



 Gii v bin lun bt phng trnh bc nht dng
iu kin

Kt qu tp nghim

 Gii v bin lun bt phng trnh dng :


. Tnh

hoc
.

Lp bng xt du chung:

T bng xt du, ta chia bi ton thnh nhiu trng hp. Trong mi trng hp ta xt du

ca

hoc

nh qui tc an du.

 Gii v bin lun h bt phng trnh bc nht cha tham s:


Gii

tm tp nghim

tng ng

H c nghim khi

H v nghim khi

H c nghim duy nht khi h c dng

Tp nghim h:

 Lu : Cn nm vng cc php ton trn tp hp phn chng I.


Page - 10 -

DeThiThuDaiHoc.com
"All the flower of tomorrow are in the seeks of today"

cng hc tp mn Ton 10 Tp IIwww.MATHVN.com

Ths. L Vn on

BA
NG
BAI T
TP AAP DU
DUNG
Bi 10.

Bi 11.

Tm tham s m bt phng trnh sau y v nghim


1/

m2 x + 4m 3 < x + m2 .

2/

m2 x + 1 m + (3m 2) x .

3/

mx m2 > mx 4 .

4/

3 mx < 2 (x m) (m + 1) .
2

Gii v bin lun cc bt phng trnh sau


1/

m (x m) x 1 .

2/

mx + 6 > 2x + 3m .

3/

(m + 1) x + m < 3m + 4 .

4/

mx + 1 > m 2 + x .

6/

3 mx < 2 (x m) (m + 1) .
x + 2m > 2 + mx .

5/

m (x 2)
6

x m x +1
>
.
3
2

7/

mx m 2 > 2x 4 .

8/

9/

m2 x 1 x + m .

10/ 2x + m2 mx + 3m 2 .

11/ m (x 2) 2mx + m 1 .

12/ 25m2 x < m2 x 5 .

13/ 2 (x m) (m + 1) 3 mx .

14/

4.

15/

(m

3m + 2 x m 1 .

17/

(m

+ 2m x + 8 < 4mx + m 3 .

19/

(m + 1)(mx 1) > 2 .

20/

21/

(m

22/ x (x m) 0 .

23/

(x 1)(x + m) 0 .

24/

(x 3)(6m 12 x) 0 .

25/

(2x 6)(x m + 1) 0 .

26/

x 3
> 0.
x + 2m + 1

27/

x 4m
> 0.
2x

28/

x 4m
0.
4x

29/

(x + m)(x + 1 m) > 0 .

30/

(2x m)(x + 2 m) 0 .

31/

mx
0.
m+2+x

32/

x + 4m
> 0.
2x m + 4

34/

2x + m 1
> 0.
x +1

(m + 1)(m 2) x m

3m + 2 x m 1 .

33/ m2 (x 1) < m 4mx 3x .


35/

mx m + 1
< 0.
x 1

"Cn c b thng minh"

16/ x + 25m2 5mx + 1 .


18/ m (x + 1) > 1 .

(m

36/

DeThiThuDaiHoc.com

3m + 2 (mx 1) m 2 1 .

x 1 (x m + 2) > 0 .

Page - 11 -

Ths. L Vn on

Bi 12.

Bi 13.

Bi 14.

www.MATHVN.com
Chng 4.

Bt ng thc v Bt phng trnh

Gii v bin lun h bt phng trnh


1/

2x + 1 0

3x + 1 m

2/

4x 1 0

x + 3m 0

3/

(x 1)(4 x) > 0

x m + 1 0

4/

7 x 0
x 5
.

x m 1 0

5/

3
4

>
1 x x + 1 .

x m 1 0

6/

2
5

>
1 x 1 2x .
x m 1 0

7/

x 2

2 < 4x 8
.

2
2

x m + 1) > (x m 1)
(

8/

x + 1 > 0

mx 2 < 0

9/

x 2 0
.
(m + 1) x 1 > 0

x + m 1
10/
.

mx + 2 m

)(

Tm tham s m h bt phng trnh sau c nghim


1/

4x 5 > 3x + 2

3x + 2m + 2 < 0

2/

3x 2 > 4x + 5

3x + m + 2 < 0

3/

4 (x 3) + 1 3 (x 3)

x + m > 1

4/

>2
.
x + 1

5/

2
.
x + m

x 1 2

6/

1
.
m x
x 3

7/

x 7 0

mx m + 12

8/

2x 1 > 0

(3m 2) x m > 0

9/

2x 1 < x + 2

m (m + 1) x + 1 > (m 2) x + 3m + 7

3x + 3 > 2x
10/
.
x 1 2m (x 2m)

x + m 1 > 0
11/
.

3m 2 x > 0

mx 1 > 0
12/
.
(3m 2) x m > 0

x + 4m2 2mx + 1
13/
.

3x + 2 > 2x 1

7x 2 4x + 19
14/
.

2x 3m + 2 < 0

Tm tham s m h bt phng trnh sau v nghim

Page - 12 -

DeThiThuDaiHoc.com
"All the flower of tomorrow are in the seeks of today"

cng hc tp mn Ton 10 Tp IIwww.MATHVN.com

Bi 15.

1/

2x + 7 < 8x 1

m + 5 < 2x

2/

2
(x 3) x + 7x + 1 .
2m 8 + 5x

3/

x m + 3 > 2m

1 < 2 (x 2)

4/

4x + 5 > 11 x
.
2
m mx > 3 (3 x)

5/

>1
.
3 x
x > 3 mx

6/

x 7 0

mx m + 12

7/

(x 1)(x 2) < 0

mx + 1 < 2x + m

8/

2x

1
.
x 1
2x m + 2 > 0

9/

3x + 5 x 1
2
2

.
(x + 2) (x 1) + 9
2
m x + 1 > (3m 2) x + m

1
1

<

10/ x 1 x + 2
.

2
2
x x 3m < x (m 3)

Tm tham s m bt phng trnh c tp nghim l D cho trc


1/

x +m 1

c tp nghim l D = 2; +) .

2/

2x m < 3 (x 1)

c tp nghim l D = (4; +) .

3/

mx 16 2 x m 3

4/

Bi 16.

Ths. L Vn on

4 x m2 + 1 x 5m 0

c tp nghim l D = 38; +) .
c tp nghim l D = 2; 4 .

5/

m 3 (x + 2) m2 (x 1)

c tp nghim l D =  .

6/

m (x + mx ) 1

c tp nghim l D = .

7/

m2 (x 1) 9x + 3m

c tp nghim l D =  .

8/

m2 (x 1) < m 4mx 3x

c tp nghim l D = .

Tm tham s m bt phng trnh tha x D cho trc


1/

xm

x D = 0;2 .

2/

2x + m 2

x D = 1; 4 .

3/

m2 x < 1

x D = (3; 4) .

4/

x 1 > 4m

x D = 2; 4 .

5/

(m

x D = 1;2 .

+1 x < 4

"Cn c b thng minh"

DeThiThuDaiHoc.com

Page - 13 -

www.MATHVN.com
Chng 4.

Ths. L Vn on

6/

5x m 2

x D = (0; 4) .

7/

m 3x 2

x D = (3;1) .

8/

3x 1 < 2 (x + 1)

x D = (m 1; 4 .

9/

x 2m 1

x D = (;2 m) .

10/ 1 2x 0

x D = m 1;2 m .

11/ x 2m + 1 > 0

x D = (m + 1;2m 3) .

12/

(x 1)(x + 3) < 0

x D = m;1 m) .

13/

(2x 1)(x + 4) > 0

x D = (2m 1; m + 3 .

14/

(m

x D = 1;2 .

+ 1 x m (x + 3) + 1 > 0

15/ mx + 2 x + m

x D = 0;2 .

16/ 2 (m 1) x + m > 0

x D = (1; 3) .

17/

(m + 1) x 3m 0

x D = 1;2 .

x D = (0; +) .

18/ mx 3m + 2 > 0
Bi 17.

Bt ng thc v Bt phng trnh

Tm tham s m h bt phng trnh sau c nghim duy nht


1/

x m

2x + 1 0

2/

2x m 0

2 (x 1) 3

3/

(x + 1)(x 2) 0

x 2m 2

4/

(x 2)(x 4) 0

x 2m + 1

5/

(x 3) x 2 + 7x + 1
.

2m 8 + 5x

6/

x y 0

mx m + 12

7/

2
x + 3 x + 1
.
x

m (x 1) 2

8/

2
x x + 3 x
.

m (x 1) 2

9/

x 2 1 x 2 0
( )

.
2
x + (m 1)(2m 1) (3m 2) x

Page - 14 -

2m (x + 1) x + 3
10/
.
4mx + 3 4x

DeThiThuDaiHoc.com
"All the flower of tomorrow are in the seeks of today"

cng hc tp mn Ton 10 Tp IIwww.MATHVN.com

Ths. L Vn on

II Du ca tam thc bc hai v bt phng trnh bc hai




Dng 1. Xt du ca tam thc bc hai Gii bt phng trnh bc hai


 Du ca tam thc bc hai

cng du vi a.

cng du vi a.

: Trong tri.
: Ngoi cng.

 Gii bt phng trnh bc hai


Bc 1. Cho

tm nghim

Bc 2. Lp bng xt du ca

(nu c).
da vo du ca tam thc bc hai.

Bc 3. T bng xt du, suy ra tp nghim ca bt phng trnh.


 Gii bt phng trnh bc hai dng:
Bc 1. Tm iu kin xc nh
Bc 2. Cho

hoc
nu c.
.

tm nghim

Bc 3. Lp bng xt du
Bc 4. T bng xt du

Du ca

.
.

tp nghim S1. Vy tp nghim bt phng trnh:

 Gii h bt phng trnh bc hai mt n dng:


Bc 1. Gii

c tp nghim tng ng l

Bc 2. Nghim ca h l

 Lu
Hai bt phng trnh

c gi l tng ng nu v ch nu

.
Cn s dng thnh tho cc php ton trn tp hp.
"Cn c b thng minh"

DeThiThuDaiHoc.com

Page - 15 -

www.MATHVN.com
Chng 4.

Ths. L Vn on

Bt ng thc v Bt phng trnh

BA
NG
BAI T
TP AAP DU
DUNG
Bi 18.

Lp bng xt du ca cc hm s sau
1/

f (x) = x2 3x + 2 .

2/

f (x) = 3x2 2x + 1 .

3/

f (x) = x2 + 4x + 5 .

4/

f (x) = x2 x 12 .

5/

f (x) = 2x2 + 5x 2 .

6/

f (x) = 4x2 + 12x 9 .

7/

f (x) = 2x2 6x +

8/

f (x) = x 2 + x + 1 .

9/

f (x) = 2x2 + 7x 8 .

9
.
2

10/ f (x) = 3x 2 2x 8 .

11/ f (x) = x2 + 2x 1 .

12/ f (x) = 2x 2 7x + 5 .

13/ f (x) = 3x2 + 2x 5 .

14/ f (x) = x2 5x 6 .

15/ f (x) = 3x2 + 2x

1
.
3

16/ f (x) = 2x2 + 6x

9
.
2

17/ f (x) = 2x2 + 2 1 2 + 6 x + 2 6 . 18/ f (x ) = 6x 2 + 2 3 + 3 2 x 6 .


19/ f (x) = 3x2 + 8 2x + 16 2 .

)(

21/ f (x) = x2 5x x2 5x + 10 + 24 .

23/ f (x) = 3x 2 10x + 3 (4x 5) .

2x 2 x 3
.
25/ f (x) =
4x x 2
27/ f (x) =

22/ f (x) = (4 2x) x2 5x + 4 .

)(

24/ f (x) = 3x 2 4x 2x 2 x 1 .
26/ f (x)

(3x
=

)(

x 3 x2

4x2 + x 3

).

x +1
1
2
.
x 1 x 1

28/ f (x) = mx 2 + (1 2m) x 2 .

30/ f (x) = mx2 m2 + 1 x + m .

29/ f (x) = m2 + 1 x2 2 (m 1) x + 4 .
Bi 19.

20/ f (x) = (x + 4) x2 + 8x + 11 + 4 .

Gii cc bt phng trnh sau


1/

x2 4x + 3 0 .

2/

2x 2 + 5x 3 0 .

3/

7x 2 4x 3 < 0 .

4/

x 2 + 6x 9 > 0 .

5/

3x2 + x + 1 0 .

6/

x2 + 7x 10 0 .

7/

2x 2 + 4x + 3 < 0 .

8/

2x2 5x + 2 0 .

9/

5x 2 + 4x + 12 < 0 .

10/ 16x 2 + 40x + 25 > 0 .

11/ 2x2 + 3x 7 0 .

Page - 16 -

12/ 3x2 4x + 4 0 .

DeThiThuDaiHoc.com
"All the flower of tomorrow are in the seeks of today"

cng hc tp mn Ton 10 Tp IIwww.MATHVN.com

13/ x2 x 6 0 .

14/

3x2 x + 4
> 0.
x2 + 3x + 5

15/

4x2 + 3x 1
> 0.
x 2 + 5x + 7

16/

5x2 + 3x 8
< 0.
x2 7x + 6

17/

x2 + 4x + 4
< 0.
2x2 x 1

18/

x 4 + x2 + 1
0.
x 2 4x 5

x2 7x + 12
19/
> 0.
2x 2 + 4x + 5

(2 x )(x
2

20/

) > 0.

2x + 1

x + 3x + 4
2

21/

x 4 4x 3 + 2x 2
> 0.
x2 x 30

22/

x2 + 6x 7
< 0.
x+7

23/

x2 1
0.
x2 + 1

24/

x2 7x + 10
0.
x2 + 6x 9

25/ 3x 4 x 3 + 4x2 x + 3 0 .

26/

(1 2x)(x

x2 + 4x 5
27/
> 0.
x +1

x2 3x 4
28/
0.
1 2x

29/
Bi 20.

Ths. L Vn on

x2 + 1
< 0.
x 2 + 3x 10

+ x 30 < 0 .

30/

x 2 3x + 2
> 0.
x2 4x + 3

Gii cc bt phng trnh sau


1/

5x 1
< 1.
x2 + 3

2/

3
> 1.
x 8x + 15

3/

4 3x2
> 1.
x2 + x + 1

4/

x 1
5+x.
x +1

5/

x 2
1
< .
2
2
x +1

6/

x2 + 6x 7
2.
x2 + 1

8/

x 1
< x.
x +1

10/

1
3
<
.
x +2 x3

7/

x +1

(x 1)

< 1.

6
.
x5

9/

11/

14x
9x 30
<
.
x +1
x4

12/

13/

1
1
1
+
> .
x 2 x 1 x

14/

1
2
3
+
<
.
x +1 x + 3 x +2

15/

1
1
2

.
x 2 x x +2

16/

x 1 x +1

< 2.
x
x 1

"Cn c b thng minh"

2 (x 4)

(x 1)(x 7)

DeThiThuDaiHoc.com

1
.
x 2

Page - 17 -

www.MATHVN.com
Chng 4.

Ths. L Vn on

Bi 21.

17/

(x 2)(x 4)(x 7) > 1


.
(x + 2)(x + 4)(x + 7)

19/

(x

2x (2x 2)

18x 18
0.
x2 2x

18/

(x 1)(x 2)(x 3) > 1


.
(x + 1)(x + 2)(x + 3)

20/

(x

+ 3x (2x + 3)

32x + 48
0.
x 2 + 3x

Gii cc bt phng trnh sau


1/

x 4 5x2 + 4 < 0 .

2/

x 4 2x 2 63 0 .

3/

x 4 3x 2 + 2 > 0 .

4/

x 6 + 19x 3 216 0 .

5/

(x

6/

(x

7/

x 2 + (x + 1)

8/

1+

10/

3x

2/

x2 + x + 5 < 0

.
2
x 6x + 1 > 0

4/

2
5x + 7x 6 0
.
2
5x 13x + 6 > 0

9/
Bi 22.

Bt ng thc v Bt phng trnh

)(

+ 3x + 1 x2 + 3x 3 5 .
2

1
x 3

<

15
.
x + x +1
2

1
.
2

)(

x 1 x2 x 7 < 5 .
12 7
< .
x
x2
1
< 2.
x

Gii cc h bt phng trnh sau


1/

x 2 > 0

3x 2 + 6x +9 0

3/

2
3x + 8x 3 0
.

6x2 + 17x 7 0

5/

(x 1)(2x 3) 0

x 1 0

6/

x 2 4x + 3 0

.
2
x 6x + 8 < 0

7/

2x2 7x 4 0

.
2
2x 15x + 22 > 0

8/

2
x x 3 3 > 0
.
2
x 2x 2 2 2 0

9/

2
2x 2 3 1 x + 3 1 0
.
2
5x 8x + 3 < 0

2x2 + 9x + 7 > 0
10/
.
2
x + x 6 < 0

2x2 + x 6 > 0
11/
.
2
3x 10x + 3 0

2x2 5x + 4 < 0
12/
.
2
x 3x + 10 > 0

x2 + 4x 7 < 0
13/
.
2
x 2x 1 0

x2 + x + 5 < 0
14/
.
2
x 6x + 1 > 0

x2 4x 5 > 0

15/ 2
.
x + x 20 < 0

x2 2x + 1 > 0

16/ 2
.
x + 2x + 3 > 0

Page - 18 -

DeThiThuDaiHoc.com
"All the flower of tomorrow are in the seeks of today"

cng hc tp mn Ton 10 Tp IIwww.MATHVN.com

Ths. L Vn on

4x 2 + 4x + 1 0

17/ 2
.
3x 9 2 x 3 2 0

2x2 + 9x + 7 > 0

18/ 2
.
2
x + x 6 x 2x + 2 < 0

2
4x 5x 6 0
19/
.
1 x2 4x 2 12x + 5 > 0

x 2 4x + 3 0

20/
.
2
(2x 1) < 4x (x + 1)

x (x + 5) 4x + 2

21/
.
(2x 1)(x + 3) 4x

22/ 4

x 2 2x 7
1.
x2 + 1

1
x2 2x 2
2
1.
13 x 5x + 7

24/ 1 <

10x 2 3x 2
<1.
x2 + 3x 2

3x2 7x + 8
2.
x2 + 1

26/ 3

x2 3x 1
< 3.
x2 + x + 1

23/

25/ 1 <

)(

)(

x2 x 2 2

27/ 2x2 11x + 9 < 0 .


3
2
x x + 2x 2 > 0

2
x + 4x + 3 0
28/ 2x2 x 10 0 .
2
2x 5x + 3 > 0

1 2x 0
29/ 3x 2
.
2
x
+
x

3
x 4x 0
30/ x 2 + x + 1
.

0
2
x 2x 3

2
3x 4x 11 1
31/
.
x2 x 6

2
(3 2x) x 4x + 3 > 0

1
2
2x + 3

+ 2
3
32/
x + 1 x x + 1 x + 1 .

(2x + 3)(4x 2) > 0

"Cn c b thng minh"

DeThiThuDaiHoc.com

Page - 19 -

www.MATHVN.com
Chng 4.

Ths. L Vn on

Bt ng thc v Bt phng trnh

Dng 2. Phng trnh Bt phng trnh cha cn, cha du tr tuyt i



 Phng trnh Bt phng trnh cha n trong du gi tr tuyt i
gii phng trnh, bt phng trnh cha n trong du gi tr tuyt i, ta thng s dng
nh ngha hoc tnh cht ca gi tr tuyt i kh du gi tr tuyt i.
Xem li cch gii phng trnh tr tuyt i (Chng 3. Phng trnh v h phng trnh).

a/ Dng 1.

b/ Dng 2.
c/ Dng 3.
 Lu
.


 Vi


, ta c:

v
.

.
.

 Phng trnh Bt phng trnh cha n trong du cn


gii phng trnh, bt phng trnh cha n trong du cn ta thng dng php nng lu tha

hoc t n ph kh du cn.
Xem li cch gii phng trnh c du cn (Chng 3. Phng trnh v h phng trnh).
a/ Dng 1.

b/ Dng 2.

 Lu : i vi cc phng trnh, bt
phng trnh, khng c dng
chun nh l thuyt, ta thc
hin:
 Bc 1. t iu kin cho cn c ngha.
 Bc 2. Chuyn v sao cho 2 v u
khng m.
 Bc 3. Bnh phng 2 v kh cn.

c/ Dng 3.

Page - 20 -

DeThiThuDaiHoc.com
"All the flower of tomorrow are in the seeks of today"

cng hc tp mn Ton 10 Tp IIwww.MATHVN.com

Ths. L Vn on

BA
NG
BAI T
TP AAP DU
DUNG
Bi 23.

Gii cc bt phng trnh sau


1/

x 1 1.

2/

2x 1 2 .

3/

4x + 2 3 .

4/

x 3 1.

5/

x 2 x 4.

6/

2x + 1 1 3x .

7/

2x 1 x .

8/

5 5x 2 x + 4 .

9/

2x2 + x + 6 2 (x + 2) .

10/

x2 x + 2 x 3 .

12/

x 2 1 2x 2 + 2x .

11/ 2 x2 + x 2 1 + 2x .

Bi 24.

13/

x + 2 3x2 x + 1 .

14/

2x 1 x + 3 .

15/

x2 x 2 x 2 + 2x + 3 .

16/

4x 2 3 2x 2 2x + 1 .

Gii cc bt phng trnh sau


1/

x 1 > x 3.

2/

3/

x 3 2x < 0 .

4/

5 x 2x 7 .

5/

x + 1 < 2x 1 .

6/

x 2 x 3 3.

7/

3x + 2 + 1 < x .

8/

7x + 11 + x + 1 0 .

9/

2x 5 x + 1 0 .

10/

x2 2x x + 1 .

12/

x2 4x > x 3 .

11/ x + 8 x2 4 .

x < 5x 4 .

13/

x2 4x + 3 < x + 1 .

14/

x2 3x + 2 < 2x 1 .

15/

x2 + x 6 x + 2 .

16/

2x 2 3x 5 < x 1 .

17/

x2 + 7x 6 < 3 + 2x .

18/

x2 + 6x 5 > 8 2x .

19/

3x2 + 13 + 2x < 1 .

20/

x2 4x + 5 + 2x 3 .

21/

x 2 + 6x 3 < x + 1 .

22/

2x2 6x + 1 x + 2 > 0 .

24/

3x 5x2 5x 2 .

23/ 2x + 4x 1 > 0 .
25/

x 2 x 12 < 7 x .

26/ 1 x + 2x2 3x 5 0 .

27/

x2 3x 10 < x 2 .

28/ 3 x2 + x + 6 > 2 (1 2x) .

29/

3x2 + 13x + 4 + 2 x < 0 .

30/ 2 3x + x2 2x 1 .

31/

2x2 6x 20 + 2 x > 0 .

32/

"Cn c b thng minh"

DeThiThuDaiHoc.com

x2 2x 15 x 3 .
Page - 21 -

www.MATHVN.com
Chng 4.

Ths. L Vn on

Bi 25.

Bt ng thc v Bt phng trnh

33/

x +2 x .

34/

x2 + x 6 < x 1 .

35/

2x 1 2x 3 .

36/

x2 + 8x 2 (x + 1) .

37/

x2 + 9x > x + 4 .

38/

2x2 1 > 1 x .

39/

x 2 4x 12 > x 4 .

40/

x2 5x 14 2x 1 .

Gii cc phng trnh sau


1/

x 4 4x 2 + 3 = 0 .

2/

x 4 + 10x 2 9 = 0 .

3/

x 4 3x2 4 = 0 .

4/

x 4 x 2 12 = 0 .

5/

x4 x2 + 3 = 0 .

6/

(1 x )(1 + x ) + 3 = 0 .
2

7/

3x 2 = 2x 1 .

8/

2x 3 = x 3 .

9/

4 6x x 2 = 4 + x .

10/

5x + 10 = 8 x .

12/

3x 2 9x + 1 = x 2 .

11/ x 2x 5 = 4 .
13/

3x2 9x + 1 = x 2 .

14/

x 2 3x 2 = 2 (x 1) .

15/

3x + 7 x + 1 = 2 .

16/

x2 + 2x + 4 = 2 x .

17/

x2 + 9 x2 7 = 2 .

18/

x2 3x + 2 = x2 3x 4 .

19/ x 2 6x + 9 = 4 x 2 6x + 6 .

20/

x 2 3x + 3 + x 2 3x + 6 = 3 .

(3 + x)(6 x) + 3 .

21/

3+x + 6x =

27/

x + 3 4 x 1 + x + 8 6 x 1 = 1. 28/

22/

(x + 1)(x + 4) 3

x2 + 5x + 2 = 6 .

(x + 1)(x + 2) = x

+ 3x 4 .

29/

x 2 + 2x 5 + x + 2 + 3 2x 5 = 7 2 .

30/

x + 5 4 x + 1 + x + 2 2 x + 1 = 1.

31/

2x 2 2x 1 2 2x + 3 4 2x 1 + 3 2x + 8 6 2x 1 = 4 .

x 2 + 4356 + x
x x2 + 4356 x 2 = 5 .
x

32/

21 + x + 21 x

33/

35/
37/

Page - 22 -

x + 5 + 3 x + 6 = 3 2x + 11 .

x + 1 + 3 3x + 1 = 3 x 1 .

36/

1+ x + 1 x = 2.

x +1 + 3 x + 2 + 3 x + 3 = 0.

38/

21 + x 21 x
3

21
.
x

34/

2x2 1 > 1 x .

DeThiThuDaiHoc.com
"All the flower of tomorrow are in the seeks of today"

cng hc tp mn Ton 10 Tp IIwww.MATHVN.com

39/

Bi 26.

3x2 + 5x + 8 3x2 + 5x + 1 = 1 .

5x + 7 3 5x 13 = 1 .

42/

9 x +1 + 3 7 + x +1 = 4.

43/

24 + x 3 5 + x = 1 .

44/

47 2x + 4 35 + 2x = 4 .

Gii cc bt phng trnh sau


1/

1 x +1
2.
x

2/

3/

1 1 4x2
< 3.
x

4/

3x2 + 16x 5
2.
x 1

5/

2 + 3x2 + x + 4
< 2.
x

6/

1 21 4x x2
1
< .
x+4
2

x3 + 8
> x 2.
x

1 8x 3
4.
4x

6 + x x2

2x + 5

8/

9/

(x 3)

x 2 4 x2 9 .

10/

11/

(x + 2)

x 2 3x 4 x 2 4 .

12/

13/

1 4x 2x + 1 .

(2x + 1)

6 + x x2
.
x+4

x + 1 < 4x2 1 .

2x + 1 x 8 > 3 .

14/ x 1 x < 0 .

Gii cc phng trnh sau


1/

x2 2
= 2.
x 1

2/

x2 5x + 4 = x + 4 .

3/

x2 8x + 12 = x2 8x + 12 .

4/

x2 5x + 4 = x2 + 6x + 5 .

5/

x 2 1 = x2 2x + 8 .

6/

2x 2 5x 2 = 0 .

7/

x2 1 + x = 1 .

8/

2 3x2 6 x2 = 0 .

9/

x2 2x + 3 = x + 1 .

10/ 2 x x 3 = 3 .

11/

x2 4 + 2x = x + 2 + 1 .

13/
14/
Bi 28.

40/

41/

7/

Bi 27.

x 2 4x 12 = x 4 .

Ths. L Vn on

x + 3 4 x 1 + x + 8 6 x 1 = 1.
2 2 x 1 1 = 3.

12/

x2 1 + x + 1
x (x 2)

= 2.

x + 14x 49 + x 14x 49 = 14 .

14/

16/

x + 1 x 2 = 2 2x2 1 .

2/

2x 4 x + 12 .

Gii cc bt phng trnh sau


1/

4 3x 8 .

"Cn c b thng minh"

DeThiThuDaiHoc.com

Page - 23 -

www.MATHVN.com
Chng 4.

Ths. L Vn on

3/

x <

4
.
x

4/

x2 4x < 5 .

5/

x2 2x < x .

6/

x2 2x 3 < 3x 3 .

7/

x2 3x + x 2 < 0 .

8/

x2 4 + 2x < 4 .

9/

x2 x 1 2x + 5 .

10/

x2 5x + 4
1.
x2 4

11/

x2 4
1.
x2 + x + 2

12/

x2 x x2 1 .

13/

x 6 > x2 5x + 9 .

14/

x2 + 4x + 3 > x2 4x 5 .

15/

x 3 x +1 < 2.

16/

3x 2 7 .

17/

x 3 > 3x + 15 .

18/ 2x 2 5x 3 < 0 .

19/

x2 2x 8 > 2x .

20/

x2 2x 3 3x 3 .

21/ 1 4x > 2x + 1 .

22/ 3 x 1 + x 2 7 > 0 .

23/ x2 + 3x + 2 + x2 + 2x 0 .

24/ x 8 > x2 + 3x 4 .

25/

x 1 + x + 2 < 3.

26/ 2 x 3 3x + 1 x + 5 .

27/

x 2 1 2x < 0 .

28/ x 2 5x 3 x < 2 .

29/ x 2 7x + 12 > x 4 .
31/

4x 3 3x 1 .

33/ x2 2x 3 2 > 2x 1 .

30/

x3 1 1 x .

32/

x 6 > x2 5x + 9 .

34/ 2 x + 1 < x 2 + 3x + 1 .

35/

2
> 1.
x4

36/

37/

x2 4x
1.
x2 + x + 2

38/

39/

41/
Bi 29.

Bt ng thc v Bt phng trnh

x 2

3.

40/

x2 5x + 4
1.
x2 4

42/

x2 5x + 6

2x 1
>2.
x 1

2x 5
x3

+ 1 > 0.

x2 + 3x + 2
1.
x2 3x + 2
2x 1
x 3x 4
2

<

1
.
2

Gii cc bt phng trnh sau

Page - 24 -

DeThiThuDaiHoc.com
"All the flower of tomorrow are in the seeks of today"

cng hc tp mn Ton 10 Tp IIwww.MATHVN.com

1/

3/

5/

7/

x+2

< 1.

x+3 +x
x +2

2/

> 1.

x3
x 5x + 6
2

4/

2.

6/

(2x 3)(2 x 1 + 2)
x 1 2

0.

8/

x +2 x
x

2x 1
x2 + x 2

< 2.

3.

2x 1

(x + 1)(x 2)

1 3x 2x 1
4x x + 1

1
.
2
> 1.

9/

x 2 + 2 x + 1 x2 .

10/

x 1 + 2x + 6 x 5 .

11/

2x + 2 + x + 2 < 3x 2 .

12/

x 1 + 2x 4 4 x < 2 .

13/

( x 1 3)( x + 2 5) < 0 .

14/

15/

x 2 2x 3 < 3x 3 .

16/ x 2 5x 3 x < 2 .

x 1 + 3
5x +x

< 4.

17/ 3x 2 x 3 > 9x 2 .

18/

x 4 + 7x x 2 + 12 .

x + 6 x2 + 6x 7 .

20/

x2 + 3x 4 2 x + 3 + 2 > 0 .

19/

21/

x 1 + 2x 4 3x + 2 3 .

22/

23/

x2 5x + 4
1.
x2 4

24/

25/
Bi 30.

x 1

Ths. L Vn on

x2 + 2x + 3
x2 + x 2

1.

26/

4x2 1
2x 1

< x2 + x 1 .

x +2 + x
x3 +x
x2 x 6
x2 4

< 2.

x.

Gii cc bt phng trnh


1/

1 x + 4 + x 3.

2/

x + 2 x 6 > 2.

3/

22 x 10 x < 2 .

4/

x2 + 9 x2 + 7 2 .

5/

x +2 x +1 x .

6/

2x + 1 2 x x 3 .

7/

x + 3 x 1 < x 2 .

8/

x + 3 7 x 2x 8 .

9/

x + 3 x 1 < x 2 .

10/

x + 2 2x 3 4x 7 .

11/

x + 11 x 4 + 2x 1 .

12/

x +3 + 2x 3.

"Cn c b thng minh"

DeThiThuDaiHoc.com

Page - 25 -

www.MATHVN.com
Chng 4.

Ths. L Vn on

Bi 31.

13/

x + 4 5 4x 1 .

14/

2x 1 + 5 x > 3 2 x .

15/

2x + 3 x + 1 > 1 .

16/

1 x + 4 + x > 1.

17/

1 x + 4 + x > 3.

18/

x + 4 2x 6 1 .

19/

3x + 7 1 + x > 2 .

20/

5 x2 + 5x2 1 > 4x .

Gii cc bt phng trnh sau


1/

5x2 + 10x + 1 7 2x x2 .

2/

(x + 1)(x + 4) < 5

3/

(4 + x)(6 x) x

4/

5/

x (x + 3) x 2 3x + 6 .

6/

(x + 1)(x + 2) < x

7/

x2 + 2x > 2x2 4x + 3 .

8/

x
x +1
2.
> 3.
x +1
x

9/

2.

2x 12 .

3x 1
x

+ 1.
x
3x 1

2x 1
x
3x
+1+
>
.
x
2x 1 2x 1

11/

13/ 3.

2x
x 1 3x 3
+ 4.

+ 10 .
x 1
2x
2x

15/ 5 x +

1
+4.
2x

< 2x +

2 x
Bi 32.

Bt ng thc v Bt phng trnh

x2 + 5x + 28 .

(4 x)(x + 2) x
2

2x 12 .

+ 3x 4 .

10/ 2.

6x 1
2x
<
+ 1.
x
6x 1

12/ 2.

x
x 1 2 (x 1)

+ 3.
x 1
x
x

x
3 2x
12 8x
+ 5.
>
+ 5.
3 2x
x
x

14/
16/

1
3x
>
1.
2
1 x
1 x2

2/

x4 <

Gii cc bt phng trnh sau (nhn lin hip)


1/

3/

4 (x + 1) < (2x + 10) 1 3 + 2x .


2

2x 2

(3

9 + 2x

< x + 4.

(3x + 2)

4/

7/
9/

Page - 26 -

4x + 1 + x 1

x +4 + x +5
16x 2

4x + 1 1

)(

< x +2.

6/

x 1 x 4 > 3 . 8/

4 (3x 2) .

10/

(1 +

1+ x

9x 2

5/

x2

1 + 3x 1

> 2x + 1 .

25x2

6x + 3 + x + 3

x +1 x 2

)(

x.

x + 6 + x 3 < 3.

9x 2

5x 1 2x 1

4x + 5 .

DeThiThuDaiHoc.com
"All the flower of tomorrow are in the seeks of today"

cng hc tp mn Ton 10 Tp IIwww.MATHVN.com

(x + 2)

Ths. L Vn on

11/

Bi 33.

3x + 1 2x 1

x +8.

13/

x+8

x + 3 x 3.

15/

x 2

3x 5 2x + 3 x 8 .

17/

2x 8

x + 3 + 7 x > 2x 4 .

19/

5x 1

3x 2 + 2x 3 > 1 + x .

21/

3x 5

x + 2 + 2x 3 < 5 x .

23/

( 3x +6 +

)( 3x +1

3x 3

12/

3x2 + 5x + 7 3x2 + 5x + 2 1 .

14/

x 1

16/

2x 3

18/

x+3

20/

2x 4

5x 1 + x 1 < 4x .

22/

1 2x

x + 4 + 1 x < 2x + 3 .

3x 2 3. 24/

x 3 8 x 2x 11 .

x 1 x 2 1.

2x 8 7 x > 3 (x 5) .

x + 12 + x 6

)(

x +2 x 4 6.

Gii cc bt phng trnh sau


1/

3/

5/

2 x2 16
x3

(x

3x

_ x3 >

7x
x3

2x 2 3x 2 0 .

6 + x x2
x2 + x + 6

.
2x + 5
x+4

2/

13 x

4/

x 1 + x 3 2 (x 3) + 2x 2 . 6/
2

4 x 2 x
3

3 x+

2 x

7/

x3 + 8
> x 2.
x

9/

x +1 + 3 x +

11/

1 x x2 + 1 < x .

13/

x2 3x + 2 + x2 4x + 3 x 4x 4 .

14/

x2 8x + 15 + x2 + 2x 15 4x 2 18x + 18 .

15/

7x + 7 + 7x 6 + 2 49x2 + 7x 42 < 181 14x .

"Cn c b thng minh"

< 2x +

1
7.
2x

x +1 + x 1 2

8/

(x 1)(3 x) 2 . 10/

x 2 + 9.

x2
.
4

x (x + 4) x 2 + 4x + (x 2) < 2 .

12/

DeThiThuDaiHoc.com

x2 + 2x + x2 x x 2 + 7x .

Page - 27 -

www.MATHVN.com
Chng 4.

Ths. L Vn on

Bt ng thc v Bt phng trnh

Bai tp qua cac ki thi


Bi 34.

Cao ng Bn Cng Hoa Sen khi D nm 2006


Gii BPT:

Bi 35.

x + 11 x 4 + 2x 1 .

x2 + x 6 x + 2 .

x + 3 2 x > 1.

x 1 > x 3.

5 2x 2 x 2x + 5 .

S:

Cao ng Giao Thng Vn Ti nm 2006


x2 4x + 3 2x2 3x + 1 x 1 .

S:

Cao ng S Phm Cn Th khi A nm 2006


Gii BPT:

Page - 28 -

S:

Cao ng S Phm Lo Cai khi A nm 2006

Gii BPT:
Bi 46.

S:

Cao ng S Phm Yn Bi khi B, D1, M nm 2006

Gii BPT:
Bi 45.

S:

Cao ng S Phm Tp. H Ch Minh khi A nm 2006

Gii BPT: 2x 2 x 1 0 .
Bi 44.

S: x (; 3 .

S:

x2 + 2x + 5 4 2x2 + 4x + 3 .

Bi 43.

S: x 4; 5 .

Cao ng S Phm Vnh Phc khi D, M, T nm 2006


Gii BPT:

Bi 42.

3
S: x 0; .
4

1 x +1
2.
x

Cao ng Ti Chnh Qun Tr Kinh Doanh khi A nm 2006


Gii BPT:

Bi 41.

1
S: x ;1 3; +) .
3

H Cao ng khi T, M nm 2004 trng i hc Hng Vng


Gii BPT:

Bi 40.

x2 4x + 3 < x + 1 .

Cao ng iu Dng H chnh quy nm 2004


Gii BPT:

Bi 39.

2
S: x ; .

Cao ng S Phm in Bin khi A, B nm 2005


Gii BPT:

Bi 38.

x2 4x + 5 + 2x 3 .

Cao ng Kinh T Cng Ngh Tp. H Ch Minh khi A nm 2006


Gii BPT:

Bi 37.

S: x (; 0) ; + .

Cao ng K Thut Y T I nm 2006


Gii BPT:

Bi 36.

x2 4x > x 3 .

(x + 5)(3x + 4) > 4 (x 1) .

S:

DeThiThuDaiHoc.com
"All the flower of tomorrow are in the seeks of today"

cng hc tp mn Ton 10 Tp IIwww.MATHVN.com

Bi 47.

Cao ng Kinh T K Thut Thi Bnh nm 2004

x2 3x + 2 + x2 4x + 3 2 x2 5x + 4 . S: x = 1 x 4 .

Cao ng A, B 2009

x + 1 + 2 x 2 5x + 1

2x 8 .

x + 6 > x + 1 + 2x 5 .

x 2 + 2x 15 < x 2 .

x2 + 6x 5 > 8 2x .

S:

S:

S: 3 < x 5 .

i hc Dn Lp Phng ng khi A nm 2000


Gii BPT: 2x2 + 4x + 3 3 2x x2 > 1 .

Bi 57.

S:

Cao ng S Phm Vnh Long khi A, B nm 2005


Gii BPT:

Bi 56.

(4 x)(2 + x) x

Cao ng Giao Thng Vn Ti nm 2005


Gii BPT:

Bi 55.

S:

Cao ng S Phm Kinh T nm 2002


Gii BPT:

Bi 54.

2x 2 1 < 0 .

Cao ng Nng Lm 2000


Gii BPT: 4

Bi 53.

S: 2 x 3 .

Cao ng S Phm Nh Tr Mu Gio TWI nm 2000


Gii BPT: x 2 + x 2

Bi 52.

S: x 1;1 .

Cao ng Hi Quan nm 1999 H phn ban

Gii BPT:
Bi 51.

x2
.
4

1+ x + 1 x 2

Gii BPT:
Bi 50.

S: x = 1 x (1; 3) .

Cao ng S Phm Tp. H Ch Minh nm 1998


Gii BPT:

Bi 49.

2 x2 1 x + 1 .

Gii BPT:
Bi 48.

Ths. L Vn on

S: x 3;1 .

i hc Dn Lp Duy Tn khi D nm 2000


Gii BPT:
1/

x2 3x + 2
x2 + 3x + 2

1.

S:

2/ x + 1 2 x2 1 .
Bi 58.

i hc Dn Lp K Thut Cng Ngh Tp. H Ch Minh (Hutech) nm2000


Gii BPT:

Bi 59.

x + 6 > x + 1 + 2x 5 .

S:

Hc Vin Chnh Tr Quc Gia Phn Vin Bo Ch Tuyn Truyn nm 2000


Gii BPT: x + x2 + 4x > 1 .

Bi 60.

S:

S:

Hc Vin Ngn Hng Tp. H Ch Minh nm 2000

"Cn c b thng minh"

DeThiThuDaiHoc.com

Page - 29 -

www.MATHVN.com
Chng 4.

Ths. L Vn on

Gii BPT: x (x 4) x 2 + 4x + (x 2) < 2 .


2

Bi 61.

i hc Thi Nguyn nm 2000


3

< 2x +

2 x

< 2x +

x + 2 3 x < 5 2x .

7x + 7 + 7x 6 + 2 49x2 + 7x 42 < 181 14x .

6
x < 6.
7

(x + 1)(4 x) > x 2 .

7
S: x 1; .

i hc Dn Lp Hng Bng nm 1999


Gii BPT:

x + 3 x 1 > 2x 1

S: 1 x

3
.
2

i hc Dn Lp K Thut Cng Ngh (HuTech) khi A, B nm 1999


Gii BPT:

Page - 30 -

S:

i hc M a Cht H Ni nm 2000
Gii BPT:

Bi 72.

S: x (9; 4) .

i hc Anh Ninh khi A nm 2000

HD: t = 7x + 7 + 7x 6 0 ...... .

Bi 71.

S: x < 0 x > 5 .

x2 8x + 15 + x2 + 2x 15 4x 2 18x + 18 .

Hc Vin Quan H Quc T nm 2000

Gii BPT:

Bi 70.

S:

x2 + 3x + 2 + x 2 + 6x + 5 2x2 + 9x + 7 .

Gii BPT: (x + 1)(x + 4) < 5 x2 + 5x + 28 .


Bi 69.

S: x 4;5) (6;7 .

i hc Dc H Ni nm 2000
Gii BPT:

Bi 68.

x + 3 7 x 2x 8 .

i hc Bch Khoa H Ni khi D nm 2000


Gii BPT:

Bi 67.

S:

i hc Thy Li H Ni i hc Thng Long nm 2000


Gii BPT:

Bi 66.

1
+2.
2x

i hc Ngoi Thng khi D nm 2000


Gii BPT:

Bi 65.

1
7.
2x

0 < x < 4 3 7

2 .
S:

3
7
x > 4 +

i hc Thy Li nm 2000
Gii BPT: 4 x +

Bi 64.

x2 + 3x + 2 + x 2 + 6x + 5 2x2 + 9x + 7 .

Gii BPT: 3 x +

Bi 63.

S: x 2 3; 2 + 3 .

i hc Bch Khoa Tp. H Ch Minh nm 2000


Gii BPT:

Bi 62.

Bt ng thc v Bt phng trnh

2x + 3 x 2 .

S: x ; 3 + 2 2 .
2

DeThiThuDaiHoc.com
"All the flower of tomorrow are in the seeks of today"

cng hc tp mn Ton 10 Tp IIwww.MATHVN.com

Bi 73.

i hc Dn Lp K Thut Cng Ngh (HuTech) khi D nm 1999


Gii BPT:

Bi 74.

2x 1 8 x .

S:
1

S: x ; + .
4

5x 1 4x 1 3 x .

2x2

(3

9 + 2x

9 7
S: x ; \ {0} .
2 2

< x + 21 .

x + 2 x 1 + x 2 x 1 >

3
.
2

S: x 1; +) .

i hc Bch Khoa H Ni nm 1999


Gii BPT:

Bi 83.

S: x 2; 4 x = 3 .

Hc Vin Ngn Hng nm 1999


Gii BPT:

Bi 82.

12 + x x2
12 + x x2
.

x 11
2x 9

i hc M a Cht nm 1999
Gii BPT:

Bi 81.

S: x 2; 1 x = 3 .

i hc An Ninh H Ni khi D nm 1999


Gii BPT:

Bi 80.

6 + x x2
.
x+4

i hc Ti Chnh K Ton Tp. H Ch Minh nm 1999


Gii BPT:

Bi 79.

6 + x x2

2x + 5

i hc Hu khi D, R, T h phn ban nm 1999


Gii BPT:

Bi 78.

3
S: x ;2 .
2

x + 2 x 1 2x 3 .

i hc Hu khi D, R h cha phn ban nm 1999


Gii BPT:

Bi 77.

1
S: x ; 0 .
12

x 1
x 1
2
3.
x
x

i hc Thy Sn nm 1999
Gii BPT:

Bi 76.

1
S: x ;5 .
2

2x 1 8 x .

i hc M H Ni khi A, B, R, V v D4 nm 1999
Gii BPT:

Bi 75.

Ths. L Vn on

x +1 > 3 x + 4 .

S: x > 0 .

i hc Quc Gia Tp. H Ch Minh t I khi D Hc Vin Ngn Hng nm 1999

Gii BPT: x (x 4) x2 + 4x + (x 2) < 2 .


2

Bi 84.

S: x 2 3; 2 + 3 .

thi chuyn Ton Tin i hc Quc Gia H Ni nm 1988


Gii BPT:

x4 + x2 + 1 + x x2 x + 1

"Cn c b thng minh"

(x

+1

DeThiThuDaiHoc.com

S: x > 0 .

Page - 31 -

www.MATHVN.com
Chng 4.

Ths. L Vn on

Bi 85.

i hc Giao Thng Vn Ti Tp. H Ch Minh nm 1998

3x2 + 6x + 4 < 2 2x x 2

Gii BPT:
Bi 86.

1 1 4x 2
< 3.
x

i hc Ti Chnh K Ton nm 1998


51 2x x2
< 1.
1 x

3x2 + x + 4 + 2
< 2.
x

i hc S Phm Tp. H Ch Minh nm 1998


2 x + 4x 3
2.
x

x2 + x 2 + x 2 + 2x 3 x 2 + 4x 5 . S:

i hc S Phm Qui Nhn nm 1998


9x2 4

Gii BPT:
Bi 94.

5x 2 1

5x 1 x 1 > 2x 4 .

2 x2 16
x3

7x
x3

S: x > 10 34 .

2x 2 3x 2 0 .

S: x

1
hoc x 3 .
2

i hc Ngoi Thng nm 2001


Gii BPT:

Page - 32 -

+ x3 >

i hc D 2002
Gii BPT: x 2 3x

Bi 97.

S: 2 x 10 .

i hc A 2004
Gii BPT:

Bi 96.

2
1 1 5
; .
S: ;
5 5 2
3

3x 2

i hc A 2005
Gii BPT:

Bi 95.

S:

i hc An Ninh nm 1998
Gii BPT:

Bi 93.

1 1
S: ; \ {0} .
2 2

1 1 4x 2
< 3.
x

Gii BPT:
Bi 92.

S:

i hc Ngoi Ng nm 1998
Gii BPT:

Bi 91.

S:

i hc Xy Dng nm 1998
Gii BPT:

Bi 90.

S: (; 2) (0; +) \ {1} .

x 4 2x 2 + 1 > 1 x .

Gii BPT:
Bi 89.

1
1
S: x .
2
2

i hc Lut nm 1998
Gii BPT:

Bi 88.

S: 2 < x < 0 .

i hc Nng Nghip nm 1998


Gii BPT:

Bi 87.

Bt ng thc v Bt phng trnh

1 + x 1 x x .

HD: Lin hip 0 x 1 .

DeThiThuDaiHoc.com
"All the flower of tomorrow are in the seeks of today"

cng hc tp mn Ton 10 Tp IIwww.MATHVN.com

Bi 98.

i hc Dn Lp Phng ng nm 2001
Gii BPT:

Bi 99.

7x 13 3x 9 5x 27

x2 3x + 2 + x2 4x + 3 2 x2 5x + 4

x2

(1 +

1+ x

> x4

8x 2 6x + 1 4x + 1 0 .

2x + 7 5 x 3x 2 .

S:

S: x 1 x 0 .

(x + 5)(3x + 4) > 4 (x 1) .

4
S: x (; 5 ; 4 .

x +1
x +1
2
3 0.
x 1
x 1

S:

i hc Kin Trc H Ni nm 2001


Gii BPT:

Bi 110.

1
x ; + .
2

i hc Giao Thng Vn Ti Tp. H Ch Minh khi A nm 2001


Gii BPT:

Bi 109.

S: x =

i hc Kinh T Quc Dn nm 2001


Gii BPT:

Bi 108.

S: x 2 x 0 .

i hc Cn Th khi D nm 2001
Gii BPT: x (x + 1) x 2 + x + 4 + 2 0 .

Bi 107.

13
.
6

D b i hc khi B nm 2005
Gii BPT:

Bi 106.

S: x 3 x

D b i hc khi D nm 2005
Gii BPT:

Bi 105.

S: 1 x < 8 .

D b i hc khi D nm 2004
Gii BPT: x 2 + 2x 2 + 4x + 3 6 2x .

Bi 104.

S: x = 1 x 4 .

i hc Dn Lp Ngoi Ng Tin Hc nm 1997


Gii BPT: (x 3) x 2 4 x2 9

Bi 103.

x > 5 + 53

2
S:
.

5
53

x <

i hc S Phm Vinh nm 2001


Gii BPT:

Bi 102.

299 + 26304
x 23 .
59

i hc Y Dc Tp. H Ch Minh nm 2001


Gii BPT:

Bi 101.

S:

i hc Y H Ni nm 2001

Gii BPT: 2x 2 + x 2 5x 6 > 10x + 15

Bi 100.

Ths. L Vn on

x2 4x + 3 2x 2 3x + 1 x 1 .

S: x = 1 hay x

1
.
2

i hc khi A nm 2010

"Cn c b thng minh"

DeThiThuDaiHoc.com

Page - 33 -

www.MATHVN.com
Chng 4.

Ths. L Vn on

Gii BPT:

x x

1 2 x2 x + 1

Bt ng thc v Bt phng trnh

1.

HD: Mu s < 0 , chuyn v, p dng k du " = " ca BT B.C.S x =


Bi 111.

i hc S Phm Tp. H Ch Minh nm 1994


2x2 6x + 1 x + 2 0 .

Gii BPT:
Bi 112.

x + 3 x 1 < x 2 .

S: x = 1 x 4 .

S: x

5
x 3.
6

i hc Quc Gia Tp. H Ch Minh nm 1997

Gii BPT: x 2 + 1 x x 2 + 2 + 1 .

S: S = 0;

2 1 .

i hc Lut Tp. H Ch Minh nm 1996


Gii BPT: 5 x +

5
2 x

Page - 34 -

28
.
3

i hc Dn Lp Vn Lang nm khi B, D nm 1997


Gii BPT: (x 3) x 2 + 4 x2 9 .

Bi 116.

S: x >

x2 3x + 2 + x2 4x + 3 2 x2 5x + 4 .

HD: S dng min nghim ca iu kin.

Bi 115.

3 7
x > 3.
2

i hc Quc Gia Tp. H Ch Minh nm 1996


Gii BPT:

Bi 114.

S: x

i hc Nng Lm Tp. H Ch Minh nm 1995


Gii BPT:

Bi 113.

3 5
.
2

< 2x +

1
+4.
2x

3 2

S: 0;

2 3 +2 2

;
+

.
2

DeThiThuDaiHoc.com
"All the flower of tomorrow are in the seeks of today"

cng hc tp mn Ton 10 Tp IIwww.MATHVN.com

Ths. L Vn on

Dng 3. Phng trnh Bt phng trnh H phng trnh c cha tham s



 Tam thc khng i du trn
T nh l v du ca tam thc bc hai, ta suy ra kt qu sau: Cho
(iu kin bt phng trnh c nghim

a/
b/
T , ta c th suy ra iu kin v nghim ca bt phng trnh nh sau:
c/ bt phng trnh

v nghim

d/ bt phng trnh

v nghim

e/ bt phng trnh

v nghim

f/ bt phng trnh

v nghim

 Lu
 Nu

c cha tham s.
.

Trng hp 1.
 Nu

Trng hp 2.

c cha tham s.

Trng hp 1.

, ta cn xt:

, ta cn xt:

Trng hp 2.

 Gii v bin lun bt phng trnh bc hai


Bc 1. Xt
Bc 2. Lp
l
Bc 3.
Bc 4.

nu h s a c tham s.
v tm nghim
(nu c nghim, th lc ny nghim
).

Lp bng xt du a v
trn cng mt bng xt du (bin s l m).
Da vo bng xt du, bin lun nghim ca bt phng trnh.

"Cn c b thng minh"

DeThiThuDaiHoc.com

Page - 35 -

www.MATHVN.com
Chng 4.

Ths. L Vn on

Bt ng thc v Bt phng trnh

BA
NG
BAI T
TP AAP DU
DUNG
Bi 117. nh m biu thc f (x) sau lun dng vi mi x
1/ f (x) = x2 (m + 2) x + 8m + 1 .

2/ f (x) = mx 2 2 (m 1) x + 4m .

3/ f (x) = (m 1) x2 (m 5) x + m 1 .

4/ f (x) = m2 + 1 x2 + 2 (m + 3) x + 1 .

5/ f (x) = (3m + 1) x2 (3m + 1) x + m + 4 . 6/ f (x) = (m + 1) x2 2 (m 1) x + 3m 3 .


Bi 118. nh m biu thc f (x) sau lun khng dng vi mi x (lun lun m)
1/ f (x) = 2x2 + 2 (m 2) x + m 2 .

2/ f (x) = (m + 4) x 2 (m 1) x 1 2m

3/ f (x) = (m 1) x2 + 2 (m + 2) x + m 6 . 4/ f (x) = (m 2) x2 + 2 (m 2) x + 2 .

5/ f (x) = m2 + 4m 5 x2 2 (m 1) x + 2 . 6/ f (x) = (m 4) x2 + (m + 1) x + 2m 1 .
Bi 119. Chng minh cc phng trnh sau lun c nghim vi mi gi tr ca m 
1/ 4x2 + 4 (m + 2) x + 3m + 2 = 0 .
3/

(3 2m) x

+ (3m 2) x + m 1 = 0 .

2/ mx2 + (5m + 6) x + m 1 = 0 .
4/

(3m 1) x

+ 3 (m + 1) x + 1 = 0 .

Bi 120. Chng minh cc phng trnh sau v nghim

(m + 1) x

1/ x 2 + 1 + m 3 x + m 2 + m 3 + 2 = 0 . 2/ x 2 + 1 m 3 x + m 2 m 3 + 2 = 0
3/

(2m

7m + 10 x2 + 2 (m 3) x + 1 = 0 . 4/

+ mx + 7m = 0 .

Bi 121. Tm tham s m cc bt phng trnh sau c v s nghim (BPT tha x  )


1/

x2 mx + m + 3 0 .

2/

3x2 + 2 (m 1) x + m + 4 > 0 .

3/

x 2 (3m 2) x + 2m 2 5m 2 0 .

4/

x 2 + (m + 1) x + 2m + 7 > 0 .

5/

x2 + 2 (1 m) x 9 0 .

6/

x 2 (m 2) x + 8m + 1 > 0 .

7/

2x 2 + (m 2) x m + 4 > 0 .

8/

mx2 + (m 1) x + m 1 < 0 .

9/

mx2 4 (m + 1) x + m 5 > 0 .

10/ mx 2 mx 5 < 0 .

11/ 5x2 4mx + m 0 .

12/ mx 2 + 2mx 1 < 0 .

13/ 4x 2 2 (m + 1) x + m > 0 .

14/

(1 3m) x

15/

1 2
x + 2 (m 2) x + m2 > 0 .
m

16/

(1 m ) x

17/

(m

18/

(m + 1) x

19/

( m + 4) x

20/

(m 1) x

Page - 36 -

+ 3 x2 + 2 (m + 1) x + 1 > 0 .
2

(m 1) x 1 2m < 0 .

2mx + 1 m 0 .

3 m2 1 x + 1 0 .

2 (m + 1) x m + 2 > 0 .

+ 2 (m 1) x 4m < 0 .

DeThiThuDaiHoc.com
"All the flower of tomorrow are in the seeks of today"

cng hc tp mn Ton 10 Tp IIwww.MATHVN.com

21/
23/

(2m 1) x

(m + 1) x + m 0 .

x 2 8x + 20

mx2 + 2 (m + 1) x + 9m + 4

< 0.

25/

x 2 + mx 1
< 1.
2x 2 2x + 3

27/

3(m + 6) x 3(m + 3) x + 2m 3 > 3 .

29/

x 2 3x + 2
3.
x 2 + mx + 1

22/
24/

Ths. L Vn on

(m 2) x

(m 4) x

26/ 4 <

+ 4mx + m 1 0 .

3x 2 5x + 4
2

+ (1 + m) x + 2m 1

2x2 + mx 4
< 6.
x 2 + x 1

x 2 + (m 1) x + 3

28/

> 0.

x2 + x + 1

2.

3x2 4x
2.
x 2 + 2mx + 3m

30/

Bi 122. Tm tham s m cc bt phng trnh sau v nghim


1/

x2 + 6x + m + 7 0 .

2/

x2 + 2 (m 1) x + 1 0 .

3/

(m 2) x

4/

mx 2 + 4x + m < 0 .

5/

mx 2 + 2 (m 1) x + 4 0 .

6/

mx2 4 (m + 1) x + m 5 < 0 .

7/

mx2 + 6mx + 8m 10 0 .

8/

(m + 1) x

9/

(m 1) x

+ 2 (m 1) x + 3m 2 > 0 .

10/

(m + 2) x

11/

(m 2) x

+ (m 2) x + m 0 .

12/

(m 2) x

13/

(m 3) x

+ (m + 2) x 4 > 0 .

14/

(m + 3) x

+ 2 (m 1) x + 4m < 0 .

15/

(m 4 ) x

+ (m + 1) x + 2m 1 0 .

16/

( m + 4) x

(m 4) x 2m + 1 < 0 .

17/

(3 m ) x

2 (2m 5) x 2m + 5 > 0 . 18/

+ 2x 4 0 .

19/ m (m + 8) x 2 2 (m + 8) + 8m + 1 0 . 20/

2 (m 1) x + 4 < 0 .

+ 2 (m 2) + m + 4 0 .

(3m + 1) x

(m

2 (m 1) x + 3m 3 > 0 .

(3m + 1) x + m + 4 0 .

+ 2m 3 x2 + 2 (m 1) x + 1 < 0 .

Bi 123. Tm tham s m cc bt phng trnh sau c nghim


1/

mx2 + 2 (m + 1) x + m 2 0 .

2/

(m + 2) x

3/

(m 2) x

+ 2 (m 2) x + 2 0 .

4/

(m 1) x

5/

2x 2 + 2 (m 2) x + m 2 < 0 .

6/

x 2 + 3x m + 1 < 0 .

7/

x 2 + (m 2) x 8m + 1 0 .

8/

(m

9/

(m 2) x

11/

(3 m ) x

+ 2 (m + 2) x + m + 4 0 .

+ 2 (m + 2) x + m 6 0 .

+ 1 x2 + 2 (m + 3) x + 1 0 .

+ 2 (2m 3) x + 5m 6 = 0 . 10/

(m 5) x

4mx + m 2 = 0 .

2 (m + 3) x + m + 2 = 0 .

(1 + m) x

2mx + 2m = 0 .

"Cn c b thng minh"

12/

DeThiThuDaiHoc.com

Page - 37 -

www.MATHVN.com
Chng 4.

Ths. L Vn on

13/

(2m 3 m ) x
2

+ 2 (2 3m) x = 3 .

Bt ng thc v Bt phng trnh

14/

(m 2) x

4mx + 2m 6 = 0 .

2mx + 2m 0 .

Bi 124. Gii v bin lun cc bt phng trnh sau


1/

x 2 mx + m + 3 > 0 .

2/

(1 + m) x

3/

mx2 2x + 4 > 0 .

4/

x2 2 (m + 3) x + m2 + 4m + 1 > 0 .

5/

x2 2 (m + 3) x + m2 + 4m + 1 < 0 .

6/

3mx2 mx + 1 0 .

7/

(m 1) x

8/

mx2 + (m 3) x + m 3 0 .

9/

mx2 + 4mx + m + 6 0 .

10/

(m + 1) x

11/

(1 m) x

+ 2mx + m + 1 0 .

12/ 2x 2 2 (m + 1) x + 1 0 .

13/

(m 3) x

4 (m 3) x + m 0 .

14/ x2 2x + 2m m2 0 .

4mx + 4 < 0 .

15/ x2 2mx + 3m < 0 .

16/

(m 1) x

17/ x2 2mx + m2 1 0 .

18/

(m + 1) x

19/

(m

+ 1 x2 + 2 (m + 3) x + 1 0 .

20/ x +

2 (m 1) x + 3m 3 0 .

+ 3 (m 1) x + 5 0 .
2mx + 4m > 0 .

1 1 m 1+ m

+
.
x 1+ m 1 m

Bi 125. Tm tham s m bt phng trnh sau tha iu kin x cho trc


1/

x 2 + 2 (m + 1) x m + 3 0

x 0 .

2/

x 2 + 2 (m + 1) x m + 3 0

x 0 .

3/

x 2 (m + 1) x + 1 > 0

x > 0 .

4/

(3 m ) x

x < 0 .

5/

x 2 2 (m 2) x + m 2 0

x 0;1 .

6/

(m 2) x

x 0;1 .

7/

x 2 2mx + 3m 2 > 0

x (1;2) .

8/

(m + 1) x

x (1;1) .

9/

(x m + 1)(x m + 3) < 0

x (1;2) .

10/

(x + 3 2m)(x + 3m 2) < 0

x 2; 3 .

11/

(x 2m)(x + m 1) > 0

x 3; +) .

12/
Page - 38 -

2 (m + 1) x + 1 > 0

2 (m 2) x + 1 0

2 (2m 1) x < 3 (1 2m)

x 2 + 2x + 4

x 2 (m + 1) x + 4

<2

x  .

DeThiThuDaiHoc.com
"All the flower of tomorrow are in the seeks of today"

cng hc tp mn Ton 10 Tp IIwww.MATHVN.com

Ths. L Vn on

Bi 126. Tm min xc nh ca hm s ty theo gi tr ca m


1/

y = f (x ) = 2x 3 +

(m 1) x

2/

y = f (x ) =

4x + 5

3/

y = f (x) = 3x 2 + mx 7

4/

y = f (x ) = 5x 2 + 2m

5/

y = f (x) =

(2 3m) x

+ 3 (m 1) x + m .

+ 2mx + m 1

+ x 2.

3x 2 4x
2 .
x 2 + mx + m

(m + 1) x

mx2 + (m + 2) x + 2
x 1

2 (m 1) x + 2 2m .

+ m2 3m + 1 .

Bi 127. Tm tham s m hai bt phng trnh sau y tng ng nhau


1/

4x 2 8x + 3 > 0

&

x2 2x + 1 m4 0 .

2/

x 2 + 8x + 7 > 0

&

mx 2 2x + 4 > 0 .

3/

2x 2 + 5x + 3 < 0

&

x 2 + 3x m + 1 < 0 .

4/

x2 x 20 0

&

x 2 + 2 (m 1) x + 1 0 .

Bi 128. Tm tham s m bt phng trnh: (1 m) x 2 + 2mx + m 6 0


1/

C nghim.

2/

C duy nht mt nghim.

3/

C nghim l mt on trn trc s c di bng 1.

Bi 129. Tm tham s m bt phng trnh: mx 2 2 (m 1) x m 5 0


1/

C nghim.

2/

C duy nht mt nghim.

3/

C nghim l mt on trn trc s c di bng 2.

Bi 130. Tm tham s m nghim cc h sau tha yu cu bi ton


1/

2
x 6x + 5 < 0
x2 2 (m + 1) x + m2 + 1 = 0

C nghim.

2/

x 3 2x 2 5x + 6 < 0

2
2
x m 1 x + 3m 1 = 0

C mt nghim m v mt nghim dng.

3/

2
x + 7x 8 < 0
x2 (3m + 1) x + m (2m + 1) = 0

V nghim.

"Cn c b thng minh"

DeThiThuDaiHoc.com

Page - 39 -

www.MATHVN.com
Chng 4.

Ths. L Vn on

Bt ng thc v Bt phng trnh

4/

x2 2mx + 2m2 + m 2 = 0

2
x 2mx + 2m > 0

C nghim duy nht.

5/

2
2
x + m = 4
x2 + (5m + 2) x + 4m2 + 2m < 0

C nghim.

6/

x2 (m + 3) x + 3m < 0

2
x + (m + 9) x + 9m < 0

C nghim.

7/

x2 2mx + 4m 0

2
x mx + 8m 0

C nghim duy nht.

8/

x2 + (m + 1) x + 1 0

(m + 1) x2 + x + 1 0

C nghim duy nht.

9/

2
2x 3x 2 0
mx 2 + 2mx + 1 0

V nghim.

2
x 5x + 4 > 0
10/ 2
x 4 (m + 1) x + 4m2 + 8m + 3 0

C nghim.

(2m + 1) x + 3 > (x 3) m
11/
(m 2) x2 4x 1 > 0

C nghim.

m2 (x 2) + (m + 1) x > 2 + mx

12/
2
(m + 1) x + x + 2 > 0

C nghim.

x2 (2m + 1) x 4m 0

13/ 2
x 2mx + 3 2m 0

C nghim duy nht.

x2 4 (m + 1) x + 4m2 + 8m + 3 0
14/
2
x 5x + 4 > 0

C nghim.

x2 + (m + 1) x + 1 0
15/

(m + 1) x2 + x + 1 0

C nghim duy nht.

2x2 3x 2 0
16/
2
mx + 2mx + 1 0

V nghim.

Bi 131. Tm tham s m cc h sau c tp nghim l 


1/

6<

3x 2 mx 6
x2 + x + 1

< 9.

2/

3x 2 mx + 5
2x 2 x + 1

<6.

Bi 132. Tm tham s m cc h phng trnh c nghim tha yu theo sau ca bi ton


Page - 40 -

DeThiThuDaiHoc.com
"All the flower of tomorrow are in the seeks of today"

cng hc tp mn Ton 10 Tp IIwww.MATHVN.com

1/

2
3
x 3x 10x + 24 > 0
x 2 + 2 m2 1 x 2m + 1 = 0

C hai nghim m.

2/

2
3
x 3x 10x + 24 > 0
2
x + 2 m2 1 x 2m + 1 = 0

C nghim duy nht.

3/

x2 + (2m + 1) x + m2 + m 2 = 0

4
x 4x2 + 4 < 0

C nghim duy nht.

4/

x2 2 (m + 3) x + m2 + 6m + 5 = 0

4
x 10x 2 + 9 < 0

C nghim duy nht.

5/

2
x 2x + m = 0
x 4 4x 3 + 4x2 + m 10 < 0

C nghim duy nht.

6/

2
x 2x 4 + m 0
x 4 6x2 8x + 20 m 0

C nghim.

(
(

)
)

Ths. L Vn on

x2 4x + 3 0

Bi 133. Cho h bt phng trnh: 2


. Tm tham s m :
x 8x + 14 + m 0
1/

H v nghim.

2/

H c nghim duy nht.

3/

H c nghim l mt on trn trc s c di bng 1.

x2 3x + 4 0

Bi 134. Cho h bt phng trnh: 2


3x 2 (m + 1) x m2 + 2m 3 0

1/

Tm m h v nghim.

2/

Tm m h c nghim duy nht.

x2 (3m 1) x + 5m + 6 = 0

Bi 135. Cho h bt phng trnh: 2


x 2x < 0

1/

Tm m h c ng mt nghim.

2/

Tm m h c hai nghim phn bit.

x2 (3m 1) x + 2m2 m < 0

Bi 136. Cho h bt phng trnh: 2


x + m2 = 4

1/

Tm m h c nghim.

2/

Tm m h c ng mt nghim.

3/

Tm m h c hai nghim phn bit.

"Cn c b thng minh"

DeThiThuDaiHoc.com

Page - 41 -

www.MATHVN.com
Chng 4.

Ths. L Vn on

Bt ng thc v Bt phng trnh

x2 2x 4m 0

Bi 137. Cho h bt phng trnh: 2


x 4x + m 0

1/

Tm m h c nghim.

2/

Tm m h c nghim duy nht.

(m 1) x 2 + (m 1) x + m 1 0

Bi 138. Cho h bt phng trnh: 2


mx 2x + 2m + 1 0

1/

Tm m h c nghim.

2/

Tm m h c nghim duy nht.

Bi 139. Xc nh m h sau c nghim tha yu bi ton


1/

2
x 2mx 0

x 1 + m 2m

C nghim duy nht.

2/

2
x 3x 4 0
3
x 3x x m2 15m 0

C nghim.

Bi 140. Gii v bin lun h bt phng trnh sau


1/

mx + 1 0

.
2
x 3x 10 0

2/

2
x x 6 0
.
x2 (m + 1) x + m > 0

3/

(x 1)(x 2m) 0

(x + 2)(x + m) 0

4/

2
x + 2x 8 0
.
x2 + (1 m) x 2 (m + 1) < 0

5/

2
x 6x + 8 0
.
x2 2 (m + 1) x + m2 + 2m > 0

6/

x2 + (1 3m) x m (1 2m) 0

.
2
x + (m 4) x 2 (m 2) 0

Bi 141. Tm tham s m cc h sau c nghim tha yu cu ca bi ton


1/

2
2

(x 1) + (y 3) 4

x m 2 + y 3 2 1
) ( )
(

C nghim.

2/

2
2

(x 1) + (y + 1) m

x + 1 2 + 3 y 2 m
) ( )
(

C nghim duy nht.

3/

2
2
x + (y m + 1) 4

x + m 2 + y + 1 2 m 2 2
) ( ) (
)
(

C nghim.

4/

2
2

(x 1) + (y 1) 2
x y + m = 0

Nghim ng x 0;2 .

Page - 42 -

DeThiThuDaiHoc.com
"All the flower of tomorrow are in the seeks of today"

cng hc tp mn Ton 10 Tp IIwww.MATHVN.com

Ths. L Vn on

x2 2 (m + 3) x + m2 + 6m + 5 y
Bi 142. Cho h bt phng trnh:

(x y m 2)(x y m 5) 0

1/

Gii h bt phng trnh khi m = 1 .

2/

Tm tham s m tp nghim ca h cha on 2; 4 .


y x x 1 0
Bi 143. Cho h bt phng trnh:
y 2 + x + 1 1 0

1/

Gii h khi y = 2 .

2/

Tm cc nghim ca h.

Bi 144. Cho bt phng trnh: mx x 3 m + 1

()

1
.
2

1/

Gii bt phng trnh () khi m =

2/

Vi gi tr no ca tham s m th bt phng trnh c nghim.

(m + 2) x m x + 1

Bi 145. Cho bt phng trnh:

()

1/

Gii bt phng trnh () khi m = 1 .

2/

Tm tt c gi tr ca m bt phng trnh () c nghim x tha: 0 x 2 .

Bi 146. Tm tham s m > 0 bt phng trnh:


Bi 147. Tm tham s m :

x x 1 > m c nghim.

(1 + 2x)(3 x) > m + (2x

1
5x + 3 tha x ; 3 .
2

Bi 148. Tm nghim ca bt phng trnh: x + 1 x2 < x 1 x 2 trn on 0;1 .

Bi 149. Cho bt phng trnh: x 2 + 1 + m x x 2 + 2 + 4

()

1/

Gii bt phng trnh () khi m = 3 .

2/

Tm tham s m bt phng trnh () c tha x 0;1 .


Bi 150. Cho bt phng trnh: x 2 x 1 m + 1

()

1/

Gii bt phng trnh () khi m = 0 .

2/

Vi gi tr no ca tham s m th bt phng trnh () c nghim.

Bi 151. Cho x 0;1 . Chng minh rng:


x + 1 x + 4 x + 4 1 x 2 + 2 2 . ng thc

xy ra khi no ?

Bi 152. Gii bt phng trnh: 2x + a 2 x2 > 0 vi a l s cho trc.


"Cn c b thng minh"

DeThiThuDaiHoc.com

Page - 43 -

www.MATHVN.com
Chng 4.

Ths. L Vn on

Bt ng thc v Bt phng trnh

Bi 153. Tm m bt phng trnh: x m x 1 > m + 1 c nghim.


Bi 154. Gii bt phng trnh:

1 x x 2 + 1 > 0 . Suy ra bng xt du ca hm s

f (x) = 1 x x2 + 1 .
Gii bt phng trnh:

1 x x2 + 1 < x .

Bi 155. Cho a thc: P (x) = 2x 3 + mx2 + nx + p Trong m, n, p l hng s.


1/

Xc nh m, n, p P (x) chia ht cho a thc Q (x) = 2x 3 + 3x2 1 .

2/

Vi cc gi tr tm c ca m, n, p , gii bt phng trnh: P x 1 > 0 .

Bi 156. Vi gi tr no ca tham s m th bt phng trnh: x 2 + 2 x m x + m2 + x 1 0 c


nghim.
Bi 157. Tm tham s m bt phng trnh: x 2 + x m < 3 c nghim m.
Bi 158. Cho bt phng trnh:

x (2 x ) + m + 1 x 2 2x + 3

()

1/

Tm m bt phng trnh () c nghim.

2/

Tm m di min nghim ca bt phng trnh () bng 2.

Bi 159. Cho bt phng trnh:

x (6 x ) x 2 6x + m + 2

()

1/

Tm tham s m bt phng trnh () c nghim.

2/

Tm tham s m min nghim ca bt phng trnh () thuc on 2; 4 .


Bi 160. Tm cc gi tr ca m sao cho bt phng trnh: x x 3 < m c tha x 1; 4 .


Bi 161. Tm tham s m h sau c nghim
1/

x2 (m + 2) x + 2m < 0

.
2
x + (m + 7) x + 7m < 0

2/

3/

x2 2x m + 1 0

.
2
2
x (2m + 1) x + m + m 0

4/

2
x + x 2 + x < 1
.
2
x 2 (m + 2) x + 1 > 0

x 4 4x 3 + 4x2 4x + 3 < 0

Bi 162. Cho h bt phng trnh: 2


x mx -3 0

1/

Gii h () khi m = 2 .

2/

Tm tham s m h () c nghim.

Page - 44 -

2
2
2
x + 2 3m x 6m < 0
.
2
x (2m + 5) x + m2 + 5m + 6 0

()

DeThiThuDaiHoc.com
"All the flower of tomorrow are in the seeks of today"

cng hc tp mn Ton 10 Tp IIwww.MATHVN.com

x 4 5x2 + 4 < 0
Bi 163. Cho h bt phng trnh:
2
x + (2m + 1) x + m2 + m 2 = 0

1/

Gii h () vi m = 1 .

2/

Tm tham s m h () c nghim.

3/

Tm tham s m h () c nghim duy nht.

Ths. L Vn on

()

2
2
4
x + 8x + 16x + 16m + 32m + 16 = 0
2

Bi 164. Cho h bt phng trnh: x2 + 4x


5

x + 2)
(

1/

Gii h () vi m = 2 .

2/

Tm tham s m h () c nghim.

()

2
2
x + (y + 1) m
c nghim duy nht.
Bi 165. Tm tham s m h:
x + 1 2 + y2 m
(
)

x + y = 1
Bi 166. Gii v bin lun h:
.
2
x + y2 a

x + 3y (x + y) + m
Bi 167. Tm tham s m h:
c nghim duy nht.
x y 2 3y x m
)
(

Bai tp qua cac ki thi


Bi 168.

Cao ng Giao Thng nm 2003

3x2 + 2x 1 < 0

Gii h bt phng trnh: 3


.
x 3x + 1 > 0

Bi 169.

Cao ng Kh Tng Thy Vn khi A nm 2003

x2 2x 0
Gii h bt phng trnh:
.
4
x 5x2 + 4 0

Bi 170.

i hc kin trc Tp. H Ch Minh nm 1994


Cho bt phng trnh: mx x 3 m + 1 .
1/ Gii bt phng trnh vi m =

"Cn c b thng minh"

1
.
2

DeThiThuDaiHoc.com

Page - 45 -

www.MATHVN.com
Chng 4.

Ths. L Vn on

Bt ng thc v Bt phng trnh

2/ Vi gi tr no ca tham s m th bt phng trnh c nghim ?


S: 1/ 3 x 7 .
Bi 171.

2/ m

1+ 3
.
4

i hc Ti Chnh K Ton Tp. H Ch Minh nm 1995


2

y x x 1 0
Cho h phng trnh:
.
y 2 + x + 1 1 0

1/ Gii h bt phng trnh khi y = 2 .


2/ Tm tt c cc nghim nguyn ca h.
S: 1/
Bi 172.

1 5
x 0.
2

2/ S =

{(0;2), (1;3)} .

i hc Y Dc Tp. H Ch Minh nm 1996


Tm a bt phng trnh

x x 1 a c nghim vi a l tham s dng.

S: 0 < a < 1 .
Bi 173.

i hc Ngoi Thng C S 2 nm 1999

x2 8x + 7 0
Vi gi tr no ca m th h bt phng trnh
c nghim ?
2
x (2m + 1) x + m2 + m 0

Xc nh m h bt phng trnh c mt nghim duy nht ?


Bi 174.

Hc Vin Quan H Quc T nm 1997

x2 (m + 2) x + 2m < 0
Tm m h
c nghim
2
x + (m + 7) x + 7m < 0

Bi 175.

S: m < 0 .

i hc Thng Mi nm 1997

2
x 2x + 1 m 0
Tm m h 2
c nghim
x (2m + 1) x + m2 + m 0

Bi 176.

i hc Thy Li nm 1998

x2 2mx < 0
Tm m h
c nghim

x 1 + m 2m

Bi 177.

i hc Thng Mi nm 1998

2
x 3x + 4 0
Tm m h 3
c nghim
x 3x x m2 15m 0

Bi 178.

i hc Kinh T Quc Dn H Ni nm 1999

2
x + 5x + 4 < 0
.
x 3 + 3x2 9x 10 > 0

Page - 46 -

S: 4 < x < 1 .

DeThiThuDaiHoc.com
"All the flower of tomorrow are in the seeks of today"

cng hc tp mn Ton 10 Tp IIwww.MATHVN.com

Ths. L Vn on

GC V CNG THC LNG GIC

Chng



A H THC LNG GIC C BN


H thc lng gic c bn
.

..


.

BA
NG
BAI T
TP AAP DU
DUNG
Bi 1.

Chng minh cc ng thc sau


1/

cos2 x sin2 x = 1 2 sin2 x .

2/

2 cos2 x 1 = 1 2 sin2 x .

3/

3 4 sin2 x = 4 cos2 x 1 .

4/

sin x cot x + cos x tan x = sin x + cos x .

5/

sin 4 x + cos4 x = 1 2 sin2 x cos2 x .

6/

cos 4 x sin 4 x = cos2 x sin2 x .

7/

4 cos2 x 3 = (1 2 sin x)(1 + 2 sin x) . 8/

9/

sin4 x cos4 x = 1 2cos2 x = 2sin2 x 1 . 10/ sin 3 x cos x + sin x cos3 x = sin x cos x .

11/ tan2 x sin2 x = tan2 x sin2 x .


Bi 2.

(1 + cos x)(sin

x cos x + cos2 x = sin2 x .

12/ cot2 x cos2 x = cot2 x cos2 x .

Chng minh cc ng thc sau


2/

1 cos x
sin x
=
.
sin x
1 + cos x

1
1
+
= 1.
1 + tan x 1 + cot x

4/

1 1 1 + 1 + tan2 x = 0 .

cos x
cos x

5/

1 + sin2 x
= 1 + 2 tan2 x .
1 sin2 x

6/

tan x tan y =

7/

1 cot4 x =

8/

tan x +

9/

(1 cos x)(1 + cot x) = 1 + cos x .

10/

1 + cos x 1 cos x
4 cot x

=
.
1 cos x 1 + cos x
sin x

11/

sin x
1 + cos x
2
+
=
.
1 + cos x
sin x
sin x

12/

sin x + cos x 1
cos x
=
.
sin x cos x + 1 1 + sin x

13/

sin2 x + 2 cos x 1
cos x
=
.
2
1 + cos x
2 + cos x cos x

14/

sin x + cos x 1
2 cos x
=
.
1 cos x
sin x cos x + 1

1/

tan x + cot x =

3/

1
.
sin x cos x

2
1

.
2
sin x sin 4 x
2

"Cn c b thng minh"

DeThiThuDaiHoc.com

tan x + tan y
.
cot x + cot y

cos x
1
=
.
1 + sin x
cos x

Page - 47 -

www.MATHVN.com
Chng 5. Cung gc lng gic Cng thc lng gic

Ths. L Vn on

Bi 3.

Bi 4.

Bi 5.

Chng minh cc ng thc sau


1/

sin 6 x + cos6 x = 1 3 sin2 x cos2 x .

2/

sin6 x cos6 x = sin2 x cos2 x 1 sin2 x cos2 x .

3/

sin 8 x + cos8 x = 1 2 sin2 x cos2 x 2 sin 4 x cos 4 x .

4/

sin 8 x cos8 x = sin2 x cos2 x 1 2 sin2 x cos2 x .

)(

)(

Chng minh cc ng thc sau


1/

1 + sin x + cos x + tan x = (1 + cos x)(1 + tan x) .

2/

(1 + tan x)(1 + cot x) sin x cos x = 1 + 2 sin x cos x .

3/

(1 + tan x) cos

4/

sin 2 x tan x + cos2 x cot x + 2 sin x cos x = tan x + cot x .

5/

sin2 x tan2 x + 4 sin2 x tan2 x + 3 cos2 x = 3 .

x + (1 + cot x ) sin2 x = (sin x + cos x ) .


2

Chng minh cc ng thc sau


1/

sin2 cos2
tan 1
=
.
1 + 2 sin cos
tan + 1

2/

tan2 sin2
= tan 6 .
cot2 cos2

3/

cos2
sin2
+
= 1 sin cos .
1 tan 1 cot

4/

tan sin
1
=
.
3
sin
cos (1 + cos )

5/

1
= tan2 + cot2 + 2 .
2
sin cos2

6/

1
3 tan2
2

tan

=
+ 1.
cos2
cos2

7/

tan2 tan2
sin2 sin2
=
.
tan2 tan2
sin2 sin2

8/

1 cos (1 + cos )

1 = 2 cot .
2

sin sin

9/

(1 cos )(1 + cot ) = 1 + cos .

10/ 1 + tan + tan2 + tan3 =

11/

sin
cos
1 + cot2

=
.
sin + cos cos sin 1 cot2

12/

1 sin
1 + sin
2
13/

= 4 tan .
1 + sin
1 sin

Bi 6.

sin + cos
.
cos3

tan2 1 + cot2
1 + tan4
.
=
.
1 + tan2 cot2
tan2 + cot2
2

1 cos
1 + cos
2
14/

= 4 cot .
1 cos
1 + cos

Rt gn cc biu thc sau


1/ P1 = sin 4 + sin2 cos2 .

2/ P2 = sin 4 cos 4 + cos2 .

3/ P3 = sin2 + sin2 cot2 .

4/ P4 = cos2 + cos2 cot2 .

5/ P5 = 1 sin2 cot2 + 1 cot2 .

Page - 48 -

6/ P6 = sin2 tan + cos2 cot + 2sin cos .

DeThiThuDaiHoc.com
"All the flower of tomorrow are in the seeks of today"

cng hc tp mn Ton 10 Tp IIwww.MATHVN.com

7/

P7 =

2 cos2 1
.
sin + cos

8/

9/

P9 =

cot + cot
.
tan + tan

10/ P10 = tan +

sin x + tan x
sin x cot x .
tan x

11/ P11 =

cos2 x cot2 x

13/ P13 =

sin x tan x

17/ P17 =
19/ P19 =
21/ P21 =

sin2 x + sin2 x tan2 x


2

cos2 x sin2 y
2

sin x sin y

1 cos
sin

12/ P12 =

16/ P16

cos x tan x
sin2 x

cot x cos x .

1 sin2 x cos2 x
2

cos x

cos2 x .

tan x sin x cos x

20/ P20 =

23/ P23 = (1 + tanx) cos2 x +(1 + cotx) sin2 x .

cos
.
1 + sin

cot2 x cot2 y .

1 cos x 1 sin x
+
+ tan x + cotx .
sin x
cos x

1
.
1 + cos

(sin x + cos x)
=

18/ P18 =

cos x + cos x tan x


2

P8 =

14/ P14 =

sin3 x + sin x cos2 x cos x


.
15/ P15 =
1 2 sin x cos x

Bi 7.

Ths. L Vn on

cot2 x cos2 x
2

cot x

+ 1 cos2 x .

1
sin x
cos x cot x

.
sin x 1 + cot x 1 + tan x

1
1
1 + cotx
.
22/ P22 = 1 + cotx +
sinx
sinx

24/ P24 =

1
sinx cot2 x cos2 x

0 < x < .

Bin i cc biu thc sau thnh tch s


1/

A = 2 cos2 x 1 .

2/

B = 3 4 sin2 x .

3/

C = sin x cos x + cos2 x 1 .

4/

D = sin2 x + sin x cos x 1 .

5/

E = 1 + sin x + cos x + tan x .

6/

F = tan x cot x + sin x + cos x .

7/

G = cos x tan2 x (1 + cos x) .

8/

H = 3 4 cos2 x sin x (2 sin x + 1) .

9/

I = sin2 x 3 cos2 x + 6 cos x 2 sin x .

10/ J = cos3 x sin3 x + sin x + cos x .


11/ K = cos3 x + cos2 x + 2 sin x 2 .
12/ L = cos2 x + sin3 x + cos x .
13/ M = 1 + cos x + cos2 x sin x (1 + cos x) .

"Cn c b thng minh"

DeThiThuDaiHoc.com

Page - 49 -

www.MATHVN.com
Chng 5. Cung gc lng gic Cng thc lng gic

Ths. L Vn on

14/ N = 2 cos3 x + 2 cos2 x + sin x 1 .


15/ O = cos3 x + sin3 x + 2 sin2 x 1 .
16/ P = (2 sin x 1)(2 cos x + 2 sin x + 1) 3 + 4 cos2 x .
17/ Q = (2 cos x 1)(sin x + cos x) 1 .
18/ R = 4 sin3 x + 3 cos3 x 3 sin x sin2 x cos x .
19/ S = (1 + sin x) tan2 x (1 + cos x) .
20/ T = 2 5 sin x + 3 (1 sin x) tan2 x .
21/ U = 2 sin x cos x 2 sin2 x + 3 sin x cos x 1 .

22/ V = tan x 3 cot x 4 sin x + 3 cos x .


23/ X = 3 sin x + 2 cos x 3 tan x 2 .
24/ Y = 2 (tan x sin x) + 3 (cot x cos x) + 5 .
25/ Z = 3 (cot x cos x) 5 (tan x sin x) 2 .
Bi 8.

Chng minh cc biu thc sau khng ph thuc vo bin x


1/

A = cos4 x sin4 x + 2 sin2 x .

2/

B = sin4 x + sin2 x cos2 x + cos2 x .

3/

C = cos4 x + sin2 x cos2 x + sin2 x .

4/

D = cos4 x 2 cos2 x 3 + sin 4 x 2 sin2 x 3 .

5/

E = sin6 x + cos6 x 2 sin4 x cos4 x + sin2 x .

6/

F = sin x.

7/

G = sin 4 x + 4 cos2 x + cos4 x + 4 sin2 x .

8/

H = cos2 x cot2 x + 5 cos2 x cot2 x + 4 sin2 x .

9/

I = (1 + cot x) sin 3 x + (1 + tan x) cos3 x sin x cos x .

1
1
+
,
1 + cos x 1 cos x

0 < x <

)(


.
4

10/ J = sin 4 x + cos4 x 1 tan2 x + cot2 x + 2 .

11/ K = 3 sin 8 x cos8 x + 4 cos6 x 2 sin6 x + 6 sin 4 x .

12/ L = sin 4 x 1 + sin2 x + cos4 x 1 + cos2 x + 5 sin2 x cos2 x + 1 .

) (
2

13/ M = 2 sin 4 x + cos4 x + sin2 x cos2 x sin 8 x + cos8 x .

Page - 50 -

DeThiThuDaiHoc.com
"All the flower of tomorrow are in the seeks of today"

cng hc tp mn Ton 10 Tp IIwww.MATHVN.com

Bi 9.

Chng minh cc biu thc sau khng ph thuc vo bin x


2
cot x + 1
.
+
tan x 1 cot x 1

1/

A=

2/

(1 tan x)
B=

3/

C=

4/

(1 cot x)
D=

5/

E=

6/

F=

7/

G=

8/

H=

4 tan x

4 sin x cos2 x

1 tan2 x
1 + tan2 x 1 + cot2 x .
tan x

cot x

1 sin6 x
cos6 x

)(

Bi 10.

Ths. L Vn on

1
2

sin x cos2 x

3 tan2 x

cos2 x

tan2 x cos2 x

sin2 x
cot2 x cos2 x
2

cot x

cot2 x sin2 x
cos2 x

sin x cos x
.
cot x

sin 4 x + cos4 x 1
sin 6 x + cos6

Cho f (x ) = 2 (1 cos x ), g (x) = 2 + cos x 3. sin x, h (x) = 2 + cos x + 3. sin x .


Chng minh cc i lng sau khng ph thuc vo bin: A = f 2 (x) + g2 (x) + h2 (x),
B = f 4 (x) + g 4 (x) + h 4 (x), C = f 2 (x) .g2 (x) + g2 (x) .h2 (x) + h2 (x) .f 2 (x) .

Bi 11.

Cho a sin x sin y b cos x cos y = 0 . Chng minh rng biu thc

Q=
Bi 12.

a sin x + b cos x
2

1
a sin y + b cos2 y
2

khng ph thuc vo bin.

Cho Q = sin 6 x + cos6 x m sin 4 x + cos4 x . Tm tham s m biu thc Q khng ph


thuc vo x v tnh gi tr ca Q vi m va tm c (nu c).
(HKII Chuyn Trn i Ngha nm 1998)

Bi 13.

Cho

1/

2/

sin 4 x cos4 x
1
+
=
. Chng minh rng:
a
b
a+b
sin 8 x
a

sin10 x
a4

cos8 x
b

cos10 x
b4

"Cn c b thng minh"

(HKII Chuyn L Hng Phong nm 2001)

(a + b)

(a + b)

(HKII H Ni Amsterdam nm 2007)

DeThiThuDaiHoc.com

Page - 51 -

www.MATHVN.com
Chng 5. Cung gc lng gic Cng thc lng gic

Ths. L Vn on

B NG TRN LNG GIC CUNG GC LNG GIC


CUNG LIN KT


Cho

. Gi s

sin

1. nh ngha cc gi tr lng gic

tang

I Gi tr lng gic ca gc (cung) lng gic


.

T
cotang

S
B
K M

cosin

 Nhn xt

tan xc nh khi

..
.

.
.

cot xc nh khi

2. Du ca cc gi tr lng gic
Phn t
Gi tr lng gic
cos
sin
tan
cot

II

III

IV

+
+
+
+

+
+

3. Mt s lu
Quan h gia v raian:
Vi

th

v ngc li

di l ca cung trn c s o

bn knh R l

S o ca cc cung lng gic c im u A, im cui l B l


.
Mi cung lng gic

ng vi mt gc lng gic

v ngc

li.
Page - 52 -

DeThiThuDaiHoc.com
"All the flower of tomorrow are in the seeks of today"

cng hc tp mn Ton 10 Tp IIwww.MATHVN.com

Ths. L Vn on

II Cung gc lin kt
Gc i nhau

Gc b nhau

Gc hn km

rad

00

300

450

600

Gc ph nhau

Gc hn km

900

1200

1350

1800

2700

3600

sin
cos
tan
cot
 Cn nm vng h thc c bn

"Cn c b thng minh"

DeThiThuDaiHoc.com

.
.

Page - 53 -

www.MATHVN.com
Chng 5. Cung gc lng gic Cng thc lng gic

Ths. L Vn on

BA
NG
BAI T
TP AAP DU
DUNG
MI LIN H GIA N V V RADIAN
2 3 5 9 7 13 3
15
,
,
,
,
,
, , 2,
, .
3
4
9
5
3
3
4
7 15

Bi 14.

i s o cc cung sau ra , pht, giy:

Bi 15.

i s o cc gc sau ra radian (rad) : 900 , 360 , 150 , 720, 2700, 2400 , 5400 , 7500 , 210 .

Bi 16.

i s o ca cc cung trn sau thnh s o radian (chnh xc n phn nghn):


21030 ', 75054 ', 120 36 ' , 15030 ', 1050 45 ' 30 '', 27 0 38 ' 49 '' .

Bi 17.

s o radian ca cc cung trn sau ra s o (chnh xc n pht):

Bi 18.

in cc gi tr thch hp vo trng

2
(rad) v 2, 5 .

00

150

300

450

600

750

900

1200

1500

1800

16

5
12

2
3

3
4

5
6

4
3

7
4

Radian

12

5
6

2
3

3
4

1350

500

8100

TNH DI CUNG TRN


Bi 19.

Mt ng trn c bn knh 10 (cm) . Tnh di cc cung c s o l


2 3 7 13
3
3
,
,
,
,
, 2,
, 450 , (rad), 750 .
3 18 5
3
6
4
2

Bi 20.

Kim gi ca ng h di 7 (cm) , kim pht di 10 (cm) . Tnh qung ng kim pht, kim gi

i c trong 30 pht.
Bi 21.

Bnh xe c ng knh (tnh c lp) l 55 (cm) . Nu xe chy vi vn tc 40 (km /h) th trong


mt giy bnh xe quay c bao nhiu vng ?

Bi 22.

Kim pht v kim gi ca ng h ln nh Bu in H Ni theo th t di 1, 75 (m) v


1,26 (m ) . Hi trong 15 pht, mi kim pht v mi kim gi vch trn cung trn c di bng

bao nhiu mt ?

BIU DIN NGN CUNG LNG GIC


Page - 54 -

DeThiThuDaiHoc.com
"All the flower of tomorrow are in the seeks of today"

cng hc tp mn Ton 10 Tp IIwww.MATHVN.com

Bi 23.

Ths. L Vn on

Biu din cc gc sau ln ng trn lng gic gc A: 300, 450 , 1200, 1200 , 3300,
6300, 7500 , 12500,

7
4
15 2010
.
,
, 7500, 11250,
,
4
3
2
4

Bi 24.

Xc nh im cui ca cung c s o: k2, k, k , k , k , (k ) .


2
3
4

Bi 25.

Xc nh im ngn ca cc h nghim sau y trn ng trn lng gic vi k  .

1/ x = k .

2/ x =

+ k2 .
2

3/ x = 4k .

4/ x =

+ k .
3

5/ x =

Bi 26.

+ k2 .
6

6/ x =

7/ x =

k
.
3

8/ x =

k2
.
+
3
3

9/ x =

k3
.
+
4
4

10/ x =

k
.
+
6
3


 = k vi k  . Tm k  :
Cho hai im M v N sao cho s AM
= v s AN
6
798

1/ M trng vi N.
Bi 27.

+ k .
4

2/ M i xng vi N qua tm O.

Hai gc lng gic c s o radian


tia cui hay khng ?

35
m
v
vi m l s nguyn c th c cng tia u v
3
5

GA TR LNG GIC CA MT CUNG (GC)


Bi 28.

Bi 29.

Xc nh du ca cc gi tr lng gic hoc biu thc


3
.
2

1/

sin x, cos x, tan x, cot x vi < x <

2/

sin x + , cos x, tan x , cot x + vi 0 < x < .

4
2
2
2
2

3/

A = sin 400.cos 2900 .

5/

C = sin 2250. tan1300.cot 1750 .

7/

E = sin 500.cos 3000 .

9/

G = cot

2
3
.sin .
3
5

4/

B = sin 250 .cos1700 .

6/

D = cos1950. tan 2690.cot 980 .

8/

F = sin 2150. tan

10/ H = cos

21
.
7

4
9
.
. sin . tan
.cot
5
3
3
5

Tnh gi tr cn li ca gc x, bit

1/

sin x =

1
vi 900 < x < 1800 .
2

"Cn c b thng minh"

2/

sin x =

DeThiThuDaiHoc.com

4
vi 2700 < x < 3600 .
5
Page - 55 -

www.MATHVN.com
Chng 5. Cung gc lng gic Cng thc lng gic

Ths. L Vn on

3/

sin x =

5/

cos x =

7/

cos x =

3
3
vi < x <
.
5
2

3
vi 0 < x < 900 .
5

vi

5
9/

sin x =

< x < 0.
2

vi
< x < .
13
2

11/ tan x = 3 vi < x <


13/ tan x =

cos x =

6/

cos x =

8/

cos x =

3
.
2

5
vi 1800 < x < 2700 .
13

4
vi 2700 < x < 3600 .
5

10/ sin x =

1
vi 1800 < x < 2700 .
3

12/ tan x = 2 vi

vi < x < .
2
2

< x < .
2

14/ cot x = 3 vi < x <

3
.
2

15/ tan x =

3
3
vi < x <
.
4
2

16/ tan x = 2 vi

< x < .
2

17/ cot x =

vi 0 < x < .
3
2

18/ cot x = 3 vi

< x < .
2

19/ cot x = 3 vi
Bi 30.

vi 0 < x < .
4
2

4/

3
< x < 2 .
2

20/ cot150 = 2 + 3

Tnh gi tr ca cc biu thc lng gic sau

1/ Cho tan x = 2 . Tnh: A1 =

5 cot x + 4 tan x
2 sin x + cos x
.
, A2 =
5 cot x 4 tan x
cos x 3 sin x

2/ Cho cot x = 2 . Tnh: B1 =

3 sin x cos x
sin x 3 cos x
, B2 =
.
sin x + cos x
sin x + 3 cos x

3/ Cho cot x = 2 . Tnh: C1 =

2 sin x + 3 cos x
2
, C2 =
.
2
3 sin x 2 cos x
cos x sin x cos x

4/ Cho tan x = 2 . Tnh: D1 =

3 sin x 2 cos x
2 sin x + 3 cos x
,
, D2 =
4 sin x 5 cos x
5 sin 3 x + 4 cos3 x

D3 =

sin 3 x cos 3 x
sin x + 5 cos x
8 cos3 x 2 sin 3 x + cos x
,
D
=
,
D
=
,
4
5
sin x + sin2 x cos x
sin 3 x 2 cos3 x
2 cos x sin 3 x

D6 =

sin 3 x 2 cos 3 x + 2 sin x + 3 cos x


.
sin 3 x cos2 x + 5 sin x

5/ Cho sin x =

cot x + tan x
.
, 0 < x < . Tnh E =
5
2
cot x tan x

6/ Cho sin x =

8 tan2 x + 3 cot x 1
1
.
, 900 < x < 1800 . Tnh F =
3
tan x + cot x

cot x + 3 tan x
2
7/ Cho cos x = . Tnh G =
.
2 cot x + tan x
3

Page - 56 -

DeThiThuDaiHoc.com
"All the flower of tomorrow are in the seeks of today"

cng hc tp mn Ton 10 Tp IIwww.MATHVN.com

8/ Cho sin x =

Ths. L Vn on

tan x cos x
tan x cos x
2
, H2 =
cos x cot x .
, 0 < x < 900 . Tnh H1 =
3
cot x
sin2 x

cot x + tan x
sin x
4
, I2 = cot x +
9/ Cho cos x = ,
.
< x < . Tnh I1 =
cot x tan x
1 + cos x
5 2
Bi 31.

Bi 32.

Bi 33.

Bi 34.

Cho sin x + cos x =

5
. Hy tnh gi tr ca biu thc sau
4

1/ A = sin x.cos x .

S: A =

2/ B = sin x cos x .

S: B =

7
.
4

3/ C = sin 3 x cos3 x .

S: C =

41 7
.
128

9
.
32

Cho tan x cot x = 3 . Hy tnh gi tr ca biu thc sau

1/ A = tan2 x + cot2 x .

S: A = 11 .

2/ B = tan x + cot x .

S: B = 13 .

3/ C = tan4 x cot4 x .

S: C = 33 13 .

Cho sin x + cos x = m . Hy tnh theo m gi tr cc biu thc

1/ A = sin x cos x .

2/ B = sin 3 x + cos3 x .

3/ C = sin 4 x + cos4 x .

4/ D = sin x cos x .

5/ E = tan2 x + cot2 x .

6/ F = sin6 x + cos6 x .

Tnh sin x, cos x, tan x, cot x . Bit rng

1/ sin x + cos x = 2 .

2/ sin x cos x = 2 .

3/ sin x + cos x =

1
.
2

(HKII, Nguyn Thng Hin nm 2005)

4/ sin x + cos x =

1
.
5

S:

5/ tan x + cot x = 4 .

S:

4
3
4
3
; ; ; .
5
5
3
4

1
2 2 3

Bi 35.

Bi 36.

2 3
; 2 + 3; 2 3 .
2

Cho tan x 2 cot x = 1 . Hy tnh

1/ A = tan2 x cot2 x .

2/ B = tan 3 x + cot3 x .

3/ C = tan 4 x + 2 cot4 x .

4/ D = tan 5 x 3 cot5 x .

Tnh gi tr ca cc biu thc sau y khi

1/ Cho 3 sin 4 x + cos4 x =

"Cn c b thng minh"

3
. Tnh A = sin 4 x + 3 cos 4 x .
4

DeThiThuDaiHoc.com

S: A =

7
.
4

Page - 57 -

www.MATHVN.com
Chng 5. Cung gc lng gic Cng thc lng gic

Ths. L Vn on

2/ Cho 3 sin 4 x cos4 x =

1
. Tnh B = sin 4 x + 3 cos4 x .
2

3/ Cho 4 sin 4 x + 3 cos4 x =

S: B = 1 .

7
7
57
. Tnh C = 3 sin 4 x + 4 cos 4 x . S: C = C =
.
4
4
28

CUNG GC LIN KT
Bi 37.

Biu din cc s o ca cung di dng + k2 vi k  .


3 5 11 14
21
.
,
,
,
,
2
4
3
5
6

Bi 38.

Biu din cc s o ca cung di dng + k3600 vi k  .

3700 , 5120, 7650 , 10000 , 12340 .


Bi 39.

Biu din cc s o ca cung di dng + k2 vi k  .


8 11 7 18
27
25
.
,
,
,
,
,
3
5
2
7
10
4

Bi 40.

Dng cung lin kt (khng dng my tnh), hy tnh cc gi tr sau

1/

sin 1500 .

2/

cot1350 .

3/

cos 2250 .

4/

tan 2100 .

5/

cot 2250 .

6/

sin 2400 .

7/

cos 3150 .

8/

tan 3000 .

9/

cot 13800 .

Bi 42.

11/ sin13 .

12/ tan10 .

13/ cot

7
.
6

14/ cos

11
.
3

15/ cot

17/ cos

29
.
6

18/ tan

45
.
4

16
.
19/ cos
3

31
.
20/ sin
2

159
.
23/ tan

115
.
24/ sin

19
.
21/ cot
4

Bi 41.

10/ cos11 .

26
.
22/ cos
3

25
.
4

16/ sin

17
.
3

Din t gi tr lng gic ca cc gc sau bng gi tr lng gic ca gc x

1/

sin x 900 .

5/

cos x + 5400 .

9/

cos 4500 + x .

sin 2700 x . 4/

sin x 5400 . 8/

2/

cos 1800 + x . 3/
cot 1800 + x . 7/

sin x 1800 .

tan 3600 x .

6/

10/ sin2 2700 + x . 11/ cos3 900 + x . 12/ cot5 1800 x .

Din t gi tr lng gic ca cc gc sau bng gi tr lng gic ca gc x

1/

cot (x ) .

2/

sin ( + x) .

3/

tan (2 x) .

4/

cot (3 x) .

5/

sin (x 7) .

6/

tan (x 5) .

7/

sin + x .
2

8/

cos + x .
2

9/

3
cot x .

5
10/ cos x .

Page - 58 -

11

7
11/ tan
+ x . 12/ sin x + .
2

DeThiThuDaiHoc.com
"All the flower of tomorrow are in the seeks of today"

cng hc tp mn Ton 10 Tp IIwww.MATHVN.com

Bi 43.

Ths. L Vn on

13/ sin2 ( + x) .

14/ cos9 ( x) .

15/ cot11 (x 3) . 16/ cos 4 (x + 3) .

17/ cot2 (x 5) .

18/ cos6 (x ) .

3
19/ cos 8 x . 20/ cos5 x .
2

21/ sin2012 x .

9
7
22/ tan2 x . 23/ cos2013 x + . 24/ sin1991 x .

2
2
2

11
.
25/ cos2015 x

11

9
26/ cot2 x . 27/ tan11
+ x . 28/ cos (x 2012) .

2
2

Rt gn (n gin) cc biu thc

1/

A = cos x + sin (x ) .

2/

B = cos x + sin x cos + x sin + x .


2

3/

C = 2 cos x + 3 cos ( x) sin x + tan x .


2

4/

D = 2 sin + x + sin(5 x) + sin + x + cos + x .


2

5/

E = 2 cos x 3 cos( x) + 5 sin x + cot x .


2

6/

F = sin (5 + x) + cos x + cot (3 x) + tan x .


2

7/

11

3
G = cos (15 x) + sin x tan + x cot
x .
2

2
2

8/

H = sin ( + x) cos x + cot (2 x) + tan x .


2
2

9/

I = cos (5 x) sin + x + tan x + cot (3 x) .


2

10/ J = cos 2700 x 2 sin x 4500 + cos x + 9000 + 2 sin 2700 x .

3
11/ K = sin2 x + + sin2 x + + sin2 x + + sin2 (x + ) .

4
2
4

12/ L = sin2

2
5
7
.
+ sin2 + sin2 + sin2
+ sin2
+ sin2
3
6
9
9
8
8

13/ M = cos2013 x + cos2013 ( + x) .sin2012 ( + x) sin2011 x .


2

"Cn c b thng minh"

DeThiThuDaiHoc.com

Page - 59 -

www.MATHVN.com
Chng 5. Cung gc lng gic Cng thc lng gic

Ths. L Vn on

14/ N = sin 6 ( + x) + cos6 (x ) 2 sin 4 (x + 2) sin 4 x + cos2 x .

2
2
19

tan
x .cos (36 x) .sin (x 5)
2

15/ O =
.
9

sin x .cos (x 99)


2

85
3
+ cos (207 + x) + sin2 (33 + x) + sin2 x .
16/ P = sin x +

2
2

Bi 44.

Rt gn v tnh gi tr ca biu thc (khng dng my tnh)

1/

A = cos 00 + cos 200 + cos 400 + ...... + cos1800 .

2/

B = cos 200 + cos 40 0 + cos 600 + ...... + cos1800 .

3/

C = cos100 + cos 400 + cos 700 + ...... + cos 1700 .

4/

D = tan 200 + tan 400 + tan 600 + ...... + tan 1800 .

5/

E = cot15 0 + cot 300 + cot 450 + ...... + cot165 0 .

6/

F = sin 50 + sin 100 + sin 150 + ...... + sin 3600 .

7/

G = cot1950 + cot210 0 + cot 2250 + ...... + cot 3450 .

8/

H = cot150.cot 350.cot 550.cot 750 .

9/

I = tan 100. tan 200. tan 300...... tan 800 .

10/ J = tan 10. tan 20. tan 30...... tan 890 .


11/ K = sin2 28 0 + sin2 360 + sin2 54 0 + cos2 1520 .
12/ L = cos2 20 + cos2 40 + cos2 60 + ...... + cos2 880 .
13/ M = sin2 100 + sin2 200 + sin2 300 + ...... + sin2 900 .
14/ N = cos2 100 + cos2 200 + cos2 300 + ... + cos2 1800 .
15/ O = sin 20 0 + sin 400 + sin 600 + ... + sin 3400 + sin 3600 .
Bi 45.

Rt gn v tnh gi tr ca biu thc (khng dng my tnh)

1/

A = cos 3150 .sin 7650 .

2/

B = sin 320.sin 148 0 sin 3020.sin 1220 .

3/

C = sin 8100.cos 5400 + tan 1350. cot 5850 .

4/

D = sin 8250.cos 150 + cos 750. sin 5550 .

5/

E = 2 tan 5400 + 2 cos 11700 + 4 sin 9900 .

6/

Page - 60 -

F=

sin 2340 cos 2160


sin 144 cos126
0

. tan 360 .

DeThiThuDaiHoc.com
"All the flower of tomorrow are in the seeks of today"

cng hc tp mn Ton 10 Tp IIwww.MATHVN.com

7/

8/

9/

G=

sin 1206 + cos 36


0

cot 5720

(
)
tan (142 ) .sin 108

cos 288 0 .cot 720


0

.cot 360 .

sin 328 0 .sin 958 0

H=

I=

sin 324 0 + cos 6660

Ths. L Vn on

) (
tan (212 )

cos 508 0 . cos 10220


0

).

tan 18 0 .

10/ J = 2 sin 7900 + x + cos 12600 x + tan 6300 + x . tan 12600 x .

2 sin 25500. cos 188 0


1
+
11/ K =
.
tan 3680
2 cos 638 0 + cos 98 0
12/

(cos 44
L=

13/ M =

14/ N =

+ tan 2260 .cos 4060


cos 316

cos 720.cot18 0 .

tan 460.sin 440 + cot 1360 .sin 4040


cos 316

sin 328 0 .sin 958 0


cos 572

tan 360. tan 54 0 .

) (
tan (212 )

cos 508 0 . cos 10220


0

).

4
17
+ cos
3
6
15/ O =
.

23
10

sin
+ cos
3
6
sin

16/ P =
Bi 46.

sin (4, 8). sin (5, 7)


cot (5,2)

cos (6, 7). cos (5, 8)


tan (6,2)

Chng minh rng nu A, B, C l ba gc ca mt tam gic th

1/

sin B = sin (A + C) .

2/

cos (A + B) = cos C .

3/

sin

A+B
C
= cos .
2
2

4/

cos (B C) = cos (A + 2C) .

5/

cos (A + B C) = cos 2C .

6/

sin (A + 2B + C) = sin B .

7/

cot (A B + C) = cot 2B .

8/

cos

9/

sin

11/ cos

3A + B + C
= sin 2A .
2

A + B + 3C
= cos C .
2

10/ tan

A + B 2C
3C
.
= cot
2
2

A + 3B + C
= sin B .
2

12/ cot

A 2B + C
3B
.
= tan
2
2

"Cn c b thng minh"

DeThiThuDaiHoc.com

Page - 61 -

www.MATHVN.com
Chng 5. Cung gc lng gic Cng thc lng gic

Ths. L Vn on

C CNG THC CNG





H qu:
.

BA
NG
BAI T
TP AAP DU
DUNG
Bi 47.

Khng dng my tnh. Hy tnh gi tr ca cc biu thc


1/ A = sin120.cos 480 + cos120.sin 480 .
2/ B = cos 38 0.cos 220 sin 38 0.sin 220 .
3/ C = sin 100.cos 550 cos 100.sin 550 .
4/ D = sin 360.cos 60 sin 1260. cos 84 0 .
5/ E = cos 1120.cos 230 sin 1120.sin 230 .
6/ F = sin 2000. sin 3100 + cos 3400.cos 500 .
7/ G = cos110.cos 210 + cos 690. cos 790 cos100 .
8/ H = cos 680. cos 780 + cos 220. cos120 sin1000 .

) (

9/ I = cos 530 . sin 337 0 + sin 307 0.sin 1130 .


Bi 48.

Tnh gi tr lng gic ca cc cung gc sau


1/ 150 .
5/

Bi 49.

19
.
12

2/

750 .

3/ 1050 .

6/

5
.
12

7/

7
.
12

2/

B=

4/

D=

6/

F=

4/ 2850 .
8/

13
.
12

Tnh gi tr ca cc biu thc sau


1/

A=

3/

C=

5/

E=

Page - 62 -

1 + tan 150
1 tan 150

sin100 cos 200 + sin 200 cos100


cos17 0 cos130 sin17 0 sin 130

sin 730 cos 30 sin 87 0 cos17 0


cos1320 cos 620 + cos 420 cos 28 0

tan 250 + tan 200


1 tan 250 tan 200

tan 2250 cot 810 cot 690

cot 2610 + tan 2010


cot 2250 cot 79o0 ot 710
cot2590 + cot 2510

DeThiThuDaiHoc.com
"All the flower of tomorrow are in the seeks of today"

cng hc tp mn Ton 10 Tp IIwww.MATHVN.com

7/

G = cos2 750 sin2 750 .

9/

I = cos2 100 + cos1100 + cos2 1300 .

8/

Ths. L Vn on

H = sin2 200 + sin2 1000 + sin2 1400 .

10/ J = tan 200. tan 800 + tan 800. tan1400 + tan1400. tan 200 .
11/ K = tan100. tan 700 + tan 700. tan1300 + tan1300. tan1900 .
12/ L = sin1600 cos1100 + sin 2500 cos 3400 + tan1100 tan 3400 .

)(

) (

)(

13/ M = cos 700 + cos 500 cos 3100 + cos2900 + cos 400 + cos1600 cos 3200 cos 3800 .
Bi 50.

Tnh gi tr ca cc biu thc sau


1/

v 0 < x < .
A = cos x x + bit sin x =

3
2
3

2/

12
3
B = sin x bit cos x =
v < x <
.

3
13
2

3/

C = cos x 300 bit tan x = 2 v 0 < x < 900 .

4/

D = tan x + bit sin x = v < x < .

3
5
2

5/

12
3
< x < 2 .
E = cos x bit sin x =
v
3

13
2

6/

4
3

F = cot x bit sin x = v < x <


.

4
5
2

7/

G = tan x + bit cot x = 2 .


4

8/

2
7
H = sin 2x + bit cot x = .

4
3

9/

I = cos (a + b) .cos (a b) bit cos a =

1
1
v cos b = .
3
4

4
8
, 00 < < 900 v sin = , 900 < < 1800 . Hy tnh gi tr ca biu
5
17
thc A = cos ( + ) v B = sin ( ) .

Bi 51.

Bit sin =

Bi 52.

Bit sin =

Bi 53.

Cho 0 < , <

8
5
, tan b =
v , l cc gc nhn. Hy tnh gi tr ca cc biu thc
17
12
A = sin ( ), B = cos ( + ) v C = tan ( + ) .

, + = v tan . tan = 3 2 2 .
2
4

1/ Hy tnh tan ( + ); tan + tan .

"Cn c b thng minh"

2/ Tnh tan ; tan . Suy ra v .

DeThiThuDaiHoc.com

Page - 63 -

www.MATHVN.com
Chng 5. Cung gc lng gic Cng thc lng gic

Ths. L Vn on

Bi 54.

Cho =

. Tnh gi tr ca cc biu thc


3

1/ A = (cos + cos ) + (sin + sin ) .


2

Bi 55.

2/ B = (cos + sin ) + (cos sin ) .


2

Rt gn cc biu thc

1/ A = sin x 3 cos x .

2/ B = 3 sin 7x cos 7x .

3/ C = a sin x + b cos x, a 2 + b2 0 .

4/ D = 3 sin x + sin x + .
3
6

5/ E = cos7x.cos5x 3 sin2x + sin7x.sin5x . 6/ F = 3 cos 2x + sin 2x + 2 sin 2x .


6

7/ G = 2 sin 2x + 4 sin x + 1 .

Bi 56.

8/ H = sin2x + 2 2 cos x + 2sin x + + 3 .

Rt gn cc biu thc

1/

A = sin x cos 5x cos x sin 5x .

2/

B = sin 4x cot 2x cos 4x .

3/

C = cos 6x tan 3x sin 6x .

4/

D = sin (x + y) cos (x y) + sin (x y) cos (x + y) .

5/
6/
7/

(
) (
) (
) (
)
F = sin (14 + 2x) cos (16 2x) + cos (14 + 2x) sin (16 2x) .
G = sin (x + 10 ) cos (2x 80 ) + sin (x + 100 ) cos (2x + 10 ) .
E = cos 400 x cos x + 200 sin 400 x sin x + 200 .
0

8/

H = sin x cos x + sin x cos x .

4
4

3
3

9/

3
I = cos x cos x + + cos x + cos x + .
3
4
6
4

10/ J = sin x cos x sin x cos + x .

4
4
3
3

11/ K = cos x cos x + cos x + cos x + .

3
4
6
4

13
13
3
+ cos x +
cos x + .
12/ L = cos x cos x +

3
4
6
4

Bi 57.

Rt gn cc biu thc sau

1/

Page - 64 -

A=

tan 3x tan x
.
1 + tan x tan 3x

2/

B=

tan 2x + 1
.
1 tan 2x

DeThiThuDaiHoc.com
"All the flower of tomorrow are in the seeks of today"

cng hc tp mn Ton 10 Tp IIwww.MATHVN.com

3/
Bi 58.

Bi 59.

Bi 60.

Bi 61.

C=

tan 2x + cot 900 + x

).

4/

1 + cot 900 2x tan x

Ths. L Vn on

D=

tan2 2x tan2 x
1 tan2 2x tan2 x

Rt gn cc biu thc sau

1/

A = sin (a + b) + sin a sin (b) .


2

2/

B = cos a cos b cos (a b) .


2

3/

1
C = cos + a cos a + sin2 a .
4
4

4/

D = sin2 a sin2 b cos2 a cos2 b .

5/

E=

2 sin (a + b)

cos (a + b) + cos (a b)

tan b .

Rt gn cc biu thc sau

1/

A = cos2 x 3 sin 2x sin2 x .

3/

C = sin x + 450 + cos x + 450 .

5/

E = tan 2x + cot x 8 cos2 x .

2/

B = 4 sin 3 x 3 sin x + 3 cos 3x .

4/

D = tan 3x tan x sin 2x .

Chng minh cc ng thc sau

1/

sin 2x = 2 sin x cos x .

2/

cos 2x = cos2 x sin2 x .

3/

tan 2x =

4/

sin 3x = 3 sin x 4 sin 3 x .

5/

cos 3x = 4 cos3 x 3 cos x .

6/

cos x + sin x = 2 cos x = 2 sin x + .

4
4

7/

cos x sin x = 2 cos x + = 2 sin x .


4
4

8/

sin (x + y) sin (x y) = sin2 x sin2 y = cos2 y cos2 x .

9/

cos (x + y) cos (x y) = cos2 x sin2 y = cos2 y sin2 x .

2 tan x
1 tan2 x

Chng minh cc ng thc sau

1/

sin + x sin x = 2 sin x .


4

"Cn c b thng minh"

DeThiThuDaiHoc.com

Page - 65 -

www.MATHVN.com
Chng 5. Cung gc lng gic Cng thc lng gic

Ths. L Vn on

Bi 62.

2/

4 sin x + sin x = 4 sin2 x 3 .

3
3

3/

sin x sin (y z) + sin y sin (z x) + sin z sin (x y) = 0 .

4/

cos x sin (y z) + cos y sin (z x) + cos z sin (x y) = 0 .

5/

tan (x + y) tan x tan y = tan (x + y) tan x tan y .

6/

tan 2x tan x + tan 2x tan x + tan x tan x = 1 .


6
3
3
6

7/

2
2
tan x. tan x + + tan x + . tan x + + tan x + . tan x = 3 .

3
3
3
3

8/

3
2
cos x . cos x + + cos x + .cos x + =
1 3 .

3
4
6
4
4

9/

(cos 70

)(

) (

)(

+ cos 50o cos230o + cos290o + cos 40o + cos160o cos 320o + cos 380o = 0 .

Chng minh cc ng thc sau

1/

2/

3/

4/
Bi 63.

cos (a b)

cos (a + b)

sin (a b)
cos a cos b

cot a cot b + 1
.
cot a cot b 1

sin (b c)
cos b cos c

sin (a + b) sin (a b)
2

cos a cos b

cos (a + b) cos (a b)
2

cos a cos b

sin (c a )
cos c cos a

= 0.

= tan2 a tan2 b .

= 1 tan2 a tan2 b .

Chng minh cc h thc sau, vi iu kin cho trc

1/

Nu cos (a + b) = 0 th sin (a + 2b) = sin a .

2/

Nu sin (2a + b) = 3 sin b th tan (a + b) = 2 tan a .

3/

Nu tan a = 2 tan b th sin (a + b) = 3 sin (a b) .

4/

Nu tan a. tan b =

5/

Nu 5 sin b = sin (2a + b) th tan (a + b) =

6/

Nu sin b = sin a cos (a + b) th 2 tan a = tan (a + b) .

Page - 66 -

1
th cos (a + b) = 2 cos (a b) .
3

b = (a + b) a
HD:
.
2a + b = (a + b) + a

HD: Khai trin gi thit.

3
tan a .
2
HD: b = (a + b) a .

DeThiThuDaiHoc.com
"All the flower of tomorrow are in the seeks of today"

cng hc tp mn Ton 10 Tp IIwww.MATHVN.com

Bi 64.

Ths. L Vn on

7/

Nu cos (2a + b) = 1 th tan (a + b) tan a = 2 tan

8/

Nu cos (a + b) = k cos (a b) th tan a tan b =

b
.
2

a + 2b = (a + b) + b
1 k

. HD:
.
a = (a + b) b
1+ k

Chng minh cc ng thc sau


sin (a + b + c)

1/

tan a + tan b + tan c tan a tan b tan c =

2/

cos2 x + cos2 (x + y) + cos2 y 2 cos x cos y cos (x + y) = 1 .

cos a cos b cos c

(Thi hc k II trng THPT chuyn L Hng Phong Tp. HCM nm 2007)

Bi 65.

Bi 66.

Chng minh cc biu thc sau c lp vi bin x

1/

A = sin2 x + cos x cos + x .


3
3

2/

B = cos2 x + cos2 + x + cos2 x .

3
3

3/

C = sin2 x + sin2 + x + sin2 x .

3
3

4/

2
2
D = cos2 x + cos2 x + + cos2 x .

3
3

5/

F=

a cos3 x cos 3x a sin 3 x + sin 3x


+
, a = const .
cos x
sin x

Chng minh rng: tan x tan x tan + x = tan 3x . T tnh gi tr ca biu thc
3
3

P = tan 100 tan 500 tan 1100 .

Bi 67.

Cho tam gic ABC vi A, B,C ln lt l ba gc ca tam gic. Chng minh

1/

sin C = sin A.cos B + sin B.cos A .

2/

sin A = sin B cos C + sin C cos B .

3/

cos A = sin B sin C cos B cos C .

4/

sin C
= tan A + tan B,
cos A. cos B

5/

tan A + tan B + tan C = tan A. tan B. tan C,

6/

cot A.cot B + cot B.cot C + cot C.cot A = 1 .

7/

sin

A
B
C
B
C
= cos cos sin sin .
2
2
2
2
2

8/

cos

A
B
C
B
C
= sin cos cos sin .
2
2
2
2
2

"Cn c b thng minh"

(A, B 90 ) .
0

DeThiThuDaiHoc.com

(A, B,C 90 ) .
0

Page - 67 -

www.MATHVN.com
Chng 5. Cung gc lng gic Cng thc lng gic

Ths. L Vn on

9/

A
B
B
C
C
A
. tan + tan . tan + tan . tan = 1 .
2
2
2
2
2
2

tan

10/ cot

A
B
C
A
B
C
+ cot + cot = cot .cot . cot .
2
2
2
2
2
2

11/ cot B +
12/ cos

cos C
cos B
= cotC +
,
sin B.cos A
sin C.cos A

(A 90 ) .
0

A
B
C
A
B
C
A
B
C
A
B
C
. cos .cos = sin sin cos + sin cos sin + cos sin sin .
2
2
2
2
2
2
2
2
2
2
2
2
A
B
C
A
B
C
+ sin2 + sin2 = 1 + 2 sin sin sin .
2
2
2
2
2
2

13/ sin2

14/ sin A cos B cos C + sin B cos C cos A + sin C cos A cos B = sin A sin B sin C .
Bi 68.

Cho tam gic ABC vi A, B,C ln lt l ba gc ca tam gic. Chng minh

1/

tan A + tan B + tan C 3 3, A, B,C nhn.

2/

tan2 A + tan2 B + tan2 C 9, A, B,C nhn.

3/

tan6 A + tan6 B + tan 6 C 81, A, B,C nhn.

4/

tan2

5/

tan

A
B
C
+ tan2 + tan2 1 .
2
2
2

A
B
C
+ tan + tan 3 .
2
2
2

HD:
1/, 2/, 3/, s dng tan A + tan B + tan C = tan A. tan B. tan C v bt ng thc
Cauchy.
4 / s dng bt ng thc b a 2 + b2 + c2 ab + bc + ca v

tan

A
B
B
C
C
A
. tan + tan . tan + tan . tan = 1 .
2
2
2
2
2
2

A
B
C

5 / khai trin tan + tan + tan v s dng cu 3 / .

2
2
2

Page - 68 -

DeThiThuDaiHoc.com
"All the flower of tomorrow are in the seeks of today"

cng hc tp mn Ton 10 Tp IIwww.MATHVN.com

Ths. L Vn on

D CNG THC NHN



Cng thc nhn i
.


.


.


Cng thc h bc

Cng thc nhn ba (m rng)

BA
NG
BAI T
TP AAP DU
DUNG
RT GN V TNH GI TR CA BIU THC
Bi 69.

Tnh gi tr ca cc biu thc sau (khng dng my tnh b ti)


1/

A = cos2

3/
5/

7/

sin2 .
8
8

2/

B = 3 cos100 4 cos3 100 .

C = sin 200 1 4 cos2 200 .

4/

D = 4 sin3 400 + 3 sin1300 .

E = 4 sin3 500 + 3 cos1400 .

6/

F=

8/

H=

G=

tan

7
8

1 tan
8

9/

I=

1
sin100

4 sin 700 .

11/ K = tan2 360 tan2 720 .

tan150
1 tan2 150
1
sin10

10/ J = tan2

3
cos100

5
+ tan2
.
12
12

12/ L = cos2 700 + sin2 400 sin1000 .

13/ M = cos2 200 + 2 sin2 550 2 sin 650 . 14/ N = sin 60.sin 420.sin 660.sin 78 0 .

"Cn c b thng minh"

DeThiThuDaiHoc.com

Page - 69 -

www.MATHVN.com
Chng 5. Cung gc lng gic Cng thc lng gic

Ths. L Vn on

Bi 70.

Tnh gi tr ca cc biu thc sau (khng dng my tnh b ti)


1/

A = cos 360 cos 720 .

2/

B = cos

2
cos
.
5
5

3/

C = sin . cos cos .


8
8
4

4/

D = sin

. cos .cos .
16
16
8

5/

E = sin100 sin 500 sin 700 .

6/

F = cos

2
4
cos
cos
.
7
7
7

7/

G = cos

8/

H = sin 60 cos120 cos 24 0 cos 48 0 .

9/

I = sin 60 sin 420 sin 660 sin 780 .

4
5
cos
cos
.
7
7
7

10/ J = sin50.sin150.sin250.... sin750.sin850 .

11/ K = cos100.cos 200.cos 300...cos 700.cos 800 .


12/ L = 8 tan180 cos180 cos 360 cos 720 .
13/ M = cos 200 cos 400 cos 600 cos 800 .
14/ N = cos

2
3
4
cos cos
cos
.
9
9
9
9

15/ O = 96 3 sin

Bi 71.

cos cos cos cos .


48
48
24
12
6

16/ P = cos

2
4
8
16
32
.cos .cos .cos
.cos
.
31
31
31
31
31

17/ Q = cos

2
4
8
16
cos cos
cos
cos
.
33
33
33
33
33

18/ R = cos

2
3
4
5
6
7
.cos .cos . cos .cos .cos .cos
.
15
15
15
15
15
15
15

Tnh gi tr ca cc biu thc sau (khng dng my tnh b ti)

cos 800 + cos 200

1/

A=

2/

B = 3 sin 150 cos150

3/

C = tan 90 tan 27 0 tan 630 + tan 810 .

4/

D = 4 tan

cos 350 cos150 sin 350 sin 150

sin 600
sin 4 150 cos4 150

+ 2 tan + tan
cot .
8
16
32
32
i hc Quc Gia H Ni khi B nm 2000

5/
Bi 72.

E=

cos2 6960 + tan 2600 tan 5300 cos2 1560


tan 252 + cot2 3420
2

Rt gn cc biu thc

Page - 70 -

DeThiThuDaiHoc.com
"All the flower of tomorrow are in the seeks of today"

cng hc tp mn Ton 10 Tp IIwww.MATHVN.com

Bi 73.

Ths. L Vn on

1/

A = (sin x + cos x) .

2/

B = 1 4 sin2 x cos2 x .

3/

C = sin x cos x cos 2x .

4/

D = cos4 2x sin 4 2x .

5/

E = cos2 x + sin2 x + .
2
2

6/

F = sin x cos x cos 2x .

7/

G = 4 sin x sin x +

8/

H = sin2 + sin2
8 2
8

9/

I = sin 8x + 2 cos2 450 + 4x .


sin 2x +
2


.
2

x
.
2

10/ J = sin2x + cos2x 2cosx(sinx + cosx) +1.

Tnh gi tr ca cc biu thc sau (khng dng my tnh b ti)


1/

A=

3/ C =

5/

2 cos2 x 1
.
sin x + cos x

(1 tan x)(1 + cot x)

E=

sin 2x cos 2x

.
sin x
cos x

1 + sin x

2
.
7/ G =

1 + sin + x
2

9/

I=

sin 4x
cos 2x
.
.
1 + cos 4x 1 + cos 2x

B=

4/

D = 1 tan2 x cot x .

6/

F=

8/

H=

10/ J =

1 + cos + x
2
x

11/ K = tan + .
.

4 2

sin + x

1 cos 2x + sin 2x
.cot x .
13/ M =
1 + cos 2x + sin 2x

1 2 sin2 2x
.
cos 2x sin 2x

2/

cot x tan x
.
cos 2x
sin2 2x 4 sin2 x
sin2 2x + 4 sin2 x 4

sin 3x cos 3x
.
sin x + cos x

12/ L = cot2x +

14/ N =

1
+ tan x .
2 sin 2x

2 sin 2x + 2 cos2 x 1

cos x sin x cos 3x + sin 3x

1
1
1

+
+
+ cot 8x .
15/ O = 1 + sin x + 1 sin x, 0 < x < . 16/ P =
4
sin 2x sin 4x sin 8x


17/ O = 1 sin 2x + 1 + sin 2x, < x <
4

Bi 74.


.
4

Tnh gi tr ca biu thc lng gic, khi bit

3
,
< x <.
5 2

1/

sin 2x, cos 2x khi sin x =

2/

sin 2x, cos 2x khi sin x + cos x = 2 .

"Cn c b thng minh"

DeThiThuDaiHoc.com

Page - 71 -

www.MATHVN.com
Chng 5. Cung gc lng gic Cng thc lng gic

Ths. L Vn on

Bi 75.

5
3
,<x<
.
13
2

3/

cos 2x, sin 2x, tan 2x khi cos x =

4/

cos 2x, sin 2x, tan 2x khi tan x = 2 .

5/

4
3
sin x, cos x khi sin 2x = ,
<x<
.
5 2
2

6/

cos 2x, sin 2x, tan 2x khi tan x =

7/

sin 2x, cos 2x, tan 2x khi tan x = 2 3, 0 < x <

7
.
8

Tnh gi tr ca biu thc lng gic, khi bit

.
16

1/

A = sin5 x cos x cos5 x sin x khi bit x =

2/

B = sin 4 x + sin3 x cos x cos3 x sin x + cos4 x khi bit x =

3/

C = cos 4x khi bit tan x + cot x = 3 .

4/

D = cos 2700 + 4x khi bit cot 450 + x = 2 .

5/

E = sin 4 x + cos4 x khi bit sin x + cos x = m vi m  .

6/

F=

3
tan x cot x
,
khi bit sin x cos x =
tan x + cot x
4

0 < x <

Bi 77.

Tnh gi tr lng gic ca (khng dng my tnh b ti)


cos150 .

Tnh tan

1/

2/


.
4

5 1
.
4

Chng minh rng sin180 =

1/

.
48

Bi 76.

Bi 78.

.
2

cos 18 0 .

3/ sin 360 .

4/

cos 360 .

x
khi bit
2

cos x =

, 0<x< .
5
2

2/

tan x =

, 0<x< .
2
6

1
Cho tan x = , 0 < x < v tan y = , 0 < y <
2
2
3

24
3
, <x<
.
7
2

3/ sin x + cos x =

Bi 79.

1/ Tnh x + y .
Bi 80.

2/


.
2

sin (x 2y) v cos (2x + y) .

Tm x khi bit

0 < x <

1/
.
2

tan x = 2 1
Page - 72 -

0 < x <

2
2/
.

6+ 2
cos x =
4

DeThiThuDaiHoc.com
"All the flower of tomorrow are in the seeks of today"

cng hc tp mn Ton 10 Tp IIwww.MATHVN.com

Bi 81.

Tnh theo cos 2x cc biu thc sau

1/

A = 1 + cos2 x .

3/ C =
5/
Bi 82.

Ths. L Vn on

2/

B = sin2 x cos2 x .

4/

D=

6/

F = sin6 x cos2 x + cos6 x sin2 x .

2/

B = cos x .

sin x
.
3 2 cos x

4/

D = tan x + cot x .

1 tan x
.
1 + cot x

6/

F=

1 + sin2 x
cos2 x

E = sin6 x + cos6 x .

Tnh theo t = tan

1/

E=

1 + cot2 x
1 cot2 x

x
cc biu thc sau
2

A = sin x .

3/ C =
5/

tan x + sin x
.
tan x sin x

CHNG MINH NG THC


Bi 83.

Chng minh cc ng thc sau

1/

cos4 x sin 4 x = cos 2x .

2/

sin 4x = 4 sin x cos x 1 2 sin2 x .

3/

cos2 2x sin2 x = cos x cos 3x .

4/

cos 4x = 8 cos4 x 8 cos2 x + 1 .

5/

8 sin 4 x = 3 4 cos 2x + cos 4x .

6/

sin4 + cos4 x =

7/

sin4 + cos4 x 6 cos2 x sin2 x = cos 4x .

8/

sin6 x + cos6 x =

9/

sin6 x cos6 x =

11/ cos8 x sin8 x =

3 1
+ cos 4x .
4 4

5 3
+ cos 4x .
8 8

15
1
x
x
1
cos 2x + cos 6x . 10/ sin6 cos6 = cos x sin2 x 4 .
16
16
2
2
4
7
1
cos 2x + cos 6x .
8
8

Bi 84.

Chng minh rng nu a + b =

Bi 85.

Chng minh cc ng thc sau

13/ cos3 xcos3xsin3 xsin3x =3 sin4 x +cos4 x 2.

th (1 + tan a )(1 + tan b) = 2 .


4

1/

1 cos 2x
= tan x .
sin 2x

2/

tan x + cot x =

3/

cot x tan x = 2 cot2x .

4/

cos x
= cot
1 sin x
4

5/

1 + cos x
x
. tan2 = 1 .
1 cos x
2

6/

1
+ cot2x = cot x
sin 2x

7/


x
2
tan + + cot + =
.

4 2
4 2 cos x

8/

(tan 2x tan x) cos 2x = tan x .

"Cn c b thng minh"

DeThiThuDaiHoc.com

2
.
sin 2x
x
.
2

Page - 73 -

www.MATHVN.com
Chng 5. Cung gc lng gic Cng thc lng gic

Ths. L Vn on

9/

tan + x tan x = 2 tan 2x .

4
4

11/ cos 3x sin3 x + sin 3x cos3 x =

10/ cos3 x sin x sin3 x cos x =

1
sin 4x .
2

3
sin 4x . 12/ tan x tan x. tan + x = tan 3x .

4
3
3

13/ tanx + tanx + + tanx + = 3tan3x . 14/ cotx tanx 2tan2x 4tan4x = 8cot8x .
3
3

Bi 86.

Chng minh cc ng thc sau


1/

3/

1 + sin x
= cot
cos x
4

2 sin 2x + sin 4x

2 (cos x + cos 3x)

x
.
2

= tan 2x cos x .

1
1 2 sin2 x
.
=
cos 2x
1 sin 2x

2/

tan 2x +

4/

6 + 2 cos 4x
= tan2 x + cot2 x .
1 cos 4x

5/

1 2 sin2 x 1 tan x
=
.
1 + sin 2x
1 + tan x

x
6/
+ 1 tan = tan x

cos x
2

7/

1 + sin 2x
= tan + x .
cos 2x
4

8/

9/

11/

13/

1 2 sin2 x
= 1.

2 tan + x cos + x

4
4

1 cos x + cos 2x
= cot x .
sin 2x sin x
sin 2x cos x

(1 + cos 2x)(1 + cos x)

= tan

6 + 2 cos 4x
15/ cot x + tan x =
.
1 cos 4x
2

17/

20/

cot2 2x 1
cos 8x cot 4x = sin 8x .
2 cot 2x

19/ tan 4x

sin x + sin
10/

x
2

1 + cos x + cos

12/

x
.
2

2 sin x sin 2x
x
= tan2 .
2 sin x + sin 2x
2

14/

x
2

= tan

x
.
2

1 + cos 4x
1
= sin 4x .
cot x tan x 2
sin4 x cos4 x + cos2 x
1 (1 cos x)

= cos2

x
.
2

x
x
4

16/ cot tan =


.

2
2
1 2 tan x cot2x

18/ tan 2x +

1
1 2 sin2 x
=
.
cos 2x
1 sin 2x

1
sin 2x cos 2x
=
.
cos 4x
sin 2x + cos 2x

cos x + sin x cos x sin x

= 2 tan2x .
cos x sin x cos x + sin x

x
x
sin
2
2 = 1 tan x .
21/
x
x
cos x
cos + sin
2
2
cos

Page - 74 -

DeThiThuDaiHoc.com
"All the flower of tomorrow are in the seeks of today"

cng hc tp mn Ton 10 Tp IIwww.MATHVN.com

22/

1 + cos x + cos 2x + cos 3x


2 cos2 x + cos x 1

Ths. L Vn on

= 2 cos x .

x
x
23/ 2
+ cot2x = cot tan .

sin 2x
2
2

24/
25/

26/

27/

sin 4 x + sin 2x cos4 x


= cos 2x .
tan 2x 1
sin2 2x 4 sin2 x
sin 2x + 4 sin x 4
2

sin2 3x
sin2 x

cos2 3x
cos2 x

= tan 4 x .

= 8 cos 2x .

cos2 x cos 3x sin 3 x + sin 3x


+
= 3.
cos x
sin x

1
1 + tan x 1 = sin 2x .
28/ 1 + tan x +

cos x
cos x cos2 x

Bi 87.

Chng minh cc ng thc sau


1/

2/

a
a
a
a
sin a
.
A = cos cos 2 cos 3 ... cos n =
2
a
2
2
2
n
2 .sin n
2
B = cos

3/ C = cos

4/
Bi 88.

D=

2
n
1
. cos
... cos
= n.
2n + 1
2n + 1
2n + 1
2

2
4
2n
1
.cos
... cos
= .
2n + 1
2n + 1
2n + 1
2

1 1 1 1 1 1
x
+
+
+ cos x = cos ,
2 2 2 2 2 2
8

0 < x < .

Chng minh cc biu thc sau y khng ph thuc vo bin s


1/

A=

sin 2x 2 sin x
x
+ tan2 .
sin 2x + 2 sin x
2

2/

B=

cos3 x cos 3x sin3 x + sin 3x


+
.
cos x
sin x

3/

1
C = 4 sin 4 x + 2 cos 2x cos 4x .
2

4/

D = 3 cos 2x + 5 sin4 x + 4 sin2 x cos2 x cos4 x .

5/

E=

tan2 x 1
cot x + cos 4x cot 2x + sin 4x .
2

"Cn c b thng minh"

DeThiThuDaiHoc.com

Page - 75 -

www.MATHVN.com
Chng 5. Cung gc lng gic Cng thc lng gic

Ths. L Vn on

6/

3
F = sin 4 x + sin 4 x + + sin 4 x + + sin 4 x + .

4
2
4

BIN I THNH TCH


Bi 89.

Bi 90.

Bin i thnh tch cc biu thc sau


1/

A = (2 sin x 1)(2 cos x + sin x) sin 2x + cos x .

2/

B = (2 cos x 1)(2 sin x + cos x) sin 2x + sin x .

3/

C = (2 sin x 1)(2 cos 2x + 2 sin x + 3) 4 sin2 x + 1 .

4/

D = (2 cos x 1)(2 cos x + 2 sin x + 1) + 4 sin2 x 3 .

5/

E = (2 sin x + 1)(3 cos 4x + 2 sin x 4) + 4 cos2 x 3 .

6/

F = 1 + sin x + cos x + sin 2x + cos 2x .

7/

G = 1 + cos x + cos2 x sin x (1 + cos x) .

8/

H = 1 + sin2 x cos x sin 2x + 1 + cos2 x sin x 1 .

Bin i thnh tch cc biu thc sau


1/ A = sin 3 x cos x cos3 x sin x .

2/ B = sin3 x + cos3 x sin x + cos x .


3/ C = sin3 x + cos3 x cos 2x .

4/ D = cos2 x + sin3 x + cos x .


5/ E = cos3 x + cos2 x + 2 sin x 2 .

6/ F = cos3 x + sin 3 x + 2 sin2 x 1 .


Bi 91.

Bin i thnh tch cc biu thc sau


1/

A = sin 2x + cos 2x + cos x sin x .

2/

B = sin 2x + cos 2x + 3 sin x cos x 2 .

3/

C = cos 2x sin 2x + 3 sin x + cos x 2 .

4/

D = sin 2x + cos 2x + 2 (cos x sin x ) 3 .

5/

E = 9 sin x + 6 cos x 3 sin 2x + cos 2x .

6/

F = sin 2x + 2 cos 2x + 4 cos x sin x 1 .

7/

G = (2 cos x 1)(sin x + cos x ) 1 .

8/

H = 2 sin 2x cos 2x 7 sin x 2 cos x + 4 .

Page - 76 -

DeThiThuDaiHoc.com
"All the flower of tomorrow are in the seeks of today"

cng hc tp mn Ton 10 Tp IIwww.MATHVN.com

Ths. L Vn on

E CNG THC BIN I



 Cng thc bin i tng thnh tch
.

H qu
.

 Cng thc bin i tch thnh tng


.

BA
NG
BAI T
TP AAP DU
DUNG
CNG THC BIN I TNG THNH TCH
Bi 92.

Tnh gi tr ca biu thc


1/

A=

cos 2x cos 4x
khi x = 200 .
sin 4x sin 2x

2/

B=

cos x.cos13x

khi x =
.
cos 3x + cos 5x
17

3/

C=

cos x.cos10x

khi x =
.
cos 2x + cos 4x
13

"Cn c b thng minh"

DeThiThuDaiHoc.com

Page - 77 -

www.MATHVN.com
Chng 5. Cung gc lng gic Cng thc lng gic

Ths. L Vn on

Bi 93.

4/

D=

tan 2x sin 2x
2
khi tan x =
.
tan 2x + sin 2x
15

5/

E=

sin x + sin 2x + sin 3x


1

khi sin x = ,
<x< .
cos x + cos 2x + cos 3x
3 4
2

Rt gn biu thc
1/

1 2 cos x
.
1 + 2 cos x

A=

1 2 cos 2x

3/ C =

5/

3 + 2 sin 2x

sin 5x sin 3x
.
2 cos 4x

E=

sin (x + y)

7/ G =

9/

I=

sin x + sin y

cos x + sin x
.
cos x sin x

B=

4/

D=

6/

F=

cos 4x cos 2x
.
sin 4x + sin 2x

8/

H=

sin x + sin y
.
cos x + cos y

10/ J =

3 + 2 cos 3x
2 2 sin 2x
2 + 2 sin 2x

sin2 4x sin2 2x
cos2 x cos2 2x

11/ K =

sin2 4x
.
2 cos x + cos 3x + cos 5x

12/ L =

sin 2x
.
tan x + cot2x

13/ M =

tan 3x + tan 5x
.
cot 3x + cot 5x

14/ N =

tan 2x + cot2x
.
1 + tan 2x. tan 4x

15/ O =

1 + sin 2x + cos 2x
.
1 + sin 2x cos 2x

16/ P =

1 + sin 4x cos 4x
.
1 + cos 4x + sin 4x

17/ Q =

sin 2x + 2 sin 3x + sin 4x


.
cos 3x + 2 cos 4x + cos 5x

18/ R =

sin x + sin 4x + sin 7x


.
cos x + cos 4x + cos 7x

19/ S =

cos 2x sin 4x cos 6x


.
cos 2x + sin 4x cos 6x

20/ T =

21/ U =

23/ X =
Bi 94.

3 2 cos 3x

2/

2 sin 2x + 2 cos2 x 1

cos x sin x cos 3x + sin 3x

22/ V =

1 + cos x + cos 2x + cos 3x


2 cos2 x + cos x 1
sin(x + y) sinx

sin(x + y) + sinx

cos(x + y) + cosx
cos (x y) cosx

cos 7x cos 8x cos 9x + cos10x


sin 4x + sin 5x + sin 6x
. 24/ Y =
.
sin 7x sin 8x sin 9x + sin 10x
cos 4x + cos 5x + cos 6x

Bin i thnh tch cc biu thc sau y

A = cos 3x + cos x .

2/

B = sin 3x + sin 2x .

3/ C = cos 4x cos x .

4/

D = sin 5x sin x .

E = 1 + sin 2x .

6/

F = 1 sin x .

8/

H = 2 sin 2x 1 .

1/
5/

7/ G = 1 + 2 cos x .
9/
Page - 78 -

I = 3 + 2 cos 2x .

10/ J = sin (a + b) sin (a b) .

DeThiThuDaiHoc.com
"All the flower of tomorrow are in the seeks of today"

cng hc tp mn Ton 10 Tp IIwww.MATHVN.com

11/ K = cos2 x cos2 y .

12/ L = 1 + sin x cos 2x .

13/ M = 1 + sin x + cos x .

14/ N = cos x + sin 2x cos 3x .

15/ O = sin 3x sin x + sin 2x .

16/ Q = cos x cos 2x sin 3x .

17/ R = sin x + sin 2x + sin 3x .

18/ S = cos x + cos 2x + cos 3x .

19/ T = 2 sin 2x + cos 5x cos 9x .

20/ U = sin 3x 2 sin 2x + sin x .

21/ V = cos x + cos 3x + 2 cos 5x .

22/ X = cos 460 cos 220 2 cos 780 .

23/ Y = cos

Bi 95.

Ths. L Vn on

2
3
cos
+ cos
.
7
7
7

24/ Z = cos

2
4
6
+ cos
+ cos
.
7
7
7

25/ W = sin 700 sin 200 + sin 500 .

26/ = cos 5x + cos 7x cos ( + 6x) .

27/ = cos + 5x + sin x cos 3x .

28/ = sin x + sin y + sin

29/ = sin x + sin y + sin (x + y) .

30/ = cos x + cos y + sin (x + y) .

31/ = cos x + cos y + cos (x + y) + 1 .

32/ = cos 600 + x + cos 600 x + cos 3x .

33/ = 1 + cos 2x + cos 4x + cos 6x .

34/ = sin 2x + sin 4x + sin 6x .

35/ = sin 5x + sin 6x + sin 7x + sin 8x .

36/ = cos 5x + cos 8x + cos 9x + cos12x .

x+y
.
2

Bin i thnh tch cc biu thc sau y


1/ 1 + cos x + cos 2x + cos 3x .

2/

sin x sin 3x + sin 7x sin 5x .

3/

sin x sin 2x + sin 5x + sin 8x .

4/

cos 7x + sin 3x + sin 2x cos 3x .

5/

cos 9x cos 7x + cos 3x cos x .

6/

cos10x cos 8x cos 6x + 1 .

7/

sin 350 + cos 400 + sin 550 + cos 200 .

8/

sin 57 0 sin 590 sin 930 + sin 610 .

9/

sin 47 0 + sin 610 sin110 sin 250 .

10/ cos 5x + 3 cos 7x + 3 cos 9x + cos11x .

11/ 1 + sin x cos 5x sin 7x 2 cos2

3x
.
2

12/ sin 3x + sin x sin2x + 2 (1 cos x) cos x .

13/ sin x + sin 2x + sin 3x 1 cos x cos 2x .


14/ 1 + sin x + cos 3x cos x sin 2x cos 2x .
15/ sin x + sin 2x + sin 3x cos x cos 2x cos 3x .
Bi 96.

Bin i thnh tch cc biu thc sau y

3
.
2

1/

cos2 x + cos2 2x + cos2 3x 1 .

2/

sin2 x + sin2 2x + sin2 3x

3/

sin2 3x sin2 2x sin 2 x .

4/

sin2 x cos2 2x cos2 3x .

5/

sin2 x 2 sin2 2x + sin2 3x .

6/

sin2 4x cos2 6x sin (10, 5 + 10x) .

7/

sin2 3x cos2 4x sin2 5x + cos2 6x .

8/

3
cos2 x + cos2 2x + cos2 3x + cos2 4x .
2

"Cn c b thng minh"

DeThiThuDaiHoc.com

Page - 79 -

www.MATHVN.com
Chng 5. Cung gc lng gic Cng thc lng gic

Ths. L Vn on

9/

5
9x
cos2 x + cos2 2x + cos2 3x + cos2 4x 2 . 10/ cos3x + sin7x 2sin2 + + 2cos2 .
4 2
2

cos3xcos 5x

2
11/ sin + 2xcot3x + sin( + 2x) 2 cos5x . 12/ 2sin2x cos + 3x
.

cos5x
2
2

Bi 97.

Chng minh
1/

sin 750 + cos 750 =

3/

6
.
2

2/

cos120 cos 48 0 = sin 18 0 .

sin 650 + sin 550 = 3 cos 50 .

4/

tan 267 0 + tan 930 = 0 .

5/

cos 850 + cos 350 + cos 250 = 0 .

6/

tan 90 tan270 tan 630 + tan 810 = 4 .

7/

cos 240 + cos 480 cos 840 cos120 =

8/

tan 90 + tan150 + cot150 + cot 90 = tan 27 0 + cot27 0 .

9/

1
1
cos x cos 3x cos 5x = 8 sin2 x cos3 x .
2
2

1
.
2

10/ sin x (1 + 2 cos 2x + 2 cos 4x + 2 cos 6x) = sin 7x .


11/ 1 + 4 cos x + 6 sin 2x 4 sin x = 16 sin 2x sin4

) (

x
.
2

12/ 8 sin2 x sin x + 600 sin x 600 = cos 4x cos 2x .

) (

13/ (sin x cos x) cos 4x = 4 sin 2x sin x + 150 cos x 150 .


2

Bi 98.

a
b
c
Cho a + b = c . Chng minh: sin a + sin b + sin c = 4 cos cos sin .
2
2
2

Bi 99.

Tnh cc gi tr ca biu thc


1/

A = sin6

+ cos6
.
24
24

2/

B = tan2

+ cot2 .
12
12

3/ C = cos

3
5
7
9
+ cos
+ cos
+ cos
+ cos
.
11
11
11
11
11
2
4
6
8
10
+ cos
+ cos
+ cos
+ cos
.
11
11
11
11
11

4/

D = cos

5/

E = cos 00 + cos

6/

F = sin 400 + cos100

Page - 80 -

2
3
4
5
6
+ cos
+ cos
+ cos
+ cos
+ cos
.
7
7
7
7
7
7

) (cos 40
2

+ sin 100

+ cos1400 .

DeThiThuDaiHoc.com
"All the flower of tomorrow are in the seeks of today"

cng hc tp mn Ton 10 Tp IIwww.MATHVN.com

Ths. L Vn on

Bi 100. Trong ABC c ba gc ln lt l A, B,C . Chng minh rng


1/

sin A + sin B + sin C = 4 cos

A
B
C
cos cos .
2
2
2

2/

sin A + sin B sin C = 4 sin

A
B
C
sin cos .
2
2
2

3/

cos A + cos B + cos C = 1 + 4 sin

4/

sin 2A + sin 2B + sin 2C = 4 sin A sin B sin C .

A
B
C
sin sin .
2
2
2

5/ 1 + cos 2A + cos 2B + cos 2C = 4 cos A cos B cos C .


6/

sin2 A + sin2 B + sin2 C = 2 (1 + cos A cos B cos C) .

7/

cos2 A + cos2 B + cos2 C = 1 2 cos A cos B cos C .

CNG THC BIN I TCH THNH TNG


Bi 101. Bin i thnh tng cc biu thc sau

A = sin

2
sin .
5
5

2/

B = sin 5x cos 3x .

3/ C = sin

cos .
4
6

4/

D = sin

E = sin (x + y) cos (x y) .

6/

F = sin x + 300 cos x 300 .

8/

H = 8 cos x sin 2x sin 3x .

1/

5/

7/ G = 2 sin x sin 2x sin 3x .


9/

I = sin x + sin x cos 2x .


6
6

7
cos
.
12
12

) (

10/ J = 4 cos (a b) cos (b c) cos (c a ) .

Bi 102. Tnh gi tr ca biu thc


1/

A = cos 750 cos150 .

2/

B = sin

5
sin
.
12
12

3/

C = sin

11
5
cos
.
12
12

4/

D = sin 200. sin 400. sin 800 .

5/

x
5x
E = sin . sin , khi x = 600 .
4
4

6/

F = sin 200 sin 400 sin 600 sin 800 .

7/

G = 24.sin

8/

H = sin 50.sin150.sin 250.......sin 650 sin 750.sin 850 .

5
7
11
. sin .sin .sin
.
24
24
24
24

"Cn c b thng minh"

DeThiThuDaiHoc.com

Page - 81 -

www.MATHVN.com
Chng 5. Cung gc lng gic Cng thc lng gic

Ths. L Vn on

9/

I = sin2 100 + cos 700 cos 500 .

1
10/ J = cos100 cos 500 cos 50 cos 250 + cos100 .
2
11/ K = sin2 500 + sin2 100 + sin 500 sin100 .
12/ L = cos2 730 + cos2 47 0 + cos 730. cos 47 0 .
13/ M = sin2 500 + sin2 700 cos 500 cos 700 .

(i hc An Ninh khi A nm 2001)

1
14/ N = cos100 cos 500 cos 50 cos 250 + sin 100 .
2

15/ O = 2 cos 220 + cos 440


16/ P =

sin 600
sin 15 sin 75
4

sin 550
sin 110

+ 3 sin150 sin 750 .

1
cos 400

17/ Q = 2 2 sin 100 2 sin 350


.

2 cos 50
sin 50
18/ R =
19/ S =

1
sin 10

1
sin10

4 sin 700 .

3
cos100

20/ T = cos

2
3
cos
+ cos
.
7
7
7

21/ U = cos

2
4
5
2
4
6
8
+ cos
+ cos
+ cos
+ cos
+ cos
+ + cos
.
7
7
7
5
5
5
5

22/ V = cos

2
4
6
8
+ cos
+ cos
+ cos
.
9
9
9
9

Bi 103. Rt gn biu thc A = 2 sin x (cos x + cos 3x + cos 5x) . T suy ra gi tr ca biu thc

B = cos

3
5
+ cos
+ cos
.
7
7
7

Bi 104. Rt gn cc biu thc sau


1/

A = cos11x cos 3x cos17x cos 9x .

2/

B = sin18x cos13x sin 9x cos 4x .

3/

C = sin x sin 3x + sin 4x sin 8x .

4/

D = sin 2x sin 6x cos x cos 3x .

5/

E = cos 3x cos 6x cos 4x cos 7x .

6/

F = sin x sin 600 x sin 600 + x .

7/

G = 8 cos x cos 600 x cos 600 + x + 1 .

Page - 82 -

) (

) (

DeThiThuDaiHoc.com
"All the flower of tomorrow are in the seeks of today"

cng hc tp mn Ton 10 Tp IIwww.MATHVN.com

1
sin 12x .
4

8/

H = cos x cos 2x sin 3x

9/

I = 4 sin 2x sin 5x sin 7x sin 4x .

10/ J = sin 2x sin 6x cos 4x +


11/ K = sin x sin 2x sin 3x

Ths. L Vn on

1
cos12x .
4

1
sin 4x .
4

12/ L = 4 cos x sin + x sin x cos 2x .


6
6

13/ M = sin 4x sin 10x sin 11x sin 3x sin 7x sin x .


14/ N = cos

x
3x
cos
sin x sin 3x sin 2x sin 3x .
2
2

15/ O = sin

7x
3x
x
5x
cos
+ sin cos
+ sin 2x cos 7x .
2
2
2
2

16/ P = cos 2x + cos 4x + cos 6x 4 cos x cos 2x cos 3x 2 .


17/ Q = sin x (1 + 2 cos 2x + 2 cos 4x + 2 cos 6x) .
18/ R =

sin 5x
2 (cos 2x + cos 4x) .
sin x

19/ S = sin2 x +

sin2 x sin cos + 2x .


4
4
12

12

Bi 105. Chng minh cc ng thc sau


1/

cos120 + cos18 0 4 cos150 cos 210 cos 24 0 =

3 +1
.
2

(i hc Quc Gia H Ni khi A nm 2001)


2/

tan 90 tan 630 + tan 810 tan 27 0 = 4 .

3/

tan 300 + tan 400 + tan 500 + tan 600 =

4/

cos 2x + 600 cos 2x 600 =

5/

(sin x + cos x)

6/

3
sin2 x + sin2 x + sin x sin x = .

4
3
3

7/

sin a sin (b c) + sin b sin (c a ) + sin c sin (a b) = 0 .

8/

cos2 x 2 cos a cos x cos (a + x) + cos2 (a + x) = sin2 a .

) (
2

"Cn c b thng minh"

8 3
cos 200 .
3

1
1
cos 4x .
2
4

) (

cos 4x = 4 sin 2x sin x + 150 cos x 150 .

DeThiThuDaiHoc.com

Page - 83 -

www.MATHVN.com
Chng 5. Cung gc lng gic Cng thc lng gic

Ths. L Vn on

9/

sin8 x cos8 x =

10/ sin8 x + cos8 x =


11/ tan

7
1
cos 2x + cos 6x .
8
8
35
7
1
+ cos 4x +
cos 8x .
64 16
64

3
2
+ 4 tan
= 11 .
11
11

12/ tan 200 tan 400 tan 800 = 3 .


13/ 8 sin 3 180 + 8 sin2 180 = 1 .
14/ sin4

(Hc Vin Ngn Hng nm 2000)

3
5
6 3
+ sin4
+ sin4
+ sin4
= .
16
16
16
16
2

15/ tan100 tan 250 + tan 250 tan 550 + tan 550 tan100 = 1 .
16/ tan 150 tan 250 tan 350 tan 850 = 1 .
17/ tan 200 tan 400 + tan 800 = 3 3 .
18/ tan 100 tan 50o + tan 600 + tan 700 = 2 3 .
19/ tan 200 + tan 400 + tan 800 + tan 600 = 8 sin 400 .
20/ tan6 200 33 tan 4 200 + 27 tan2 200 3 = 0 .
Bi 106. Tnh cc gc ca ABC bit rng
1/

BC =

1
v sin B.sin C = .
3
2

S: B =

, C= , A=
.
2
6
3

2/

B+C =

2
1+ 3
v sin B.cos C =
.
3
4

S: A =

, B=
, C= .
3
12
4

Bi 107. Chng minh iu kin cn v ABC vung


1/

cos 2A + cos 2B + cos 2C = 1 .

2/

tan 2A + tan 2B + tan 2C = 0 .

3/

b
c
a
+
=
.
cos B cos C sin B.sin C

4/

cot

B a+c
=
.
2
b

Bi 108. Chng minh iu kin cn v ABC cn:

A+B
. 2/
2

1/

a tan A + b tan B = (a + b) tan

3/

sin A + sin B
1
= (tan A + tan B) .
cos A + cos B 2

4/

2 tan B + tan C = tan2 B. tan C .


cot

C 2 sin A. sin B
=
.
2
sin C

Bi 109. Chng minh bt ng thc, t suy ra iu kin cn v ABC u:


1/

sin A + sin B + sin C

3 3
.
2

HD: Cng sin

vo VT.
3

2/

cos A + cos B + cos C

3
.
2

HD: Cng cos

vo VT.
3

Page - 84 -

DeThiThuDaiHoc.com
"All the flower of tomorrow are in the seeks of today"

cng hc tp mn Ton 10 Tp IIwww.MATHVN.com

3/

tan A + tan B + tan C 3 3 .

4/

cos A.cos B.cos C

Ths. L Vn on

1
.
8

HD: Bin i cos A.cos B. cos C


Bi 110. Cho cos = cos a cos b vi a, b

1
v dng hng ng thc.
8

+a
a
b
+ k . Chng minh: tan
tan
= tan2 .
2
2
2
2

Bi 111. Cho sin (2a + b) = 5 sin b . Chng minh:

a + k
2
= 3 vi
.

a + b + l
2

2 tan (a + b)
tan a

Bi 112. Cho tan (a + b) = 3 tan a . Chng minh: sin (2a + 2b) + sin 2a = 2 sin 2b .
Bi 113. Cho

sin (x ) A
cos (x )
a
;
=
=
v aB + bA 0 . Chng minh:
b
sin (x ) B
cos (x )

aA + bB
= cos ( ) .
aB + bA
Bi 114. Chng minh rng nu tan

x
a
= th A = a sin x + b cos x khng ph thuc vo a v x.
2
b

Bi 115. Cho a, b, c l ba cnh ca tam gic, tng ng cc gc ln lt l A, B, C . Cc gc nhn


, , c xc nh bi cos =

1/

tan2

a
b
c
, cos =
, cos =
. Chng minh:
b+c
c+a
a+b

+ tan2 + tan2 = 1 .
2
2
2

Bi 116. Chng minh:

2/

tan

A
B
C
tan tan = tan tan tan .
2
2
2
2
2
2

1
1
1
tan 8x tan x
+
+ ... +
=
.
cos x cos 2x cos 2x cos 3x
cos 7x cos 8x
sin x

Bi 117. Tnh gi tr ca biu thc: M = sin8 200 + sin 8 400 + sin8 800 .
( thi hc sinh gii tnh Ninh Bnh khi 11 nm 2006)

Bi 118. Tnh gi tr ca biu thc: N =

2 sin 20 + 4 sin 40 + ... + 178 sin 178 0 + 180 sin1800


cot10

Bi 119. Rt gn cc biu thc sau

sin 2n
.
2 sin

1/

A = cos + cos 3 + cos 5 + ... + cos (2n 1) .

2/

B = sin

(n 1) .

2
3
+ sin
+ sin
+ ... + sin
n
n
n
n

S: B = cot

3/

C = cos

(2n 1)

3
5
+ cos
+ cos
+ ... cos
.
n
n
n
n

S: C = cos

"Cn c b thng minh"

DeThiThuDaiHoc.com

S: A =

.
2n

.
n

Page - 85 -

www.MATHVN.com
Chng 5. Cung gc lng gic Cng thc lng gic

Ths. L Vn on

1
1
1

+
+ ... +
, a = . S: D = 1 5 .
cos a.cos 2a cos 2a.cos 3a
cos 4a. cos 5a
5

4/

D=

5/

1
1
1 + 1 1 + 1 ... 1 +
.
E = 1 +

cos x
cos 2x
cos 3x
cos 2n1 x

6/

F=

7/

G = sin3

S: E =

tan 2n1 x
.
x
tan
2

1
1
1
1
+
+
+ ... +
.
sin 2x sin 4x sin 8x
sin 2n x

x
x
x
x
+ 3sin 3 + 32 sin3 + ...... + 3n1 sin3 n .
3
3
3
3

Bi 120. Tm phn nguyn ca biu thc: M = tan 4

3
2
+ tan 4 + 2 tan2
+ tan2 .
4
4
7
7

1
a
3 sin x sin 3x) () . Khi thay x = n vo biu thc () . Hy
(
4
3
a
a
a
tnh Sn = sin 3 + 3 sin 3 2 + ... + 3n1 sin 3 n .
3
3
3

Bi 121. Chng minh: sin3 x =

S: Sn =

1 n
a
3 sin n sin a .

4
3

Bi 122. Chng minh: cos a =

sin x

S: Pn =

2 sin

sin 2a
x
x
x
. T suy ra gi tr ca Pn = cos cos 2 ... cos n .
2 sin a
2
2
2

2n

1
x
= cot cot x . T tnh:
sin x
2
1
1
1
S=
+
+ ... +
, 2n1 k .
n 1
sin sin 2
sin 2

Bi 123. Chng minh:

S: S = cot

cot2n 1 .
2

Bi 124. Chng minh: tan2 x. tan 2x = tan 2x 2 tan x . T tnh:

Sn = tan2

a
a
a
a
a
. tan a + 2 tan2 2 . tan + ... + 2n 1 tan2 n . tan n1 .
2
2
2
2
2

S: Sn = tan a 2n tan

Bi 125. Tnh sin2 2x, bit:


S:

Page - 86 -

a
2n

1
2

tan x

1
2

cot x

1
2

sin x

1
cos2 x

= 7.

8
.
9

DeThiThuDaiHoc.com
"All the flower of tomorrow are in the seeks of today"

cng hc tp mn Ton 10 Tp IIwww.MATHVN.com

Ths. L Vn on

Bi 126. Cho bit sin a = m v 2700 < x < 4500 . Chng minh:
1/

cos

a
1
=
2
2

Bi 127. Cho 0 < <

1+ m + 1 m .

2/

sin

a
1
=
2
2

1+ m 1 m .

. Chng minh: tan + cot 2 .


2

Bi 128. Chng minh cc bt ng thc sau


1/

sin 750 + sin150 > 1 .

2/

cos 360 > tan 360 .

Bi 129. Chng minh cc bt ng thc sau


1/
2/

sin 3x cos x cos 3x sin x cos 2x 2 .


3 sin 3x + cos 2x cos x sin 2x sin x 2 .

) (

1
.
4

3/

sin x sin x + 600 sin x 600

4/

cos x cos 3x sin 2x sin 4x 1 .

5/

cos 2x cos x + sin x sin 3x sin2 x cos 3x 1 .

6/

2 sin x (cos x + cos 3x + cos 5x + cos 7x + cos 9x) 1 .

Bi 130. Tm gi tr ln nht v gi tr nh nht ca cc biu thc sau


1/

y = 3 sin x + cos x + 2 .

2/

y = cos x +

3/

y = sin x cos x cos 2x cos 4x .

4/

y = sin 4 x + cos4 x, 0 x

5/

y = sin x + 2 sin2 x .

6/

y = cos 4 x + sin 4 x + cos2 x sin2 x .

7/

y = sin 4 4x cos2 x +

8/

y = 2 sin6 x + 2 cos6 x sin4 x cos4 x + cos 2x .

1
,
cos x

"Cn c b thng minh"

< x < .
2
2


.
2

5
cos 2x .
4

DeThiThuDaiHoc.com

Page - 87 -

Ths. L Vn on

www.MATHVN.com
Chng 5. Cung gc lng gic Cng thc lng gic

PHN II

HNH HC

Page - 88 -

DeThiThuDaiHoc.com
"All the flower of tomorrow are in the seeks of today"

cng hc tp mn Ton 10 Tp IIwww.MATHVN.com

Chng

Ths. L Vn on

PHNG PHP TA TRONG MT PHNG & NG DNG


A TA VCT TA IM
 Ta Oxy
Trong mt phng ta Oxy cho ba im

v hai vct

. Khi :
.

Vct

di on

Gi I l trung im ca on thng AB. Lc :


Gi

l trng tm ABC, lc ny:

Gi

chia on AB theo t s
(honh

vi

.
.

. Khi :
honh tung

tung)

mt vct.

.
(honh nhn honh

tung nhn tung)

mt s.
.

cng phng

iu kin

vung gc nhau

iu kin

bng nhau

.
(honh

ba im A, B, C thng hng th

Gc gia hai vct

Cho im

th ta ca im

i xng vi M qua trc honh

i xng vi M qua trc tung

i xng vi M qua gc ta

"Cn c b thng minh"

tung)

vi

honh, tung

.
.
.
.

v
.

DeThiThuDaiHoc.com

Page - 89 -

www.MATHVN.com

Ths. L Vn on

Phn hnh hc

 Mt s dng ton c bn
a/ Dng ton 1. Xc nh im tha mn mt ng thc vct hay di
Bc 1. Gi s
.
Bc 2. Ta ha cc vct c trong ng thc hoc s dng cng thc v khong
cch gia hai im, chuyn ng thc v biu thc i s.
Bc 3. Gii phng trnh hoc h trn, ta nhn c ta im M.
 Lu

D l nh th t ca hnh bnh hnh


.
xc nh tm I v bn knh ng trn R ngoi tip ABC
+ Tm I tha

. Gii h tm

. A

+ Bn knh

I
C

Ta chn ng phn gic


+ D l chn ng phn gic trong ca ABC
. (theo vng trn)

B
+ E l chn ng phn gic ngoi ca ABC

B
A

b/ Dng ton 2. Vct cng phng (thng hng) Tm im


thng hng

cng phng

.
Tm im

tng

t gi tr nh nht.

y l bi ton bt ng thc tam gic, cn phn bit hai trng hp:


+ Trng hp 1. Hai im A, B nm khc bn so vi ng thng d.
Cch 1. S dng vct cng phng
Gi

tng

thng hng

Mo

Cch 2. S dng bt ng thc tam gic


B
Trong ABM, ta c
.
Vit phng trnh ng thng AB: i qua A v B.

Page - 90 -

DeThiThuDaiHoc.com
"All the flower of tomorrow are in the seeks of today"

cng hc tp mn Ton 10 Tp IIwww.MATHVN.com

Ths. L Vn on

+ Trng hp 2. Hai im A, B nm cng bn so vi ng thng d


A
Dng A' i xng vi A qua d
.
Trong AMB, ta c:
.
Do ,

B
M

Mo

.
 Lu

A'
nm cng bn hay nm hai bn so vi ng

xt xem hai im
thng
Nu
Nu

th ta cn tnh:
A

Hai im A, B nm cng bn so vi ng thng d.


Hai im A, B nm hai bn so vi ng thng d.

Tm im

+ Trng hp 1. Hai im A, B nm cng bn so vi ng thng d

Mo

A
+ Trng hp 2. Hai im A, B nm hai bn so vi ng thng d
thng hng
Dng A' l im i xng ca im A qua d, khi :
M
.

Mo
B

.
c/ Dng ton 3. Tm hnh chiu vung gc ca
Gi

ln BC vi

A'

l hnh chiu ca A ln ng thng BC.

Ta im H tha h phng trnh:


B
A
A'
tm ta im A' i xng vi A qua BC
l trung im AA'.
H
d/ Dng ton 4. Phng php ta ha
C
Phng php ta ha thng c s dng ph bin trong hai loi ton:
thng hng
cng phng
Loi 1. Ta thc hin php ta ha cc im trong hnh v a bi ton hnh hc v
dng gii tch.
Loi 2. Lc chn cc im thch hp bin i biu thc i s v dng di hnh
hc. Phng php ny t ra rt hiu qu i vi bi ton tm gi tr ln nht v
gi tr nh nht ca cc biu thc i s.

 Lu
Du

. Du

. Du

"Cn c b thng minh"

xy ra

xy ra
xy ra

cng phng v hng.


cng phng.

cng phng v hng.

DeThiThuDaiHoc.com

Page - 91 -

www.MATHVN.com

Ths. L Vn on

Phn hnh hc

e/ Dng ton 5. Tm qu tch mt im trong mt phng ta Oxy.


Bc 1. Gi
l im cn tm qu tch v da vo gi thit v rng buc iu
kin tm quan h:

vi

Bc 2. Kh m h phng trnh
ca xo hoc yo h
Bc 3. Kt lun: t

: tp cha iu kin

. Gii hn khong chy

ta c
.

v iu kin

ta c qu tch ca im M l

+ C ng cong

nu

+ Mt phn ng cong

l tp

trn D nu

 Lu :

l
.

BA
NG
BAI T
TP AAP DU
DUNG
Bi 1.

Trong mt phng vi h trc ta Descarter vung gc Oxy, cho ABC bit A (1; 0),
B (3; 5), C (0; 3) .



1/ Xc nh ta im E sao cho AE = 2BC .
2/ Xc nh ta im F sao cho AF = CF = 5 .
 

 
3/ Tm tp hp im M sao cho 2 MA + MB 3MC = MB MC .

Bi 2.

Tm I (4; 19)

S: 1/ E (7;16) . 2/ F (4; 0) F (5; 3) . 3/ l ng trn


.
Bk : R = 73

Trong mt phng vung gc Oxy, cho ABC c A (4;1), B (2; 4),C (2; 2) .

1/ Chng minh rng ba im A, B, C khng thng hng (to thnh mt tam gic).

2/ Tnh cos CBA .
3/ Tnh chu vi v din tch ABC. Tnh bn knh ng trn ni tip tam gic.

  
4/ Tm im M sao cho: 2MA + 3MB MC = 0 .

Bi 3.

5
6
.
2/ Chu vi = 6 5 + 1 ; SABC =
.
3/ M (1; 4) .
5
5 +1
Cao ng C Kh Luyn Kim nm 2004 (cu III 2)
Trong mt phng vi h trc ta Descarter vung gc Oxy, cho ba im A (2; 3), B (2;1),

S: 1/ cos CBA =

C (2; 1) . Tm ta nh D t gic ABCD l hnh bnh hnh.

S: D (2; 5) .
Bi 4.

Trong mt phng vi h trc ta vung gc Oxy, cho A (4; 3), B (2; 7), C (3; 8) .

Page - 92 -

DeThiThuDaiHoc.com
"All the flower of tomorrow are in the seeks of today"

cng hc tp mn Ton 10 Tp IIwww.MATHVN.com

Tm im D sao cho ABCD l hnh bnh hnh.


Tm giao im I ca hai ng thng OA v BC.
Tm ta trng tm, trc tm ABC.
Tm tm ng trn ngoi tip ABC.
4 1
2
S: 1/ D (1; 12) . 2/ I ; . 3/ G 1; , H (13; 0) .
9
3
3

Ths. L Vn on

1/
2/
3/
4/

Bi 5.

4/ J (5;1) .

Trong mt phng vi h trc ta vung gc Oxy, cho A (1;5), B (4; 5), C (4; 1) . Tm
tm ng trn ni tip ABC.
S: I (1; 0) .

Bi 6.

Bi 7.

i hc Giao Thng Vn Ti Tp. H Ch Minh 2 nm 1997


Trong mt phng vi h trc ta Descarter vung gc Oxy, tm ta trc tm ca ABC,
bit ta cc nh A (1;2), B (5;7), C (4; 3) .
1 21
S: H ; .
11 11
i hc S Phm K Thut Tp. H Ch Minh nm 2001
Trong mt phng vi h trc ta vung gc Oxy, cho A (1;2), B (2; 0), C (3;1) .

1/ Xc nh tm ng trn ngoi tip ABC.


2/ Tm im M trn ng thng BC sao cho din tch ABM bng

Bi 8.

Bi 9.

1
din tch ABC.
3

11 13
1 1
11 1
S: 1/ I ; .
2/ M ; M ; .
4
3
14
3 3
3
i hc Bch Khoa H Ni nm 2001
Trong mt phng vi h trc ta Descarter vung gc Oxy, cho ABC c ba nh thuc
1
th (C) ca hm s y = . Chng minh trc tm H ca ABC cng thuc (C) .
x
 

1 1 1
BC
AH

; abc H (C) .
HD: Gi A a; , B b; ,C c; (C) . T
  H

BH AC
abc
a b c

Cao ng S Phm KomTum nm 2004


Trong mt phng vi h trc ta Oxy cho hai im A (1;2), B (3; 4) . Tm im C trn

ng thng d : x 2y + 1 = 0 sao cho ABC vung ti C.


3 4
S: C (3;2) C ; .
5 5
Bi 10. Cao ng Cng Nghip IV nm 2004
Trong mt phng vi h trc ta Descarter vung gc Oxy, cho ABC vung ti A vi
B (3; 0), C (7; 0), bn knh ng trn ni tip tam gic l r = 2 10 5 . Tm ta tm I

ca ng trn ngoi tip ABC, bit im I c tung dng.

S: I 2 + 10; 2 20 5

I 2 10; 2 10 5 .

Bi 11. i hc M a Cht nm 2001 (Cu IV 2)


Trong mt phng vi h trc ta trc chun Oxy, cho A (10;5), B (15; 5), C (20; 0) l ba
nh ca mt hnh thang cn ABCD. Tm ta im C, bit rng AB // CD.
S: C (7; 26) .
Bi 12. i hc Lut H Ni nm 1998 (Cu IV 2)
"Cn c b thng minh"

DeThiThuDaiHoc.com

Page - 93 -

Ths. L Vn on

www.MATHVN.com

Phn hnh hc

Trong mt phng vi h trc ta Descarter vung gc Oxy, tm im C thuc ng thng


x y + 2 = 0 sao cho ABC vung ti C vi A (1; 2), B (3; 3) .
7
3
S: C (1; 3) C ; .
2 2
Bi 13. i hc Nng Nghip I 1 nm 1995

Cho im A (1;1) trn mt phng ta Oxy. Hy tm im B trn ng thng y = 3 v im


C trn trc honh sao cho ABC l tam gic u.

4
5
4
5
B 1 +
, C 1 +
.
;3, C 1
;
3
S: B 1

3
3
3
3
Bi 14. i hc Tng Hp nm 1976
Trong mt phng vi h trc ta Descarter vung gc Oxy, cho A (1; 0), B (1; 0) v ly
MA2
MA
= k, (k > 0) .
v tm M sao cho
2
MB
MB

MA2
x2 + 2x + 2
k2 + 1 k4 + 6k2 1
=
M
;1
S:
v
d .
1;2

k2 1
MB2
x2 2x + 2

Bi 15. i hc Ngoi Thng nm 1993


Trong mt phng vi h trc ta Descarter vung gc Oxy, cho hai im A (3 cos t; 0) v

 
B (0; 2 sin t) . Tm tp hp cc im M (x o ; y o ) sao cho: 2AM + 5MB = 0 khi t thay i.

im di ng trn ng thng d : y = 1 . Hy tnh

S: Tp hp im M l elip (E) :

x2 9y2
+
= 1.
4
100

Bi 16. i hc M a Cht nm 1999


Trong mt phng vi h trc ta Descarter vung gc Oxy, cho ABC v im M bt k.




1/ Chng minh rng: u = 3MA 5MB + 2MC khng ph thuc vo v tr ca im M.



 
2/ Tm tp hp im M trn mt phng sao cho: 3MA + 2MB 2MC = MB MC .




S: 1/ u = 2AC 5AB .

2/ ng trn tm (C) tm I, bn knh R =

CB
.
3

Bi 17. Hc Vin Ngn Hng Tp. H Ch Minh nm 2000


Trong mt phng vi h trc ta Descarter vung gc Oxy, cho ABC c nh C (2; 4) v
trng tm G (0; 4) .
1/ Gi s M (2; 0) l trung im ca cnh BC. Xc nh ta cc nh A, B.
2/ Gi s M di ng trn ng thng d : x + y + 2 = 0 . Hy tm qu tch im B. Xc nh M
di cnh AB l ngn nht.
1 9
S: 1/ B (6; 4), A (4;12) .
2/ Qu tch l d : x + y 2 = 0 . 2/ M ; .
4 4
Bi 18. i hc Ngoi Thng nm 2000
Trong mt phng vi h trc ta Descarter vung gc Oxy, cho Parabol (P) : y = x2 v
ng thng d : y = mx + 1 . Chng minh rng khi m thay i, ng thng lun lun ct
Parabol (P) ti hai im phn bit A v B. Hy tm qu tch tm vng trn ngoi tip OAB khi
m thay i vi O l gc ta .
S: A (x1; mx1 + 1), B (x2 ; mx2 + 1) v qu tch tm l Parabol (P ') : y = 2x 2 + 1 .
Page - 94 -

DeThiThuDaiHoc.com
"All the flower of tomorrow are in the seeks of today"

cng hc tp mn Ton 10 Tp IIwww.MATHVN.com

Ths. L Vn on

Bi 19. i Hc Nng Nghip nm 1997


Trong mt phng ta Oxy cho ba im A (1;1), B (3; 3), C (2; 0) .
1/ Tnh din tch ABC.

2/ Hy tm tt c cc im M trn trc honh Ox sao cho gc AMB nh nht.


S: SABC = 2 (.v.d.t) v M O .
Bi 20. Trong mt phng Oxy cho ba im A (1; 3), B (3;1), C (2; 4 ) .
a/ Tnh din tch ABC.

b/ Tm tt c cc im M Ox sao cho gc AMB nh nht.


Bi 21. Trch b tuyn sinh i hc Cao ng 97 cu Va
Tm trn trc honh Ox im P sao cho tng cc khong cch t P n cc im A v B l nh

) ) . Bit rng:

nht hay PA + PB

min

1/ A (1;1), B (2; 4) .
2/ A (1;2), B (3; 4 ) .
6
S: 1/ P Po ; 0 .
5

5
2 / P Po ; 0 .
3
Bi 22. Tm trn ng thng d : x + y = 0 im M sao cho tng cc khong cch t M n cc im
A v B l nh nht trong cc trng hp sau

1/ A (1;1), B (2; 4) .

2/ A (1;1), B (3; 2) .
Bi 23. Cho im M (4;1) v hai im A (a; 0), B (0; b) vi a, b > 0 sao cho A, B, M thng hng. Xc
nh ta im A, B sao cho
1/ Din tch tam gic OAB l nh nht (SOAB min ) .
2/ OA + OB nh nht.
1
1
3/
+
nh nht.
2
OA
OB2
S: 1/ A (8; 0), B (0;2) .

2/ A (6; 0), B (0; 3) .

17

;0, B (0;17) .
4

3/ A

Bi 24. Cho im M (2;1) v hai im A (a; 0), B (0; b) vi a, b > 0 sao cho A, B, M thng hng. Xc
nh ta im A, B sao cho:
1/ Din tch tam gic OAB l nh nht (SOAB min ) .
2/ OA + OB nh nht.
1
1
3/
+
nh nht.
2
OA
OB2
S: 1/ A (4; 0), B (0;2) .
Bi 25. Trong mt phng vi h trc ta Descarter vung gc Oxy, cho A (1; 2), B (3; 4) .
1/ Tm im M trn trc honh sao cho tng khong cch t M n hai im A, B l ngn nht.
2/ Tm im N trn trc honh sao cho NA NB l di nht.
3/ Tm im I trn trc tung sao cho (IA + IB)

min

"Cn c b thng minh"

DeThiThuDaiHoc.com

Page - 95 -

www.MATHVN.com

Ths. L Vn on

Phn hnh hc

 
4/ Tm im J trn trc tung sao cho JA + JB ngn nht.
5
S: 1/ M ; 0 .
3

2/ NA NB

3/ (IA + IB)

min

1
= 2 13 khi I 0; .
2

max

 
4/ JA + JB
min

= 2 2 khi N (1; 0) .

= 4 khi J (0;1) .

Bi 26. Cho ba im A (0;6), B (2;5), M (2t 2; t) . Tm ta im M sao cho


2/ MA MB .
(MA + MB) .
Cho ba im A (1;2), B (2;5), M (2t + 2; t) . Tm ta im M sao cho
 
1/ (MA + MB) .
2/ MA + MB .
1/
Bi 27.

3/ MA MB

min

max

min

max

max

4/ MA MB

min

Bi 28. Hc Vin K Thut Mt M nm 2000


Trong mt phng vi h trc ta Descarter vung gc Oxy, tm qu tch im M sao cho
khong cch t M n A (1;2) v khong cch t M n Ox lun bng nhau.
Bi 29. Cao ng khi M, T nm 2003
Trn mt phng Oxy, cho hai im A (1;2), B (3; 4) . Tm trn tia Ox mt im P sao cho

AP + PB l nh nht.
5
S: P ; 0 .
3
Bi 30. i hc Quc Gia Tp. H Ch Minh nm 1997 (cu IVa 1)
Trong mt phng vi h trc ta vung gc Oxy, cho A (0;2), Parabol (P) : y = x 2 . Xc
nh im M trn (P) sao cho AM min .

6 3

S: M1;2
; .
2 2

Page - 96 -

DeThiThuDaiHoc.com
"All the flower of tomorrow are in the seeks of today"

cng hc tp mn Ton 10 Tp IIwww.MATHVN.com

Ths. L Vn on

B PHNG TRNH NG THNG



 Vect ch phng ca ng thng
Vect

c gi l vct ch phng

ca ng thng nu gi ca n

song song hoc trng vi . K hiu

Nhn xt
+ Nu l mt VTCP ca th

cng l mt VTCP ca .

+ Mt ng thng hon ton c xc nh nu bit mt im v mt VTCP.


 Vect php tuyn ca ng thng
Vect

c gi l vct php tuyn

vung gc vi . K hiu

ca ng thng nu gi ca n

Nhn xt
+ Nu l mt VTPT ca th

cng l mt VTPT ca .

+ Mt ng thng hon ton c xc nh nu bit mt im v mt VTPT.


+ Nu l mt VTCP v l mt VTPT ca th
.
 Phng trnh tham s ca ng thng
Cho ng thng i qua
(

v c

. Phng trnh tham s ca


.

l tham s) v

Nhn xt
hay

+ Gi k l h s gc ca th

vi
vi

 Phng trnh chnh tc ca ng thng

Cho ng thng i qua


v c

. Phng trnh chnh tc ca

.
Trong trng hp

hoc

th ng thng khng c phng

trnh chnh tc.


"Cn c b thng minh"

DeThiThuDaiHoc.com

Page - 97 -

www.MATHVN.com

Ths. L Vn on

Phn hnh hc

Phng trnh tng qut ca ng thng


Phng trnh:
vi
phng trnh tng qut ca ng thng.
Nhn xt
+ Nu c phng trnh:

+ Nu i qua

(a, b khng n thi

th c

v c

) c gi l

th phng trnh ca l
.

+ ng thng i qua hai im

. Phng trnh ca

. c gi l phng trnh ng thng theo on chn.


+ ng thng i qua im

v c h s gc k. Phng trnh ca

. c gi l phng trnh ng thng theo h s gc k.


+ Mt s trng hp t bit:
Cc h s

Phng trnh ng thng

Tnh cht ng thng


i qua gc to O
// Ox hoc Ox
// Oy hoc Oy

V tr tng i ca hai ng thng


Cho hai ng thng

To giao im ca 1 v 2 l nghim ca h phng trnh


t

ct

c mt nghim

v nghim

v s nghim

 Lu : Trong cc biu thc t s:

.
.

.
th

Page - 98 -

DeThiThuDaiHoc.com
"All the flower of tomorrow are in the seeks of today"

cng hc tp mn Ton 10 Tp IIwww.MATHVN.com

Ths. L Vn on

Gc gia hai ng thng


Cho hai ng thng

c VTPT
c VTPT

v ng thng

Lc :

v
1
.

Lu

2
.

+ Nu
+ Nu

th

Khong cch t mt im n mt ng thng


Khong cch t mt im n mt ng thng:
Cho ng thng

V tr tng i ca hai im i vi mt ng thng:


Cho ng thng
v hai im

+ M, N nm cng pha i vi

+ M, N nm khc pha i vi

Phng trnh cc ng phn gic ca cc gc to bi hai ng thng:


Cho hai ng thng
v
ct nhau.
Phng trnh cc ng phn gic ca cc gc to bi hai ng thng 1 v 2 l:
.
Ta c th phn bit ng phn gic trong hoc ngoi da vo du ca tch
Du ca tch

Trong :

"Cn c b thng minh"

Phng trnh gc nhn

nh sau:

Phng trnh gc t

DeThiThuDaiHoc.com

Page - 99 -

www.MATHVN.com

Ths. L Vn on

Phn hnh hc

Dng ton 1. Lp phng trnh ng thng v mt s bi ton lin quan



 Lp phng trnh ng thng

Lp phng
trnh ng
thng d

 Mt s lu :
ng thng qua im
ng thng qua

.
v c h s gc k

ng thng
c phng trnh:
.
ng thng
c phng trnh:
.
Trong nhiu trng hp c th, xc nh phng trnh ng thng chng ta cn s dng:
+ Phng trnh chm ng thng.
+ Phng trnh qu tch.
Ta c th chuyn i gia cc phng trnh tham s, chnh tc, tng qut ca 1 ng thng.

l mt phng trnh ng thng th
.
 Mt s bi ton thng gp khc
a/ Tm im c nh ca h ng cong (thng)
.

Bc 1. Gi
Bc 2. Bin i

v mt trong cc dng

(bin s l m).

Bc 3. Ta im c nh:

Page - 100 -

+ Nu

c bin i v

th ta tha

+ Nu

c bin i v

th ta tha

DeThiThuDaiHoc.com
"All the flower of tomorrow are in the seeks of today"

cng hc tp mn Ton 10 Tp IIwww.MATHVN.com

Ths. L Vn on

b/ Cho h ng thng ph thuc tham s m c phng trnh


ng cong c nh lun tip xc vi h

. Hy tm

gii bi ton ny, ta c th s dng theo hai phng php


 Phng php 1. Thc hin theo hai bc:
.

Bc 1. nh dng cho th c nh, chng hn nh parabol

Bc 2. S ng iu kin tip xc ca hai th vi mi gi tr ca tham s, ta xc nh


c
l ng cong c nh tip xc vi h
cn tm.
 Phng php 2. Thc hin theo hai bc:
Bc 1. Tm tp hp cc im m h

khng i qua. Tp hp c xc nh bi
.

bt phng trnh c dng


Bc 2. Ta i chng minh h

lun tip xc vi ng cong

c phng trnh

.
c/ Tm im M i xng vi im M qua ng thng
gii bi ton ny, ta c th s dng theo hai phng php
 Phng php 1
Bc 1. Vit phng trnh ng thng qua M v vung gc vi d.
Bc 2. Xc nh
(H l hnh chiu ca M trn d).
Bc 3. Xc nh
sao cho H l trung im ca
.
M

 Phng php 2
Bc 1. Gi H l trung im ca

Bc 2. M i xng ca M qua

(s dng ta ).

H
M'

d/ Lp phng trnh ng thng d i xng vi ng thng d qua ng thng


gii bi ton ny, trc tin ta nn xem xt chng ct nhau hay song song.
 Nu d //
Bc 1. Ly A d. Xc nh A i xng vi A qua .
Bc 2. Vit phng trnh ng thng d qua A v song song vi d.
I

 Nu d

Bc 1. Ly A d (A I). Xc nh A i xng vi A qua .


Bc 2. Vit phng trnh ng thng d qua A v I.
A
A

I
H

A'

A'
"Cn c b thng minh"

DeThiThuDaiHoc.com

Page - 101 -

Ths. L Vn on

www.MATHVN.com

Phn hnh hc

e/ Lp phng trnh ng thng d i xng vi ng thng d qua im I


Bc 1. Ly A d. Xc nh A i xng vi A qua I.
Bc 2. Vit phng trnh ng thng d qua A v song song vi d.

BA
NG
BAI T
TP AAP DU
DUNG
Bi 31.

Lp phng trnh tham s, phng trnh chnh tc (nu c) v phng trnh tng qut ca

ng thng i qua im A v c vct ch phng u :


1/ A O (0; 0), u = (1; 3) .
2/ A (2; 3), u = (5; 1) .


3/ A (3; 1), u = (2; 5) .
4/ A (2; 0), u = (3; 4) .


5/ A (1;2), u = (4;6) .
6/ A (1;1), u = (1; 5) .


7/ A (2; 3), u = (4; 1) .
8/ A (3; 5), u = (0; 2) .


9/ A (7; 3), u = (0; 3) .
10/ A (1;2), u = (5; 0) .

Bi 32.

Lp phng trnh tham s, phng trnh chnh tc (nu c) v phng trnh tng qut ca

ng thng i qua im A v c vct ch phng n :


1/ A (0;1), n = (1;2) .
2/ A (2; 3), n = (5; 1) .


3/ A (3; 4), n = (4; 3) .
4/ A (1;2), n = (2; 3) .


5/ A (1; 3), n = (3; 4) .
6/ A (3; 1), n = (2; 5) .


7/ A (2; 0), n = (1; 1) .
8/ A (1;2), n = (5; 0) .


9/ A (7; 3), n = (0; 3) .
10/ A O (0; 0), n = (2;5) .

Bi 33.

Cho ng thng c phng trnh d : 2x 3y + 1 = 0 .


1/ Hy tm vct php tuyn v vct ch phng ca ng thng d.
2/ Vit phng trnh tham s v phng trnh chnh tc ca ng thng d.

Bi 34.

Bi 35.

Lp phng trnh tham s, phng trnh chnh tc (nu c) v phng trnh tng qut ca
ng thng i qua im A v c h s gc k
1/ A (2; 4), k = 2 .

2/ A (3;1), k = 2 .

3/ A (5; 8), k = 3 .

4/ A (3; 4), k = 3 .

5/ A (5;2), k = 1 .

6/ A (3; 5), k = 1 .

7/ A (2; 4), k = 0 .

8/ A (4; 0), k = 9 .

9/ A O (0; 0), k = 4 .

10/ A (0; 30), k = 7 .

Lp phng trnh tham s, phng trnh chnh tc (nu c) v phng trnh tng qut ca
ng thng i qua hai im A v B

Page - 102 -

DeThiThuDaiHoc.com
"All the flower of tomorrow are in the seeks of today"

cng hc tp mn Ton 10 Tp IIwww.MATHVN.com

Bi 36.

1/ A (2; 1), B (4; 5) .

2/ A ( 2; 4), B (1; 0) .

3/ A (5; 3), B ( 2; 7) .

4/ A (3; 5), B (3; 8) .

5/ A (3; 5), B (6; 2) .

6/ A (4; 0), B (3; 0) .

7/ A (0; 3), B (0; 2) .

8/ A (3; 0), B (0; 5) .

9/ A (0; 4), B ( 3; 0) .

10/ A ( 2; 0), B (0; 6) .

Lp phng trnh tham s, phng trnh chnh tc (nu c) v phng trnh tng qut ca
ng thng d i qua im A v song song vi ng thng

1/ A (2; 3), : 4x 10y + 1 = 0 .

2/ A (5; 7), : x 2y + 6 = 0 .

3/ A (1; 2), : 5x + 1 = 0 .

4/ A (1; 7), : y 2 = 0 .

x = 1 2t
5/ A (2; 3), :
.

y = 3 + 4t

x = 1 3t
6/ A (5; 3), :
.

y = 3 + 5t

7/ A (0; 3), :

x 1 y + 4
=
.
3
2

9/ A (1; 2), Ox .
Bi 37.

Bi 39.

8/ A (5; 2), :

x +2 y 2
=
.
1
2

10/ A (4; 3), Oy .

Lp phng trnh tham s, phng trnh chnh tc (nu c) v phng trnh tng qut ca
ng thng d i qua im A v vung gc vi ng thng

1/ A (4; 1), : 3x 5y + 2013 = 0 .

2/ A (2; 3), : x + 3y 7 = 0 .

3/ A (4;5), : x + 5y 4 = 0 .

4/ A (5;5), Ox .

5/ A (4; 1), Oy .

6/ A (7;2012), : 2012x 3y + 11 = 0 .

7/ A (1; 4), :

Bi 38.

Ths. L Vn on

x 1 y + 3
=
.
1
2

8/ A (4; 6), :

x +2
y3
=
.
3
10

x = 2t
x = 2 + t
9/ A (1; 0), :
.
10/ A (0;7), :
.

y = 1 4t
y = t

Trong mt phng vi h trc ta Descarters vung gc Oxy, cho ABC c cc nh tng


ng sau. Hy lp:
a/ Phng trnh ba cnh ABC.
b/ Phng trnh cc ng cao. T suy ra trc tm ca ABC.
c/ Phng trnh cc ng trung tuyn. Suy ra trng tm ca ABC.
d/ Phng trnh cc ng trung bnh trong ABC.
e/ Phng trnh cc ng trung trc. Suy ra bn knh ng trn ni tip ABC.
1/ A (1; 1), B (2;1), C (3;5) .

2/ A (2; 0), B (2; 3), C (0; 1) .

3/ A (4;5), B (1;1), C (6; 1) .

4/ A (1; 4), B (3; 1), C (6;2) .

5/ A ( 1; 1), B (1;9), C (9;1) .

6/ A (4; 1), B ( 3;2), C (1;6) .

Cho ABC, bit phng trnh ba cnh ca tam gic. Vit phng trnh cc ng cao AA ',
BB ', CC ' ca tam gic, vi

1/ AB : 2x 3y 1 = 0, BC : x + 3y + 7 = 0, CA : 5x 2y + 1 = 0 .
"Cn c b thng minh"

DeThiThuDaiHoc.com

Page - 103 -

Ths. L Vn on

www.MATHVN.com

Phn hnh hc

2/ AB : 2x + y + 2 = 0, BC : 4x + 5y 8 = 0, CA : 4x y 8 = 0 .
Bi 40.

Bi 41.

Bi 42.

Bi 43.

Bi 44.

Bi 45.

Vit phng trnh cc cnh v cc trung trc ca tam gic ABC bit trung im ca cc cnh
BC, CA, AB ln lt l cc im M, N, P vi

1/ M (1;1), N (5;7), P (1; 4) .

2/ M (2;1), N (5; 3), P (3; 4) .

3
1
3/ M 2; , N 1; , P (1; 2) .

2
2

3
7
4/ M ;2, N ; 3, P (1; 4) .
2
2

3 5
5 7
5/ M ; , N ; , P (2; 4)
6/ M ( 1; 1), N (1;9), P (9;1) .
2 2
2 2
Vit phng trnh ng thng i qua im M v chn trn hai trc to 2 on bng nhau
(to vi hai trc ta mt tam gic vung cn) vi

1/ M (4;10) .

2/ M (2;1) .

3/ M (3; 2) .

4/ M (2; 1) .

Vit phng trnh ng thng i qua im M v cng vi hai trc to to thnh mt tam
gic c din tch S, vi

1/ M ( 4;10), S = 2 .

2/ M (2;1), S = 4 .

3/ M ( 3; 2), S = 3 .

4/ M (2; 1), S = 4 .

Tm hnh chiu ca im M ln ng thng d v im M i xng vi M qua ng thng d


vi

1/ M (2;1), d : 2x + y 3 = 0 .

2/ M (3; 1), d : 2x + 5y 30 = 0 .

3/ M (4;1), d : x 2y + 4 = 0 .

4/ M (5;13), d : 2x 3y 3 = 0 .

Lp phng trnh ng thng d i xng vi ng thng d qua ng thng , vi


1/ d : 2x y + 1 = 0, : 3x 4y + 2 = 0 . 2/ d : x 2y + 4 = 0, : 2x + y 2 = 0 .

3/ d : x + y 1 = 0, : x 3y + 3 = 0 .

4/ d : 2x 3y + 1 = 0, : 2x 3y 1 = 0 .

Cho phng trnh: mx + (m 2) y m = 0

(1) .

1/ Chng minh: m phng trnh (1) l phng trnh ca mt ng thngm gi l h (dm ) .


2/ Tm im c nh m h (dm ) lun i qua.
S: 2/ M (1; 0) .
Bi 46.

Cho h ng thng c phng trnh: (dm ) : (2m + 1) x y m2 = 0 . Chng minh rng h

ng thng (dm ) lun tip xc vi mt parabol c nh.


S: Tip xc vi parabol (P) : y = x2 + x .
Bi 47.

Cho hai im A (0;2), B (m; 2) .

1/ Hy vit phng trnh ng trung trc d ca AB.


2/ Chng minh rng d lun tip xc vi mt ng cong c nh khi m thay i.
S: Tip xc vi parabol (P) : x =
Page - 104 -

1 2
y .
4

DeThiThuDaiHoc.com
"All the flower of tomorrow are in the seeks of today"

cng hc tp mn Ton 10 Tp IIwww.MATHVN.com

Ths. L Vn on

Dng 2. Cc bi ton dng tam gic S tng giao Khong cch Gc



 Cc bi ton dng tam gic
l cc bi ton xc nh to cc nh hoc phng trnh cc cnh ca mt tam gic khi bit
mt s yu t ca tam gic . gii loi bi ton ny ta thng s dng n cc cch dng tam
gic. Ta thng gp mt s loi c bn sau y
a/ Loi 1. Dng ABC, khi bit cc ng thng cha cnh BC v hai ng cao BB, CC.
A
Xc nh ta cc im
.
B'
Dng AB qua B v vung gc vi CC.
C'
Dng AC qua C v vung gc vi BB.
Xc nh ta
.
B
C
b/ Loi 2. Dng ABC, khi bit nh A v hai ng thng cha hai ng cao BB, CC.
Dng AB qua A v vung gc vi CC.
Dng AC qua A v vung gc vi BB.
Xc nh

c/ Loi 3. Dng ABC, khi bit nh A, 2 ng thng cha 2


ng trung tuyn BM, CN.

Xc nh trng tm
.
Xc nh A i xng vi A qua G ( BA // CN, CA // BM).
Dng dB qua A v song song vi CN.
Dng dC qua A v song song vi BM.
B
Xc nh
.

M
G
C

d/ Loi 4. Dng ABC, khi bit hai ng thng cha hai cnh AB, AC vA'
trung im M
ca cnh BC

Xc nh
.
Dng d1 qua M v song song vi AB.
Dng d2 qua M v song song vi AC.
Xc nh trung im I ca

A
.

Xc nh trung im J ca
Xc nh B, C sao cho

B
Mim C sao choC
Ngoi cch gii trn, ta c th dng theo: Trn AB ly im B, trn AC ly
.

 V tr tng i Khong cch Gc


Xem li l thuyt.
chng minh ba ng thng ng qui, ta c th thc hin nh sau
+ Tm giao im ca hai trong ba ng thng.
+ Chng t ng thng th ba i qua giao im .

"Cn c b thng minh"

DeThiThuDaiHoc.com

Page - 105 -

www.MATHVN.com

Ths. L Vn on

Phn hnh hc

BA
NG
BAI T
TP AAP DU
DUNG
CC BI TON DNG TAM GIC
Bi 48.

Bi 49.

Bi 50.

Bi 51.

Bi 52.

Bi 53.

Cho tam gic ABC, bit phng trnh mt cnh v hai ng cao. Vit phng trnh hai cnh
v ng cao cn li, vi
1/ BC : 4x + y 12 = 0,

BB ' : 5x 4y 15 = 0,

CC ' : 2x + 2y 9 = 0 .

2/ BC : 5x 3y + 2 = 0,

BB ' : 4x 3y + 1 = 0,

CC ' : 7x + 2y 22 = 0 .

3/ BC : x y + 2 = 0,

BB ' : 2x 7y 6 = 0,

CC ' : 7x 2y 1 = 0 .

4/ BC : 5x 3y + 2 = 0,

BB ' : 2x y 1 = 0,

CC ' : x + 3y 1 = 0 .

Cho tam gic ABC, bit to mt nh v phng trnh hai ng cao. Vit phng trnh cc
cnh ca tam gic , vi
1/ A (3; 0),

BB ' : 2x + 2y 9 = 0,

CC ' : 3x 12y 1 = 0 .

2/ A (1; 0),

BB ' : x 2y + 1 = 0,

CC ' : 3x + y 1 = 0 .

Cho tam gic ABC, bit to mt nh v phng trnh hai ng trung tuyn. Vit phng
trnh cc cnh ca tam gic , vi
1/ A (1; 3),

BM : x 2y + 1 = 0,

CN : y 1 = 0 .

2/ A (3;9),

BM : 3x 4y + 9 = 0,

CN : y 6 = 0 .

Cho tam gic ABC, bit phng trnh mt cnh v hai ng trung tuyn. Vit phng trnh
cc cnh cn li ca tam gic , vi
1/ AB : x 2y + 7 = 0,

AM : x + y 5 = 0,

BN : 2x + y 11 = 0 .

2/ AB : x y + 1 = 0,

AM : 2x + 3y = 0,

BN : 2x + 6y + 3 = 0 .

Cho tam gic ABC, bit phng trnh hai cnh v to trung im ca cnh th ba. Vit
phng trnh ca cnh th ba, vi
1/ AB : 2x + y 2 = 0,

AC : x + 3y 3 = 0,

M (1;1) .

2/ AB : 2x y 2 = 0,

AC : x + y + 3 = 0,

M (3; 0) .

3/ AB : x y + 1 = 0,

AC : 2x + y 1 = 0,

M (2;1) .

4/ AB : x + y 2 = 0,

AC : 2x + 6y + 3 = 0,

M (1;1) .

Cho tam gic ABC, bit to mt nh, phng trnh mt ng cao v mt trung tuyn. Vit
phng trnh cc cnh ca tam gic , vi
1/ A (4; 1),

BH : 2x 3y + 12 = 0,

BM : 2x + 3y = 0 .

2/ A (2; 7),

BH : 3x + y + 11 = 0,

CN : x + 2y + 7 = 0 .

3/ A (0; 2),

BH : x 2y + 1 = 0,

CN : 2x y + 2 = 0 .

4/ A (1;2),

BH : 5x 2y 4 = 0,

CN : 5x + 7y 20 = 0 .

V TR TNG I CA HAI NG THNG


Page - 106 -

DeThiThuDaiHoc.com
"All the flower of tomorrow are in the seeks of today"

cng hc tp mn Ton 10 Tp IIwww.MATHVN.com

Bi 54.

Xt v tr tng i ca cc cp ng thng sau, nu chng ct nhau th tm to giao im


ca chng
1/ d1 : 2x + 3y + 1 = 0

&

d2 : 4x + 5y 6 = 0 .

2/ d1 : 4x y + 2 = 0

&

d2 : 8x + 2y + 1 = 0 .

x = 5 + t
3/ d1 :

y = 3 + 2t

x = 1 t
4/ d1 :

y = 2 + 2t

x = 5 + t
5/ d1 :

y = 1

&

Bi 57.

Bi 58.

d2 : x + y 5 = 0 .

&

d2 : x + 2y 4 = 0 .

&

c/ Trng nhau.
: 2x + y 3 = 0 .

2/ d : 2mx + (m 1) y 2 = 0

&

: (m + 2) x + (2m + 1) y (m + 2) = 0

3/ d : (m 2) x + (m 6) y + m 1 = 0

&

: (m 4) x + (2m 3) y + m 5 = 0 .

4/ d : (m + 3) x + 2y + 6 = 0

&

: mx + y + 2 m = 0 .

b/ Song song.

Tm m ba ng thng sau ng qui


1/ d1 : y = 2x 1

d2 : 3x + 5y = 8

d3 : (m + 8) x 2my = 3m .

2/ d1 : y = 2x m

d2 : y = x + 2m

d3 : mx (m 1) y = 2m 1 .

3/ d1 : 5x + 11y = 8

d2 : 10x 7y = 74

d3 : 4mx + (2m 1) y + m + 2 .

4/ d1 : 3x 4y + 15 = 0

d2 : 5x + 2y 1 = 0

d3 : mx (2m 1) y + 9m 13 = 0 .

Vit phng trnh ng thng d i qua giao im ca hai ng thng d1 v d2 v


1/ d1 : 3x 2y + 10 = 0

d2 : 4x + 3y 7 = 0

d qua A (2;1) .

2/ d1 : 3x 5y + 2 = 0

d2 : 5x 2y + 4 = 0

d song song d3 : 2x y + 4 = 0 .

3/ d1 : 3x 2y + 5 = 0

d2 : 2x + 4y 7 = 0

d vung d3 : 4x 3y + 5 = 0 .

Tm im m cc ng thng sau lun i qua vi mi m


1/

(m 2) x y + 3 = 0 .

3/ mx y 2m 1 = 0 .
Bi 59.

&

Cho hai ng thng d v . Tm m hai ng thng


a/ Ct nhau.
1/ d : mx 5y + 1 = 0

Bi 56.

x = 4 + 2t
d2 :
.
y = 7 + 3t

x = 2 + 3t
d2 :
.
y = 4 6t

&

6/ d1 : x = 2
Bi 55.

Ths. L Vn on

2/ mx y + (2m + 1) = 0 .
4/

(m + 2) x y + 1 = 0 .

Cho tam gic ABC vi A (0; 1), B (2; 3), C (2; 0) .


1/ Vit phng trnh cc ng trung tuyn, phng trnh cc ng cao, phng trnh cc
ng trung trc ca tam gic.

"Cn c b thng minh"

DeThiThuDaiHoc.com

Page - 107 -

Ths. L Vn on

www.MATHVN.com

Phn hnh hc

2/ Chng minh cc ng trung tuyn ng qui, cc ng cao ng qui, cc ng trung


trc ng qui.
Bi 60.

Hai cnh ca hnh bnh hnh ABCD c phng trnh x 3y = 0, 2x + 5y + 6 = 0 , nh

C (4; 1) . Vit phng trnh hai cnh cn li.


Bi 61.

Vit phng trnh ng thng i qua im M v cch u hai im P, Q vi


1/ M (2; 5), P ( 1; 2), Q (5; 4) .

2/ M (1; 5), P ( 2; 9), Q (3; 2) .

KHONG CCH PHNG TRNH NG PHN GIC


Bi 62.

Bi 63.

Tnh khong cch t im M n ng thng d, vi


1/ M (4; 5), d : 3x 4y + 8 = 0 .

2/ M (3;5), d : x + y + 1 = 0 .

x = 2t
.
3/ M (4; 5), d :

y = 2 + 3t

4/ M (3;5), d :

x 2 y +1
=
.
2
3

Trong mt phng vi h trc ta Descarter vung gc Oxy:


1/ Cho ng thng : 2x y + 3 = 0 . Tnh bn knh ng trn tm I (5;3) v tip xc
vi ng thng .
2/ Cho hnh ch nht ABCD c phng trnh 2 cnh l: 2x 3y + 5 = 0, 3x + 2y 7 = 0
v nh A (2; 3) . Tnh din tch hnh ch nht .

3/ Tnh din tch hnh vung c 4 nh nm trn 2 ng thng song song:


d1 : 3x 4y + 6 = 0 v d2 : 6x 8y 13 = 0 .
Bi 64.

Cho tam gic ABC. Tnh din tch tam gic ABC, vi

1/ A ( 1; 1), B (2; 4), C (4; 3) .


Bi 65.

Vit phng trnh ng thng d song song v cch ng thng mt khong h, vi

1/ : 2x y + 3 = 0, h = 5 .
3/ : y 3 = 0, h = 5 .
Bi 66.

Bi 67.

Bi 68.

2/ A ( 2;14), B (4; 2), C (5; 4) .

x = 3t
2/ :
, h = 3.

y = 2 + 4t

4/ : x 2 = 0, h = 4 .

Vit phng trnh ng thng d song song vi ng thng v cch im A mt khong


bng h, vi

1/ : 3x 4y + 12 = 0, A (2; 3), h = 2 .

2/ : x + 4y 2 = 0, A (2; 3), h = 3 .

3/ : y 3 = 0, A (3; 5), h = 5 .

4/ : x 2 = 0, A (3;1), h = 4 .

Vit phng trnh ng thng i qua A v cch B mt khong bng h, vi

1/ A ( 1; 2), B (3; 5), d = 3 .

2/ A ( 1; 3), B (4; 2), d = 5 .

3/ A (5; 1), B (2; 3), d = 5 .

4/ A (3; 0), B (0; 4), d = 4 .

Vit phng trnh ng thng i qua im M v cch u hai im P, Q, vi

1/ M (2; 5), P ( 1; 2), Q (5; 4) .


Page - 108 -

2/ M (1; 2), P (2; 3), Q (4; 5) .

DeThiThuDaiHoc.com
"All the flower of tomorrow are in the seeks of today"

cng hc tp mn Ton 10 Tp IIwww.MATHVN.com

3/ M (10; 2), P (3; 0), Q ( 5; 4) .


Bi 69.

4/ M (2; 3), P (3; 1), Q (3; 5) .

Vit phng trnh ng thng d cch im A mt khong bng h v cch im B mt khong


bng k, vi

1/ A (1; 1), B (2; 3), h = 2, k = 4 .


Bi 70.

Ths. L Vn on

2/ A (2; 5), B ( 1; 2), h = 1, k = 3 .

Cho ng thng : x y + 2 = 0 v cc im O (0; 0), A (2; 0), B ( 2; 2) .

1/
2/
3/
4/

Chng minh ng thng ct on thng AB.


Chng minh rng hai im O, A nm cng v mt pha i vi ng thng .
Tm im O i xng vi O qua
Trn , tm im M sao cho di ng gp khc OMA ngn nht.

Bi 71.

Cho hai im A (2; 2), B (5; 1) . Tm im C trn ng thng : x 2y + 8 = 0 sao cho

Bi 72.

din tch tam gic ABC bng 17 (vdt).


76 18
S: C (12;10), C ; .
5
5
Tm tp hp im

1/ Tm tp hp cc im cch ng thng : 2x + 5y 1 = 0 mt khong bng 3.


2/ Tm tp hp cc im cch u 2 ng thng d : 5x + 3y 3 = 0, : 5x + 3y + 7 = 0 .
3/ Tm tp hp cc im cch u hai ng thng d : 4x 3y + 2 = 0, : y 3 = 0 .
4/ Tm tp hp cc im c t s cc khong cch n hai ng thng sau bng
d : 5x 12y + 4 = 0 v : 4x 3y 10 = 0 .

Bi 73.

Bi 74.

5
:
13

Vit phng trnh cc ng phn gic ca cc gc to bi hai ng thng

1/ 3x 4y + 12 = 0, 12x + 5y 20 = 0 .

2/ 3x 4y 9 = 0, 8x 6y + 1 = 0 .

3/ x + 3y 6 = 0, 3x + y + 2 = 0 .

4/ x + 2y 11 = 0, 3x 6y 5 = 0 .

Cho tam gic ABC. Tm tm v bn knh ng trn ni tip tam gic ABC, vi

1/ A ( 3; 5), B (4; 6), C (3; 1) .

2/ A (1; 2), B (5; 2), C (1; 3) .

3/ AB : 2x 3y + 21 = 0, BC : 2x + 3y + 9 = 0, CA : 3x 2y 6 = 0 .
4/ AB : 4x + 3y + 12 = 0, BC : 3x 4y 24 = 0, CA : 3x + 4y 6 = 0 .

GC
Bi 75.

Bi 76.

Tnh gc gia hai ng thng

1/ x 2y 1 = 0, x + 3y 11 = 0 .

2/ 2x y + 5 = 0, 3x + y 6 = 0 .

3/ 3x 7y + 26 = 0, 2x + 5y 13 = 0 .

4/ 3x + 4y 5 = 0, 4x 3y + 11 = 0 .

Tnh s o ca cc gc trong tam gic ABC, vi

1/ A ( 3; 5), B (4; 6), C (3; 1) .

2/ A (1; 2), B (5; 2), C (1; 3) .

3/ AB : 2x 3y + 21 = 0, BC : 2x + 3y + 9 = 0, CA : 3x 2y 6 = 0 .
4/ AB : 4x + 3y + 12 = 0, BC : 3x 4y 24 = 0, CA : 3x + 4y 6 = 0 .
"Cn c b thng minh"

DeThiThuDaiHoc.com

Page - 109 -

www.MATHVN.com

Ths. L Vn on

Bi 77.

Phn hnh hc

Cho hai ng thng d v . Tm m gc gia hai ng thng bng , vi

1/ d : 2mx + (m 3) y + 4m 1 = 0, : (m 1) x + (m + 2) y + m 2 = 0, = 450 .
2/ d : (m + 3) x (m 1) y + m 3 = 0, : (m 2) x + (m + 1) y m 1 = 0, = 900 .
Bi 78.

Bi 79.

Vit phng trnh ng thng d i qua im A v to vi ng thng mt gc , vi

1/ A (6;2), : 3x + 2y 6 = 0, = 450 .

2/ A (2; 0), : x + 3y 3 = 0, = 450 .

3/ A (2;5), : x + 3y + 6 = 0, = 600 .

4/ A (1; 3), : x y = 0, = 300 .

Cho hnh vung ABCD c tm I (4; 1) v phng trnh mt cnh l 3x y + 5 = 0 .

1/ Vit phng trnh hai ng cho ca hnh vung.


2/ Tm to 4 nh ca hnh vung.

BI TP QUA CC K THI
Bi 80.

Cao ng S Phm Nh Tr Mu Gio TW1 nm 2000


Trong mt phng ta Oxy, cho ng thng () : 2x 3y + 3 = 0 . Vit phng trnh
ng thng i qua M (5;13) v vung gc vi ng thng () .
S: d : 3x + 2y 11 = 0 .

Bi 81.

Cao ng S Phm H Ni nm 1997


Trong mt phng ta Oxy, cho ABC vi A (1; 1), B (2;1), C (3;5) .
1/ Vit phng trnh ng vung gc AH k t A n trung tuyn BK ca ABC.
2/ Tnh din tch ABK.
S: 1/ AH : 4x + y 3 = 0 .

Bi 82.

2/ SABK = 11 (vdt) .

Cao ng K Ngh Tp. H Ch Minh nm 1998


Trong mt phng ta Oxy, cho hai ng thng: (1 ) : 4x 3y 12 = 0 v

( ) : 4x + 3y 12 = 0 .
2

1/ Xc nh nh ca tam gic c ba cnh thuc (1 ), (2 ) v trc Oy .


2/ Tm ta tm v bn knh ng trn ni tip tam gic ni trn.

A (0; 4) = Oy
Tm I 4 ; 0

3
S: 1/ B (0; 4) = 2 Oy .
2/
.

4
Bk : R = d (I; AB) =
C (3; 0) = 1 2
3

Bi 83.

Cao ng S Phm H Ni khi A nm 1999


Trong mt phng ta Oxy, cho ABC, cnh BC, cc ng cao BI, CK c phng trnh ln
lt l 7x + 5y 8 = 0, 9x 3y 4 = 0, x + y 2 = 0 . Vit phng trnh cc cnh AB,
AC v ng cao AH.
S: AB : x y = 0, AC : x + 3y 8 = 0, AH : 5x 7y + 4 = 0 .

Bi 84.

Cao ng Cng Nghip Tp. H Ch Minh nm 2000

Page - 110 -

DeThiThuDaiHoc.com
"All the flower of tomorrow are in the seeks of today"

cng hc tp mn Ton 10 Tp IIwww.MATHVN.com

Ths. L Vn on

Trong mt phng ta Oxy, cho ABC c cc ng cao (BH) : x + y 1 = 0 ,

(CK) : 3x + y + 1 = 0 v cnh (BC) : 5x y 5 = 0 . Vit phng trnh ca cc cnh cn


li ca tam gic v ng cao AL ?
S: AB : x + 3y 1 = 0, AC : x y + 3 = 0, AL : x + 5y 3 = 0 .
Bi 85.

Cao ng Kim St Pha Bc nm 2000


Trong mt phng ta Oxy, cho ABC c A (1; 3) v hai trung tuyn l x 2y + 1 = 0 v
y 1 = 0 . Vit phng trnh cc cnh ca tam gic ?

S: AB : x y + 2 = 0, AC : x + 2y 3 = 0, BC : x 4y + 1 = 0 .
Bi 86.

Cao ng S Phm Nh Tr Mu Gio TWI nm 2001


Trong mt phng ta Oxy, cho cc im A (1;2), B (1;2) v ng thng d c phng
trnh (d) : x 2y + 1 = 0 . Hy tm ta ca im C thuc ng thng d sao cho ba im A,
B, C to thnh tam gic v tha mn mt trong cc iu kin sau
1/ CA = CB .
2/ AB = AC .
1
1 2
S: 1/ C 0; .
2/ C (3;2) C ; .
2
5 5

Bi 87.

Cao ng S Phm Vnh Phc khi A nm 2002


Trong mt phng ta Oxy, cho ABC v im M (1;1) l trung im ca AB. Hai cnh
AC v BC theo th t nm trn hai ng thng 2x + y 2 = 0 v x + 3y 3 = 0 .
1/ Xc nh ta ba nh A, B, C ca ABC v vit phng trnh ng cao CH.
2/ Tnh din tch ABC.

3 4

S: 1/ A (1;0), B (3;2), C ; v CH : 10x 5y 2 = 0 .


5 5

Bi 88.

2/ SABC =

6
(vdt) .
5

Cao ng Nng Lm nm 2003


Trong mt phng vi h trc ta Oxy, cho hai ng thng x + y 1 = 0 v
3x y + 5 = 0 . Hy tm din tch hnh bnh hnh c hai cnh nm trn hai ng thng
cho, mt nh l giao im ca hai ng thng v giao im ca hai ng cho l I (3; 3) .
S: SABCD = 55 (vdt) .

Bi 89.

Cao ng S Phm Ph Th khi A nm 2003


Trong mt phng vi h trc ta cac Oxy cho tam gic ABC c nh A (2; 3),
B (3; 2) v din tch tam gic ABC bng
d : 3x y 8 = 0 . Tm ta im C.

3
. Bit trng tm G ca ABC thuc ng thng
2

S: C (1; 1) C (4; 8) .
Bi 90.

Cao ng khi D, M nm 2004 i hc Hng Vng


Trong mt phng vi h trc ta Descarter vung gc Oxy, cho ABC bit nh A (3;9) v
phng trnh cc ng trung tuyn BM, CN ln lt l 3x 4y + 9 = 0, y 6 = 0 . Vit
phng trnh ng trung tuyn AD ca tam gic cho.

"Cn c b thng minh"

DeThiThuDaiHoc.com

Page - 111 -

www.MATHVN.com

Ths. L Vn on

Phn hnh hc

S: AD : 3x + 2y 27 = 0 .
Bi 91.

Cao ng iu Dng chnh quy nm 2004 i hc iu dng


Trong mt phng vi h trc ta Descarter vung gc Oxy, cho ABC c nh A (0;1) v
hai ng thng cha cc ng cao v t B v C c phng trnh tng ng l
2x y 1 = 0 v x + 3y 1 = 0 . Tnh din tch ABC.
S: SABC = 14 (vdt) .

Bi 92.

Cao ng khi A nm 2004


Cho tam gic ABC c A (6; 3), B (4; 3), C (9;2) .
1/ Vit phng trnh cc cnh ca ABC.
2/ Vit phng trnh ng phn gic trong ca gc A ca tam gic ABC.
3/ Tm im M trn cnh AB v tm im N trn cnh AC sao cho MN // BC v AM = CN .

AB : 3x y + 15 = 0
32 9 33 4
S: 1/
AC : x 3y 3 = 0 . 2/ d A : y = x + 3 . 3/ M ; , N ; .

7 7 7 7
BC : x + 13y 35 = 0

Bi 93.

Cao ng S Phm Hi Phng nm 2004


Trong mt phng vi h trc ta Oxy, cho hai ng thng 1 : x y + 1 = 0,

2 : 2x + y 1 = 0 v im P (2;1) .

1/ Vit phng trnh ng thng i qua im P v giao im I ca hai ng thng 1 v 2.


2/ Vit phng trnh ng thng i qua im P v ct hai ng thng 1, 2 ln lt ti hai
im A, B sao cho P l trung im AB.
2/ d AB : 4x y 7 = 0 (c th gii theo 3 cch).
S: 1/ y 1 = 0 .
Bi 94.

Cao ng S Phm Kom Tum nm 2004


Trong mt phng vi h trc ta Descarter vung gc Oxy, cho hai im A (1;2) v

B (3; 4) . Tm im C trn ng thng d : x 2y + 1 = 0 sao cho ABC vung C.

3 4

S: C (3;2) C ; .
5 5

Bi 95.

Cao ng Kinh T K Thut Cng Nghip I khi B nm 2004


Trong mt phng vi h trc ta Oxy cho ng thng d : 2x + 3y + 1 = 0 v im
M (1;1) . Vit phng trnh ca cc ng thng i qua im M v to vi ng thng d mt

gc 45 0 .
S: x 5y + 4 = 0 . C th gii theo hai cch.
Bi 96.

Cao ng Kinh T K Thut Cng Nghip I khi A nm 2004


Trong mt phng vi h trc ta Descarter vung gc Oxy, cho hai im A (3; 1) v

B (3;5) . Hy vit phng trnh ng thng i qua im I (2; 3) v cch u hai im A, B.


S: x + 2 = 0 x + 5y 13 = 0 .
Bi 97.

Cao ng Mu Gio TW 1 nm 2004

Page - 112 -

DeThiThuDaiHoc.com
"All the flower of tomorrow are in the seeks of today"

cng hc tp mn Ton 10 Tp IIwww.MATHVN.com

Ths. L Vn on

Trong mt phng vi h trc ta Decac Oxy, xt ABC vi AB : x 2y + 7 = 0 , cc


ng trung tuyn k t A, B ln lt c phng trnh x + y 5 = 0 v 2x + y 11 = 0 .
Hy tnh din tch ca ABC v lp phng trnh hai ng thng AC v BC.
S: SABC =
Bi 98.

45
(vdt) v AC : 16x + 13y 68 = 0,
2

BC : 17x + 11y 106 = 0 .

Cao ng khi T M trng i hc Hng Vng nm 2004


Trong mt phng vi h trc ta Descarter vung gc Oxy, cho ABC bit nh A (3;9) v
phng trnh cc ng trung tuyn BM, CN ln lt l : 3x 4y + 9 = 0 v y 6 = 0 . Vit
phng trnh ng trung tuyn AD.
S: AD : 3x + 2y 27 = 0 .

Bi 99.

Cao ng Cng Nghip IV nm 2004


Trong mt phng vi h trc ta Descarter vung gc Oxy, cho ABC vung ti A vi

B (3; 0), C (7; 0), bn knh ng trn ni tip r = 2 10 5 . Tm ta tm I ca ng


trn ni tip ABC, bit im I c honh dng.

S: I 2 + 10; 2 10 5

I 2 10; 2 10 5 .

Bi 100. Cao ng Ti Chnh K Ton nm 2004


Trong mt phng vi h trc ta Oxy, cho ba im A (2;1), B (2; 3), C (4;5) . Hy vit
phng trnh cc ng thng cch u ba im A, B, C.

MN : x 3y + 6 = 0

S: L cc ng trung bnh ABC


NP : x + 2y 9 = 0 .

MP : 2x y + 2 = 0

Bi 101. Cao ng khi A, B nm 2005


Mt hnh thoi c: mt ng cho phng trnh l x + 2y 7 = 0 , mt cnh c phng trnh
l x + 3y 3 = 0 , mt nh l (0;1) . Tm phng trnh cc cnh ca hnh thoi.
Bi 102. Cao ng S Phm KomTum nm 2005
5
Trong mt phng vi h trc ta Oxy cho im M ;2 v hai ng thng
2

( ) : x 2y = 0 , ( ) : 2x y = 0 . Lp phng trnh ng thng d qua M ct ( ), ( )


1

ln lt ti A, B sao cho M l trung im ca on AB.


Bi 103. Cao ng S Phm Vnh Long khi A, B nm 2005
Trong mt phng vi h trc ta Oxy, cho ABC c A (1; 3) v hai ng trung tuyn xut
pht t B v C ln lt c phng trnh: x 2y + 1 = 0 v y 1 = 0 . Hy lp phng trnh
cc cnh ca ABC.
S: AB : x y + 2 = 0, BC : x 4y + 1 = 0, CA : x + 2y 7 = 0 .
Bi 104. Cao ng S Phm H Ni nm 2005

"Cn c b thng minh"

DeThiThuDaiHoc.com

Page - 113 -

Ths. L Vn on

www.MATHVN.com

Phn hnh hc

Trong mt phng vi h trc ta Oxy, cho ABC c im A (1;2) , ng trung tuyn BM


v ng phn gic trong CD tng ng c phng trnh 2x + y + 1 = 0 , x + y 1 = 0 . Hy
vit phng trnh ng thng BC.
S: BC : 4x + 3y + 4 = 0 .
Bi 105. Cao ng Bn Tre nm 2005
Trong mt phng vi h trc ta Oxy, vit phng trnh cc cnh ca ABC bit nh
A (4; 1), phng trnh mt ng cao v mt ng trung tuyn v cng mt nh ln lt l

d1 : 2x 3y + 12 = 0 v d2 : 2x + 3y = 0 .
S: AB : 3x + 7y 5 = 0, AC : 3x + 2y 10 = 0, BC : 9x + 11y + 5 = 0 .
Bi 106. Cao ng Kinh T K Thut Cn Th nm 2005
Trong mt phng vi h trc ta Oxy, cho ABC c nh A (1; 3), phng trnh ng cao
BH : 2x 3y 10 = 0 v phng trnh ng thng BC : 5x 3y 34 = 0 . Xc nh ta
cc nh B v C.

S: B (8;2), C (5; 3) .
Bi 107. Cao ng S Phm H Nam khi H nm 2005
Trong mt phng vi h trc ta Descarter vung gc Oxy, cho A (1;2), B (5; 4) v
 
ng thng : x + 3y 2 = 0 . Tm im M trn ng thng sao cho MA + MB
ngn nht.
5 3
S: M ; .
2 2
Bi 108. Cao ng S Phm Qung Ninh khi A nm 2005
Trong mt phng vi h trc ta Descarter vung gc Oxy, cho ABC c im A (2; 1)
v hai ng phn gic trong ca hai gc B, C ln lt c phng trnh
(B ) : x 2y + 1 = 0, (C ) : x + y + 3 = 0 . Vit phng trnh cnh BC.
S: BC : 4x y + 3 = 0 .
Bi 109. Cao ng S Phm in Bin khi A, B nm 2005
Trong mt phng vi h trc ta Descarter vung gc Oxy, cho ABC vung A. Bit ta
A (3;5), B (7;1) v ng thng BC i qua im M (2; 0) . Tm ta nh C.
S: C (3; 1) .
Bi 110. Cao ng S Phm C Mau khi A nm 2005
Trong mt phng vi h trc ta Descarter vung gc Oxy, cho hai im A (1;1), B (2;1)
v ng thng d : x 2y + 2 = 0 .
1/ Chng t rng hai im A, B v cng mt pha ca d.
2/ Tm ta im M thuc d sao cho tng khong cch (MA + MB) b nht.

Page - 114 -

DeThiThuDaiHoc.com
"All the flower of tomorrow are in the seeks of today"

cng hc tp mn Ton 10 Tp IIwww.MATHVN.com

Ths. L Vn on

23 16
S: M ; .
15 13

Bi 111. Cao ng Truyn Hnh khi A nm 2005


 = 900 . Bit
Trong mt phng vi h trc ta Oxy, cho ABC c AB = AC, BAC
2
M (1; 1) l trung im cnh BC v G ; 0 l trng tm ca ABC. Tm ta nh A, B, C.
3

Bi 112. Cao ng Cng ng Vnh Long khi A, B nm 2005


Trong mt phng vi h trc ta Oxy, cho ABC c nh A (3; 0) v phng trnh hai
ng cao (BB ') : 2x + 2y 9 = 0 v (CC ') : 3x 12y 1 = 0 . Vit phng trnh cc cnh
ca tam gic ABC.
Bi 113. Cao ng S Phm H Ni khi D1, T nm 2005
Trong mt phng vi h trc ta Descarter vung gc Oxy, cho ABC c A (2; 4),

B (0;2) v im C thuc ng thng: 3x y + 1 = 0, din tch ABC bng 1 (n v din


tch). Hy tm ta im C.
1 1
S: C ; C (1; 2) .
2 2
Bi 114. Cao ng Kinh T K Thut Cng Nghip I khi A nm 2006
Trong mt phng vi h trc ta Descarter vung gc Oxy, cho ba im A (1;2), B (3;1),

C (4; 3) . Chng minh rng ABC l tam gic cn. Vit phng trnh cc ng cao ca tam
gic .
S: AH : x + 2y 5 = 0, BI : 3x + y 10 = 0, CK : 2x y 5 = 0 .
Bi 115. Cao ng Xy Dng s 2 khi A nm 2006
Trong mt phng vi h trc ta Descarter vung gc Oxy, cho mt tam gic c mt nh l
A (4; 3), mt ng cao v mt ng trung tuyn i qua hai nh khc nhau c phng trnh
ln lt l 3x y + 11 = 0 v x + y 1 = 0 . Hy vit phng trnh cc cnh tam gic.
S: AC : x + 3y 13 = 0, AB : x 2y + 2 = 0, BC : 7x + y + 29 = 0 .
Bi 116. Cao ng Giao Thng Vn Ti III Tp. H Ch Minh khi A nm 2006
Trong mt phng vi h trc ta Descarter vung gc Oxy, cho hnh thoi ABCD c phng
trnh hai cnh v mt ng cho l AB : 7x 11y + 83 = 0, CD : 7x 11y 53 = 0,
BD : 5x 3y + 1 = 0 . Tm ta B v D. Vit phng trnh ng cho AC, ri suy ra ta
ca A v C.
S: AC : 3x + 5y 13 = 0 A (4;5), C (6; 1) .
Bi 117. Cao ng Bn Cng Hoa Sen khi A nm 2006
Trong mt phng vi h trc ta Descarter vung gc Oxy, cho hai ng thng c phng
trnh: d1 : 2x 3y + 1 = 0, d2 : 4x + y 5 = 0 . Gi A l giao im ca d1 v d2. Tm im
B trn d1 v im C trn d2 sao cho ABC c trng tm l im G (3;5) .

"Cn c b thng minh"

DeThiThuDaiHoc.com

Page - 115 -

www.MATHVN.com

Ths. L Vn on

Phn hnh hc

61 43 5 55
; , C ; .
7 7 7 7

S: A (1;1), B

Bi 118. Cao ng Kinh T K Thut Cn Th khi A nm 2006


Trong mt phng vi h trc ta Descarter vung gc Oxy, cho ABC vi A (2;1),

B (4; 3) v C (m; 2) . nh m ABC vung ti C.


S: m = 1 m = 5 .
Bi 119. Cao ng in Lc Tp. H Ch Minh nm 2006
Trong mt phng vi h trc ta Descarter vung gc Oxy, cho ng thng d c phng

trnh x + y 3 = 0 v hai im A (1;1), B (3; 4 ) . Tm ta im M thuc ng thng d


sao cho khong cch t M n ng thng AB bng 1.
S: M (0; 3) M (10; 7 ) .

Bi 120. Cao ng Kinh T Cng Ngh Tp. H Ch Minh khi D1 nm 2006


Trong mt phng vi h trc ta Descarter vung gc Oxy, cho ABC vung cn ti
A (4;1) v cnh huyn BC c phng trnh: 3x y + 5 = 0 . Vit phng trnh hai cnh gc
vung AC v AB.
S: AC : x 2y 2 = 0 v AB : 2x + y 9 = 0 .
Bi 121. Cao ng S Phm Bnh Phc nm 2006
Trong mt phng vi h trc ta Descarter vung gc Oxy, cho A (1;1), B (4; 3) . Tm
im C thuc ng thng x + 2y + 1 = 0 sao cho khong cch t im C n ng thng
AB bng 6.
43 27
S: C ; C (7; 3) .
11 11
Bi 122. Cao ng S Phm Tr Vinh khi M nm 2006
Trong mt phng vi h trc ta Descarter vung gc Oxy, cho ABC bit C (2; 4),
trong tm G (0; 4 ) v M (2; 0) l trung im cnh BC. Hy vit phng trnh ng thng
cha cnh AB.
S: AB : 4x + 5y 44 = 0 .
Bi 123. Cao ng K Thut Cao Thng nm 2006
Trong mt phng vi h trc ta Oxy, cho ng thng d : 3x 4y + 1 = 0 . Hy vit
phng trnh ng thng song song vi d v c khong cch n d bng 1.
S: 1 : 3x 4y 4 = 0 2 : 3x 4y + 6 = 0 .
Bi 124. Cao ng Kinh T Tp. H Ch Minh nm 2007
Trong mt phng vi h trc ta Descarter vung gc Oxy, cho hai ng thng
d1 : x + y + 1 = 0, d2 : 2x y 1 = 0 v im M (2; 4) . Vit phng trnh ng thng
i qua I v ct d1, d2 ln lt ti A v B m I l trung im ca AB.
S: AB : x + 4y 14 = 0 .

Page - 116 -

DeThiThuDaiHoc.com
"All the flower of tomorrow are in the seeks of today"

cng hc tp mn Ton 10 Tp IIwww.MATHVN.com

Ths. L Vn on

Bi 125. Cao ng Kinh T i Ngoi nm 2007


Trong mt phng ta Oxy, cho ABC. Bit im B (4; 1), ng cao AH c phng trnh
l : 2x 3y + 12 = 0, ng trung tuyn AM c phng trnh : 2x + 3y = 0 . Vit phng
trnh cc ng thng cha cc cnh ca tam gic ABC.
S: A (3;2), B (4;1), C (8; 7) .
Bi 126. Cao ng Xy Dng s 2 nm 2007
Vit phng trnh cc cnh ca ABC bit nh A (1;1), ng trung tuyn v ng cao i
qua nh B ln lt c phng trnh: 3x + 4y 27 = 0, 2x + y 8 = 0 .
S: AB : x = 1, AC : x 2y + 1 = 0, BC : x + 8y 49 = 0 .
Bi 127. Cao ng Cng Nghip Thc Phm nm 2007
Trong mt phng vi h trc ta Descarter vung gc Oxy, cho ABC c nh A (2; 7 ),
trung tuyn CM, ng cao BK c phng trnh ln lt l x + 2y + 7 = 0 v
3x + y + 11 = 0 . Vit phng trnh cc ng thng AC v BC.
S: AC : x 3y 23 = 0 v BC : 7x + 9y + 19 = 0 .
Bi 128. Cao ng khi A, B, D nm 2008
Trong mt phng vi h trc ta Oxy, tm im A thuc trc honh v im B thuc trc
tung sao cho A v B i xng vi nhau qua ng thng d : x 2y + 3 = 0 .
S: A (2; 0), B (0; 4) .
Bi 129. Cao ng A, B, D nm 2011 (Chng Trnh C Bn)
Trong mt phng vi h to Oxy, cho ng thng d : x + y + 3 = 0 . Vit phng trnh
ng thng i qua im A (2; 4) v to vi ng thng d mt gc bng 45 0 .
S: 1 : y + 4 = 0 2 : x 2 = 0 .
Bi 130. Cao ng A, B, D nm 2011 (Chng Trnh Nng Cao)
Trong mt phng vi h to Oxy, cho tam gic ABC c phng trnh cc cnh l
AB : x + 3y 7 = 0, BC : 4x + 5y 7 = 0, CA : 3x + 2y 7 = 0 . Vit phng trnh
ng cao k t nh A ca tam gic ABC.
S: AH : 5x 4y + 3 = 0 .
Bi 131. i hc S PhmKinh tTi ChnhNng Nghip Tp. H Ch Minh nm 1977
Trong mt phng ta vung gc Oxy, hy vit phng trnh ng thng i qua giao im
ca hai ng thng : 3x 5y + 2 = 0, 5x 2y + 4 = 0 v song song vi ng thng
2x y + 4 = 0 .
S: d : 38x 19y + 30 = 0 .
Bi 132. i hc Th Dc Th Thao Tp. H Ch Minh nm 1977
Trong mt phng ta vung gc Oxy, hy lp phng trnh ng phn gic ca gc t to
bi hai ng thng 1 : 3x 4y + 12 = 0, 2 : 12x + 3y 7 = 0 .

) (

S: d : 60 9 17 x + 15 12 17 y 35 + 36 17 = 0 .
Bi 133. i hc Tng Hp Tp. H Ch Minh khi B nm 1978
"Cn c b thng minh"

DeThiThuDaiHoc.com

Page - 117 -

www.MATHVN.com

Ths. L Vn on

Phn hnh hc

Trong mt phng ta vung gc Oxy, cho hai ng thng d1 v d2 ln lt c phng trnh:


d1 : x + y = 1, d2 : x 3y + 3 = 0 . Hy vit phng trnh ng thng d i xng vi d2
qua ng thng d1.
S: d : 3x y + 1 = 0 .
Bi 134. i hc Th Dc Th Thao Tp. H Ch Minh nm 1978
Trong mt phng ta vung gc Oxy, cho tam gic cn PRQ, bit phng trnh cnh y
PQ : 2x 3y + 5 = 0, cnh bn PR : x + y + 1 = 0 . Tm phng trnh cnh bn RQ bit
rng n i qua im D (1;1) .
S: RQ : 17x + 7y 24 = 0 .
Bi 135. i hc Bch Khoa i hc Tng Hp Tp. H Ch Minh nm 1979
Trong mt phng ta vung gc Oxy, cho ng cong (C) : y = x 2 + 9 v ng thng
d : ax 5y 32 = 0 .

1/ V ng cong cho.
2/ Tnh khong cch z t mt im M ty ca ng cong n ng thng d theo honh
x ca M.
3/ Tnh khong cch ngn nht gia ng cong v ng thng.
x 2 y2
S: 1/ V (H) : 2 2 = 1 trn Ox. 2/ z =
3
3

4x 5 x2 + 9 32
41

16
. 3/ M 4; .
5

Bi 136. i hc Y Nha Dc Tp. H Ch Minh nm 1980


Trong mt phng ta vung gc Oxy, hy vit phng trnh ng thng i qua im
A (1;2) m khong cch t im M (2; 3) v im N (4; 5) n ng thng y bng nhau.
S: d : 3x + 2y 7 = 0 d : 4x + y 6 = 0 .
Bi 137. Hc Vin Ngn Hng Tp. H Ch Minh nm 1991
Trong mt phng ta Descartes vung gc, cho ABC c nh A (2;2) . Lp phng trnh
cc cnh ca ABC. Bit rng cc ng thng 9x 3y 4 = 0 v x + y 2 = 0 ln lt l
cc ng cao ca tam gic xut pht t B v C.
S: AC : x + 3y 8 = 0, AB : x y = 0, BC : 7x + 5y 8 = 0 .
Bi 138. i hc Cn Th 1993 i hc Hng Hi 1995 Trung Tm o To Cn B Y T Tp.
H Ch Minh nm 1997 Hc Vin Hng Khng 2001
Lp phng trnh cc cnh ca tam gic ABC nu bit B (2; 1), ng cao qua A v ng
phn gic trong gc C c phng trnh ln lt l 3x 4y + 27 = 0; x + 2y 5 = 0 .
S: AB : 4x + 7y 1 = 0, BC : 4x + 3y 5 = 0, AC : y = 3 .
Li bnh
Phng trnh ng thng x + 2y 5 = 0 l phng trnh ng phn gic ngoi ca gc C,
khng phi l phng trnh ng phn gic trong gc C. ra thiu chnh xc. Mt s trng
i hc ra ny tuyn sinh m khng pht hin ra, y, ti i li ng phn
gic ngoi gc C l x + 2y 5 = 0 v gii ra kt qu nh trn.
Bi 139. Trung Tm o To Cn B Y T Tp. H Ch Minh nm 1993

Page - 118 -

DeThiThuDaiHoc.com
"All the flower of tomorrow are in the seeks of today"

cng hc tp mn Ton 10 Tp IIwww.MATHVN.com

Ths. L Vn on

Trong mt phng ta vung gc Oxy, cho hai im P (2;5) v Q (5;1) . Lp phng trnh
ng thng qua P cch Q mt on c di bng 3 .
S: d : x 2 = 0 d : 7x + 24y 134 = 0 .
Bi 140. i hc Php L Tp. H Ch Minh nm 1994
Trong mt phng vi h trc ta Descarter vung gc Oxy, cho ba ng thng:
d1 : 3x + 4y 6 = 0, d2 : 4x + 3y 1 = 0, d3 : y = 0 . Gi A = d1 d2 , B = d2 d3 ,

C = d3 d1 .
1/ Vit phng trnh ng phn gic trong ca gc A ca ABC v tnh din tch ABC.
2/ Vit phng trnh ng trn ni tip ABC.
S: 1/ dA : x + y 1 = 0 v SABC =

21
(vdt) .
4

73 9
1 73

73 1
2/ x
+ y
=
.
8
8

8

2

Bi 141. i hc Tng Hp Tp. H Ch Minh khi A, B nm 1994


Trong mt phng ta vung gc Oxy, cho hai ng thng d1 v d2 ln lt c phng trnh:
d1 : kx y + k = 0, d2 : 1 k2 x + 2ky 1 + k2 = 0 .

1/ Chng minh rng khi k thay i, ng thng d1 lun i qua mt im c nh.


2/ Vi mi gi tr k, hy xc nh giao im ca d1 v d2.
3/ Tm qu tch ca giao im khi k thay i.
1 k2 2k2
3/ ng trn: x2 + y2 = 1 loi M o (1; 0) .
;
S: 1/ M o (1; 0) . 2/ M
2
2
1 + k 1 + k
Bi 142. i hc Bch Khoa H Ni nm 1994
Phng trnh hai cnh mt tam gic trong mt phng ta l: 5x 2y + 6 = 0;
4x + 7y 21 = 0 . Vit phng trnh cnh th ba ca tam gic, bit trc tm H trng vi gc
ta .
S: BC : y + 7 = 0 .
Bi 143. i hc M a Cht nm 1995
Lp phng trnh cc cnh ABC nu bit A (1; 3) v hai ng trung tuyn c phng trnh
l x 2y + 1 = 0 v y 1 = 0 .
S: AB : x + 2y 7 = 0, AC : x y + 2 = 0, BC : x 4y 1 = 0 .
Bi 144. i hc Quc Gia H Ni nm 1995
Trn mt phng ta trc chun cho cc im P (2; 3), Q (4; 1), R (3; 5) l cc trung
im ca cc cnh ca mt tam gic. Hy lp phng trnh ca cc ng thng cha cc cnh
ca tam gic .
S: BC : 6x 7y 3 = 0, AB : 2x + y + 1 = 0, AC : 2x + 5y 3 = 0 .
Bi 145. i hc S Phm H Ni 2 Khi A v i hc S Phm Quy Nhn nm 1995
Lp phng trnh cc cnh ca ABC trong mt phng vi h trc ta trc chun Oxy, nu
cho C (4; 5) v hai ng cao c phng trnh 5x + 3y 4 = 0 v 3x + 8y + 13 = 0 .
"Cn c b thng minh"

DeThiThuDaiHoc.com

Page - 119 -

www.MATHVN.com

Ths. L Vn on

Phn hnh hc

S: BC : 3x 5y 13 = 0, AC : 8x 3y + 17 = 0, AB : 5x + 2y 1 = 0 .
Bi 146. i hc Vn Ha H Ni nm 1995
Lp phng trnh cc cnh ca hnh vung bit rng hnh vung c nh l (4; 8) v mt
ng cho c phng trnh : 7x y + 8 = 0 .
S: AB : 3x 4y + 32 = 0, AD : 4x + 3y + 1 = 0, BC : 4x + 3y 24 = 0,
CD : 3x 4y + 7 = 0 .
Bi 147. i hc Y Khoa H Ni nm 1995
Trong mt phng ta vung gc Oxy, cho hai ng thng d1 v d2 ln lt c phng trnh:
d1 : (a b) x + y = 1 v d2 : a 2 b2 x + ay = b vi a 2 + b2 > 0 . Xc nh giao im ca

d1 v d2, bin lun theo a, b s giao im y.


b = 0 (a b) d // d
1 a
1
2
S: TH1.
. TH2. a 0 b 0 d1 d2 = A ; .
b b
b = a d1 d2
Bi 148. i hc Cn Th nm 1995
Trong mt phng ta vung gc Oxy, cho A (2; 3), B (3; 2) . Trng tm G ca ABC
nm trn ng thng d : 3x y 8 = 0, din tch ABC bng

3
. Tm ta im C.
2

S: C (1; 1) C (2; 10) .


Bi 149. i hc Ti Chnh H Ni nm 1996
Trong mt phng ta vung gc Oxy, cho ABC c M (2;2) l trung im ca BC, cnh
AB c phng trnh: x 2y 2 = 0, cnh AC c phng trnh: 2x + 5y + 3 = 0 . Xc nh
ta cc nh ca ABC.
4 7
40 11
76 25
S: A ; , B ; , C ; .
9
9 9
9 9
9
Bi 150. i hc Vn Lang t 1 khi B, D nm 1997
Trong mt phng ta Oxy, cho ng thng d c phng trnh 3x + 4y 12 = 0 .
1/ Xc nh ta qua cc giao im A, B ca d ln lt vi trc Ox, Oy.
2/ Tnh ta hnh chiu H ca gc O trn ng thng d.
3/ Vit phng trnh ng thng d' i xng vi d qua O.
36 48
S: 1/ A (4; 0), B (0; 3) . 2/ H ; . 3/ d ' : 3x + 4y + 12 = 0 .
25 25
Bi 151. i hc An Ninh 2 khi D nm 1997
Trong mt phng ta cho im A (0;2) v im B (m; 2) . Hy vit phng trnh ng
thng trung trc d ca AB. Chng minh rng d lun tip xc vi ng cong (C) c nh khi
m thay i.
S: d : y =

m
m2
1
x
, lun tip xc vi parabol (P) : y = x2 .
4
8
8

Bi 152. i hc Hu khi D nm 1997


Page - 120 -

DeThiThuDaiHoc.com
"All the flower of tomorrow are in the seeks of today"

cng hc tp mn Ton 10 Tp IIwww.MATHVN.com

Ths. L Vn on

Trong mt phng ta Oxy, cho hai ng thng 1 : 4x 3y 12 = 0,

2 : 4x + 3y 12 = 0 .
1/ Tm ta cc nh ca tam gic c ba cnh ln lt nm trn cc ng thng 1, 2 v
trc tung.
2/ Xc nh tm v bn knh ng trn ni tip ca tam gic ni trn.
4
4
2/ E ; 0, r = .
S: 1/ A (0; 4), B (0; 4), C (3; 0) .
3
3
Bi 153. i hc S Phm H Ni 2 nm 1997
Trong mt phng ta Oxy, cho A (2;1), B (0;1), C (3;5), D (3; 1) .
1/ Tnh din tch t gic ABCD.
2/ Vit phng trnh cc cnh ca hnh vung c hai cnh song song i qua A v C v hai cnh
cn li i qua B v D.

MN : 7x + y 15 = 0
MN : x 3y + 1 = 0
PQ : 7x + y 26 = 0
PQ : x 3y + 12 = 0
S: 1/ SABCD = 6 (vdt) . 2/
.

NP : x 7y + 7 = 0
NP : 3x + y 1 = 0

MQ : x 7y 4 = 0
MQ : 3x + y + 10 = 0
Bi 154. i hc Y Dc Tp. H Ch Minh h C nhn nm 1997
Cho ABC, cnh BC c trung im M (0; 4), cn hai cnh kia c phng trnh l
2x + y 11 = 0 v x + 4y 2 = 0 .

1/ Xc nh nh A.
2/ Gi C l nh nm trn ng thng x 4y 2 = 0 v N l trung im AC. Tm ta im
N ri tnh ta B, C.
S: 1/ A (6; 1) .

2/ C (2;1), B (2;7) .

Bi 155. i hc Quc Gia Tp. H Ch Minh khi A i hc Lut nm 1997


Trong mt phng ta vung gc Oxy, cho ng thng d : 2x + y 4 = 0 v hai im
M (3; 3), N (5; 19) . H MK d v gi P l im i xng ca M qua d.

1/ Tm ta im K v P.
2/ Tm im A trn d sao cho AM + AN c gi tr nh nht v tnh gi tr .
S: 1/ K (1;2), P (1;1) .

2/ A (3;10) (AM + AN)

min

= 2 85 .

Bi 156. i hc Lt nm 1998
Trong mt phng ta vung gc Oxy, cho hai ng thng d1 : (a + 1) x 2y a 1 = 0
v d2 : x + (a 1) y a 2 = 0 .
1/ Tm giao im I ca d1 v d2.
2/ Tm a ng thng qua M (0;a ), N (a; 0) cng i qua im I.
2
2

3a 1 (a + 1) (a 1)

S: 1/ I 2
;
.
a + 1

a2 + 1

"Cn c b thng minh"

1
2/ a = 1 a = .
2

DeThiThuDaiHoc.com

Page - 121 -

www.MATHVN.com

Ths. L Vn on

Phn hnh hc

Bi 157. i hc K Thut Cng Ngh Tp. H Ch Minh khi B, D nm 1998


Trong mt phng ta Oxy, cho ABC c A (2;2) , bit tam gic c hai ng cao l:
9x 3y 4 = 0 v x + y 2 = 0 .

1/ Vit phng trnh cc ng trung trc ca tam gic ABC.


2/ Vit phng trnh ng trn ngoi tip ABC v xc nh ta trng tm ABC.
d : 3x + 3y 8 = 0
AB
S: 1/ dAC : 9x 3y + 3 = 0 .

dBC : 15x 21y + 41 = 0

2
2

5 17
5
9
185

2/ (C) : x + y =
v G ; .

9 9
12
4
72

Bi 158. i hc Giao Thng Vn Ti Tp. H Ch Minh 1 nm 1998


Trong mt phng ta Oxy, cho hai im A (1;2) v B (3; 4) . Tm ta im C trn
ng thng: x 2y + 1 = 0 sao cho ABC vung C.
3 4
S: C (3;2) C ; .
5 5

Bi 159. i hc Nng khi A nm 1998


Trong mt phng ta vung gc Oxy, cho im P (3; 0) v hai ng thng:

d1 : 2x y 2 = 0 v d2 : x + y + 3 = 0 . Gi d l ng thng qua P v ct d1, d2 ln lt


A v B. Vit phng trnh ca d bit rng PA = PB .
S: d : 4x 5y 12 = 0 d : 8x y 24 = 0 .
Bi 160. i hc Vn Lang khi B, D nm 1998
Trong mt phng ta vung gc Oxy, cho ABC c nh B (3;5), ng cao k t A c
phng trnh: 2x 5y + 3 = 0 v ng trung tuyn k t nh C c phng trnh:
x + y 5 = 0 . Tm ta nh A v vit phng trnh cc cnh ca tam gic.
S: A (1;1), BC : 5x + 2y 25 = 0, AB : 2x y 1 = 0, AC : x + 4y 5 = 0 .
Bi 161. i hc Quc Gia Tp. H Ch Minh t 3 nm 1998
Trong mt phng ta vung gc Oxy, cho ABC c trng tm G (2; 1) v cc cnh
AB : 4x + y + 15 = 0 v AC : 2x + 5y + 3 = 0 .

1/ Tm ta nh A v ta trung im M ca BC.
2/ Tm ta nh B v vit phng trnh ng thng BC.
S: 1/ A (4;1), M (1;2) .

2/ B (3; 3), BC : x 2y 3 = 0 .

Bi 162. i hc Hng Hi nm 1998


Trong mt phng ta vung gc Oxy, cho A (1;1), B (1; 3) v ng thng d c phng
trnh d : x + y + 4 = 0 .

1/ Tm trn d im C cch u hai im A, B.


2/ Vi C va tm c, tm D sao cho ABCD l hnh bnh hnh. Tnh din tch hnh bnh hnh.
S: 1/ C (3; 1) .

2/ D (1; 3) v SABCD = 12 (vdt) .

Bi 163. i hc Cn Th nm 1998
Page - 122 -

DeThiThuDaiHoc.com
"All the flower of tomorrow are in the seeks of today"

cng hc tp mn Ton 10 Tp IIwww.MATHVN.com

Ths. L Vn on

Trong mt phng ta vung gc Oxy, cho ABC c nh A (1; 3) .


1/ Bit ng cao BH : 5x + 3y 25 = 0, ng cao CK : 3x + 8y 12 = 0 . Tm ta
nh B, C.
2/ Bit ng trung trc ca AB l : 3x + 2y 4 = 0 v trong tm G (4; 2) . Tm ta
nh B, C.
S: 1/ B (2; 5), C (4; 0) .

2/ B (5;1), C (8; 4) .

Bi 164. i hc Vn Ha H Ni nm 1998
Trong mt phng ta vung gc Oxy, cho ABC bit nh C (4; 1) v ng cao, ng
trung tuyn k t mt nh c phng trnh ln lt l 2x 3x + 12 = 0 v 2x + 3y = 0 .
Tm phng trnh cc cnh ca tam gic ABC.
S: AC : 3x + 7y 5 = 0, BC : 3x + 2y 10 = 0, AB : 9x + 11y + 5 = 0 .
Bi 165. i hc Hu khi D nm 1998
Trong mt phng ta vung gc Oxy, hy vit phng trnh ng thng song song vi
d : 3x 4y + 1 = 0 v c khong cch n ng ng thng d bng 1 .
S: : 3x 4y 4 = 0 : 3x 4y + 6 = 0 .
Bi 166. i hc Kin Trc H Ni nm 1998
Trong mt phng ta vung gc Oxy, cho ba im A (2; 4), B (3;1), C (1; 4) v ng thng
d c phng trnh: d : x y 1 = 0 .

1/ Tm M d sao cho AM + MB nh nht.


2/ Tm N d sao cho AN + CN nh nht.
11 7
S: 1/ M ; (AM + BM) = 10 .
min
4 4

23 16
2/ N ; (AN + CN) = 5 .
min
7 7

Bi 167. i hc Dn Lp K Thut Cng Ngh khi D nm 1999


Trong mt phng ta vung gc Oxy, cho im M (2; 3) . Tm phng trnh ng thng d
qua M v cch u hai im A (1; 0), B (2;1) .
S: d : x 3y + 11 = 0 d : x + y 1 = 0 .
Bi 168. i hc Cn Th khi A nm 1999
Trong mt phng ta vung gc Oxy, cho ba im A (3; 4), B (5; 1), C (4; 3) .
1/ Tnh di AB, BC, AC . Hy cho bit tnh cht (nhn, t, vung) ca cc gc trong
ABC.
2/ Tnh di ng cao AH ca ABC v vit phng trnh ng thng AH.
S: 1/ AB = 29, AC = 50, BC = 97 nhn.
2/ AH =

37

, AH : 9x + 4y + 11 = 0 .

97
Bi 169. i hc M Thut Cng Nghip khi A nm 1999

"Cn c b thng minh"

DeThiThuDaiHoc.com

Page - 123 -

www.MATHVN.com

Ths. L Vn on

Phn hnh hc

Trong mt phng ta vung gc Oxy, cho hai ng thng c phng trnh ln lt l:


d1 : x y 1 = 0, d2 : 3x y + 1 = 0 v im M (1;2) . Vit phng trnh ng thng d i
qua im M, ct d1 v d2 ln lt ti M1, M2 v tha mt trong cc iu kin sau:
1/ MM1 = MM2 .
2/ MM1 = 2MM2 .
S: 1/ d MM2 : x 1 = 0 .

2/ d MM2 : x + y 3 = 0 .

Bi 170. i hc Dc H Ni nm 1999
Trong mt phng ta vung gc Oxy, cho hai ng thng d1 : (a b) x + y = 1 v

d2 : a 2 b2 x + ay = b vi b2 = 4a 2 + 1 .

1/ Xc nh giao im ca d1 v d2.
2/ Tm tp hp (E) cc giao im ca d1 v d2 khi a, b thay i.
1 a
S: 1/ M ; .
b b

x2
y2
= 1.
2/ Ellipse (E) : 2 +
1
1 2

2

Bi 171. i hc Nng khi A i hc Kinh T Tp. H Ch Minh nm 1999


Trong mt phng ta vung gc Oxy, cho hai ng thng d1 : 2x y 2 = 0 v

d2 : 2x + 4y 7 = 0 .
1/ Vit phng trnh ng phn gic ca gc to bi d1 v d2.
2/ Vit phng trnh ng thng qua im P (3;1) cng vi d1, d2 to thnh tam gic cn c
nh l giao ca d1 v d2.
d : 2x 6y + 3 = 0
S: 1/
.
d : 6x + 2y 11 = 0

: 3x + y 10 = 0
2/
.
: x 3y = 0

Bi 172. i hc S Phm H Ni 2 nm 1999


Trong mt phng ta vung gc Oxy, cho tam gic ABC vi cc nh A (6; 3),
B (4; 3) v C (9;2) .

1/ Vit phng trnh ng thng d cha ng phn gic trong ca gc A ca ABC.


2/ Tm im P trn ng thng d sao cho t gic ABCP l hnh thang.
S: 1/ d : x y + 3 = 0 .

2/ P (14;17) P (2;5) .

Bi 173. i hc Kinh T Quc Dn H Ni nm 1999


Trong mt phng ta vung gc Oxy, hy vit phng trnh ng thng i qua im
A (0;1) v to vi ng thng x + 2y + 3 = 0 mt gc bng 45 0 .
S: d : 3x + y 1 = 0 d : x 3y + 1 = 0 .
Bi 174. i hc Hng Hi nm 1999
Cho ABC c A (2; 1) v phng trnh cc ng cao l 2x y + 1 = 0, 3x + y + 2 = 0 .
Lp phng trnh ng trung tuyn ca tam gic qua nh A.
Page - 124 -

DeThiThuDaiHoc.com
"All the flower of tomorrow are in the seeks of today"

cng hc tp mn Ton 10 Tp IIwww.MATHVN.com

Ths. L Vn on

S: AM : x + 32y + 30 = 0 .
Bi 175. i hc M Bn Cng Tp. H Ch Minh khi A, B nm 2000
Trong mt phng ta vung gc Oxy, cho im M (1;6) v d : 2x 3y + 3 = 0 .
1/ Vit phng trnh ng thng d1 i qua M v song song vi d.
2/ Vit phng trnh ng thng d2 i qua M, vung gc vi d v xc nh ta hnh chiu
vung gc ca M ln ng thng d.
S: 1/ d1 : 2x 3y + 16 = 0 .

2/ d2 : 3x + 2y 15 = 0 v H (3; 3) .

Bi 176. i hc Ty Nguyn khi D nm 2000


Trong mt phng ta vung gc Oxy, hy lp phng trnh tng qut ca ng thng i
qua im I (2; 3) v cch u hai im A (5; 1) v B (3;7) .
S: d : 4x + y + 5 = 0 d : y 3 = 0 .
Bi 177. i hc S Phm H Ni 2 khi A nm 2000
Trong mt phng vi h trc ta Descarter vung gc Oxy, cho ABC v nh A (1;1) . Cc
ng cao h t B v C ln lt nm trn cc ng thng d1 v d2 theo th t c phng trnh
2x + y 8 = 0 v 2x + 3y 6 = 0 . Hy vit phng trnh ng thng cha ng cao h
t nh A v xc nh ta nh B, C ca ABC.
9 7
S: I ; = d1 d2 , AI : 10x + 13y 23 = 0, B (17; 26), C (3; 0) .
4 2
Bi 178. i hc Thng Mi nm 2000
Trong mt phng ta vung gc Oxy, cho ABC c A (2; 1) v phng trnh hai ng
phn gic trong ca gc B v gc C ln lt c phng trnh: dB : x 2y + 1 = 0 v

dC : x + y + 3 = 0 . Tm phng trnh ca ng thng cha cnh BC.


S: BC : 4x y + 3 = 0 .
Bi 179. Hc Vin Ngn Hng Tp. H Ch Minh nm 2000
Trong mt phng vi h trc ta Descarter vung gc Oxy, cho ABC c nh C (2; 4)
v trng tm G (0; 4) .
1/ Gi s M (2; 0) l trung im ca cnh BC. Xc nh ta cc nh A v B.
2/ Gi s M di ng trn ng thng (D) : x + y 2 = 0, tm qu tch im B. Hy xc nh
M di cnh AB l ngn nht.



MA = 3MG A (4;12)

S: 1/ 
.
CB = 2CM
B (6; 4)

1 9
2/ Qu tch l d : x + y 10 = 0 v M ; .
4 4

Bi 180. i hc Giao Thng Vn Ti khi A nm 2001


Trong mt phng vi h trc ta Descarter vung gc Oxy, cho hnh bnh hnh ABCD c s
o din tch bng 4. Bit ta cc nh A (1; 0), B (2; 0) v giao im I ca hai ng cho
AC v BD nm trn ng thng y = x . Hy tm ta cc nh C v D.

S: C (3; 4), D (2; 4) C (5; 4), D (6; 4) .


"Cn c b thng minh"

DeThiThuDaiHoc.com

Page - 125 -

www.MATHVN.com

Ths. L Vn on

Phn hnh hc

Bi 181. Hc Vin K Thut Qun S nm 2001


Trong mt phng vi h trc ta Oxy, cho ABC cn, cnh y BC c phng trnh
x + 3y + 1 = 0 . Cnh bn AB c phng trnh x y + 5 = 0 . ng thng cha cnh AC i
qua im M (4;1) . Tm ta nh C.
S: Ba cnh ABC ng quy ti M V l Bi ton khng xc nh C tha yu
cu bi ton.
Bi 182. i hc Nng Nghip I nm 2001
Trong mt phng Oxy cho im A (1;1) v ng thng d c phng trnh: 4x + 3y = 12 .
1/ Gi B v C ln lt l giao im ca d vi cc trc Ox v Oy. Xc nh ta trc tm ca
ABC.
2/ im M chy trn ng thng d. Trn na ng thng i qua hai im A v M, ly im
 
N sao cho AM.AN = 4 . im N chy trn ng cong no ? Vit phng trnh ng
cong .
S: 1/ H (3; 2) .

2
2

13
11
2/ N chy trn ng trn (C) : x + y = 4 .

5
5

Bi 183. i hc Hng Hi nm 2001


5
Trong mt phng vi h trc ta Oxy, cho im M ;2 v hai ng thng c phng
2

x
; y 2x = 0 . Lp phng trnh ng thng d qua M v ct hai ng thng ni
2
trn ti hai im A v B sao cho M l trung im ca AB.
S: y = 2 .
trnh y =

Bi 184. i hc Hu khi A,B,V nm 2001


Vit phng trnh ba cnh ca ABC trong mt phng ta Oxy, cho bit nh C (4; 3),
ng phn gic trong v ng trung tuyn k t mt nh ca tam gic c phng trnh ln
lt l x + 2y 5 = 0 v 4x + 13y 10 = 0 .
S: AC : x + y 7 = 0, AB : x + 7y + 5 = 0, BC : x 8y + 20 = 0 .
Bi 185. i hc Kinh T Quc Dn nm 2001
Trong mt phng vi h trc ta trc chun Oxy, hy lp phng trnh cc cnh ca ABC
nu cho B (4;5) v hai ng cao h t hai nh cn li ca tam gic ln lt c phng
trnh: 5x + 3y 4 = 0 v 3x + 8y + 13 = 0 .
S: BC : 3x 5y + 37 = 0, AB : 8x 3y 47 = 0, AC : 2535x 3016y + 29033 = 0 .
Bi 186. i hc khi A nm 2002
Trong mt phng vi h ta cc vung gc Oxy , xt tam gic ABC vung ti A, phng
trnh ng thng BC l 3x y 3 = 0 , cc nh A v B thuc trc honh v bn knh ng
trn ni tip bng 2. Tm ta trng tm G ca tam gic ABC.

7 + 4 3 6 + 3

, G 4 3 1 ; 6 2 3 .
;
S: G1

3
3 2
3
3

Bi 187. i hc khi B nm 2002


Page - 126 -

DeThiThuDaiHoc.com
"All the flower of tomorrow are in the seeks of today"

cng hc tp mn Ton 10 Tp IIwww.MATHVN.com

Ths. L Vn on

Trong mt phng vi h ta cc vung gc Oxy , cho hnh ch nht ABCD c tm


1
I ; 0 , phng trnh ng thng AB l x 2y + 2 = 0 v AB = 2AD . Tm ta cc
2
nh A, B, C, D bit rng nh A c honh m.
S: A (2; 0), B (2;2), C (3; 0), D (1; 2) .
Bi 188. i hc khi B nm 2003
Trong mt phng vi h ta cc vung gc Oxy , cho tam gic ABC c AB = AC,
 = 900 . Bit M 1; 1 l trung im cnh BC v G 2 ; 0 l trng tm tam gic ABC.
BAC

( )

3
Tm ta cc nh A, B, C.
S: A (0;2), B (4; 0), C (2 2) .
Bi 189. D b 1 i hc khi D nm 2003
Trong mt phng vi h ta Oxy , cho tam gic ABC c nh A (1; 0) v hai ng thng
ln lt cha cc ng cao v t B v C c phng trnh tng ng l: x 2y + 1 = 0,
3x + y 1 = 0 . Tnh din tch tam gic ABC.

S: B (5; 2), C (1; 4) SABC = 14 .


Bi 190. i hc khi A nm 2004

Trong mt phng vi h ta Oxy , cho hai im A (0;2) v B 3; 1 . Tm ta trc tm


v ta tm ng trn ngoi tip ca tam gic OAB.

S: H

) (

3; 1 , I 3;1 .

Bi 191. i hc khi B nm 2004


Trong mt phng vi h ta Oxy , cho hai im A (1;1), B (4; 3) . Tm im C thuc ng
thng x 2y 1 = 0 sao cho khong cch t C n ng thng AB bng 6.

43 27
S: C1 (7; 3), C2 ; .
11 11
Bi 192. i hc khi D nm 2004
Trong mt phng vi h ta Oxy , cho ABC c cc nh A (1; 0), B (4; 0), C (0; m) vi

m 0 . Tm ta trng tm G ca ABC theo m. Xc nh m tam gic GAB vung ti G.


m
S: G 1; , m = 3 6 .
3
Bi 193. D b 2 i hc khi A nm 2004
Trong mt phng vi h ta Oxy , cho im A (0;2) v ng thng d : x 2y + 2 = 0 . Tm
trn d hai im B, C sao cho tam gic ABC vung B v AB = 2BC .
2 6
S: B ; .
5 5

Bi 194. D b 1 i hc khi B nm 2004


"Cn c b thng minh"

DeThiThuDaiHoc.com

Page - 127 -

Ths. L Vn on

www.MATHVN.com

Phn hnh hc

Trong mt phng vi h ta Oxy , cho im I (2; 0) v hai ng thng d1 : 2x y + 5 = 0,

d2 : x + y 3 = 0 . Vit phng trnh ng thng d i qua im I v ct hai ng thng d1, d2




ln lt ti A, B sao cho IA = 2IB .
S: d : 7x 3y + 6 = 0 .
Bi 195. D b 1 i hc khi D nm 2004
Trong mt phng vi h ta Oxy , cho tam gic ABC vung A. Bit A (1; 4), B (4; 1),
7
ng thng BC i qua im K ;2 . Tm to nh C.
3
69 1579
.
S: C ;
50
250

Bi 196. D b 2 i hc khi D nm 2004


Trong mt phng vi h ta Oxy, cho im A (2; 3) v hai ng thng d1 : x + y + 5 = 0,

d2 : x + 2y 7 = 0 . Tm to cc im B trn d1 v C trn d2 sao cho tam gic ABC c trng

tm G (2; 0) .

Bi 197. i hc khi A nm 2005


Trong mt phng vi h ta Oxy, cho hai ng thng d1 : x y = 0 v d2 : 2x + y 1 = 0 .
Tm to cc nh hnh vung ABCD bit rng nh A thuc d1, nh C thuc d2 v cc nh B, D
thuc trc honh.
S: A (1;1), B (0, 0),C (1; 1), D (2; 0) hoc A (1;1), B (2, 0),C (1; 1), D (0; 0) .
Bi 198. D b 1 i hc khi A nm 2005

4 1
Trong mt phng vi h ta Oxy, cho tam gic ABC cn ti nh A c trng tm G ; ,
3 3
phng trnh ng thng BC l x 2y 4 = 0 v phng trnh ng thng BG l
7x 4y 8 = 0 .Tm ta cc nh A, B, C.
S: A (0; 3), B (0; 2), C (4; 0) .
Bi 199. i hc khi A nm 2006
Trong mt phng vi h ta Oxy, cho cc ng thng ln lt c phng trnh:
d1 : x + y + 3 = 0, d2 : x y 4 = 0, d3 : x 2y = 0 . Tm to im M nm trn ng
thng d3 sao cho khong cch t M n ng thng d1 bng hai ln khong cch t M n
ng thng d2.
S: M (22; 11) M (2;1) .
Bi 200. D b 2 i hc khi A nm 2006
Trong mt phng vi h ta Oxy, cho tam gic ABC c nh A thuc ng thng
d : x 4y 2 = 0 , cnh BC song song vi d. Phng trnh ng cao (BH) : x + y + 3 = 0
v trung im ca cnh AC l M (1;1) . Tm to cc nh A, B, C.

2 2
8 8
S: A ; , B (4;1), C ; .
3 3
3 3
Page - 128 -

DeThiThuDaiHoc.com
"All the flower of tomorrow are in the seeks of today"

cng hc tp mn Ton 10 Tp IIwww.MATHVN.com

Ths. L Vn on

Bi 201. D b 1 i hc khi B nm 2006


Trong mt phng vi h ta Oxy, cho tam gic ABC cn ti B, vi A (1; 1), B (3;5) . im
B nm trn ng thng d : 2x y = 0 . Vit phng trnh cc ng thng AB, BC.

S: (AB) : 23x y 24 = 0 v (BC) : 19x 13y + 8 = 0 .


Bi 202. D b 2 i hc khi B nm 2006
Trong mt phng vi h ta Oxy, cho tam gic ABC c nh A (2;1) , ng cao qua nh B
c phng trnh x 3y 7 = 0 v ng trung tuyn qua nh C c phng trnh
x + y + 1 = 0 . Xc nh to cc nh B v C ca tam gic.
S: B (2; 3), C (4; 5) .
Bi 203. i hc khi B nm 2007
Trong mt phng vi h ta Oxy, cho im A (2;2) v cc ng thng:

d1 : x + y 2 = 0, d2 : x + y 8 = 0 . Tm to cc im B v C ln lt thuc d1 v d2
sao cho tam gic ABC vung cn ti A.
S: B (1; 3), C (3;5) B (3; 1), C (5; 3) .
Bi 204. D b 2 i hc khi A nm 2007
Trong mt phng vi h ta Oxy, cho tam gic ABC c trng tm G (2; 0), phng trnh
cc cnh AB : 4x + y + 14 = 0, AC : 2x + 5y 2 = 0 . Tm to cc nh A, B, C.
S: A (4;2), B (3; 2), C (1; 0) .
Bi 205. D b 1 i hc khi D nm 2007
Trong mt phng vi h ta Oxy, cho im A (2;1) . Trn trc Ox, ly im B c honh

x B 0 , trn trc Oy, ly im C c tung yC 0 sao cho tam gic ABC vung ti A. Tm
cc im B, C sao cho din tch tam gic ABC ln nht.
S: B (0; 0), C (0;5) .
Bi 206. D b 2 i hc khi A nm 2007
Trong mt phng vi h ta Oxy, cho cc im A (0;1), B (2; 1) v cc ng thng
d1 : (m 1) x + (m 2) y + 2 m = 0, d2 : (2 m) x + (m 1) y + 3m 5 = 0 . Chng
minh d1 v d2 lun ct nhau. Gi P l giao im ca d1 v d2. Tm m sao cho PA + PB ln nht.

S: Ch : (PA + PB) 2 PA2 + PB2 = 2AB2 = 16 . Do (PA + PB)


2

max

= 4 khi P

l trung im ca cung AB. Khi P (2;1) hay P (0; 1) m = 1 m = 2 .

Bi 207. i hc khi B nm 2008


Trong mt phng vi h ta Oxy, hy xc nh to nh C ca tam gic ABC bit rng
hnh chiu vung gc ca C trn ng thng AB l im H (1; 1), ng phn gic trong
gc A c phng trnh x y + 2 = 0 v ng cao k t B c phng trnh 4x + 3y 1 = 0 .

10 3
S: C ; .
3 4
"Cn c b thng minh"

DeThiThuDaiHoc.com

Page - 129 -

www.MATHVN.com

Ths. L Vn on

Phn hnh hc

Bi 208. i hc khi A nm 2009 (Chng trnh c bn)


Trong mt phng vi h ta Oxy, cho hnh ch nht ABCD c im I (6;2) l giao im ca
hai ng cho AC v BD. im M (1;5) thuc ng thng AB v trung im E ca cnh CD
thuc ng thng : x + y 5 = 0 . Vit phng trnh ng thng AB.

y 5 = 0
.
S: AB :
x 4y + 19 = 0
Bi 209. i hc khi B nm 2009 (Chng trnh nng cao)
Trong mt phng vi h ta Oxy, cho tam gic ABC cn ti A c nh A (1; 4) v cc nh
B, C thuc ng thng : x y 4 = 0 . Xc nh to cc im B v C, bit din tch
tam gic ABC bng 18.
11 3 3 5
3 5 11 3
S: B ; , C ; B ; , C ; .
2 2 2 2
2 2 2 2
Bi 210. i hc khi D nm 2009 (Chng trnh c bn)
Trong mt phng vi h ta Oxy, cho tam gic ABC c M (2; 0) l trung im ca cnh AB.
ng trung tuyn v ng cao qua nh A ln lt c phng trnh l 7x 2y 3 = 0,
6x y 4 = 0 . Vit phng trnh ng thng AC.
S: (AC) : 3x 4y + 5 = 0 .
Bi 211. i hc khi A nm 2010 (Chng trnh nng cao)
Trong mt phng ta Oxy, cho tam gic ABC cn ti A c nh A (6; 6) ; ng thng i qua
trung im ca cc cnh AB v AC c phng trnh x + y 4 = 0 . Tm to cc nh B v
C, bit im E (1; 3) nm trn ng cao i qua nh C ca tam gic cho.
S: B (0; 4),C (4; 0) B (6;2), C (2; 6) .
Bi 212. i hc khi B nm 2010 (Chng trnh c bn)
Trong mt phng ta Oxy, cho tam gic ABC vung ti A, c nh C (4;1), phn gic
trong gc A c phng trnh x + y 5 = 0 . Vit phng trnh ng thng BC, bit din tch
tam gic ABC bng 24 v nh A c honh dng.
S: BC : 3x 4y + 16 = 0 .
Bi 213. i hc khi D nm 2010 (Chng trnh nng cao)
Trong mt phng ta Oxy, cho im A (0;2) v l ng thng i qua O. Gi H l hnh
chiu vung gc ca A trn . Vit phng trnh ng thng , bit khong cch t H n
trc honh bng AH.
S: 12 :

5 1 x 2

5 2.y = 0 .

Bi 214. i hc khi B nm 2011 (Chng trnh c bn)


Trong mt phng to Oxy, cho hai ng thng : x y 4 = 0 v d : 2x y 2 = 0 .
Tm ta im N thuc ng thng d sao cho ng thng ON ct ng thng ti im
M tha mn OM.ON = 8 .
Page - 130 -

DeThiThuDaiHoc.com
"All the flower of tomorrow are in the seeks of today"

cng hc tp mn Ton 10 Tp IIwww.MATHVN.com

Ths. L Vn on

6 2
S: N (0; 2) N ; .
5 5

Bi 215. i hc khi D nm 2011 (Chng trnh c bn)


Trong mt phng ta Oxy, cho tam gic ABC c nh B (4;1), trng tm G (1;1) v
ng thng cha phn gic trong ca gc A c phng trnh x y 1 = 0 . Tm ta cc
nh A v C.
S: A (4; 3), C (3; 1) .
Bi 216. i hc khi A nm 2012 (Chng trnh c bn)
Trong mt phng vi h ta Oxy, cho hnh vung ABCD. Gi M l trung im ca cnh
11 1
BC, N l im trn cnh CD sao cho CN = 2ND . Gi s M ; v ng thng AN c
2 2
phng trnh 2x y 3 = 0 . Tm ta im A.
S: A (1; 1) A (4; 5) .

"Cn c b thng minh"

DeThiThuDaiHoc.com

Page - 131 -

www.MATHVN.com

Ths. L Vn on

Phn hnh hc

C PHNG TRNH NG TRN



 Xc nh tm v bn knh ng trn
Nu phng trnh ng trn c dng

th

c tm l

v bn knh bng R.
th tm I c xc

Nu phng trnh ng trn c dng


nh

v bn knh

 Lu
Nu

l phng trnh ng trn nu tha mn iu

kin:
.
iu kin ng thng tip xc vi ng thng l

 Lp phng trnh ng trn


ta thng cn phi xc nh tm

lp phng trnh ng trn


. Khi phng trnh ng trn
a/ Dng 1.

c tm
Tm

v tip xc vi ng thng

R
I

c ng knh AB
I

Bn knh

i qua hai im A, B v c tm I nm trn ng thng

Bn knh
Page - 132 -

Vit phng trnh ng trung trc d ca on AB

tho mn:

i qua hai im A, B v tip xc vi ng thng

Tm I ca

Vit phng trnh ng trung trc d ca on AB.


Xc nh tm
.
Bn knh
.
e/ Dng 5.

Tm I l trung im AB.

d/ Dng 4.

Bn knh
c/ Dng 3.

c tm
Tm

v i qua im

Bn knh
b/ Dng 2.

v bn knh R ca

A
R I
B

DeThiThuDaiHoc.com
"All the flower of tomorrow are in the seeks of today"

cng hc tp mn Ton 10 Tp IIwww.MATHVN.com

f/ Dng 6.

i qua im A v tip xc vi ng thng ti im B

g/ Dng 7.

Ths. L Vn on

Vit phng trnh ng trung trc d ca on AB.


Vit phng trnh ng thng i qua B v
.
Xc nh tm
.
Bn knh
.

B
I

i qua im A v tip xc vi hai ng thng 1 v 2


Tm I ca

tho mn:

.
I

Bn knh
.
Lu
o Mun b du gi tr tuyt i trong

, ta xt du min mt phng nh bi 1

v 2 hay xt du khong cch i s t A n 1 v 2.


o Nu 1 // 2, ta tnh
h/ Dng 8.

, v

tip xc vi hai ng thng 1, 2 v c tm nm trn ng thng d


Tm I ca

tho mn:

Bn knh
i/ Dng 9.

c thay th bi

i qua ba im khng thng hng A, B, C (ng trn ngoi tip tam gic)
Cch 1
Phng trnh ca

c dng:

Ln lt thay to ca A, B, C vo

ta c h

Gii h phng trnh ny ta tm c

phng trnh

phng trnh ca

Cch 2
Tm I ca

tho mn:

Bn knh
j/ Dng 10.

B
I

ni tip tam gic ABC

B
Vit phng trnh ca hai ng phn gic trong ca hai gc A
trong tam gic.
Xc nh tm I l giao im ca hai ng phn gic trn.
Bn knh
.
A
F
I
B
"Cn c b thng minh"

DeThiThuDaiHoc.com

C
Page - 133 -

www.MATHVN.com

Ths. L Vn on

Phn hnh hc

 Tp hp cc tm ng trn (qu tch tm I ca ng trn)


tm tp hp cc tm I ca ng trn

ta c th lm theo cc bc sau

Bc 1. Tm gi tr ca m tn ti tm I.
Bc 2. Tm to tm I. Gi s: I

Bc 3. Kh m gia x v y ta c phng trnh

Bc 4. Da vo iu kin ca m bc 1 gii hn min ca x hoc y.


Bc 5. Phng trnh tp hp im l
cng vi phn gii hn bc 4.
 Lu : tm tp hp im l ng trn, ta cng thc hin tng t nh cc bc trn.
 V tr tng i ca ng thng d v ng trn (C)
bin lun s giao im ca ng thng

v ng trn

ta c th thc hin nh sau


 Phng php 1. So snh khong cch t tm I n
Xc nh tm I v bn knh R ca
Tnh khong cch t I n
+
ct

vi bn knh R.

ti hai im phn bit.

tip xc vi

khng c im chung.

 Phng php 2. To giao im (nu c) ca

+ H

c 2 nghim

ct

+ H

c 1 nghim

tip xc vi

+ H

v nghim

V tr tng i ca hai ng trn

l nghim ca h phng trnh:

ti hai im phn bit.


.

khng c im chung.
v

bin lun s giao im ca hai ng trn

, ta c th thc hin theo hai phng php


 Phng php 1. So snh di on ni tm I1I2 vi cc bn knh R1, R2.
ct

ti hai im.

tip xc ngoi vi

tip xc trong vi

ngoi nhau.

trong nhau.

Page - 134 -

DeThiThuDaiHoc.com
"All the flower of tomorrow are in the seeks of today"

cng hc tp mn Ton 10 Tp IIwww.MATHVN.com

Ths. L Vn on

 Phng php 2. Phng php i s: To cc giao im (nu c) ca

nghim ca h phng trnh


H

c hai nghim

c mt nghim

v nghim

ct

ti 2 im.

tip xc vi
v

khng c im chung.

Tip tuyn ca ng trn


Cho ng trn

c tm I, bn knh R v ng thng .

tip xc vi

a/ Dng 1. Tip tuyn ti mt im


Phng php 1. Tip tuyn

i qua

v c VTPT

Phng php 2. Phn i ta :


.

Phng trnh tip tuyn c dng :


b/ Dng 2. Tip tuyn c phng cho trc.
Bc 1. Vit phng trnh ca c phng cho trc :
Bc 2. Da vo iu kin tip xc
tm thnh phn cn li.
T suy ra phng trnh ca .
Lu : Cc dng phng cho trc thng gp l
Tip tuyn
Tip tuyn
Tip tuyn
Tip tuyn
Tip tuyn c h s gc bng k
Tip tuyn

.
Tip tuyn to vi ng thng

.
.

mt gc . Khi ta linh hot s

dng mt trong hai cng thc

hoc

c/ Dng 3. Tip tuyn v t mt im

ngoi ng trn

Vit phng trnh ca i qua A (cha 2 tham s).


Da vo iu kin:
ta tm c cc tham s. T suy ra phng trnh ca .

d/ Dng 4. Lp phng trnh tip tuyn chung ca hai ng trn


Bc 1. Gi s tip tuyn l
Bc 2. Theo iu kin tip xc ca

"Cn c b thng minh"

vi
vi

DeThiThuDaiHoc.com

Page - 135 -

www.MATHVN.com

Ths. L Vn on

Bc 3. Kt lun v tip tuyn chung ca


e/ H tip tuyn ca ng trn
Cho ng trn

Phn hnh hc

.
.

c phng trnh :

Tip tuyn vi

ti

c dng :
.

. Do c th t :

Khi , mi tip tuyn

ca

Ta gi cc tip tuyn
ca

vi

c dng :

vi tham s t l h tip tuyn ca

. Ta tip im

BA
NG
BAI T
TP AAP DU
DUNG
XC NH TM V BN KNH NG TRN
Bi 217. Trong cc phng trnh sau, phng trnh no l phng trnh ng trn. Tm tm v bn
knh ca ng trn
1/ x2 + y2 2x 2y 2 = 0 .

2/ x2 + y2 6x + 4y 12 = 0 .

3/ x2 + y2 + 2x 8y + 1 = 0 .

4/ x2 + y2 6x + 5 = 0 .

5/ 16x2 + 16y2 + 16x 8y = 11 .

6/ 7x2 + 7y2 4x + 6y 1 = 0 .

7/ 2x2 + 2y2 4x + 12y + 11 = 0 .

8/ 4x2 + 4y2 + 4x 5y + 10 = 0 .

Bi 218. Tm m cc phng trnh sau l phng trnh ng trn


1/ x2 + y2 + 4mx 2my + 2m + 3 = 0 .
2/ x2 + y2 2 (m + 1) x + 2my + 3m2 2 = 0 .
3/ x2 + y2 2 (m 3) x + 4my m2 + 5m + 4 = 0 .

4/ x2 + y2 2mx 2 m 2 1 y + m 4 2m 4 2m 2 4m + 1 = 0 .
5/ x2 + y2 6x + 2y ln m + 3 ln m + 7 = 0
6/ x2 + y2 2x + 4y + ln (m 2) + 4 = 0
7/ x2 + y2 2e2m x + 2em y + 6e2m 4 = 0
Page - 136 -

( ) .
( ) .
( ) .

DeThiThuDaiHoc.com
"All the flower of tomorrow are in the seeks of today"

cng hc tp mn Ton 10 Tp IIwww.MATHVN.com

8/ x 2 + y2 2x cos m + 4y + cos2 m 2 sin m + 5 = 0


9/ x2 + y2 4x cos m + 2y sin m 4 = 0

Ths. L Vn on

( ) .

( ) .

LP PHNG TRNH NG TRN


Bi 219. Vit phng trnh ng trn c tm I v i qua im A, vi
1/ I (2; 4), A ( 1; 3) .

2/ I ( 3; 2), A (1; 1) .

3/ I ( 1; 0), A (3; 11) .

4/ I (1; 2), A (5; 2) .

5/ I (3; 5), A (7; 2) .

6/

I (0; 0), A (4; 4) .

Bi 220. Vit phng trnh ng trn c tm I v tip xc vi ng thng , vi


1/ I (3; 4), : 4x 3y + 15 = 0 .

2/ I (2; 3), : 5x 12y 7 = 0 .

3/ I (3;2), Ox .

4/ I (3; 5), Oy .

5/ I (1;2), : x 2y + 7 = 0 .

6/

I (0; 0), : y 2x = 0 .

Bi 221. Vit phng trnh ng trn c ng knh AB, vi


1/ A ( 2; 3), B (6; 5) .

2/ A (0; 1), C (5; 1) .

3/ A ( 3; 4), B (7; 2) .

4/

A (5; 2), B (3; 6) .

5/ A (1; 1), B (7; 5) .

6/

A (1; 5), B (1; 1) .

Bi 222. Vit phng trnh ng trn i qua hai im A, B v c tm I nm trn ng thng , vi


1/ A (2;3), B (1;1), : x 3y 11 = 0 .

2/ A (0; 4), B (2;6), : x 2y + 5 = 0 .

3/ A (2;2), B (8;6), : 5x 3y + 6 = 0 .

4/ A (1; 0), B (1;2), : x y 1 = 0 .

5/ A (1;2), B (3; 0), : 7x + y 6 = 0 .

6/ A (0; 0), B (1;2), : x y = 0 .

Bi 223. Vit phng trnh ng trn i qua hai im A, B v tip xc vi ng thng , vi


1/ A (1;2), B (3;4), : 3x + y 3 = 0 .

2/ A (6;3), B (3;2), : x + 2y 2 = 0 .

3/ A (1; 2), B (2;1), : 2x y + 2 = 0 .

4/ A (2; 0), B (4;2), Oy .

Bi 224. Vit phng trnh ng trn i qua im A v tip xc vi ng thng ti im B, vi


1/ A (2;6), : 3x 4y = 15, B (1; 3) .

2/ A (2;1), : 3x 2y = 6, B (4; 3) .

3/ A (6; 2), Ox, B (6; 0) .

4/ A (4; 3), : x + 2y 3 = 0, B (3; 0) .

Bi 225. Vit phng trnh ng trn i qua im A v tip xc vi hai ng thng 1 v 2, vi


1/ A (2; 3),

1 : 3x 4y + 1 = 0,

2 : 4x + 3y 7 = 0 .

2/ A (1; 3),

1 : x + 2y + 2 = 0,

2 : 2x y + 9 = 0 .

3/ A O (0; 0),

1 : x + y 4 = 0,

2 : x + y + 4 = 0 .

"Cn c b thng minh"

DeThiThuDaiHoc.com

Page - 137 -

Ths. L Vn on

4/ A (3; 6),

www.MATHVN.com
1 Ox,

Phn hnh hc

2 Oy .

Bi 226. Vit phng trnh ng trn tip xc vi hai ng thng 1, 2 v c tm nm trn ng


thng d, vi
1/ 1 : 3x + 2y + 3 = 0,

2 : 2x 3y + 15 = 0,

d : x y = 0.

2/ 1 : x + y + 4 = 0,

2 : 7x y + 4 = 0,

d : 4x + 3y 2 = 0 .

3/ 1 : 4x 3y 16 = 0,

2 : 3x + 4y + 3 = 0,

d : 2x y + 3 = 0 .

4/ 1 : 4x + y 2 = 0,

2 : x + 4y + 17 = 0,

d : x y + 5 = 0.

Bi 227. Vit phng trnh ng trn ngoi tip tam gic ABC, vi
1/ A (2; 0), B (0; 3), C (5; 3) .

2/ A (5; 3), B (6; 2), C (3; 1) .

3/ A (1; 2), B (3; 1), C ( 3; 1) .

4/ A ( 1; 7), B ( 4; 3), C O (0; 0) .

5/ AB : x y + 2 = 0,

BC : 2x + 3y 1 = 0,

CA : 4x + y 17 = 0 .

6/ AB : x + 2y 5 = 0,

BC : 2x + y 7 = 0,

CA : x y + 1 = 0 .

Bi 228. Vit phng trnh ng trn ni tip tam gic ABC, vi


1/ A (2; 6), B ( 3; 4), C (5; 0) .

2/ A (2; 0), B (0; 3), C (5; 3) .

3/ AB : 2x 3y + 21 = 0, BC : 3x 2y 6 = 0, CA : 2x + 3y + 9 = 0 .
4/ AB : 7x y + 11 = 0, BC : x + y 15, CA : 7x + 17y + 65 = 0 .
Bi 229. Lp phng trnh ng trn (C) trong cc trng hp sau

(C) i qua ba im: O (0; 0), M (1;2), N (2; 4) .


2/ (C) i qua ba im: A (1;2), B (5;2), C (1; 3) .
3/ (C) i qua ba im: A (4;5), B (3; 2), C (1; 4) .
4/ (C) i qua ba im: A (1; 3), B (5; 6), C (7; 0) .
5/ (C) i qua ba im: A (2;1), B (2; 5), C (2;1) .
6/ (C) i qua ba im: A (2; 4), B (5;5), C (6; 2) .
7/ (C) c tm I (2; 5) v tip xc vi Ox .
8/ (C) c tm I (1; 3) v tip xc vi Oy .
9/ (C) qua A (9;9) v tip vi trc Ox ti im M (6; 0) .
10/ (C) tip xc vi trc Ox ti im A (2; 0) v khong cch t tm ca (C) n B (6; 4)
1/

bng 5.
11/ (C) tip xc vi c hai trc ta v qua M (2;1) .

12/ (C) tip xc vi c hai trc ta v c tm nm trn ng thng d : 4x 2y 8 = 0 .


13/ (C) qua hai im A (2; 3), B (2;1) v c tm nm trn trc honh.
14/ (C) qua hai im A (2; 0), B (3;1) v bn knh R = 5 .
15/ (C) qua hai im A (1;1), B (0;2) v c tm nm trn ng thng d : 2x + 3y = 0 .

Page - 138 -

DeThiThuDaiHoc.com
"All the flower of tomorrow are in the seeks of today"

cng hc tp mn Ton 10 Tp IIwww.MATHVN.com

Ths. L Vn on

16/ (C) tip xc vi ng thng d1 : x 2y + 3 = 0 ti M (1;2) v c tm thuc ng


thng d2 : x 5y 5 = 0 .

17/ (C) tip xc vi ng thng d1 : 3x 4y 31 = 0 ti im M (1; 7) v c bn knh


R = 5.
18/ (C) tip xc vi ng trn (C ') : x 2 + y2 2x = 0 ti im A (2; 0) v R = 5 .

19/ (C) qua A (5; 3) v tip xc vi ng thng d : x + 3y + 2 = 0 ti M (1; 1) .


20/ (C) qua cc giao im ca (C1 ) : x2 + y2 9 = 0 v (C2 ) : x 2 + y2 2x 2y = 14 v
c tm nm trn ng thng d : x + 6y 6 = 0 .

21/ (C) i xng vi ng trn (C ') : (x 1) + (y 2) = 4 qua d : x y 1 = 0 .


2

22/ (C) i xng vi ng trn (C ') : (x 2) + (y 3) = 3 qua d : x + y 1 = 0 .


2

23/ (C) i xng vi ng trn (C ') : x2 + y2 2x 4y + 3 = 0 qua d : x 2 = 0 .

TP HP IM (Qui tch tm ng trn)


Bi 230. Tm tp hp cc tm I ca ng trn (C) c phng trnh (m v t l tham s)
1/ x2 + y2 2 (m 1) x 4my + 3m + 11 = 0 .
2/ x 2 + y2 2mx 4 (m + 1) y + 3m + 14 = 0 .
3/ x2 + y2 2mx 2m2 y + 2 = 0 .
4/ x 2 + y2 + mx m (m + 2) y 2m 2 4 = 0 .
5/ x 2 + y2 2 (cos 2t + 4) x 2y sin 2t + 6 cos 2t 3 = 0
6/ x 2 + y2 4x sin t + 4 (cos 2t sin t) y 2 cos2 t = 0

7/ x 2 + y2 2 2 e t x + 4 e2t 1 y e t 3 = 0
8/

(t

)(

( ) .
( ) .

( ) .

+ 1 x2 + y2 + 8 t2 1 x 4 t2 + 4t + 1 y 3t2 3 = 0

( ) .

Bi 231. Tm tp hp cc tm I ca ng trn (C) , bit


1/
2/
3/
4/
5/

(C) tip xc vi ng thng d : 6x 8y + 15 = 0 v c bn knh R = 3 .


(C) tip xc vi hai ng thng d : x + 2y 3 = 0, d : x + 2y + 6 = 0 .
(C) tip xc vi hai ng thng d : 2x + 3y 6 = 0, d : 3x 2y + 9 = 0 .
(C) tip xc vi ng trn (C ') : x + y 4x + 6y 3 = 0 v c bn knh R = 2 .
(C) i qua im A (2; 3) v tip xc vi ng thng d : y 5 = 0 .
1

Bi 232. Cho (Cm ) : x 2 + y2 2mx 4 (m 2) y + 6 m = 0 .


1/ Tm iu kin ca m (Cm ) l ng trn.
2/ Tm tp hp tm ca h ng trn (Cm ) khi m thay i.

"Cn c b thng minh"

DeThiThuDaiHoc.com

Page - 139 -

www.MATHVN.com

Ths. L Vn on

Phn hnh hc

Bi 233. Cho (Cm ) : x 2 + y2 + (m + 2) x (m + 4) y + 1 = 0 .


1/ Chng minh (Cm ) l h ng trn.
2/ Chng minh rng trong cc ng trn ca (Cm ) c mt ng trn qua gc ta . Tm
phng trnh ng trn .
Bi 234. Cho (Cm ) : x2 + y2 2 (m + 1) x + 4my + 17 = 0 .

1/ Tm iu kin m (Cm ) l ng trn.


2/ Vi gi tr no ca m th ng trn (Cm ) tip xc vi ng thng d : x + y 1 = 0 .
3/ Tm tp hp tm ca h (Cm ) khi m thay i.
Bi 235. Cho (Cm ) : x2 + y2 4mx 2 (m + 1) y 1 = 0 .
1/ Tm iu kin m (Cm ) l ng trn.
2/ Tm im c nh ca h (Cm ) khi m thay i.
3/ Tm tp hp tm ca h (Cm ) khi m thay i.
Bi 236. Cho (Cm ) : x2 + y2 + (m + 3) x (m + 2) y + m 13 = 0 .
1/ Chng minh (Cm ) l h ng trn.
2/ Chng minh rng khi m thay i th (Cm ) lun i qua hai im c nh.
Bi 237. Cho (Cm ) : x2 + y2 + (m + 2) x (m + 4) y + m + 1 = 0 .
1/ nh m (Cm ) l ng trn c bn knh nh nht.
2/ Tm tp hp tm ca h ng trn (Cm ) .
3/ Chng minh rng khi m thay i th (Cm ) lun i qua hai im c nh.

Bi 238. Cho (Cm ) : x2 + y2 2mx 4 (2m 1) y + 5 = 0 .


1/ Tm iu kin ca m (Cm ) l ng trn.
2/ Tm m ng trn (Cm ) ct trc honh ti hai im A, B sao cho AB = 4 .
Bi 239. Cho (Cm ) : x2 + y2 2mx + 2my + m2 2m + 3 = 0 .
1/ nh m (Cm ) l ng trn. Khi tm ta tm v bn knh.
2/ nh m (Cm ) tip xc vi hai trc ta .
3/ nh m (Cm ) ct trc Ox ti hai im A, B sao cho AB = 2 .
Bi 240. Cho hai im A (2; 4), B ( 6; 2) . Tm tp hp cc im M (x; y) sao cho
2/ MA = 3MB .

1/ AM 2 + BM 2 = 100 .
3/ AM + BM = k ,
2

(k > 0) .

4/ 2MA2 3MB2 = OM2 .

Bi 241. Cho hai im A (2; 3), B ( 2;1) . Tm tp hp cc im M (x; y) sao cho


 
 
1/ AM.BM = 0 .
2/ AM.BM = 4 .
Bi 242. Tm tp hp cc im M sao cho tng bnh phng cc khong cch t n hai ng thng
d v d bng k, vi

d : x y + 3 = 0, d ' : x + y = 1 = 0, k = 9 .
Page - 140 -

DeThiThuDaiHoc.com
"All the flower of tomorrow are in the seeks of today"

cng hc tp mn Ton 10 Tp IIwww.MATHVN.com

Ths. L Vn on

Bi 243. Cho bn im A (4; 4), B ( 6; 4), C ( 6; 2), D (4; 2) .


1/ Chng t rng ABCD l hnh ch nht.
2/ Tm tp hp cc im M sao cho tng bnh phng cc khong cch t M n cc cnh ca
hnh ch nht bng 100 .

V TR TNG I CA NG THNG V NG TRN


Bi 244. Xt v tr tng i ca ng thng d v ng trn (C) trong cc trng hp
1/ d : x + y 2 = 0

&

(C) : x

+ y2 2x + 6y 6 = 0 .

x = t
2/ d :

y = 5 2t

&

(C) : x

+ y2 4x + 2y 20 = 0 .

Bi 245. Cho ng trn (C) : x2 + y2 4x + 6y + 3 = 0 v ng thng : 3x y + m = 0 . Tm


cc gi tr ca m
1/ ng thng tip xc vi ng trn (C) .
2/ ng thng ct ng trn (C) .
3/ ng thng v ng trn (C) khng c im chung.
Bi 246. Bin lun theo m s giao im ca ng thng d v ng trn (C), vi
1/ d : mx y 3m 2 = 0

&

2/ d : 2x y + m = 0

&

3/ d : x + y 1 = 0

&

4/ d : mx + y 4m = 0

&

5/ d : x my + 2m + 3 = 0

&

6/ d : mx y + 2 = 0

&

7/ d : mx y + 7 4m = 0

&

8/ mx y + 3m 1 = 0

&

(C) : x
(C) : x
(C) : x
(C) : x
(C) : x
(C) : x
(C) : x
(C) : x

+ y2 4x 2y = 0 .

+ y2 6x + 2y + 5 = 0 .

+ y2 2 (2m + 1) x 4y + 4 m = 0 .

+ y2 2x 4y 4 = 0 .

+ y2 + x 2y 2 = 0 .

+ y2 + 2x 4y + 4 = 0 .

+ y2 + 2x 4y = 0 .

+ y2 4x 3y = 0 .

Bi 247. Cho ng trn (C) : x 2 + y2 2x 2y + 1 = 0 v ng thng d i qua im A (1; 0) v


c h s gc k
1/ Vit phng trnh ng thng d.
2/ Bin lun theo k v tr tng i ca d v (C) .
3/ Suy ra phng trnh cc tip tuyn ca (C) xut pht t A.
Bi 248. Cho ng thng d v ng trn (C)
a/ Chng t d ct (C) .

b/ Tm to cc giao im ca d v (C) .

1
1/ d i qua M (1; 5) v c h s gc k = ,
3
"Cn c b thng minh"

(C) : x

DeThiThuDaiHoc.com

+ y2 6x 4y + 8 = 0 .
Page - 141 -

www.MATHVN.com

Ths. L Vn on

(C) : x

2/ d : 3x y 10 = 0,

Phn hnh hc

+ y2 4x 2y 20 = 0 .

Bi 249. Cho ng trn (C) : x 2 + y2 2x 6y + 6 = 0 v im M (4;2) . Vit phng trnh ng


thng d qua M v ct ng trn (C) ti hai im A v B sao cho M l trung im ca AB.
Bi 250. Cho ng trn (C) : x2 + y2 4x + 2y 1 = 0 v ng thng d : y = x .
1/ Chng minh rng d ct (C) ti hai im phn bit A v B.
2/ Tnh di on AB.
Bi 251. Vit phng trnh ng thng qua A (2; 3) v ct ng trn (C) : (x + 1) + y2 = 9 ti hai
2

im M v N sao cho MN = 6 .
Bi 252. Cho ng trn (C) : (x 1) + (y + 2) = 4 v ng thng : 3x 4y + 5 = 0 . Vit
2

phng trnh ng thng d song song vi ng thng v ct (C) ti hai im A v B sao


cho AB c di ln nht.
Bi 253. Cho ng trn (C) : x 2 + y2 4x 6y + 5 = 0 . Vit phng trnh ng thng d i qua
im A (3;2) v ct ng trn (C) theo mt dy cung c di
1/ Ln nht.

2/ Nh nht.

Bi 254. Cho ng trn (C) : (x + 1) + (y + 2) = 2 v ng thng d : 3x 2y 1 = 0 .


2

1/ Xc nh v tr tng i ca d v (C) .

2/ Tm trn ng thng d im M (x o ; yo ) sao cho x2o + y2o t gi tr nh nht.


Bi 255. Tm trn ng trn (C) : x + y 2x + 2y 23 = 0 im M c khong cch n ng
2

thng : 3x 4y + 23 = 0
1/ Nh nht.

2/ Ln nht.

Bi 256. Cho ng trn (Cm ) : x + y 2mx + 2y + m + 7 = 0 c tm l I. Xc nh m ng


2

thng d : x + y + 1 = 0 ct (Cm ) ti hai im phn bit A v B sao cho IAB u.


Bi 257. Cho (Cm ) : x2 + y2 + (m + 2) x (m + 4) y + m + 1 = 0 . Chng minh rng (Cm ) lun i
qua hai im c nh. Suy ra gi tr ca m (Cm ) l ng trn c bn knh nh nht.

Bi 258. Cho ng trn (C) : (x 2) + (y 3) = 2 v ng thng : x y 2 = 0 . Tm ta


2

im M trn (C) sao cho khong cch t M n t gi tr ln nht, nh nht.

V TR TNG I CA HAI NG TRN


Bi 259. Xt v tr tng i ca hai ng trn (C1 ) v (C2 ), tm to giao im (nu c) vi
1/
2/
3/
4/

Page - 142 -

(C ) : x
(C ) : x

(C ) : x
(C ) : x

+ y2 + 6x 10y + 24 = 0
+ y2 4x 6y + 4 = 0
+ y2 6 x 3y = 0

+ y2 + 2x 4y 5 = 0

(C ) : x
(C ) : x

+ y2 6x 4y 12 = 0 .

+ y2 10x 14y + 70 = 0 .

(C ) c tm I 5; 2 v bn knh R
(C ) : x + y x 5y + 4 = 0 .
2

5
.
2

DeThiThuDaiHoc.com
"All the flower of tomorrow are in the seeks of today"

cng hc tp mn Ton 10 Tp IIwww.MATHVN.com

Ths. L Vn on

Bi 260. Bin lun s giao im ca hai ng trn (C1 ) v (C2 ), vi

(C ) : x 2 + y2 6x 2my + m2 + 4 = 0

1/ 1
.
2
2
2
(C2 ) : x + y 2mx 2 (m + 1) y + m + 4 = 0
2
2

(C1 ) : x + y + 4mx 2my + 2m + 3 = 0


2/
.
(C2 ) : x 2 + y2 + 4 (m + 1) x 2my + 6m 1 = 0

Bi 261. Cho hai ng trn (C1 ) : x2 + y2 7x 7 = 0 v (C2 ) : x 2 + y2 x 7y 18 = 0 .


1/ Chng t (C1 ) v (C2 ) ct nhau ti hai im A v B.
2/ Vit phng trnh ng thng i qua hai im A v B.
Bi 262. Cho hai ng trn (C1 ) : x 2 + y2 1 = 0 v (Cm ) : x2 + y2 2 (m + 1) x + 4my 5 = 0 .
Tm m (C1 ) v (Cm ) tip xc vi nhau.

Bi 263. Cho h ng trn (Cm ) : x 2 + y2 2mx + 4my + 5m2 1 = 0 .


1/ Tm m (Cm ) ct ng trn (C1 ) : x2 + y2 = 1 ti hai im phn bit A v B.
2/ Tm m (Cm ) v (C2 ) : x2 + y2 2x + 4y + 1 = 0 tip xc trong vi nhau.
Bi 264. Cho hai ng trn (C1 ) : (x + 2) + (y 2) = 2 v (C2 ) : (x 3) + (y 2) = 1 .
2

1/ Chng minh rng (C1 ) v (C2 ) nm ngoi nhau.


2/ Cho M (1;2) . Hy tm hai im A (C1 ), B (C2 ) sao cho M l trung im ca AB.
Bi 265. Cho hai im A (8; 0), B (0; 6) .
1/ Vit phng trnh ng trn ni tip tam gic OAB.
2/ Gi M, N, P ln lt l trung im ca OA, AB, OB. Vit phng trnh ng trn ngoi
tip tam gic MNP.
3/ Chng minh rng hai ng trn trn tip xc nhau. Tm to tip im.
Bi 266. Cho ng trn (C) : x2 + y2 = 1 . ng trn (C ') c tm I (2;2) ct (C) ti hai im A, B
sao cho AB = 2 . Hy vit phng trnh ng thng AB.
2

2
1
Bi 267. Cho ABC c A (1; 0), B (0;2) v ng trn (C) : (x 1) + y = 1 . Vit phng

trnh ng thng i qua cc giao im ca ng trn (C) v ng trn ngoi tip ABO.

TIP TUYN CA NG TRN


Bi 268. Vit phng trnh tip tuyn ca ng trn (C) ti im M (C)
1/
2/
3/
4/

(C) : x
(C) : x

+ y2 = 25

&

M (3; 4) .

+ y2 = 50

&

M (5; 5) .

&

M (8; 16) .

&

M (1;2) .

(C) : (x 3) + (y + 4) = 169
(C) : x + y + 4x 9 = 0
2

"Cn c b thng minh"

DeThiThuDaiHoc.com

Page - 143 -

www.MATHVN.com

Ths. L Vn on

5/
6/

(C) : x
(C) : x

Phn hnh hc

+ y2 + 4x + 4y + 3 = 0

&

M (3; 0) .

+ y2 2x 8y 8 = 0

&

M (4; 0) .

Bi 269. Cho ng trn (C) : x 2 + y2 4x 2y = 0 .


1/ Lp phng trnh tip tuyn ca (C) ti A c honh l 0 .
2/ Lp phng trnh tip tuyn ca (C) cc giao im ca n vi trc tung Oy .
3/ Lp phng trnh tip tuyn ca (C) ti giao im ca (C) v ng thng d : x + y = 0 .
Bi 270. Cho ng trn (C) : x 2 + y2 8x 6y + 17 = 0 .
1/ Chng t M (6; 5) nm trn ng trn (C) . Vit phng trnh tip tuyn d ca (C) qua M
2/ Chng t N (0; 1) nm ngoi ng trn (C) . Vit phng trnh tip tuyn d' ca (C)
qua im N.
Bi 271. Vit phng trnh tip tuyn (C) k t mt im cho trc
1/
2/
3/
4/
5/
6/

(C) : x
(C) : x
(C) : x
(C) : x
(C) : x
(C) : x

+ y2 4x + 2y + 2 = 0

&

A (3;1) .

+ y2 + 4x 4y 1 = 0

&

A (0; 1) .

+ y2 + 2x 4y + 4 = 0

&

A (3;5) .

+ y2 2x 8y 8 = 0

&

A (4; 6) .

+ y2 + 2x 8y + 13 = 0

&

A (1;1) .

+ y2 6x 4y + 8 = 0

&

A (8;7) .

&

A (2; 3) .

7/ (C) : (x 3) + (y 1) = 5 .
2

Bi 272. Cho ng trn (C) : (x 2) + (y 1) = 25 .


2

1/ Tm ta tm v bn knh ca ng trn (C) .


2/ Vit phng trnh tip tuyn ca (C) ti im M (5; 3) .
3/ Lp phng trnh tip tuyn ca (C) song song vi ng thng d1 : 5x 12y + 2 = 0 .
4/ Lp phng trnh tip tuyn ca (C) vung gc vi ng thng d2 : 3x + 4y 7 = 0 .
5/ Lp phng trnh tip tuyn ca (C) bit tip tuyn i qua im A (3;6) .
Bi 273. Cho ng trn (C) : x 2 + y2 6x + 2y + 5 = 0 .
1/ Tm ta tm v bn knh ca ng trn (C) .
2/ Lp phng trnh tip tuyn ca (C) song song vi ng thng d1 : 4x + 2y 1 = 0 .
3/ Lp phng trnh tip tuyn ca (C) vung gc vi ng thng d2 : 2x y + 7 = 0 .
Bi 274. Cho ng trn (C), im A v ng thng d.
a/ Chng t im A ngoi (C) .
b/ Vit phng trnh tip tuyn ca (C) k t A.

Page - 144 -

DeThiThuDaiHoc.com
"All the flower of tomorrow are in the seeks of today"

cng hc tp mn Ton 10 Tp IIwww.MATHVN.com

Ths. L Vn on

c/ Vit phng trnh tip tuyn ca (C) vung gc vi d.


d/ Vit phng trnh tip tuyn ca (C) song song vi d.
1/
2/

(C) : x
(C) : x

+ y2 4x 6y 12 = 0

A (7;7)

d : 3x + 4y 6 = 0 .

+ y2 + 4x 8y + 10 = 0

A (2;2)

d : x + 2y 6 = 0 .

Bi 275. Cho ng trn (C) v ng thng d.


a/ Vit phng trnh cc tip tuyn ca (C) ti cc giao im ca (C) vi cc trc to .
b/ Vit phng trnh tip tuyn ca (C) vung gc vi d.
c/ Vit phng trnh tip tuyn ca (C) song song vi d.
1/
2/
3/

(C) : x + y 6x 2y + 5 = 0
(C) : x + y 4x 6y = 0
(C) : x2 + y2 2x 6y + 9 = 0
2

&

d : 2x y + 3 = 0 .

&

d : 2x 3y + 1 = 0 .

&

d : 3x 4y + 12 = 0 .

Bi 276. Cho ng trn (C) . Hy lp phng trnh tip tuyn vi (C), bit tip tuyn to vi ng
thng d mt gc trong cc trng hp sau:
1/ (C) : (x 1) + (y + 1) = 10

= 45 0

&

d : 2x + y 4 = 0 .

2/ (C) : x 2 + y2 + 4x 8y + 10 = 0

= 600

&

d : 2x 3y + 1 = 0 .

Bi 277. Cho hai ng trn (C1 ) : x2 + y2 = 9 v (C2 ) : x2 + y2 2x 3 = 0 .


1/ Tm tm v bn knh ca ng trn (C1 ) v (C2 ) .
2/ Xt v tr tng i ca (C1 ) v (C2 ) .
3/ Vit phng trnh tip tuyn chung ca (C1 ) v (C2 ) .
Bi 278. Cho hai ng trn (C1 ) : x2 + y2 2x 2y 2 = 0 v (C2 ) : x 2 + y2 8x 4y + 16 = 0 .
1/ Tm tm v bn knh ca ng trn (C1 ) v (C2 ) .
2/ Xt v tr tng i ca (C1 ) v (C2 ) .
3/ Vit phng trnh tip tuyn chung ca (C1 ) v (C2 ) .
Bi 279. Cho hai ng trn (C1 ) : x2 + y2 2x 2y 2 = 0 v (C2 ) : x 2 + y2 8x 4y + 16 = 0 .
1/ Tm tm v bn knh ca ng trn (C1 ) v (C2 ) .
2/ Xt v tr tng i ca (C1 ) v (C2 ) .
3/ Vit phng trnh tip tuyn chung ca (C1 ) v (C2 ) .
Bi 280. Cho hai ng trn (C1 ) : x2 + y2 10x = 0 v (C2 ) : x 2 + y2 + 4x 2y 20 = 0 .
1/ Tm tm v bn knh ca ng trn (C1 ) v (C2 ) .
2/ Xt v tr tng i ca (C1 ) v (C2 ) .
3/ Vit phng trnh tip tuyn chung ca (C1 ) v (C2 ) .
"Cn c b thng minh"

DeThiThuDaiHoc.com

Page - 145 -

www.MATHVN.com

Ths. L Vn on

Phn hnh hc

Bi 281. Cho hai ng trn (C1 ) : x2 + y2 4x 5 = 0 v (C2 ) : x2 + y2 6x + 8y + 16 = 0 .


1/ Tm tm v bn knh ca ng trn (C1 ) v (C2 ) .
2/ Xt v tr tng i ca (C1 ) v (C2 ) .
3/ Vit phng trnh tip tuyn chung ca (C1 ) v (C2 ) .
Bi 282. Lp phng trnh tip tuyn chung ca cc ng trn (C1 ) v (C2 ) sau:
&
(C ) : x + y = 1
(C ) : (x 8) + (y 6) = 16 .
2/ (C ) : (x 1) + (y 1) = 1
&
(C ) : (x 2) + (y + 1) = 4 .
3/ (C ) : (x 1) + (y + 3) = 25
&
(C ) : (x + 1) + (y 3) = 9 .
4/ (C ) : x + y 4x 2y + 4 = 0
&
(C ) : x + y + 4x + 2y 4 = 0 .
5/ (C ) : x + y 2x + 4y 4 = 0
&
(C ) : x + y + 4x 4y 56 = 0 .
6/ (C ) : x + y 10x = 0
&
(C ) : x + y + 4x 2y 2 = 0 .
7/ (C ) : x + y 10x + 24y 56 = 0
&
(C ) : x + y 2x 4y 20 = 0 .
8/ (C ) : x + y + 2x 6y + 6 = 0
&
(C ) : x + y 4x + 2y 4 = 0 .
9/ (C1 ) : x2 + y2 8x 4y 29 = 0
&
(C2 ) : x2 + y2 2x 12y + 33 = 0 .
10/ (C1 ) : x2 + y2 2x 3 = 0
&
(C2 ) : x2 + y2 8x 8y + 28 = 0 .
11/ (C1 ) : x2 + y2 2x 2y 2 = 0
&
(C2 ) : x2 + y2 6x 4y + 19 = 0 .
12/ (C1 ) : x2 + y2 6x + 5 = 0
&
(C2 ) : x2 + y2 12x 6y + 44 = 0 .
Cho hai im A (1; 2), B (3; 4) v ng thng d : 3x + y + 3 = 0 .

1/

Bi 283.

1/ Vit phng trnh cc ng trn (C1 ) v (C2 ) qua A, B v tip xc vi d.


2/ Vit phng trnh tip tuyn chung (khc d) ca hai ng trn .
Bi 284. Cho ng trn (C) : x 2 + y2 6x 4y + 4 = 0 v im A (8; 1) .
1/ Vit phng trnh tip tuyn ca (C) k t A.
2/ Gi M v N l cc tip im ca cc tip tuyn k t A ca (C) . Vit phng trnh ng
thng MN v tnh di on thng MN.
Bi 285. Cho ng trn (C) : x2 + y2 + 2x 4y + 4 = 0 v im A (3;5) .

1/ Vit phng trnh tip tuyn ca (C) k t A.


2/ Gi E v F l cc tip im ca cc tip tuyn k t A ca (C) . Vit phng trnh ng
thng EF v tnh di on thng EF.

Bi 286. Cho ng trn (C) : (x 2) + (y 4) = 4 . Lp phng trnh tip tuyn ca ng trn


2

bit rng tip tuyn:


1/ To vi hai trc ta mt tam gic vung cn.
2/ To vi hai trc ta mt tam gic c din tch bng 4 .
Bi 287. Cho ng trn (C) : x 2 + y2 6x 2my + m2 + 4 = 0 .

1/ Tm m t A (2; 3) c th k c hai tip tuyn vi (C) .


Page - 146 -

DeThiThuDaiHoc.com
"All the flower of tomorrow are in the seeks of today"

cng hc tp mn Ton 10 Tp IIwww.MATHVN.com

Ths. L Vn on

2/ Vit phng trnh cc tip tuyn khi m = 6 .


Bi 288. Cho ng trn (C) : x2 + y2 + 2x 4y = 0 v ng thng d : x y + 1 = 0 . Tm ta
im M thuc d m t c th k c hai ng thng tip xc vi ng trn (C) ti A v


B sao cho AMB = 600 .
Bi 289. Cho ng trn (C) : (x 1) + (y + 2) = 9 v ng th d : 3x 4y + m = 0 . Tm m
2

trn d c duy nht mt im P m t c th k c hai tip tuyn PA, PB n ng trn


sao cho PAB u.

Bi 290. Cho (C ) : x2 + y2 sin = 2 (x cos + y sin cos ), k .


1/ Chng minh rng (C ) lun l ng trn . nh tm v bn knh ng trn (C ) .
2/ Chng minh rng (C ) lun c mt tip tuyn c nh v xc nh tip tuyn .
Bi 291. Cho hai ng trn (C1 ) : x2 + y2 2x 4y 4 = 0 v (C2 ) : x2 + y2 + 6x 2y + 1 = 0 .
1/ Chng minh (C1 ) v (C2 ) ct nhau ti hai im A v B.
2/ Vit phng trnh ng trn (C) qua A, B v im C (3; 1) .
3/ Cho im M (4;1) . Chng minh qua M c hai tip tuyn n (C) . Gi E, F l hai tip im
ca hai tip tuyn trn vi (C) . Hy lp phng trnh ng trn (C ') ngoi tip MEF.

Bi 292. Lp phng trnh ng trn (C) c bn knh nh nht, tip xc vi ng thng

d : 3x + 4y + 20 = 0 v ng trn (C ') : (x 1) + (y + 2) = 1 .
2

Bi 293. Tm trn ng thng d : 3x + 4y + 20 = 0 nhng im m t c th k n ng trn

Bi 294.

(C) : x2 + y2 = 1 nhng tip tuyn c di nh nht.


Cho (C1 ) : x2 + y2 4x + 2y 4 = 0 v (C2 ) : x2 + y2 10x 6y + 30 = 0 c tm ln
lt l I v J.
1/ Chng minh rng (C1 ) v (C2 ) tip xc ngoi nhau. Tm ta tip im H.

2/ Gi d l tip tuyn chung ca (C1 ) v (C2 ) khng i qua H. Tm giao im K ca d v IJ.


3/ Vit phng trnh ng trn (C ') i qua K v tip xc vi (C1 ) v (C2 ) ti H.

XET V TR TNG I CA NG THNG V NG TRN GII


H PHNG TRNH H BT PHNG TRNH
mx + (m + 1) y = 2
Bi 295. nh m h phng trnh sau c nghim
.
2
x + y2 = 4

x + my m = 0

Bi 296. nh m h phng trnh sau c gnhim 2


.
x + y2 x = 0

x2 + y2 + 2x 4y + 4 = 0

Bi 297. nh m h phng trnh sau c ng mt nghim


.
mx y + 2 = 0

"Cn c b thng minh"

DeThiThuDaiHoc.com

Page - 147 -

www.MATHVN.com

Ths. L Vn on

Phn hnh hc

x2 + y2 = 2 1 + m
(
)

Bi 298. nh m h phng trnh sau c ng hai nghim


.

2
(x + y) = 4

x2 + y2 = 9

Bi 299. Cho h phng trnh


.
(2m + 1) x + my + m 1 = 0
Xc nh tham s m h phng trnh c hai nghim (x1; y1 ), (x2 ; y2 ) sao cho biu thc
A = (x1 x2 ) + (y1 y2 ) t gi tr ln nht.
2

x 1 + y + 1 = 1
Bi 300. Cho h phng trnh
. nh m h c nghim nhiu nht.
2
x + y2 = m

Bi 301. nh m phng trnh sau c nghim: x + y + 2x (y 1) + m = 2 .

x2 + y2 + 4x + 6y 12 0

.
Bi 302. Cho h bt phng trnh
x y + m 0

1/ Gii h khi m = 1 .
2/ nh m h c nghim duy nht.
2
2

(x 1) + (y + 1) m
Bi 303. nh m h bt phng trnh sau c nghim duy nht
.
x + 1 2 + y 1 2 m
(
)
(
)

Page - 148 -

DeThiThuDaiHoc.com
"All the flower of tomorrow are in the seeks of today"

cng hc tp mn Ton 10 Tp IIwww.MATHVN.com

Ths. L Vn on

BI TP QUA CC K THI
Bi 304. Cc Trng Cao ng nm 1984
Trong mt phng vi h trc ta vung gc Oxy, cho hai im A (a; 0) v B (0;a) .

m 2
(m l tham s).
2
Vit phng trnh ng trn ny. Tm ta giao im P ca (C) v ng thng AB.

1/ Gi (C) l ng trn tip xc vi Ox ti A v tm C c tung l

2/ Vit phng trn ng trn (C ') i qua P v tip xc vi Oy ti B. Hai ng trn (C) v

(C ') ct nhau ti P v Q. Vit phng trnh ng thng PQ. Chng minh rng khi m thay
i, ng thng PQ lun lun i qua mt im c nh.

m 2

;a .
S: 1/ P a

2
m 2
m 2

2/ (C ') : x a +
+ (y a ) = a
,

2
2

PQ : m 2y 2a m 2 y = 0 v PQ lun i qua gc ta O khi m thay i.

Bi 305. Cao ng S Phm Tp. H Ch Minh nm 1996


Trong mt phng ta Oxy, cho hai ng trn: (C) : x 2 + y2 1 = 0 v

(C ) : x
m

+ y2 2 (m + 1) x + 4my 5 = 0 .

1/ Tm qu tch tm (Cm ) khi m thay i.


2/ Chng minh rng: c hai ng trn (Cm ) tip xc vi (C) ng vi hai gi tr ca m.
S: 1/ Qu tch tm I (m + 1; 2m ) l d : 2x + y 2 = 0 . 2/ m = 1; m =

3
.
5

Bi 306. Cao ng Kinh T i Ngoi Tp. H Ch Minh nm 1998


Trong mt phng ta Oxy, cho h (Cm ) : x2 + y2 + 4mx 2my + 2m + 3 = 0 .
1/ Xc nh m (Cm ) l ng trn.
2/ Tm tp hp tm cc ng trn (Cm ) .
Bi 307. Cao ng Kinh T i Ngoi nm 1999 H cha phn ban
Trong mt phng vi h trc ta Descarter vung gc Oxy, cho h ng cong
(Cm ) : x2 + y2 2x 2y + m = 0 vi m l tham s.
1/ Vi iu kin no ca m th (Cm ) l ng trn ? Xc nh tm v bn knh ca (Cm ) trong
trng hp ny.
2/ nh m (Cm ) l ng trn c bn knh bng 1. Gi ng trn ny l (C) . Vit phng

2
2

;1
trnh ng thng d tip xc vi ng trn (C) ti im A 1 +
.

2
2
3/ Vit phng trnh tt c cc tip tuyn vi ng trn (C) bit chng vung gc ng
thng d.
"Cn c b thng minh"

DeThiThuDaiHoc.com

Page - 149 -

www.MATHVN.com

Ths. L Vn on

S: 1/ m < 2, I (1;1), R = 2 m .

Phn hnh hc

2/ m = 1, d : x y 2 = 0 .

Bi 308. Cao ng S Phm H Ni khi A nm 2000


Trong mt phng ta Oxy, cho ba im A (2; 6), B (3; 4), C (5; 0) .
1/ Vit phng trnh ng trn ni tip ABC.
2/ Tm ta im D i xng vi B qua AC.

Bi 309. Cao ng Nng Lm nm 2000


Trong mt phng ta Oxy, cho hai im A (1; 0), B (2;1) v ng thng d c phng trnh

(d) : 2x y + 3 = 0 .
1/ Tm phng trnh ng trn c tm ti A, tip xc vi ng thng d. Hy xt xem im B
nm pha trong hay pha ngoi ng trn tm ?
2/ Tm trn ng thng d im M sao cho tng (MA + MB) l nh nht so vi mi im
cn li trn ng thng d. Vit ta im M ?

S: 1/ (x 1) + y2 = 5 v im B nm trong ng trn.
2

8 17
2/ M ; .
11 11

Bi 310. Cao ng S Phm K Thut nm 2000


Trong mt phng ta Oxy, cho phng trnh ng trn: (C) : x 2 + y2 6x 2y + 8 = 0 .
Hy vit phng trnh tip tuyn vi (C) c h s gc k = 1 .
Bi 311. Cao ng Kinh T i Ngoi nm 2000
Trong mt phng ta Oxy, h ng cong (Cm ) : x 2 + y2 2x 2y + m = 0 .
1/ Vi iu kin no ca m th (Cm ) l ng trn ? Xc nh tm v bn knh ca (Cm )
trong trng hp ny ?
2/ nh m (Cm ) l ng trn c bn knh bng 1. Gi ng trn ny l (C) . Vit phng

2
2

trnh ng thng d tip xc vi ng trn (C) ti im A 1 +


;1
.

2
2
3/ Vit phng trnh tt c cc tip tuyn vi ng trn (C) bit chng vung gc vi ng
thng d.

m = 1
m 2

d : x y 2 = 0

2
2
S: 1/ Tm I (1;1) . 2/ (C) : x + y 2x 2y 1 = 0 . 3/ 1
.

d2 : x y + 2 = 0
R = 2 m
TT d : x y 2

Bi 312. Cao ng Y T Nam nh nm 2000 (H Cao ng iu Dng Chnh Qui)


Trong mt phng ta Oxy:
1/ Vit phng trnh ng trn tm Q (1;2), bn knh R = 13 , gi l ng trn (Q) .

Page - 150 -

DeThiThuDaiHoc.com
"All the flower of tomorrow are in the seeks of today"

cng hc tp mn Ton 10 Tp IIwww.MATHVN.com

Ths. L Vn on

2/ Tm ta giao im ca ng trn (Q) vi ng thng c phng trnh

() : x 5y 2 = 0 , gi cc giao im l A, B. Tm ta im C soa cho ABC l


tam gic vung v ni tip trong ng trn (Q) .
S: 1/ (Q) : (x + 1) + (y 2) = 13 .
2

2/ A (2; 0), B (3; 1) . Nu BC: ng knh th

C (1; 5) . Nu AC: ng knh th C (4; 4) .

Bi 313. Cao ng Lao ng X Hi nm 2000


Trong mt phng ta Oxy, cho ABC c A (3; 7), B (9; 5), C (5;9) .
1/ Vit phng trnh ng phn gic gc ln nht ca ABC.
2/ Vit phng trnh tip tuyn i qua im M (2; 7) n ng trn ngoi tip ca ABC.
Tm ta tip im ?

Bi 314. Cao ng S Phm Tp. H Ch Minh khi A nm 2002


Trong mt phng ta Oxy, cho ng trn (C) : x2 + y2 = 9 v im A (1;2) . Hy lp
phng trnh ca ng thng cha dy cung ca (C) i qua A sao cho di dy cung
ngn nht ?
Bi 315. Cao ng S Phm H Tnh khi A, B nm 2002
Trong mt phng ta Oxy cho ng trn (C) : x 2 + y2 2x 6y + 6 = 0 v im
M (2; 4) .

1/ Chng t rng im M nm trong ng trn.


2/ Vit phng trnh ng thng i qua im M, ct ng trn ti hai im A, B sao cho M
l trung im ca AB.
3/ Vit phng trnh ng trn i xng vi ng trn cho qua AB.
S: 1/ Chng minh IM < IR .

2/ x + y 6 = 0 .

2/ (x 3) + (y 5) = 4 .
2

Bi 316. Cao ng Ti Chnh K Ton IV nm 2003


Trong mt phng vi h trc ta cc vung gc Oxy, cho ng trn

(C) : (x 3)

+ (y 1) = 4 . Vit phng trnh tip tuyn vi (C) , bit rng tip tuyn ny i
2

qua im Mo (6; 3) .
Bi 317. Cao ng Mu Gio TW3 nm 2003
Cho tam gic ABC c A (1;5) v phng trnh ng thng BC : x 2y 5 = 0 vi

x B < x C v I (0;1) l tm ng trn ngoi tip tam gic ABC.


1/ Vit phng trnh cc cnh AB, AC ca ABC.
2/ Gi A1, B1,C1 ln lt l chn ng cao v t cc nh A, B, C ca tam gic. Tm ta
cc im A1, B1,C1 .
3/ Gi E l tm ng trn ni tip A1B1C1 . Tm ta im E.
Bi 318. Cao ng Lng Thc Thc Phm nm 2004 (i hc Lng Thc Thc Phm)

"Cn c b thng minh"

DeThiThuDaiHoc.com

Page - 151 -

www.MATHVN.com

Ths. L Vn on

Phn hnh hc

Trong mt phng vi h trc ta Descarter vung gc Oxy, cho im A (2; 4) . Vit phng
trnh ng thng trung trc d ca on OA. T suy ra phng trnh ng trn (C) c tm
I trn Ox v i qua hai im O v A vi O l gc ta .
S: d : x + 2y 5 = 0, (C) : (x 5) + y2 = 25 .
2

Bi 319. Cao ng S Phm Mu Gio TW3 nm 2004


Trong mt phng Oxy cho ng trn (C) : x2 + y2 + 2x 4y = 0 v ng thng
d : x y +1 = 0.

1/ Vit phng trnh ng thng vung gc vi d v tip xc vi ng trn.


2/ Vit phng trnh ng thng song song vi d v ct ng trn ti hai im M, N sao cho
di MN = 2 .
3/ Tm ta im T trn d sao cho qua T k c hai ng thng tip xc vi (C) ti hai


im A, B v ATB = 600 .

x y + 3 + 2 2 = 0
S: x + y 1 + 10 = 0,
, T (3; 4) T (3; 2) .
x y + 3 2 2 = 0

Bi 320. Cao ng Cng Nghip H Ni nm 2004


Trong mt phng vi h trc ta Oxy cho ABC, AB : x + y 2 = 0,
AC : 2x + 6y + 3 = 0, cnh BC c trung im M (1;1) . Vit phng trnh ng trn ngoi

tip ABC.

1
3
85
S: (C) : x + y + =
.

2
2
8

Bi 321. Cao ng Giao Thng Vn Ti II nm 2004


Trong mt phng vi h trc ta Descarter vung gc Oxy, cho hai ng trn:
(C1 ) : x2 + y2 + 6x 8y = 0 v (C2 ) : x2 + y2 + 6x 4y 4 = 0 . Chng minh rng hai
ng trn ct nhau ti hai im phn bit. Hy vit phng trnh trc ng phng ca hai
ng trn .

x = 1 + 8t
S: (C1 ) (C2 ) = A (7;1), B (1;1) Trc ng phng AB :

(t  ) .
y = 1

Bi 322. Cao ng Giao Thng Vn Ti III nm 2004


Trong mt phng vi h trc ta Descarter vung gc Oxy, cho h ng cong
(Cm ) : x2 + y2 4mx + 2 (m + 2) y + 6m2 1 = 0 .
1/ Xc nh m (Cm ) l ng trn. Khi , tnh theo m ta tm I v bn knh R ca

(C ) .
m

2/ Tm m (Cm ) l ng trn c tm nm trn ng cong (P) : y = x 2 7 .


S: 1/ 1 < m < 5 v I (2m; m 2), R = m2 + 4m + 5 . 2/ m =
Page - 152 -

5
m = 1.
4

DeThiThuDaiHoc.com
"All the flower of tomorrow are in the seeks of today"

cng hc tp mn Ton 10 Tp IIwww.MATHVN.com

Ths. L Vn on

Bi 323. Cao ng Kinh T K Thut Thi Bnh nm 2004


Trong mt phng vi h trc ta Oxy, cho ABC c A (1;2), B (2; 4), C (3;1) .
1/ Lp phng trnh ng trn qua A, B, C.
2/ Tm ta im M trn cnh BC sao cho din tch ABM bng

5
5
5
S: 1/ (C) : x + y = .

2
2
2
2

1
din tch ABC.
3

7
3

2/ M ;3 .

Bi 324. Cao ng Cng Nghip H Ni nm 2005


Trong mt phng vi h trc ta Oxy, cho ABC, bit cc cnh AB, BC, CA ln lt c
phng trnh: 2x + y 5 = 0, x + 2y + 2 = 0, 2x y + 9 = 0 . Tm ta tm ng trn
ni tip ABC.
S: I (1;2) .
Bi 325. Cao ng Kinh T K Thut Cng Nghip I khi A nm 2005
Trong mt phng vi h trc ta Descarter vung gc Oxy, cho ba im A (1;2),

B (2; 3), C (2; 1) . Tm ta tm I ca ng trn qua ba im A, B, C.


S: I (1;1) .
Bi 326. Cao ng S Phm H Nam khi B nm 2005
Trong mt phng vi h trc ta Descarter vung gc Oxy, cho hai im A (5; 0), B (1; 4)
v ng thng d : x + y 3 = 0 . Vit phng trnh ng trn (C) i qua A, B v c tm
nm trn ng thng d.
S: (C) : (x 2) + (y 1) = 10 .
2

Bi 327. Cao ng S Phm Sn La khi A, B, T, M nm 2005


Trong mt phng vi h trc ta vung gc Oxy, cho ng thng : 2x + y + 3 = 0 v
hai im A (5;1), B (2; 4) .
1/ Vit phng trnh ng trn (C) i qua A, B v c tm I .
2/ Vit phng trnh ng tip tuyn ti A vi ng trn (C) .
3/ Vit phng trnh cc tip tuyn vi (C), bit tip tuyn i qua D (1;2) . Tm ta tip
im.
S: 1/ (C) : (x + 2) + (y 1) = 9 .
2

x 1 = 0 M (1;1)

2 14 .
2/ x + 5 = 0 . 3/

4x + 3y 10 = 0 M ;
5 5

Bi 328. Cao ng S Phm C Mau khi B nm 2005


Trong mt phng vi h trc ta Descarter vung gc Oxy, cho A (2; 1) v ng trn

(C) : x

+ y2 4x 6y 12 = 0 .

"Cn c b thng minh"

DeThiThuDaiHoc.com

Page - 153 -

www.MATHVN.com

Ths. L Vn on

Phn hnh hc

1/ Chng minh rng A l mt im nm ngoi ng trn.


2/ Vit phng trnh cc ng thng i qua im A v tip xc vi ng trn (C) .
S: x

16 + 5 7
34 5 7
y+
= 0.
9
9

Bi 329. Cao ng S Phm Tr Vinh khi B, M nm 2005


Trong mt phng vi h trc ta Descarter vung gc Oxy, cho ABC bit A (4; 2),

B (2;2), C (4; 1) . Vit phng trnh ng trn (C) ngoi tip ABC v phng trnh
tip tuyn vi (C) ti B.
2

3
65

S: (C) : x + y + =
v 4x 7y + 22 = 0 .

2
4

Bi 330. Cao ng Ti Chnh K Ton nm 2005


Trong mt phng vi h trc ta Oxy, cho ba im : A (2; 2), B (0; 4), C (2;2) . Tm ta
trc tm v tm ng trn ngoi tip ABC.
Bi 331. Cao ng S Phm Sc Trng nm 2005
Trong mt phng vi h trc ta Oxy, cho hai im A (4;2), B (1; 1) . Vit phng trnh
ng trn qua A, B v c tm nm trn ng thng 2x y = 0 .
Bi 332. Cao ng Y T Thanh Ha nm 2005
Lp phng trnh tip tuyn chung ca hai ng trn (C1 ) : x2 + y2 4x 2y + 4 = 0 v

(C ) : x
2

+ y2 + 4x + 2y 4 = 0 trong mt phng ta vung gc Oxy.

Bi 333. Cao ng S Phm Vnh Phc nm 2005


Trong mt phng vi h trc ta Oxy, cho ng trn (C) : x 2 + y2 + 2x 4y 20 = 0 .
Vit phng trnh tip tuyn vi (C) , bit rng tip tuyn vung gc vi ng thng
d: x + y = 0.

Bi 334. Cao ng S Phm Qung Bnh nm 2005


Trong mt phng vi h trc ta Oxy, vit phng trnh ng trn (C) qua ba im

M1 (2; 3), M2 (4;5), M3 (4;1) . Chng t im K (5;2) thuc min trong ng trn (C) . Vit
phng trnh ng thng d qua K sao cho d ct (C) theo dy cung AB nhn K lm trung
im.
Qua K

  .
S: (C) : x 2 + y2 8x 6y + 21 = 0, K(5;2)/(C) = 2 < 0, d :
VTPT n = IK
d

Bi 335. Cao ng Kinh T K Thut Cng Nghip II khi A nm 2006


Trong mt phng vi h trc ta Descarter vung gc Oxy, cho ng trn
(C) : x2 + y2 2x 4y + 3 = 0 . Lp phng trnh ng trn (C ') i xng vi ng
trn (C) qua ng thng d : x 2 = 0 .
Page - 154 -

DeThiThuDaiHoc.com
"All the flower of tomorrow are in the seeks of today"

cng hc tp mn Ton 10 Tp IIwww.MATHVN.com

Ths. L Vn on

S: (C ') : ( x 3) + ( y 2) = 2 .
2

Bi 336. Cao ng Xy Dng s 3 khi A nm 2006


Trong mt phng vi h trc ta Descarter vung gc Oxy, cho ng trn
(C) : x2 + y2 4x + 2y + 1 = 0 . Vit phng trnh cc tip tuyn ca (C) i qua F (0; 3) .
S: d : x = 0 v d : 3x + 4y 12 = 0 .
Bi 337. Cao ng Kinh T i Ngoi khi A, D nm 2006
Trong mt phng vi h trc ta Descarter vung gc Oxy, cho ng trn
(C) : x2 + y2 + 4x 2y 20 = 0 . Tm tt c cc tip tuyn ca (C) song song vi ng
thng 3x + 4y = 0 .
S: d : 3x + 4y + 27 = 0 d : 3x + 4y 23 = 0 .
Bi 338. Cao ng S Phm Hng Yn khi B nm 2006
Trong mt phng vi h trc ta Descarter vung gc Oxy, tm di dy cung xc nh bi
ng thng 4x + 3y 8 = 0 v ng trn tm I (2;1) tip xc vi ng thng

5x 12y + 15 = 0 .
S: (C) : ( x 2) + (y 1) = 1 v di dy cung AB =
2

8
.
5

Bi 339. Cao ng S Phm Mu Gio TW 3 nm 2006


Trong mt phng vi h trc ta Descarter vung gc Oxy, cho cho hai ng thng c
phng trnh: d1 : x y + 2 = 0, d 2 : 2x + y 5 = 0 v im M (1; 4) .
1/ Vit phng trnh ng thng ct d1, d2 ln lt ti A v B sao cho M l trung im ca
on AB.
2/ Vit phng trnh ng trn (C) qua M v tip xc vi ng thng d1 ti giao im ca
d1 vi trc tung.

5
17
25
2/ (C) : x + + y =
.

6
6
18

S: 1/ AB : x + 1 = 0 .

Bi 340. Cao ng S Phm Trung ng nm 2006


Trong mt phng vi h trc ta Descarter vung gc Oxy, cho ng trn
(C) : x2 + y2 4x + 2y + 1 = 0 v im A (0; 3) . Xc nh ta tm v bn knh ng
trn (C) . Vit phng trnh tip tuyn ca ng trn (C) k t A.
S: Tm I (2; 1) v bn knh R = 2 . C hai tip tuyn: x = 0 3x + 4y 12 = 0 .
Bi 341. D b khi A Cao ng S Phm H Nam nm 2006
Trong mt phng vi h trc ta Descarter vung gc Oxy.
1/ Vit phng trnh ng thng i qua A (2; 4) v tip xc vi ng trn

(C) : x

+ y2 2x 4y + 4 = 0 .

"Cn c b thng minh"

DeThiThuDaiHoc.com

Page - 155 -

www.MATHVN.com

Ths. L Vn on

Phn hnh hc

2/ Cho im A (2;3), B ;0, C (2; 0) . Xc nh ta im I l tm ng trn ni tip

tam gic ABC.


S: 1/ x 2 = 0 y =

3
5
x+ .
4
2

1 1

2/ I ; .
2 2

Bi 342. Cao ng K Thut Y T I nm 2006


Trong mt phng vi h trc ta Descarter vung gc Oxy, cho hai ng thng d1 v d2 c
phng trnh d1 : 2x + y + 2 = 0, d 2 : 2x + y 1 = 0 . Vit phng trnh ng trn c tm
nm trn trc Ox, ng thi tip xc vi d1 v d2.

S: (C) : x +

1
9
+ y2 =
.
4
20
2

Bi 343. Cao ng Ti Chnh Hi Quan khi A nm 2006


Trong mt phng vi h trc ta Descarter vung gc Oxy, cho ng trn
(C) : x2 + y2 2x + 4y 5 = 0 . Lp phng trnh tip tuyn vi ng trn (C) bit rng
tip tuyn i qua A (4; 3) .
S: 3x + y + 9 = 0 x + 3y 5 = 0 .
Bi 344. Cao ng K Thut Cao Thng nm 2006
Trong mt phng vi h trc ta Descarter vung gc Oxy, cho ng trn (C) c phng
trnh: x 2 + y2 4x + 8y 5 = 0 .
1/ Vit phng trnh tip tuyn ca ng trn, bit cc tip tuyn ny vung gc vi ng
thng x + 2y = 0 .
2/ Tm iu kin ca m ng thng x + (m 1) y + m = 0 tip xc vi ng trn.
S: 1/ d : 2x y 8 + 5 5 = 0 d : 2x y 8 5 5 = 0 .
2/ d (I, ) = R 8m2 7m + 7 = 0 v nghim nn khng c m tha YCBT.
Bi 345. Cao ng K Thut Tp. H Ch Minh kha II nm 2006
Trong mt phng vi h trc ta Descarter vung gc Oxy, cho im I (2;1) v ng
thng d : 3x 4y = 0 .
1/ Vit phng trnh ng trn (C) c tm I v tip xc vi d.
2/ Vit phng trnh tp hp cc im m qua cc im v c hai tip tuyn n (C) sao
cho hai tip tuyn vung gc nhau.
S: 1/ (C) : (x + 2) + (y 1) = 4 .
2

2/ Tp hp l (C) : (x + 2) + (y 1) = 8 .
2

Bi 346. Cao ng Kinh T Cng Ngh Tp. H Ch Minh khi A nm 2006


Trong mt phng vi h trc ta vung gc Oxy, cho im A (3; 0) v ng thng d c
phng trnh: 3x 4y + 16 = 0 . Vit phng trnh ng trn tm A v tip xc vi d.
Page - 156 -

DeThiThuDaiHoc.com
"All the flower of tomorrow are in the seeks of today"

cng hc tp mn Ton 10 Tp IIwww.MATHVN.com

Ths. L Vn on

S: (C) : (x 3) + y2 = 25 .
2

Bi 347. Cao ng Kinh T Tp. H Ch Minh nm 2007


Trong mt phng vi h trc ta Descarter vung gc Oxy, cho ng trn c phng trnh
(C) : x2 + y2 2x + 4y 5 = 0 . Lp phng trnh tip tuyn vi (C) qua im M (4; 3) .
S: 1 : 3x + y + 9 = 0 2 : x + 3y 5 = 0 .
Bi 348. Cao ng S Phm TW Tp. H Ch Minh nm 2007
Trong mt phng vi h trc ta Oxy, cho ABC c A (2;2), B (8;6), C (1; 1) .
1/ Tm ta im H l chn ng cao k t nh A ca ABC.
2/ Vit phng trnh ng trn i qua hai im A, B v c tm nm trn ng thng
d : 5x 3y + 6 = 0 .
3/ Vit phng trnh ng thng i qua im A v cch im B mt khong bng 6 .

S: 1/ H (3;1) . 2/ (C) : (x 3) + (y 7) = 26 .
2

3/ x 2 = 0 5x + 12y 34 = 0 .

Bi 349. Cao ng K Thut Cao Thng nm 2007


Trong mt phng vi h trc ta vung gc Oxy, cho ng trn (C) c phng trnh:

x2 + y2 6x 4y 28 = 0 . Vit phng trnh cc tip tuyn vi ng trn (C), bit cc


tip tuyn song song vi ng thng 5x + 4y = 0 .

S: d1 : 5x + 4y + 18 = 0 d2 : 5x + 4y 6 = 0 .
Bi 350. Cao ng A, A1, B, D nm 2012 (Chng trnh nng cao)
Trong mt phng vi h trc ta Oxy, cho ng thng d : 2x y + 3 = 0 . Vit phng
trnh ng trn c tm thuc d, ct trc Oxy ti A v B, ct trc Oy ti C v D sao cho
AB = CD = 2 .
S: (C) : (x + 3) + (y + 3) = 10 .
2

Bi 351. i hc S Phm Tp. H Ch Minh nm 1976


Trong mt phng vi h trc ta Descarter vung gc Oxy, cho bn im: A (1; 1),
B (4;2), C (1; 5) v D (2;2) .


   
1/ Tnh mun ca AB, AD, CD, CB . Chng minh rng AB v AD vung gc. T suy
ra t gic ABCD l hnh g ?
2/ Vit phng trnh ng trn ngoi tip t gic ABCD.




2
2
S: 1/ AB = AD = CD = CB = 3 2; Hnh vung. 2/ (C) : (x 1) + (y 2) = 3 .

Bi 352. i hc Gio Dc Tp. H Ch Minh khi A nm 1976


Ngi ta cho hai trc ta trc chun x'x v y'y v mt s dng . Vng trn (C) vi
phng trnh: x2 + y2 2 = 0 ct na trc Ox A v na trc Ox' A'.
1/ Vit phng trnh vng trn ng knh OA'. Ta gi l tm ca vng trn .
2/ Gi Az l na tip tuyn bn trong gc xOy ca vng trn (C) . Vit phng trnh ca
vng trn tip xc vi y'y, x'x v Az. Gi l tm ca vng trn .

"Cn c b thng minh"

DeThiThuDaiHoc.com

Page - 157 -

www.MATHVN.com

Ths. L Vn on

Phn hnh hc

3/ Tm phng trnh ca nhng tip tuyn chung cho cc vng trn tm v .


2

2

2

S: 1/ (C ) : x + + y = .
2

3/ d1 : x = 0 d23 : y =

2
2

2/ C : x + y = .
2
2

( )

1
1 5
3

d4 : y = x + .
x+
2
4
4
4

Bi 353. i hc Y Nha Dc Tp. H Ch Minh nm 1977


Trong mt phng vi h trc ta Descarter vung gc Oxy, cho cc ng trn
(C) : x2 + y2 x y = 0 v (C ') : x2 + y2 x + y = 0 .
1/ Xc nh ta tm ca (C) v (C ') . Tnh bn knh ca tng ng trn ni trn.
2/ Mt ng thng d c phng trnh y = mx ct vng trn (C) ti hai im O v M, ng
thi ct ng trn (C ') ti hai im O v M'. Tnh ta im M v M' theo m.

1 1
1 1
2
2
v I2 ; , R2 =
.
S: 1/ I1 ; , R1 =

2 2

2
2
2 2

m 1 m (m 1)

M
;
m2 + 1 m2 1

2/
.

1 m m (1 m)

M '
;
m2 + 1 m2 1

Bi 354. i hc Bch Khoa Tp. H Ch Minh nm 1978


Trong mt phng vi h trc ta Descarter vung gc Oxy, cho ng trn (C) c phng
trnh: x2 + y2 + 2x 4y = 0 . Vit phng trnh tip tuyn (C) xut pht t im M (4;7) .
S: d1 : y = 2x 1 d2 : y =

x
+5.
2

Bi 355. i hc Kinh T Tp. H Ch Minh nm 1978


Trong mt phng vi h trc ta Descarter vung gc Oxy, cho ng trn (C) c phng
trnh: x2 + y2 + 2x 4y = 0 . ng trn ny ct Ox ti A v O, ct Oy ti B v O.
1/ Vit phng trnh cc tip tuyn vi (C) ti O, A, B.
2/ Vit phng trnh cc tip tuyn vi (C) xut pht t M (4;7) .

tO : x 2y = 0
S: 1/ tA : x + 2y + 2 = 0 .

tB : x + 2y 8 = 0

2/ d1 : y = 2x 1 d2 : y =

x
+5.
2

Bi 356. i hc Bch Khoa H Ni nm 1978


Trong mt phng vi h trc ta vung gc Oxy, hy vit phng trnh ng trn qua hai
1 3 3 1

, B ; v tip xc vi ng thng: : x 3y 2 = 0 .
im: A ;
2 2 2 2
S: (C1 ) : x 2 + y2 = 1

Page - 158 -

(C ) : (x 2
2

) (
2

3 + 2 + y 2 3 1 = 25 .

DeThiThuDaiHoc.com
"All the flower of tomorrow are in the seeks of today"

cng hc tp mn Ton 10 Tp IIwww.MATHVN.com

Ths. L Vn on

Bi 357. i hc Nng Nghip Tng Hp S Phm K Thut nm 1980


Trong mt phng vi h trc ta vung gc Oxy, hy vit phng trnh ng trn tm
A (2; 1), tip xc vi ng thng 4y 3x = 5 v xc nh tip im vi ng .
29 53
2
2
S: (C) : (x 2) + (y + 1) = 9 v H ; .
25 25

Bi 358. i hc Kinh TS PhmNng NghipBch KhoaTng HpYNhaDc nm 1982


Trong mt phng vi h trc ta Descarter vung gc Oxy, cho hai im A (8; 0), B (0;6) .
1/ Vit phng trnh ng trn (C) ngoi tip OAB.
2/ Vit phng trnh cc tip tuyn vi ng trn (C) v qua S (14; 8) .
3/ Vit phng trnh ng trn ni tip OAB.
S: 1/ (C) : x2 + y2 8x 6y = 0 .

2/ T1 : y 8 = 0 T2 : 4x 3y + 4 = 0 .

3/ (C ') : (x 2) + (y 2) = 4 .
2

Bi 359. i hc Bch Khoa Tp. H Ch Minh nm 1986


Trong mt phng vi h trc ta Descarter vung gc Oxy, lp phng trnh ng trn i
qua cc im A (1;1) v B (1; 3) c tm nm trn ng thng c phng trnh:
2x y + 1 = 0 .
2
2

4
5
65

S: x + + y + =
.

3
3
9

Bi 360. i hc Nng Lm Tp. H Ch Minh 1991


Trong mt phng vi h trc ta Descarter vung gc Oxy, cho h ng cong (Cm ) c
phng trnh: (Cm ) : x2 + y2 (m 2) x + 2my 1 = 0 .
1/ Tm tp hp tm cc ng trn (Cm ) .
2/ Chng t rng cc ng trn ny u qua hai im c nh khi m thay i.
3/ Cho m = 2 v im A (0; 1) . Vit phng trnh cc tip tuyn ca ng trn (C2 ) v
t im A.
S: 1/ Qu tch tm I l d : 2x + y + 2 = 0 .
3/ C hai tip tuyn: (T1 ) : y + 1 = 0

2 1
2/ Hai im c nh A (2; 1), B ; .
5 5

(T ) : 12x 5y = 0 .
2

Bi 361. i hc Kin Trc Tp. H Ch Minh nm 1995


Trong mt phng vi h trc ta Descarter vung gc Oxy, cho hai im A (a; 0) v B (0;a)
vi a > 0 . Cho tham s thc m 0 v m a .
1/ Hy vit phng trnh ng trn (C) tip xc vi trc honh ti A v c tm tha y1 = m
(y1 l tung ca I).
2/ (C) ct ng thng AB ti A v P. Hy tm ta ca im P. T hy vit phng
trnh ng trn (E) i qua P v tip xc vi trc tung ti B.
"Cn c b thng minh"

DeThiThuDaiHoc.com

Page - 159 -

www.MATHVN.com

Ths. L Vn on

Phn hnh hc

3/ (C) v (E) ct nhau ti P v Q. Chng minh rng ng thng PQ lun lun i qua mt
im c nh.
S: 1/ (C) : (x a ) + (y m ) = m 2 .
2

2/ P (a m; m ) v (E) : (x a + m) + (y a ) = (a m) .
2

3/ PQ lun i qua im c nh O (0; 0) .


Bi 362. i hc Cng on nm 1995
Trong mt phng vi h trc ta Descarter vung gc Oxy, cho h ng thng ph thuc
tham s : (x 1) cos + (y 1) sin 1 = 0 .
1/ Tm tp hp cc im ca mt phng khng thuc bt c ng thng no ca h.
2/ Chng minh rng mi ng thng ca h u tip xc vi mt ng trn c nh.
S: 1/ Tp hp cc im m (C ) khng th i qua l min trong ca hnh trn (C) c
phng trnh: (C) : (x 1) + (y 1) < 1 .
2

2/ Tip xc vi ng trn (C) : (x 1) + (y 1) = 1 .


2

Bi 363. i hc Dn Lp Duy Tn nm 1995


Trong mt phng vi h trc ta Descarter vung gc Oxy, cho h ng trn (Cm ) c
phng trnh l (Cm ) : x2 2mx + y2 + 2 (m + 1) y 12 = 0 .
1/ Tm qu tch tm ca h ng trn trn.
2/ Vi gi tr no ca m th bn knh ca h ng trn cho l nh nht ?
3/ Vi m = 2 .
a/ V ng trn (C2 ) .
b/ Cho ng thng d c phng trnh l d : 3x 3y + 12 = 0 . Tm khong cch ngn
nht gia d v (C2 ) .
S: 1/ Qu tch tm l d : x + y + 1 = 0 .

2/ R min =

5 2
1
m= .
2
2

3/ d d; (C2 ) = 1 .

Bi 364. i hc Dn Lp Hng Vng nm 1995


Trong mt phng vi h trc ta Descarter vung gc Oxy, cho hai ng trn c phng
trnh ln lt l (C) : x2 + y2 = 1 v (C ') : x 2 + y2 6x + 6y + 17 = 0 .
1/ Tm tm v bn knh ca (C) v (C ') .
2/ Tm phng trnh ng thng tip xc vi c hai ng trn trn.
S: 1/ (C) c tm O (0; 0), R1 = 1 v (C ') c tm I (3; 3), R 2 = 1 .
2/ 1,2 : x y 2 = 0 .

Bi 365. i hc Y Dc Tp. H Ch Minh nm 1995

Page - 160 -

DeThiThuDaiHoc.com
"All the flower of tomorrow are in the seeks of today"

cng hc tp mn Ton 10 Tp IIwww.MATHVN.com

Ths. L Vn on

Trong mt phng vi h trc ta Descarter vung gc Oxy, xt hai im A (a; 0) v B (0; b)


vi ab 0 . Gi (C) l ng trn tip xc vi Ox ti A v c tm C vi tung yC = m,
trong m l tham s , m 0 v m

a 2 + b2
.
ab

1/ ng thng AB ct ng trn (C) ti giao im th hai l P. Hy xc nh ta im P.


2/ Xc nh tm K ca ng trn (K) tip xc vi trc Oy ti B v i qua P.
3/ Cc ng trn (C) v (K) ct nhau ti P v Q. Chng t khi m thay i ng thn PQ
lun i qua mt im c nh.

a2 + b2 2bm
a 3 ab2 ba 2 b3
2abm 2b2m
. 2/ K
. 3/ Mo

;
;
b
S: 1/ P a 2

a2 + b2 ; a 2 + b2 .

2a
a + b2 a 2 + b2

Bi 366. i hc Quc Gia H Ni nm 1995


Trong mt phng vi h trc ta Descarter vung gc Oxy, vit phng trnh ng trn
tip xc vi Ox v ct Oy ti im (0;1) . Tm qu tch tm ca ng trn .

S: (C) : x2 + y2 2ax a 2 + 1 y + a 2 = 0 v qu tch l Parabol (P) : y =

x2 + 1
.
2

Bi 367. i hc Ngoi Thng nm 1996


Trong mt phng vi h trc ta Descarter vung gc Oxy, lp phng trnh ng trn
ngoi tip tam gic c ba nh l A (1;1), B (1;2), C (0; 1) .
S: (C) : x2 + y2 + x y 2 = 0 .
Bi 368. i hc Quc Gia H Ni nm 1996
Trong mt phng vi h trc ta Descarter vung gc Oxy, lp phng trnh ng trn
x 2
ngoi tip tam gic c ba cnh nm trn ba ng thng: d1 : y = , d2 : y = x + 2,
5 5
d3 : y = 8 x .
S: (C) : (x 2) + y2 = 26 .
2

Bi 369. i hc Y Dc Tp. H Ch Minh nm 1997 H cha phn ban


Trong mt phng vi h trc ta vung gc Oxy, cho h ng trn c phng trnh:
(Cm ) : x2 + y2 2mx 2 (1 m) y + 2m2 2m 3 = 0 . Cho m = 2 v im A (0; 3) . Hy
vit phng trnh cc tip tuyn ca (C2 ) k t A.
S: d1 Oy : x = 0 d2 : 3x + 4y 12 = 0 .
Bi 370. i hc Ngoi Thng Tp. H Ch Minh khi A, D nm 1997
Trong mt phng vi h trc ta Descarter vung gc Oxy, cho ng trn:
(C) : x2 + y2 + 2x 4y 4 = 0 v im A (3;5) . Hy tm phng trnh cc tip tuyn k t
A n ng trn. Gi s cc tip tuyn tip xc vi ng trn ti M v N, tm d di MN .
S: d1 : y 5 = 0 d2 : 24x 7y 37 = 0 .
Bi 371. i hc Ti Chnh K Ton H Ni nm 1997
"Cn c b thng minh"

DeThiThuDaiHoc.com

Page - 161 -

www.MATHVN.com

Ths. L Vn on

Phn hnh hc

Trong mt phng vi h trc ta Descarter vung gc Oxy, cho ng trn:


(C) : x2 + y2 2x 6y + 6 = 0 v im M (2; 4) .
1/ Vit phng trnh ng thng i qua M v ct ng trn ti hai im A, B sao cho M l
trung im ca on AB.
2/ Vit phng trnh cc tip tuyn ca ng trn c h s gc k = 1 .
2/ 1 : x + y + 2 2 + 4 = 0 2 : x + y 2 2 4 = 0 .

S: 1/ d : x + y 6 = 0 .

Bi 372. i hc Vn Lang khi A nm 1997


Trong mt phng vi h trc ta Descarter vung gc Oxy, cho h ng cong
(Cm ) : x2 + y2 (m 2) x + 2my 1 = 0 .
1/ Tm tp hp tm cc ng trn (Cm ) .
2/ Tm cc im c nh m (Cm ) lun i qua khi m thay i.
m 2

2 1
S: 1/ I
; m tp hp tm l d : 2x + y + 2 = 0 . 2/ A (2; 1), B ; .

2
5 5
Bi 373. i hc Y Dc Tp. H Ch Minh nm 1997 (cha phn ban)

Trong mt phng vi h trc ta Descarter vung gc Oxy, cho h ng trn:


(Cm ) : x2 + y2 2mx 2 (1 m) y + 2m2 2m 3 = 0 . Tm qu tch tm ca ng trn.
Cho m = 2 v im A (0; 3) . Vit cc phng trnh tip tuyn ca (C2 ) k t A.
S: Qu tch l d : y = x + 1 . Cc tip tuyn: x = 0 3x + 4y 12 = 0 .
Bi 374. i hc An Ninh ( 1) khi A nm 1997
Trong mt phng vi h trc ta Descarter vung gc Oxy, cho hai ng trn:
(C1 ) : x2 + y2 2x + 4y 4 = 0 v (C2 ) : x2 + y2 + 2x 2y 14 = 0 .
1/ Xc nh cc giao im ca (C1 ) v (C2 ) .
2/ Vit phng trnh ng trn i qua hai giao im v A (0;1) .

7 18 3 17 12 3 7 + 18 3 17 + 12 3
B
;
,
C
;
S: 1/

.
13
13
13
13

2/ (C) : 16x 2 + 16y2 28x + 58y 74 = 0 .


Bi 375. i hc Bch Khoa H Ni i hc Lut H Ni nm 1997
Trong mt phng vi h trc ta Descarter vung gc Oxy, hy vit phng trnh ng
trn i qua im A (2; 1) v tip xc vi hai trc ta Ox v Oy.
S: (C) : (x 1) + (y + 1) = 1
2

(C) : (x 5)

+ (y + 5) = 25 .
2

Bi 376. i hc Thy Sn nm 1997


Trong mt phng vi h trc ta Descarter vung gc Oxy, cho A (1; 3), B (1;1), M (2; 4)
v ng thng d : y = 2x .
1/ Tm ta im C d sao cho ABC cn.
2/ Vit phng trnh ng trn (ABM) .

Page - 162 -

DeThiThuDaiHoc.com
"All the flower of tomorrow are in the seeks of today"

cng hc tp mn Ton 10 Tp IIwww.MATHVN.com

Ths. L Vn on

5 15 10 2 15

C 3 39 ; 6 2 39 .
;
S: 1/ C 4 im tha yu cu: C

5
5
5
5

2/ (CABM ) : x 2 + y2

3
11
x y +5 = 0.
2
2

Bi 377. i hc Thi Nguyn khi A, B nm 1997


Trong mt phng vi h trc ta Descarter vung gc Oxy, cho hc ng trn
(C) : x2 + y2 2my = 0 v h ng thng d : y = mx + m .
1/ Chng minh rng d lun i qua tm ca ng trn (C) v h ng thng d qua mt im
c nh.
2/ Tm qu tch giao im ca h ng trn (C) v h ng thng d.
S: 1/ im c nh M o (1; 0) .

2/ Qu tch im l ng cong y =
2

x 2 (1 x )
1+ x

Bi 378. Cao ng Kinh T i Ngoi nm 1998 h phn ban


Trong mt phng vi h trc ta Descarter vung gc Oxy, cho h ng cong c phng
trnh: (Cm ) : x2 + y2 + 4mx 2my + 2m + 3 = 0 .
1/ nh m (Cm ) l ng trn.
2/ Tm tp hp tm cc ng trn (Cm ) .

5
S: 1/ m < m > 1 .
3

x + 2y = 0
2/ L mt phn ng thng d :
.
y < 5 y > 1
3

Bi 379. i hc Thy Sn nm 1998


Trong mt phng vi h trc ta Descarter vung gc Oxy, hy lp phng trnh ng trn
qua A (3; 3), B (1;1), C (5;1) .
S: (ABC) : x2 + y2 6x 2y + 6 = 0 .
Bi 380. i hc Nng Nghip I khi A nm 1998
Trong mt phng vi h trc ta Descarter vung gc Oxy, cho A (3;1), B (0;7), C (5;2) .
1/ Chng minh ABC vung v tnh din tch ca n.
2/ Gi s M l im chy trn ng trn ngoi tip ABC. Chng minh khi trng tm G
ca MBC chy trn mt ng trn. Vit phng trnh chnh tc ng trn .
S: 1/ SABC

15
= (vdt) .
2

2
2

5
9
25

2/ (C) : x + y =
.

2
2
18

Bi 381. i hc Thi Nguyn khi D nm 1998


Trong mt phng vi h trc ta Descarter vung gc Oxy, cho A (0;6), B (4; 0), C (3; 0) .
ng thng () : x = m (m thay i) ct AB v AC ti M v N. Gi hnh chiu ca M, N
trn trc honh l P v Q. Gi H v E ln lt l trung im AO v BC. Gi I l tm hnh ch
nht MNQP.
"Cn c b thng minh"

DeThiThuDaiHoc.com

Page - 163 -

Ths. L Vn on

www.MATHVN.com

Phn hnh hc

1/ Chng minh ba im H, E, I thng hng.


2/ Tm tm ng trn (ABC) .
3/ Tm im T AC sao cho OT BT .
7
18 6
S: 2/ J ; 4 .
3/ T (2;2) T ; .
5
2
5
Bi 382. i hc M Thut Cng Nghip nm 1998
Trong mt phng vi h trc ta Descarter vung gc Oxy, cho A (8; 0), B (0;6) .
1/ Vit phng trnh ng trn ngoi tip OAB.
2/ Vit phng trnh ng trn ni tip OAB.

S: 1/ (C) (ABC) : x2 + y2 8x 6y = 0 .

2/ (C ') : (x 2) + (y 2) = 4 .
2

Bi 383. i hc An Ninh nm 1998


Trong mt phng vi h trc ta Descarter vung gc Oxy, cho A (4; 0), B (0; 3) .
1/ Vit phng trnh ng trn ngoi tip OAB.
2/ Vit phng trnh ng trn ni tip OABC.
2

2
3
25
S: 1/ (C) : (x 2) + x =
.

2
4

2/ (C ') : (x 1) + (y 1) = 1 .
2

Bi 384. i hc Kin Trc H Ni nm 1998


Trong mt phng vi h trc ta Descarter vung gc Oxy, cho ng trn c phng trnh:
(C) : x2 + y2 + 2x 4y 20 = 0 v im A (3; 0) . Vit phng trnh ng thng cha dy
cung ca ng trn qua A khi:
1/ Dy cung c di l ln nht.
2/ Dy cung c di l nh nht.
S: 1/ : x + 2y 3 = 0 .
2/ d : 2x y 6 = 0 .
Bi 385. i hc K Thut Cng Ngh Tp. H Ch Minh khi A nm 1998
Trong mt phng vi h trc ta Descarter vung gc Oxy:
1/ Cho im A (3; 2) v ng trn (C) : x2 + y2 4x 2y = 0 . Vit phng trnh nhng
tip tuyn vi (C) v t A v tm ta cc tip im.
2/ Lp phng trnh ng trn tm I (4; 3) v tip xc vi ng thng d : x + 2y 5 = 0 .

d1 : y = 2x 8
S: 1/
d : y = 1 x 1
2
2
2

v M (4; 0), N (1; 1) .

2/ (C ') : (x 4) + (y 3) = 5 .
2

Bi 386. Hc Vin Bu Chnh Vin Thng H Trung Cp nm 1998


Trong mt phng vi h trc ta Descarter vung gc Oxy, cho ng trn
(C) : x2 + y2 + 4x 4y 1 = 0 v im M (0; 1) . Vit phng trnh tip tuyn vi ng
trn (C) xut pht t M.
S: d1 : y + 1 = 0 d2 : 12x 5y 5 = 0 .
Page - 164 -

DeThiThuDaiHoc.com
"All the flower of tomorrow are in the seeks of today"

cng hc tp mn Ton 10 Tp IIwww.MATHVN.com

Ths. L Vn on

Bi 387. Cao ng Kinh T i Ngoi nm 1998 H phn ban


Trong mt phng vi h trc ta Descarter vung gc Oxy, cho ng trn
(C) : x2 + y2 6x + 2y + 5 = 0 . Tm phng trnh tip tuyn vi (C) c h s gc l 2 .
S: d1 : 2x + y = 0 d2 : 2x + y 10 = 0 .
Bi 388. i hc Xy Dng H Ni nm 1998
Trong mt phng vi h trc ta Descarter vung gc Oxy, cho hai ng trn tm A (1; 0),
bn knh R A = 4 v tm B (1; 0), bn knh R B = 2 . Tm tp hp tm I (x; y) ca cc
ng trn tip xc vi c hai ng trn trn.
TH2. I (E) :

S: TH1. I (x ' Ox) AB : y = 0 .

x 2 y2
+
= 1.
9
8

Bi 389. i hc Kin Trc H Ni nm 1998


Trong mt phng vi h trc ta Descarter vung gc Oxy, cho hai ng thng:
d1 : 3x + 4y + 5 = 0, d2 : 4x 3y 5 = 0 . Vit phng trnh ng trn c tm nm trn
ng thng : x 6y 10 = 0 v tip xc vi hai ng thng d1 v d2.
2
2

10
70
49

S: (C1 ) : (x 10) + y = 49 (C2 ) : x + y + =


.
43
43
1849

Bi 390. i hc Lt khi B nm 1999


Trong mt phng vi h trc ta Descarter vung gc Oxy, cho ng cong (Cm ) c
phng trnh: x 2 + y2 2 (a + 1) x 4 (a 1) y + 5 a = 0 .
1/ Tm iu kin ca a (Cm ) l ng trn.
2/ Tm a ng trn (Cm ) tip xc vi ng thng y = x .
S: 1/ a 0 a 1 .

2/ a =

2 85

Bi 391. i hc Ngoi Thng khi D nm 1999

x2
+ y2 = 1 .
9
1/ Chng minh rng parabol v elip ct nhau ti bn im phn bit A, B, C, D.
2/ Chng minh rng bn im A, B, C, D cng nm trn mt ng trn. Xc nh tm v bn
knh ca ng trn .
Cho Parabol y = x2 2x v elp

S: 1/ PTHG: 9x 4 36x 3 + 37x 2 9 = 0 .

8 4
161
2/ Tm I ; , R =
.
9
9 9

Bi 392. i hc Ngoi Thng khi A nm 1999


Trong mt phng vi h trc ta Descarter vung gc Oxy, cho h vng trn
(Cm ) : x2 + y2 2mx 2 (m + 1) y + 2m 1 = 0 .
1/ Chng minh rng khi m thay i, h vng trn lun lun i qua hai im c nh.
2/ Chng minh rng vi mi m, h vng trn lun ct trc tung ti hai im phn bit.
S: 1/ A (1;2), B (1; 0) .
2/ Hai im A v B lun nm v hai pha trc tung pcm.
"Cn c b thng minh"

DeThiThuDaiHoc.com

Page - 165 -

www.MATHVN.com

Ths. L Vn on

Phn hnh hc

Bi 393. i hc Ngoi Ng H Ni khi D nm 1999


Trong mt phng vi h trc ta vung gc Oxy, lp phng trnh ng thng qua gc ta
v ct ng trn (C) : (x 1) + (y + 3) = 25 thnh mt dy cung c di = 8 .
2

S: d : y = 0 d : y =

3
x.
4

Bi 394. i hc Dn Lp K Thut Cng Ngh khi A, B nm 1999


Trong mt phng vi h trc ta Descarter vung gc Oxy, cho ng trn
(C) : x2 + y2 = 1 v ng thng d : Ax + By + 1 = 0 .
1/ Tm iu kin ca A, B d tip xc vi (C) .
2/ Gi s d tip xc vi (C) v M, N l hai im thuc (C) sao cho x M = 1, y N = 1 . Hy
tnh A, B tng khong cch t M v N n d l nh nht.

A = cos x S
S: 1/ A 2 + B2 = 1 . 2/ t
min
B = sin x

A = 2
7
= 2 2 khi x =

.
4
B = 2

Bi 395. i hc Y Dc Tp. H Ch Minh nm 1999


Trong mt phng vi h trc ta Descarter vung gc Oxy, cho phng trnh cc ng
trn: (C) : x2 + y2 1 = 0 v (Cm ) : x2 + y2 2 (m + 1) x + 4my 5 = 0 .
1/ Chng minh rng: c hai ng trn thuc h (Cm ) tip xc vi (C) .
2/ Vit phng trnh cc tip tuyn chung ca hai ng trn va tm c cu 1/.
(C ) : Tm I (0;2),
R1 = 3
1
1

8 6
. 2/ d1,2 : 2x + y 2 3 5 = 0 .
S: 1/
C3/5 : Tm I2 ; , R 2 = 3
5 5

Bi 396. i hc Quc Gia H Ni khi A nm 1999 H cha phn ban


Trong mt phng vi h trc ta Descarter vung gc Oxy, cho h ng trn c phng
trnh: x2 + y2 2 (m + 1) x 2 (m + 2) y + 6m + 7 = 0 vi m l tham s.
1/ Tm qu tch tm cc ng trn c h .
2/ Xc nh ta tm ca ng trn thuc h cho m tip xc vi trc Oy.

I m (m + 1; m + 2)
S: 1/
Qu tch tm l mt phn ca ng thng : y = x + 1 vi
R = 2 m2 1

x < 0

x > 2 .

2/ Tm I3 (4;5) tip xc vi trc tung Oy.

Bi 397. i hc Quc Gia Tp. H Ch Minh t 1 khi A nm 1999


Trong mt phng vi h trc ta Descarter vung gc Oxy, cho hai ng trn c phng
trnh: (C1 ) : x2 + y2 4x + 2y 4 = 0 v (C2 ) : x2 + y2 10x 6y + 30 = 0 c tm ln
lt l I v J.
1/ Chng minh rng (C1 ) tip xc ngoi vi (C2 ) v tm ta tip im H.
Page - 166 -

DeThiThuDaiHoc.com
"All the flower of tomorrow are in the seeks of today"

cng hc tp mn Ton 10 Tp IIwww.MATHVN.com

Ths. L Vn on

2/ Gi d l mt tip tuyn chung khng i qua H ca (C1 ) v (C2 ) . Tm ta giao im K


ca d v ng thng IJ. Vit phng trnh ng trn (C) i qua K v tip xc vi hai
ng trn (C1 ) v (C2 ) ti H.

19 7
HI
3
S: 1/  = H ; .
5 5
2
HJ

2
2

37
31

2/ K (11;11) v (C) : x + y = 36 .

5
5

Bi 398. i hc Ngoi Ng H Ni nm 2000


Trong mt phng cho ba im: A (1;7), B (4; 3), C (4;1) . Hy vit phng trnh ng
trn ni tip ABC.
S: (C) : (x + 1) + (y 2) = 5 .
2

Bi 399. i hc Dn Lp Hng Vng Ban B nm 2000


Trong mt phng vi h trc ta Descarter vung gc Oxy, cho hai im A (3; 0), B (0; 4) .
1/ Hy vit phng trnh ng cao ca OAB h t nh O v ng phn gic ca gc
.
OAB
2/ Hy vit phng trnh ng trn ngoi tip v ng trn ni tip ca OAB.
2

3
25

+ (y 2) =
C
:
x
( )

2
4 .
S: 1/ OH : 3x 4y = 0, At : x + 2y 3 = 0 . 2/

2
2
C ' : x 1) + (y 1) = 1
( ) (
Bi 400. i hc Dn Lp Hng Vng ban C nm 2000
Trong mt phng vi h trc ta Descarter vung gc Oxy, cho h ng trn
(Cm ) : x2 y2 2 (1 + m) x + 2m2 y + m4 = 0, (m 1) .
1/ Hy tm qu tch ca tm h ng trn ny khi tham s thc m bin thin.
2/ Hy chng t rng cc ng trn trong h lun lun tip xc vi mt ng thng c
nh. Hy tm ng thng c nh ?

S: 1/ Qu tch tm I 1 m; m 2 l ng Parabol y = x2 2x + 1, x 0 . 2/ Oy.


Bi 401. i hc Hng Hi nm 2000
Trong mt phng vi h trc ta Descarter vung gc Oxy, cho ng trn
(C) : x2 + y2 6x + 2y 6 = 0 v im A (1; 3) .
1/ Xc nh tm I v bn knh R ca ng trn (C) v chng t A nm ngoi (C) .
2/ Lp phng trnh tip tuyn ca (C) xut pht t A.
S: 1/ I (3; 1) ; R = 2; IA = 20 > 2 .

3
15
2/ x = 1 y = x +
.
4
4

Bi 402. i hc Quc Gia Tp. H Ch Minh t 2 nm 2000


Trong mt phng vi h trc ta Descarter vung gc Oxy, cho h ng cong (Cm ) c
phng trnh: x2 + y2 + 2 (m 1) x 2 (m 2) y + m2 8m + 13 = 0 .

"Cn c b thng minh"

DeThiThuDaiHoc.com

Page - 167 -

www.MATHVN.com

Ths. L Vn on

Phn hnh hc

1/ Tm tt c cc gi tr ca m (Cm ) l ng trn. Tm qu tch tm I ca ng trn (Cm )


khi m thay i.
2/ Cho m = 4 . Vit phng trnh cc tip tuyn k t im A (1; 5) n ng trn (C4 ) .

x > 5
S: 1/ m < 4 m > 2 v qu tch tm I l hai tia nm trn d : y = x 1 vi
.

x < 1

2/ d1 : 7x + 24y 127 = 0 d2 : x 1 = 0 .
Bi 403. i hc Thy Sn Nha Trang nm 2000
Trong mt phng vi h trc ta Descarter vung gc Oxy, cho h ng trn (Cm ) c
phng trnh: x 2 + y2 (m 2) x + 2my 1 = 0 .
1/ Chng minh rng h ng trn (Cm ) i qua hai im c nh khi m thay i.
2/ Cho m = 2 v im A (0; 1) . Vit phng trnh cc tip tuyn ca (C2 ) k t A.
2 1
S: 1/ M1 (2; 1), M2 ; .
5 5

2/ d1 : y + 1 = 0 d2 : 12x 5y 5 = 0 .

Bi 404. i hc Dc H Ni nm 2000
Trong mt phng vi h trc ta Descarter vung gc Oxy, cho cc ng trn:
(C) : x2 + y2 = 1 v (Cm ) : x2 + y2 2 (m + 1) x + 4my = 5 .

( ) ( ) tip xc vi ng trn (C) ng vi hai

1/ Chng minh rng c hai ng trn Cm , Cm


1

gi tr m1 v m2 ca m.

( ) ( )

2/ Xc nh phng trnh cc ng thng tip xc vi c hai ng trn Cm , Cm .


1

S: 1/ m = 1 m =

d : 2x + y + 3 5 2 = 0

2/ 1
.
d2 : 2x + y 3 5 2 = 0

3
.
5

Bi 405. i hc Ty Nguyn khi A, B nm 2000


Trong mt phng vi h trc ta vung gc Oxy, cho hai ng trn (C1 ) v (C2 ) c
phng trnh: (C1 ) : x2 + y2 2x 9y 2 = 0 v (C2 ) : x 2 + y2 8x 9y + 16 = 0 .
1/ Chng minh rng hai ng trn (C1 ) v (C2 ) tip xc vi nhau.
2/ Vit phng trnh cc tip tuyn chung ca (C1 ) v (C2 ) .
S: 1/ Tip xc ngoi ti T (3;1) .

d1 : x 3 = 0

2/ d2 : x 2 2y + 2 2 7 = 0 .

d : x + 2 2y 2 2 7 = 0
3

Bi 406. i hc Dn Lp Hng Vng Ban C nm 2000


Trong mt phng vi h trc ta Descarter vung gc Oxy, cho h ng trn:
(Cm ) : x2 + y2 2 (1 m) x 2my + 4 = 0, (m 1) .
1/ Hy tm qu tch tm h ng trn ny khi tham s thc m bin thin.
Page - 168 -

DeThiThuDaiHoc.com
"All the flower of tomorrow are in the seeks of today"

cng hc tp mn Ton 10 Tp IIwww.MATHVN.com

Ths. L Vn on

2/ Hy chng t rng cc ng trn trong h lun lun tip xc vi mt ng thng c nh.


Hy tm ng thng c nh .

S: 1/ Qu tch tm Im l parabol (P) : y = x2 2x + 1 tr i im M (0;1) (P) .


2/ (Cm ) lun tip xc vi trc tung Oy : x = 0 .

Bi 407. i hc Cn Th nm 2000
Trong mt phng vi h trc ta Descarter vung gc Oxy, cho h ng trn c phng
trnh: (Cm ) : x2 + y2 (2m + 5) x + (4m 1) y 2m + 4 = 0 .
1/ Chng minh rng (Cm ) lun i qua hai im c nh m  .
2/ Xc nh tt c cc gi tr ca m (Cm ) tip xc vi trc tung.
S: 1/ A (1;1), B (3;2) .

2/ m =

15
.
4

Bi 408. i hc Ngoi Thng c s II khi A nm 2000


Cho Parabol y = x2 v ng thng y = mx + 1 . Chng minh rng khi m thay i, ng
thng lun lun ct parabol ti hai im phn bit A v B. Hy tm qu tch tm vng trn
ngai tip OAB khi m thay i vi O l gc ta .
S: PTHG : x 2 mx 1 = 0 = m 2 + 4 > 0 . Tp hp tm I l (P) : y = 2x2 + 1 .
Bi 409. i hc Qun S B Quc Phng nm 2001
4 2
Trong mt phng vi h trc ta vung gc Oxy, cho A (2; 4), B ; , C (6; 0) . Tm tm
3 3
v bn knh ng trn ni tip ABC.
Bi 410. i hc S Phm K Thut Tp. H Ch Minh nm 2001
Trong mt phng vi h trc ta Descarter vung gc Oxy, cho ABC bit A (1;2),

B (2; 0), C (3;1) .

1/ Xc nh tm ng trn ngoi tip ABC.


2/ Tm im M trn ng thng BC sao cho din tch ABM bng

1
din tch ABC.
3

11 13
1 1
11 1
S: 1/ I ; .
2/ M ; M ; .
14 14
3 3
3
3
Bi 411. i hc M Bn Cng nm 2001
Trong Trong mt phng vi h trc ta vung gc Oxy, cho A (1;2), B (0;1), C (2;1) .

1/ Vit phng trnh ng thng AB.


2/ Vit phng trnh ng cao CH ca ABC.
3/ Vit phng trnh ng trn ngoi tip ABC.
Bi 412. i hc Dn Lp Vn Lang nm 2001
3
Trong mt phng vi h trc ta Descarter vung gc Oxy, cho M 2; .
2

1/ Vit phng trnh ng trn (C) ng knh OM.


2/ Vit phng trnh ng thng d qua M v ct hai na trc dng Ox, Oy ti A v B sao
cho din tch OAB bng 6.
3/ Tm ta tm I ca ng trn ni tip OAB v vit phng trnh ng trn .
Bi 413. i hc Ti Chnh K Ton H Ni nm 2001
"Cn c b thng minh"

DeThiThuDaiHoc.com

Page - 169 -

www.MATHVN.com

Ths. L Vn on

Phn hnh hc

Trong mt phng vi h trc ta Descarter vung gc Oxy, cho h ng trn


(Cm ) : x2 + y2 + 2mx 6y + 4 m = 0 .
1/ Chng minh rng (Cm ) l ng trn vi mi m. Hy tm tp hp tm cc ng trn

(C ) khi m thay i.
m

2/ Vi m = 4 . Hy vit phng trnh ng thng vung gc vi ng thng


: 3x 4y + 10 = 0 v ct ng trn ti hai im A v B sao cho AB = 6 .
S: 1/ I (m; 3) Qu tch tm l ng y = 3 .

d : 4x + 3y + 27 = 0
2/ 1
.
d2 : 4x + 3y 13 = 0

Bi 414. i hc Y H Ni nm 2001
Cho ng trn c phng trnh: x2 + y2 + 8x 4y 5 = 0 . Vit phng trnh tip tuyn
ca ng trn i qua im A (0; 1) .
S: 4x 3y 3 = 0 .
Bi 415. i hc Y Thi Bnh H di hn
Cho M (4; 3) l mt im trn Parabol (P) : y = x2 4x + 3 . Vit phng trnh ng trn
tip xc vi vi (P) ti M v c tm nm trn trc honh.
S: (C) : (x 16) + y2 = 153 .
2

Bi 416. i hc Th Dc Th Thao I nm 2001


Trong mt phng vi h trc ta Oxy, cho ng trn (C) : x2 + y2 2x + 4y + 4 = 0 .
Qua im A (1; 0), hy vit phng trnh hai tip tuyn vi ng trn cho v tnh gc to
bi hai tip tuyn .
Bi 417. i hc M a Cht khi A nm 2001
Trong mt phng vi h trc ta trc chun Oxy, hy vit phng trnh ng trn ngoi
tip tam gic ABC, bit phng trnh ng thng AB : y x 2 = 0, phng trnh ng
thng BC : 5y x + 2 = 0 v phng trnh ng thng AC l y + x 8 = 0 .
S: Nhn thy AB AC Tm l trung im BC (C) : (x 2) + y 2 = 26 .
2

Bi 418. i hc Quc Gia Tp. H Ch Minh khi A nm 2001


Trong mt phng vi h to Oxy, cho ng thng d : 2x + my + 1 2 = 0 v hai
ng trn: (C1 ) : x2 + y2 2x + 4y 4 = 0 v (C2 ) : x2 + y2 + 4x 4y 56 = 0 .
1/ Gi I l tm ng trn (C1 ) . Tm m sao cho d ct (C1 ) ti hai im phn bit A v B. Vi
gi tr no ca m th din tch IAB ln nht v tnh gi tr ln nht .
2/ Chng minh rng (C1 ) tip xc vi (C2 ) . Vit phng trnh tng qut ca tt c cc tip
tuyn chung ca (C1 ) v (C2 ) .
S: 1/ m , (SIAB )

max

9
m = 4 .
2

2/ Tip xc trong v : 3x 4y 26 = 0 .

Bi 419. Hc Vin Ngn Hng Tp. H Ch Minh khi D nm 2001


Page - 170 -

DeThiThuDaiHoc.com
"All the flower of tomorrow are in the seeks of today"

cng hc tp mn Ton 10 Tp IIwww.MATHVN.com

Ths. L Vn on

Trong mt phng vi h trc ta Descarter vung gc Oxy, cho ng trn c phng trnh
+ (y b) = R 2 . Chng minh rng tip tuyn ca ng trn ti im (x o ; yo )

(C) : (x a)

c phng trnh: (x o a )(x a ) + (yo b)(y b) = R 2 .

  
HD: Gi E (a; b) l tm, F (x o ; y o ) (C) v M (x; y) tt FM EF FM.EF = 0 .

Bi 420. i hc Kin Trc Tp. H Ch Minh nm 2001


1/ cho ng trn c phng trnh (C) : (x a ) + (y b) = R 2 . Chng minh rng tip
2

tuyn ca ng trn ti im (x o ; yo ) c phng trnh:

(x

a )(x a ) + (yo b)(y b) = R 2 .

2/ Chng minh rng tch cc khong t mt im bt k ca Hyperbol:


tim cn ca n l mt hng s khng i.
S: 1/ Xem Hc Vin Ngn Hng 2001.

x 2 y2

= 1 n cc
a 2 b2

2/ d (Mo ; 1 ).d (Mo ; 2 ) =

a2b2
.
a2 + b2

Bi 421. D b 1 i hc khi A nm 2002


Trong mt phng vi h ta Oxy , cho ng thng d : x y + 1 = 0 v ng trn

(C) : x

+ y2 + 2x 4y = 0 . Tm to im M thuc ng thng d m qua ta k c


hai ng thng tip xc vi (C) ti A v B sao cho AMB = 600 .
S: M1 (3;4), M2 (3; 2) .
Bi 422. D b 1 i hc khi B nm 2002
Trong mt phng vi h ta Oxy , cho hai ng trn (C1 ) : x2 + y2 4y 5 = 0 v

(C ) : x + y 6x + 8y + 16 = 0 . Vit phng trnh tip tuyn chung ca hai ng trn


(C ) v (C ) .
2

S: (12 ) : 2x + y 3 5 2 = 0;

( ) : y = 1; ( ) : y = 3 x 3 .
3

Bi 423. D b 2 i hc khi D nm 2002


Trong mt phng vi h ta Oxy , cho hai ng trn (C1 ) : x2 + y2 10x = 0 v

(C ) : x
2

+ y2 + 4x 2y 20 = 0 .

1/ Vit phng trnh ng trn i qua cc giao im ca (C1 ), (C2 ) v c tm nm trn

ng thng d : x + 6y 6 = 0 .
2/ Vit phng trnh tip tuyn chung ca cc ng trn (C1 ), (C2 ) .

S: 1/

(x 12)

+ (y + 1) = 125.
2

2 / x + 7y 5 25 2 = 0 .

Bi 424. i hc khi D nm 2003


Trong mt phng vi h ta cc vung gc Oxy , cho ng trn

(C) : (x 1)

+ (y 2) = 4 v ng thng (d) : x y 1 = 0 . Vit phng trnh ng

"Cn c b thng minh"

DeThiThuDaiHoc.com

Page - 171 -

www.MATHVN.com

Ths. L Vn on

Phn hnh hc

trn (C ') i xng vi ng trn (C) qua ng thng d. Tm ta cc giao im ca (C)


v (C ') .
S: (C ') : (x 3) + y2 = 4, A (1; 0), B (3;2) .
2

Bi 425. D b 1 i hc khi B nm 2003


Trong mt phng vi h ta Oxy , cho ng thng d : x 7y + 10 = 0 . Vit phng trnh
ng trn c tm thuc ng thng : 2x + y = 0 v tip xc vi ng thng d ti im
A (4;2) .

S: (C) : (x 6) + (y + 12) = 200 .


2

Bi 426. D b 1 i hc khi A nm 2004


Trong mt phng ta Oxy , cho im A (1;1) v ng thng d : x y + 1 2 = 0 . Vit
phng trnh ng trn i qua A, qua gc to O v tip xc vi ng thng d.

S: (C) : x 2 + ( y 1) = 1
2

(C) : (x + 1)

+ y2 = 1 .

Bi 427. i hc khi B nm 2005


Trong mt phng vi h ta Oxy, cho hai im A (2; 0), B (6; 4) . Vit phng trnh ng trn

(C) tip xc vi trc honh ti im A v khong cch t tm ca (C) n im B bng 5.


S: (C1 ) : (x 2) + (y 1) = 1, (C2 ) : (x 2) + (y 7) = 49 .
2

Bi 428. D b 2 i hc khi A nm 2005


Trong mt phng vi h ta Oxy, cho ng trn (C) : x 2 + y2 12x 4y + 36 = 0 . Vit
phng trnh ng trn (C1 ) tip xc vi hai trc ta Ox, Oy ng thi tip xc ngoi vi
ng trn (C) .
2
2

(C1 ) : (x 2) + (y 2) = 4

2
2
S: (C2 ) : (x 18) + (y 18) = 18 .

2
2
(C3 ) : (x 6) + (y + 6) = 36

Bi 429. D b 1 i hc khi B nm 2005


Trong mt phng vi h ta Oxy, cho 2 ng trn ln lt c phng trnh:
(C1 ) : x 2 + y2 = 9 v (C2 ) : x2 + y2 2x 2y 23 = 0 . Vit phng trnh trc ng
phng d ca 2 ng trn (C1 ) v (C2 ) . Chng minh rng nu K thuc d th khong cch t
K n tm ca (C1 ) nh hn khong cch t K n tm ca (C2 ) .

S: d : x + y + 7 = 0 . Xt OK 2 IK 2 = 16 < 0 OK < IK .
Bi 430. D b 1 i hc khi D nm 2005

Page - 172 -

DeThiThuDaiHoc.com
"All the flower of tomorrow are in the seeks of today"

cng hc tp mn Ton 10 Tp IIwww.MATHVN.com

Ths. L Vn on

Trong mt phng vi h ta Oxy, cho ng trn (C) c phng trnh:

(C) : x

+ y2 4x 6y 12 = 0 . Tm ta im M thuc ng thng d c phng trnh:

2x y + 3 = 0 sao cho MI = 2R , trong I l tm v R l bn knh ca ng trn (C) .

24 63
S: M (4; 5) M ; .
5 5
Bi 431. D b 2 i hc khi D nm 2005
Trong mt phng vi h ta Oxy, cho hai im A (0;5), B (2; 3) . Vit phng trnh ng
trn i qua hai im A, B v c bn knh R = 10 .

S: (C) : (x + 1) + (y 2) = 10
2

(C) : (x 3)

+ (y 6) = 10 .
2

Bi 432. i hc khi B nm 2006


Trong mt phng vi h ta Oxy, cho ng trn (C) : x 2 + y2 2x 6y + 6 = 0 v im
M (3;1) . Gi T1 v T2 l cc tip im ca cc tip tuyn k t M n (C) . Vit phng trnh

ng thng T1T2.
S: Chng t ta (x o ; yo ) ca T1, T2 tha phng trnh: 2x + y 3 = 0 .
Bi 433. i hc khi D nm 2006
Trong mt phng vi h ta Oxy, cho ng trn (C) v ng thng d ln lt c phng
trnh: (C) : x 2 + y2 2x 2y + 1 = 0, d : x y + 3 = 0 . Tm to im M nm trn d sao
cho ng trn tm M, c bn knh gp i bn knh ng trn (C) , tip xc ngoi vi ng
trn (C) .
S: M (1; 4) M (2;1) .
Bi 434. D b 1 i hc khi D nm 2006
Trong mt phng vi h ta Oxy, cho im A (1;1) v ng thng
d : x y + 1 2 = 0 . Vit phng trnh ng trn (C) i qua im A, gc to O v

tip xc vi ng thng d.
S:

(C ) : x
1

+ y2 2y = 0,

(C ) : x
2

+ y2 + 2x = 0 .

Bi 435. i hc khi A nm 2007


Trong mt phng vi h ta Oxy, cho tam gic ABC c A (0;2), B (2; 2), C (4; 2) . Gi
H l chn ng cao k t B; M v N ln lt l trung im ca cc cnh AB v BC. Vit
phng trnh ng trn i qua cc im H, M, N.
S: H (1;1) v (C) : x2 + y2 x + y 2 = 0 .
Bi 436. i hc khi D nm 2007
Trong mt phng vi h ta Oxy, cho ng trn (C) v ng thng d c phng trnh:

(C) : (x 1)

+ (y + 2) = 9 v d : 3x 4y + m = 0 . Tm m trn d c duy nht mt im

"Cn c b thng minh"

DeThiThuDaiHoc.com

Page - 173 -

www.MATHVN.com

Ths. L Vn on

Phn hnh hc

P m t c th k c hai tip tuyn PA, PB ti (C) (A, B l cc tip im) sao cho tam
gic PAB u.
S: m = 19 m = 41 .
Bi 437. D b 1 i hc khi A nm 2007
Trong mt phng vi h ta Oxy, cho ng trn (C) : x 2 + y2 = 1 . ng trn (C ') tm
I (2;2) ct (C) ti cc im A, B sao cho AB = 2 . Vit phng trnh ng thng AB.

S: Ch AB OI. Phng trnh AB : y = x 1 .


Bi 438. D b 1 i hc khi B nm 2007
Trong mt phng vi h ta Oxy, cho ng trn (C) v ng thng d ln lt c phng
trnh: (C) : x2 + y2 8x + 6y + 21 = 0, d : x + y 1 = 0 . Xc nh to cc nh hnh
vung ABCD ngoi tip ng trn (C), bit A nm trn d.
S: A (2; 1), B (2; 5), C (6; 5), D (6; 1) A (6; 5), B (6; 1), C (2; 1), D (2; 5) .
Bi 439. D b 2 i hc khi B nm 2007
Trong mt phng vi h ta Oxy, cho ng trn (C) c phng trnh

(C) : x + y 2x + 4y + 2 = 0 . Vit phng trnh ng trn (C ') c tm M (5;1) v (C ')


ct (C) ti cc im A, B sao cho AB = 3 .
2

S:

(C ) : (x 5) + (y 1)
2

'
1

( )

= 13 C1' : (x 5) + (y 1) = 43 .
2

Bi 440. i hc Si Gn khi B nm 2007


Trong mt phng vi h trc ta Descarter vung gc Oxy, cho im A (2;1) v hai ng
thng d1 : x y 1 = 0, d2 : x 2y 6 = 0 . Vit phng trnh ng trn (C) tip xc vi
d1 ti A v c tm thuc d2.
S: (C) : (x 4) + (y + 1) = 8 .
2

Bi 441. i hc khi A nm 2009 (Chng trnh nng cao)


Trong mt phng vi h ta Oxy, cho ng trn (C) : x2 + y2 + 4x + 4y + 6 = 0 v
ng thng : x + my 2m + 3 = 0 , vi m l tham s thc. Gi I l tm ca ng trn

(C) . Tm m ct (C) ti hai im phn bit A, B sao cho din tch IAB ln nht.
S: m = 0 m =

8
.
15

Bi 442. i hc khi B nm 2009 (Chng trnh c bn)

4
v hai ng
5
thng 1 : x y = 0, 2 : x 7y = 0 . Xc nh to tm K v tnh bn knh ca ng

Trong mt phng vi h ta Oxy, cho ng trn (C) : ( x 2) + y2 =


2

trn (C1 ) ; bit ng trn (C1 ) tip xc vi cc ng thng 1, 2 v tm K (C) .

Page - 174 -

DeThiThuDaiHoc.com
"All the flower of tomorrow are in the seeks of today"

cng hc tp mn Ton 10 Tp IIwww.MATHVN.com

Ths. L Vn on

8 4
2 5
S: K ; , R =
.
5 5
5

Bi 443. i hc khi D nm 2009 (Chng trnh nng cao)


Trong mt phng vi h ta Oxy, cho ng trn (C) : (x 1) + y2 = 1 . Gi I l tm ca
2


O = 30
(C) . Xc nh to im M thuc (C) sao cho IM

3
3

S: M ;
.
2
2

Bi 444. i hc khi A nm 2010 (Chng trnh c bn)


Trong mt phng ta Oxy, cho hai ng thng d1 : 3x + y = 0 v d2 : 3x y = 0 .

Gi (T) l ng trn tip xc vi d1 ti A, ct d2 ti hai im B, C sao cho tam gic ABC


vung ti B. Vit phng trnh ca (T), bit tam gic ABC c din tch bng
c honh dng.

3
v im A
2

1
3
+ y + = 1 .
S: (T) : x +

2
2 3
2

Bi 445. i hc khi D nm 2010 (Chng trnh c bn)


Trong mt phng ta Oxy, cho tam gic ABC c nh A (3; 7 ), trc tm l H (3; 1), tm
ng trn ngoi tip l I (2; 0) . Xc nh to nh C, bit C c honh dng.

S: C 2 + 65; 3 .
Bi 446. i hc khi A nm 2011 (Chng trnh c bn)
Trong mt phng ta Oxy, cho ng thng : x + y + 2 = 0 v ng trn

(C) : x

+ y2 4x 2y = 0 . Gi I l tm ca (C), M l im thuc . Qua M k cc tip

tuyn MA v MB n (C) (A v B l cc tip im). Tm ta im M, bit t gic MAIB c


din tch bng 10.

S: M (2; 4) M (3;1) .
Bi 447. i hc khi B nm 2011 (Chng trnh nng cao)
1
Trong mt phng ta Oxy, cho tam gic ABC c nh B ;1 . ng trn ni tip tam
2

gic ABC tip xc vi cc cnh BC, CA, AB tng ng ti cc im D, E, F. Cho D (3;1) v


ng thng EF c phng trnh y 3 = 0 . Tm ta nh A, bit A c tung dng.
13
S: A 3; .
3

Bi 448. i hc khi D nm 2011 (Chng trnh nng cao)

"Cn c b thng minh"

DeThiThuDaiHoc.com

Page - 175 -

www.MATHVN.com

Ths. L Vn on

Phn hnh hc

Trong mt phng Oxy, cho im A (1; 0) v ng trn (C) : x 2 + y2 2x + 4y 5 = 0 . Vit


phng trnh ng thng ct (C) ti im M v N sao cho AMN vung cn ti A.
S: : y = 1 : y = 3 .
Bi 449. i hc khi B nm 2012
1/ Trong mt phng ta Oxy, cho ng trn (C1 ) : x2 + y2 = 4 ,

(C ) : x
2

+ y2 12x + 8 = 0 v ng thng d : x y 4 = 0 . Vit phng trnh ng

trn c tm thuc (C2 ) , tip xc vi d v ct (C1 ) ti hai im phn bit A v B sao cho AB
vung gc vi d.
2/ Trong mt phng h trc ta Oxy, cho hnh thoi ABCD c AC = 2BD v ng trn
tip xc vi cc cnh ca hnh thoi c phng trnh x2 + y2 = 4 . Vit phng trnh chnh
tc ca elip (E) i qua cc nh A, B, C, D ca hnh thoi, bit A Ox .
S: 1/ (C) : (x 3) + (y 3) = 8.
2

Page - 176 -

2 / ( E) :

x 2 y2
+
= 1.
20
5

DeThiThuDaiHoc.com
"All the flower of tomorrow are in the seeks of today"

cng hc tp mn Ton 10 Tp IIwww.MATHVN.com

Ths. L Vn on

D PHNG TRNH NG ELLIPSE



 Kin thc c bn
a/ nh ngha
Cho

c nh vi

v mt di

im M sao cho

khng i

. Tp hp cc

c gi l mt elip.
.

Hai tiu im ca elip


Trong :

Tiu c ca elip

Bn knh qua tiu im ca M.


b/ Phng trnh chnh tc, phng trnh tham s ca elp
 Phng trnh chnh tc:

. Trong :
.

 Ta cc tiu im:

: Bn knh qua tiu im bn tri.

: Bn knh qua tiu im bn phi.


.

 Phng trnh tham s:


c/ Hnh dng ca elip
 Elip

nhn cc trc to lm cc trc i xng v gc to lm tm i xng.

 To cc nh:
 di cc trc:
 Tm sai ca

Trc ln
Trc nh

 Hnh ch nht c s to bi cc ng thng

A1

 Phng trnh cc ng chun i ng vi cc tiu im Fi l:


ta c:

"Cn c b thng minh"

F1

F2

A2

d/ ng chun ca elip

 Vi

B2

B1

DeThiThuDaiHoc.com

Page - 177 -

www.MATHVN.com

Ths. L Vn on

Phn hnh hc

(E)
x2

(E ) :

(E )

a2

y2
b2

=1

a>b

(E)

Ox,

2a

F1 (c; 0), F2 (c; 0)

(E)

A1

2b

c
e=
a

Oy,

A2

F2

O
B1

c2 = b2 a 2

Ox

e=

2b

F1 (0; c), F2 (0; c)

(E)

F1

a < b,

(E)

B2

c2 = a 2 b2

2a

c
b

(x )

(E ) :

OI
X = x
x = X +

Y = y
y = Y +

IX Y
(E )

a2

(y )

b2

=1

I (; )

(E ) :

X2
a2

Y2
b2

=1

(E )
(E )

(E ) :
a, b
a, b

x2
a2

y2
b2

=1

a2, b2
a2, b2

M (x; y) (E)

Bc 2. Chuyn

thnh biu thc gii tch nh


.

Page - 178 -

DeThiThuDaiHoc.com
"All the flower of tomorrow are in the seeks of today"

cng hc tp mn Ton 10 Tp IIwww.MATHVN.com

MF1 MF2 =

MF12 MF22
MF1 + MF2

(x
=

(1) + (4)

2
1

) (

Ths. L Vn on

x22 + y12 y22 2x (x1 x2 ) 2y (y1 y2 )


2a

(2)

MF1,

( 4)

(E)

c
a
A1 (a;0),A2 (a;0),B1 (0;b),B2 (0;b)

e=

b2 = a 2 c2

F1 (c;0), F2 (c; 0)

(E)
M (x o ; y o ) (E )

x 2o
a2

y2o
b2

=1

x o , yo

M (E) M (a sin t; b cos t)

x = a sin t
E
:
( ) y = b cos t , t 0;2)

x o , yo

F M = a + cx o
1
a

cx o
F2M = a
a

MF1 = a +

c
c
x, MF2 = a x
a
a
M (x; y)

Dng 1.

Tp hp l elip

Dng 2.

c hai tiu im F1, F2, trc ln 2a.

Tp hp l elip

c di trc ln 2a,

trc nh 2b.

"Cn c b thng minh"

DeThiThuDaiHoc.com

Page - 179 -

www.MATHVN.com

Ths. L Vn on

(E ) : x2 + y2
a

xox
a

yo y
b2

Phn hnh hc

M (x o ; y o ) (E)

=1

=1
: Ax + By = 0

a 2 A2 + b2 B2 = C2

Elip (E) :

M (x M ; yM )

(E )

x2
a2

y2
b2

PM/(E) =

=1
x 2M
a2

y2M
b2

PM/(E) < 1
PM/(E) = 1
PM/(E) > 1

(E )

(E )
(E )

(E)

(E )

(E)

(E )
(E)

(E )

(E1 ) :

x2
a12

y2
b12

=1

(E 2 ) :

x2
a 22

y2
b22

(E1 ) (E2 ) = {A, B,C, D}

=1

 ABCD l hnh ch nht.


 Phng trnh ng trn ngoi tip hnh ch nht ABCD l
ng trn
tm O bn knh
c phng trnh
.

Page - 180 -

DeThiThuDaiHoc.com
"All the flower of tomorrow are in the seeks of today"

cng hc tp mn Ton 10 Tp IIwww.MATHVN.com

Ths. L Vn on

BA
NG
BAI T
TP AAP DU
DUNG
TM CC THUC TNH V LP PHNG TRNH ELP
Bi 450. V th cc elp (E) sau
1/

( E) :

x 2 y2
+
= 1.
9
4

2/

x2

(E) : 25 +

y2
= 1.
9

Bi 451. Cho elip (E) . Xc nh di cc trc, tiu c, to cc tiu im, to cc nh, tm sai,
phng trnh cc ng chun ca (E), vi (E) c phng trnh

x 2 y2
+
= 1.
9
4

1/

( E) :

3/

(E) : 25 +

x2

y2
= 1.
9

(E) : 16x + 25y = 400 .


7/ (E) : 4x + 9y = 5 .
9/ (E) : x2 + 4y2 = 4 .
11/ (E) : 9x2 + 16y2 = 144 .
5/

2/

( E) :

x 2 y2
+
= 1.
16
9

4/

( E) :

x 2 y2
+
= 1.
4
1

(E) : x + 4y = 1 .
8/ (E) : 9x + 25y = 1 .
10/ (E) : 6x2 + 9x2 = 54 .
12/ (E) : 4x2 + 9y2 = 1 .

6/

Bi 452. Lp phng trnh chnh tc ca elip (E), bit


1/
2/
3/
4/

di trc ln bng 6, trc nh bng 4.


C di trc nh v trc ln ln lt l 8 v 6.
di trc ln bng 10, tiu c bng 6.
di trc ln bng 8, di trc nh bng tiu c.

5/

Mt tiu im F1 (1; 0) v di trc ln bng 2.

6/

Tiu c bng 8 v i qua im M

7/

di trc nh bng 6 v i qua im M 2 5; 2 .

8/

Mt tiu im l F1 (2; 0) v di trc ln bng 10.

9/

.
Mt tiu im l F1 3; 0 v i qua im M 1;
2

15; 1 .

;1 .
10/ i qua hai im M (1; 0), N
2

) (

11/ i qua hai im M 4; 3 , N 2 2; 3 .

12
12/ i qua hai im M (0; 3), N 3; .

"Cn c b thng minh"

DeThiThuDaiHoc.com

Page - 181 -

www.MATHVN.com

Ths. L Vn on

Phn hnh hc

3
.
5

12/ di trc ln bng 10, tm sai bng

13/ Mt tiu im l F1 (8; 0) v tm sai bng

4
.
5

14/ di trc nh bng 6, phng trnh cc ng chun l x 7 16 = 0 .


15/ Mt nh l A1 (8; 0) , tm sai bng

3
.
4

5
2
16/ i qua im M 2; v c tm sai bng .

3
3
17/ C tiu c bng 4 v t s di hai trc bng

5
.
3

18/ i qua im M (8;12) v c bn knh qua tiu im bn tri ca M bng 20.

19/ i qua im M 3; 2 3 v c bn knh qua tiu im bn tri ca M bng 4 3 .


20/ C phng trnh cc cnh hnh ch nht c s l x = 9, y = 3 .

3 4
21/ i qua im M ; v MF1F2 vung ti M.
5 5
22/ Hnh ch nht c s ca (E) c mt cnh nm trn ng thng x 2 = 0 v c di

ng cho bng 6.
23/ C nh l A1 (5; 0) v phng trnh ng trn ngoi tip hnh ch nht c s c dng
l x 2 + y2 = 34 .
24/ C nh l B1 (0;6) v phng trnh ng trn ngoi tip hnh ch nht c s c dng l

x2 + y2 = 61 .
25/ C di trc ln bng 4 2, cc nh trn trc nh v cc tiu im ca (E) cng nm
trn mt ng trn.

TM IM TRN ELIP THA IU KIN CHO TRC


Bi 453. Tm nhng im trn elip (E) :

5
x2
y2
+
= 1 c bn knh qua tiu im bng .
16
7
2

Bi 454. Tm nhng im M trn elip (E) :


bng

x2
y2
+
= 1 sao cho hiu s hai bn knh qua tiu im
25
9

32
.
5

Bi 455. Cho elp (E) :


1/ 90 0 .

x2
y2

MF2 l
+
= 1 . Tm nhng im M nm trn (E) sao cho s o F
1
25
4
2/ 120 0 .

3/ 30 0 .


MF2 l
Bi 456. Cho elp (E) : 4x2 + 9y2 = 36 . Tm nhng im M nm trn (E) sao cho s o F
1

1/ 90 0 .
Page - 182 -

2/ 60 0 .

3/ 30 0 .

DeThiThuDaiHoc.com
"All the flower of tomorrow are in the seeks of today"

cng hc tp mn Ton 10 Tp IIwww.MATHVN.com

Ths. L Vn on

Bi 457. Cho elip (E) . Tm nhng im M (E) nhn hai tiu im di 1 gc 300, 450, 600, 1200 .
1/
2/
3/

(E ) :
(E ) :
(E ) :

9x 2 + 25y2 = 225 .
9x 2 + 16y2 = 144 .
7x 2 + 16y2 = 112 .

x2
y2
+
= 1 . Tm im M nm trn (E) sao cho
100 36
1/ MF2 = 4MF1 .
2/ Nhn F1 v F2 di mt gc vung.
Bi 459. Cho elip (E) . Tm nhng im M (E) sao cho
Bi 458. Cho elp (E) :

a/ MF1 = MF2 .
1/
2/
3/

(E ) :
(E ) :
(E ) :

b/ MF2 = 3MF1 .

c/ MF1 = 4MF2 .

9x 2 + 25y2 = 225 .
9x 2 + 16y2 = 144 .
7x 2 + 16y2 = 112 .

Bi 460. Cho elp (E) : x2 + 9y2 = 9 .


1/

Tm M trn (E) sao cho MF1 = 2MF2 .

2/

Tm M trn (E) sao cho 3MF1 = MF2 .

3/

Tm M trn (E) sao cho

1
1
6
+
=
.
MF1 MF2
F1F2

Bi 461. Cho elip (E) . Tm nhng im M (E) nhn hai tiu im di mt gc vung, vi
1/
2/
3/

(E ) :
(E ) :
(E ) :

9x 2 + 25y2 = 225 .
9x 2 + 16y2 = 144 .
7x 2 + 16y2 = 112 .

Bi 462. Cho elip (E) v ng thng d vung gc vi trc ln ti tiu im bn phi F2 ct (E) ti hai
im M, N.
a/ Tm to cc im M, N.
1/
3/

(E ) :
(E ) :

b/ Tnh MF1, MF2, MN .

9x 2 + 25y2 = 1 .

2/

9x 2 + 16y2 = 144 .

4/

(E ) :
(E ) :

9x 2 + 25y2 = 225 .
7x 2 + 16y2 = 112 .

Bi 463. Tm trn ng thng d : x + 5 = 0 im M cch u tiu im tri v nh trn ca elp

x2

y2
= 1.
9
x2
y2
+
= 1.
Bi 464. Cho elp (E) :
25 16
1/ Bit M (E) sao cho MF1 = 3 . Tm MF2 v tm ta im M.

(E) : 25 +

"Cn c b thng minh"

DeThiThuDaiHoc.com

Page - 183 -

www.MATHVN.com

Ths. L Vn on

2/

Phn hnh hc

Dy cung AB thay i i qua tiu im F1 nhng khng i qua tiu im F2 ca (E) .


Chng minh rng chu vi tam gic ABF2 khng i.

x 2 y2
+
= 1 v ng thng d : x 2y + 12 = 0 . Tm trn (E) im M sao
25
9
cho khong cch t im M n ng thng d l ln nht, nh nht.

Bi 465. Cho elp (E) :

Bi 466. Cho elp (E) : x2 + 4y2 = 25 v ng thng d : 3x + 4y 30 = 0 . Tm trn (E) im M


sao cho khong cch t im M n ng thng d l ln nht, nh nht.

x 2 y2
+
= 1 v ng thng d : x 2y + 2 = 0 . ng thng d ct (E) ti
8
4
hai im B, C. Tm ta im A trn (E) sao cho ABC c din tch ln nht.

Bi 467. Cho elp (E) :

Bi 468. Cho elp (E) : x2 + 2y2 = 2 v ng thng d : 3x 2y 3 = 0 . ng thng d ct (E) ti


hai im B, C. Tm ta im A trn (E) sao cho ABC c din tch ln nht.

x2
+ y2 = 1 v im C (2; 0) .
4
Tm im M thuc elp (E) sao cho c bn knh qua tiu im ny bng 7 ln bn knh qua

Bi 469. Cho elp (E) :


1/

tiu im kia.
2/

M nhnh hai tiu im di mt gc 600, 900 .

3/

Tm ta cc im A, B trn (E) sao cho ABC l tam gic u.

Bi 470. Cho elp (E) : 13x2 + 16y2 = 208 . Tm ta cc im A, B trn (E) sao cho ABF1 l tam
gic u.

1/

x2
y2
+
= 1 . Tm cc im M thuc elp (E) sao cho
2
8
C ta nguyn thuc (E) .

2/

C tng hai to t gi tr ln nht, nh nht.

Bi 471. Cho elp (E) :

1/

x2
y2
+
= 1 v ng thng d : 3x + 4y 12 = 0 .
16
9
Chng minh rng d lun ct (E) ti hai im phn bit A, B. Tnh di on AB.

2/

Tm ta im C (E) sao cho

Bi 472. Cho elp (E) :

a/ ABC c din tch bng 6.


b/ ABC c din tch ln nht.
c/ ABC vung.

TIP TUYN TNG GIAO


Bi 473. Cho elp (E) :

x2

y2

= 1 v ng thng : Ax + By + C = 0 . Chng minh rng iu


a2
b2
kin cn v ng thng tip xc vi elp (E) l a 2A2 + b2B2 = C2 .

Bi 474. Cho im M (1;2) . Lp phng trnh tip tuyn ca elp (E) :

x2
y2
+
= 1 i qua M.
2
8

x2
y2
+
= 1.
9
4
Chng minh rng qua M k c 2 tip tuyn n (E) .

Bi 475. Cho im M (3; 4) v elp (E) :


1/
Page - 184 -

DeThiThuDaiHoc.com
"All the flower of tomorrow are in the seeks of today"

cng hc tp mn Ton 10 Tp IIwww.MATHVN.com

2/

Ths. L Vn on

Xc nh phng trnh hai tip tuyn v lp phng trnh ng thng i qua hai tip im
ca (E) vi hai tip tuyn trn.

Bi 476. Cho elp (E) : 4x2 + 9y2 = 36 . Vit phng trnh cc tip tuyn ca (E) . Bit tip tuyn
1/

Tip xc vi (E) ti im M (0; 4) .

2/

i qua im A (3; 0) .

3/

i qua im B (2; 3) .

4/

Song song vi ng thng d1 : x 2y + 6 = 0 .

5/

Vung gc vi ng thng d2 : x y = 0 .

6/

To vi ng thng d3 : 2x y = 0 mt gc bng 60 0 .

x 2 y2
x2
y2
+
= 1 v (E2 ) :
+
= 1.
Bi 477. Lp phng trnh tip tuyn chung ca hai elp (E1 ) :
9
4
4
9
Bi 478. Vit phng trnh chnh tc ca elp, bit rng elp (E) tip xc vi hai ng thng
d1 : x + y 5 = 0 v d2 : x 4y 10 = 0 .
x2
y2
+
= 1 . Xt hnh vung ngoi tip elp (E) . Vit phng trnh cc ng
24 12
thng cha cc cnh ca hnh vung .
Bi 480. Chng minh rng tch khong cch t hai tiu im ca elp n mt tip tuyn bt k ca elp
l mt i lng khng i.

Bi 479. Cho elp (E) :

x 2 y2
+
= 1 v im M (1;1) .
9
4
Chng minh rng mi ng thng i qua M lun ct elp (E) ti hai im phn bit.

Bi 481. Cho elp (E) :


1/
2/

Lp phng trnh ng thng d i qua M v ct elp ti hai im phn bit A, B sao cho
di on MA = MB .

x2
y2
+
= 1 . Xt v tr tng i ca elp (E) trong cc trng hp
Bi 482. Cho elp (E) :
16
9
1/ M (4; 0) .
2/ M (3; 3) .

Bi 483. Xt v tr tng i ca ng thng d v elp (E) , bit:

x 2 y2
+
= 1.
4
9
x2
y2
2/ d : 2x y = 0 v (E) :
+
= 1.
2
8
x2
y2
2
2
Bi 484. Cho hai elp (E1 ) :
+ y = 1 v (E2 ) : x +
= 1.
4
9
1/ Chng minh rng (E1 ) (E2 ) = {A, B,C, D} v ABCD l hnh ch nht.
1/

d : 2x + y 5 = 0 v (E) :

2/

Lp phng trnh ng trn ngoi tip hnh ch nht ABCD.

1/

x2
y2
x2
y2
+
= 1 v (E2 ) :
+
= 1.
9
4
16
1
Chng minh rng (E1 ) (E2 ) = {A, B,C, D} v ABCD l hnh ch nht.

2/

Lp phng trnh ng trn ngoi tip hnh ch nht ABCD.

Bi 485. Cho hai elp (E1 ) :

"Cn c b thng minh"

DeThiThuDaiHoc.com

Page - 185 -

www.MATHVN.com

Ths. L Vn on

Bi 486. Cho elip (E) :


ti A v B.
1/

x2
a2

y2
b2

Chng minh rng

Phn hnh hc

= 1 . Mt gc vung nh O quay quanh O, c 2 cnh ct (E) ln lt


1

khng i.
OA
OB2
2/ Tnh khong cch t O n ng thng AB. Suy ra ng thng AB lun tip xc vi
mt ng trn (C) c nh. Tm phng trnh ca (C) .
2

TNH CHT V MT S DNG TON KHC


Bi 487. Tnh di dy cung ca elp (E) i qua mt tiu im v vung gc vi Ox trong cc trng
hp sau
1/

2/

(E) : 9x2 + 25y2 = 225 .

4/

x2

y2
= 4.
9

x2

y2

(E) : 25 +

(E) : 24 + 12 = 1 .
Trong mt phng Oxy, gi M l im bt k trn elp (E) . Tm gi tr ln nht v gi tr nh
3/

Bi 488.

x2
y2
+
= 1.
9
4

(E ) :

nht ca OM trong cc trng hp sau


1/
3/

x2

y2
= 1.
9
(E) : 9x2 + 16y2 = 144 .

(E) : 36 +

2/
4/

(E) : x2 + 4y2 = 4 .
(E) : 8x2 + 16y2 = 32 .

Bi 489. Tm gi tr ln nht v gi tr nh nht ca cc biu thc sau

x2
y2
+
= 1.
25
4
x
x2
2/ B = + 3y bit
+ y2 = 1 .
2
4
Bi 490. Cho elp (E) : 9x2 + 25y2 = 225 .
1/

A = x + 2y bit

1/

Tm ta tiu im, tm sai v di cc trc ca (E) .

2/

ng thng d qua tiu im F1, vung gc vi Ox v ct (E) ti hai im M, N. Tnh

di on thng MN.
3/ Ly P, Q thuc elp (E) sao cho PF1 + QF2 = 6 . Tnh tng S = QF1 + PF2 .
4/

Cho R (E) tnh gi tr ca biu thc W = RF1.RF2 + OR 2 .

Bi 491. Tm tm sai ca (E) trong cc trng hp sau:


1/
2/
3/
4/

Mi nh trn trc nh nhn hai tiu im di mt gc vung.


Mi tiu im nhn trc nh di mt gc vung.
Mi tiu im nhn trc nh di mt gc 60 0 .
di trc ln bng k ln di trc nh (k > 1) .

5/

Khong cch t mt nh trn trc ln n mt nh trn trc nh bng tiu c.

Bi 492. Cho elip (E) :

x2

y2

= 1 . Gi F1, F2 l 2 tiu im, A1, A2 l 2 nh trn trc ln, M l 1


a2
b2
im tu thuc (E) v P l hnh chiu ca M trn trc ln.
1/

Page - 186 -

Chng minh: b OM a .

DeThiThuDaiHoc.com
"All the flower of tomorrow are in the seeks of today"

cng hc tp mn Ton 10 Tp IIwww.MATHVN.com

2/

Chng minh: MF1.MF2 + OM2 = a 2 + b2 .

3/

Chng minh:

(MF1 MF2 )

4/

Chng minh:

MP2
b2
= 2.
A1P.A2 P
a

Ths. L Vn on

= 4 OM2 b2 .

Bi 493. Cho elp (E) : 9x2 + 16y2 = 144 .


1/

Tm tm sai ca (E) .

2/

Gi M l im di ng trn (E) . Chng minh OM2 + MF1.MF2 l mt hng s.

3/

Tm im N (E) sao cho NF1F2 vung ti N.

4/

Cho A, B l hai im nm trn (E) vi AF1 + BF2 = 8 . Tnh AF2 + BF1 .

QU TCH TP HP IM
Bi 494. Tm tp hp nhng im M (x; y) trong mt phng ta sao cho: x = 4 sin t, y = 3 cos t .

 
Bi 495. Cho im A (3 cos t; 0) v B (0; sin t) . Tm tp hp im M (x; y) sao cho 2AM + 5MB = 0
khi t thay i.

Bi 496. Cho ng trn (C): (C) : x2 + y2 6x 55 = 0 v im F1 (3; 0) :


1/

Tm tp hp cc tm M ca ng trn (C ') di ng lun i qua F1 v tip xc vi (C) .

2/

Vit phng trnh ca tp hp trn.

1/

Chng minh (C) v (C ') tip xc nhau.

2/

Tm tp hp cc tm M ca ng trn (T) di ng v tip xc vi hai ng trn trn.

Bi 497. Cho hai ng trn (C) : x2 + y2 + 4x 32 = 0 v (C ') : x2 + y2 4x = 0 :

3/ Vit phng trnh ca tp hp .


Bi 498. Tm tp hp cc im M c t s cc khong cch t n im F v n ng thng
bng e, vi:
1
1
1/ F (3; 0), : x 12 = 0, e = .
2/ F (2; 0), : x 8 = 0, e = .
2
2
4
3
3/ F (4; 0), : 4x + 25 = 0, e = .
4/ F (3; 0), : 3x 25 = 0, e = .
5
5
Bi 499. Cho hai im A, B ln lt chy trn hai trc Ox v Oy sao cho AB = 12 .
1/ Tm tp hp cc trung im I ca on AB.
1
2/ Tm tp hp cc im N chia on AB theo t s k = .
2
Bi 500. Cho elp (E) di ng c tm l gc ta v di trc ln bng 16 . Bit rng (E) lun i
qua im M (4; 0) . Chng minh rng hai tiu im F1 v F2 ca (E) lun di ng trn mt elp
c nh.
Bi 501. Cho im A di ng trn trc honh v im B di ng trn trc tung sao cho di on
thng AB lun bng a khng i. Tm tp hp cc im M thuc on AB sao cho
MB = 2MA .
Bi 502. Cho hai im A v B c nh. Xc nh hnh thang ABCD c y ln AB sao cho
AB
AD + BC = AB v CD =
. Tm tp hp trung im M ca on CD.
3
"Cn c b thng minh"

DeThiThuDaiHoc.com

Page - 187 -

www.MATHVN.com

Ths. L Vn on

Phn hnh hc

x2
y2
+
= 1 v (E2 ) : x2 + 4y = 4 . Cc giao im ny c
6
3
cng nm trn mt ng trn hay khng ? Nu c th vit phng trnh ng trn ?

Bi 503. Tm giao im ca hai elp (E1 ) :

Bi 504. Cho hai elp (E1 ) : 4x2 + 9y2 = 36 v (E2 ) : x2 + 16y2 = 16 . Hy vit phng trnh ng
trn i qua cc giao im ca (E1 ) v (E2 ) .

BI TP QUA CC K THI
Bi 505. Cao ng Kinh T i Ngoi nm 1997
Cho elp (E) : 4x2 + y2 = 36 .
1/ Tnh di trc ln, trc nh, ta cc tiu im v phng trnh cc ng chun.
2/ Lp phng trnh tip tuyn vi (E) song song vi ng phn gic gc phn t th II.
3/ Lp phng trnh parabol c nh trng vi gc ta v c tiu im l tiu im pha trn
ca elip (E) .

2/ x + y 3 5 = 0 .

S: 1/ F1,2 0; 3 3 , y = 4 3 .

3/ (P) : y =

x2 3
.
36

Bi 506. Cao ng S Phm K Thut Vinh nm 2001


Trong mt phng ta Oxy, cho hai ng elp c phng trnh ln lt l
x 2 y2
x 2 y2
(E1 ) : 3 + 2 = 1 v (E2 ) : 2 + 3 = 1 .
1/ Vit phng tnh ca ng trn i qua giao im ca hai elp.
2/ Vit phng trnh ca cc tip tuyn chung ca hai elp.
S: 1/ x 2 + y2 =

12
.
5

2/ x + y 5 = 0 x y 5 = 0 .

Bi 507. Cao ng S Phm Th Dc TWII nm 2002

) (

Trong mt phng ta Oxy, cho elp (E) c hai tiu im l F1 3; 0 , F2


ng chun c phng trnh x =

3; 0 , mt

3
1/ Vit phng trnh chnh tc ca elp (E) .
2/ Gi M (E) . Tnh gi tr ca biu thc P = F1M 2 + F2 M 2 3O M 2 F1M.F2 M .
3/ Vit phng trnh ng thng d song song vi trc honh v ct (E) ti hai im A, B sao
cho OA OB .
Bi 508. Cao ng C Kh Luyn Kim nm 2004
Vit phng trnh tip tuyn ca elip (E) :
thng d : x + 2y 2 = 0 .

x 2 y2
+
= 1 , bit rng song song vi ng
32
8

S: 1,2 : x + 2y 8 = 0 .
Bi 509. Cao ng Kinh T K Thut Cng Nghip II nm 2004
Page - 188 -

DeThiThuDaiHoc.com
"All the flower of tomorrow are in the seeks of today"

cng hc tp mn Ton 10 Tp IIwww.MATHVN.com

Ths. L Vn on

) (

Trong mt phng vi h trc ta Oxy, cho hai im A 4; 3 , B 2 2;3 .


1/ Vit phng trnh chnh tc elp i qua hai im A v B.
2/ Xc nh tiu im, tiu c, tm sai ca elip trn.
S: 1/ (E) :

x 2 y2
+
= 1.
20 15

) (

2/ F1 5;0 , F2

1
5;0 , 2c = 2 5, e = .
2

Bi 510. Cao ng Ha Cht nm 2004


Trong mt phng vi h trc ta Descarter vung gc Oxy, cho elip (E) :

x2
+ y2 = 1 .
9

1/ Xc nh tm sai ca (E) .
2/ Qua im M (1;1) k cc tip tuyn MT, MT ' vi T, T ' l cc tip im vi (E) . Hy
xc nh ta T, T ' .
S: 1/ e =

c 2 2
=
.
a
3

9 4
5 5

2/ T (0;1), T ' ; .

Bi 511. Cao ng S Phm Bnh Phc nm 2004

x2
y2
x2
y2
+
= 1 v
+
= 1.
4
5
5
4
S: C bn tip tuyn: x + y 3 = 0 x y 3 = 0 .
Vit phng trnh tip tuyn chung ca hai elip:

Bi 512. Cao ng S Phm Tp. H Ch Minh nm 2005

x 2 y2
+
= 1 . Chng minh tch cc
25 16
khong cch t cc tiu im ca elip (E) n mt tip tuyn bt k ca n l mt hng s.
Trong mt phng vi h trc ta Oxy, cho elip (E) :

Bi 513. Cao ng S Phm Qung Nam nm 2005


Trong mt phng vi h trc ta cc vung gc Oxy, cho ng trn (C) : x2 + y2 = 1
v elip (E) : x2 +

y2
= 1 . Vit phng trnh cc tip tuyn chung ca (C) v (E) .
4

Bi 514. Cao ng Kinh T K Hoch Nng nm 2005

x2
+ y2 = 1 . Vit phng trnh tip tuyn ca (E) song song vi ng thng
4
d c phng trnh x + 2y 8 = 0 .

Cho elip (E) :

Bi 515. Cao ng Bn Cng Hoa Sen khi D nm 2006


Trong mt phng vi h trc ta Descarter vung gc Oxy, vit phng trnh cc tip tuyn

x 2 y2
ca elip (E) :
+
= 1 , bit rng tip tuyn i qua A (4; 3) .
16
9
S: x 4 = 0 y + 3 = 0 .
Bi 516. Cao ng S Phm H Nam khi A nm 2006

"Cn c b thng minh"

DeThiThuDaiHoc.com

Page - 189 -

www.MATHVN.com

Ths. L Vn on

Phn hnh hc

Trong mt phng vi h trc ta Descarter vung gc Oxy, cho elip (E) :

x 2 y2
+
= 1.
16
9

Vit phng trnh ng thng i qua im M 0; 3 3 v tip xc vi elip (E) .


S: 3x 2 2y 6 6 = 0 .
Bi 517. Cao ng S Phm H Nam khi M nm 2006
Trong mt phng vi h trc ta Descarter vung gc Oxy, cho elip (E) :

x 2 y2
+
= 1.
9
4

1/ Xc nh tiu c, tm sai, tiu im, ng chun ca elip (E) .


2/ Lp phng trnh tip tuyn ca (E) , bit tip tuyn i qua im M (4;1) .
S: 1/ Tiu c 2c = 2 5, tm sai e =

5
3
, tiu im F1,2 5; 0 , ng chun x =
.
3
5

2/ C hai phng trnh tip tuyn: 1;2 : y =

4 37
4 37
x +1
.
7
7

Bi 518. Cao ng S Phm Tr Vinh khi A, B nm 2006

x 2 y2
+
= 1 . Vit phng trnh tip
9
4
tuyn vi (E) bit tip tuyn i qua im M (6; 0) . Tm ta tip im.

Trong mt phng vi h trc ta Oxy, cho elp (E) :

S: d1;2 : x

3 3
y 6 = 0 . A ; 3 .
2
2

Bi 519. Cao ng Nguyn Tt Thnh khi A, D nm 2007


Trong mt phng vi h trc ta Oxy, cho elip (E) : 4x 2 + 9y2 = 36 .
1/ Tm ta cc tiu im ca (E) .
2/ Tm im M trn (E) nhn cc tiu im ca (E) di mt gc vung.

S: 1/ F1,2 5; 0 .

3
3
4
4
, M3,4
.
;

2/ C bn im: M1,2 ;

5
5
5
5

Bi 520. i hc Bch Khoa Tng Hp nm 1980


1/ Vit phng trnh chnh tc ca Ellipse (E) c tiu c 2c = 8, tm sai e =
im nm trn trc honh.

4
v cc tiu
5

45
2/ Vit phng trnh cc tip tuyn vi Ellipse (E) xut pht t im A 0; .
12

3/ Tnh din tch hnh phng gii hn bi Ellipse v cc tip tuyn nn trn.
S: 1/ (E) :

x 2 y2
+
= 1.
25
9

2/ d1,2 : 9x 20y 75 = 0 .

3/ Shp = 15 (vdt) .

Bi 521. i hc Bch Khoa Tp. H Ch Minh nm 1988 Y Dc Tp. H Ch Minh nm 1991


Page - 190 -

DeThiThuDaiHoc.com
"All the flower of tomorrow are in the seeks of today"

cng hc tp mn Ton 10 Tp IIwww.MATHVN.com

Ths. L Vn on

x2
y2
+
= 1, (a > 0, b > 0) v im M (x o ; y o ) (E) vi a < x o < a .
a2
b2
Tip tuyn ti M ct hai tip tuyn ti A (a; 0) v A ' (a; 0) ln lt ti T v T'.

Cho Ellipse (E) :

1/ Chng minh rng tch s P = AT.AT ' khng ph thuc vo v tr ca im M.


2/ Tm cc im M sao cho t gic ATT'A' c din tch nh nht v tnh din tch nh nht .
3/ Gi N (x; y) l giao im ca AT' v A'T. Tm qu tch cc giao im N khi M chy trn
Ellipse (E) .

S: 1/ P = b . 2/ Smin
2

= 2ab M B (0; b) B ' (0; b) . 3/ (E ') :

x2
y2
+
= 1.
a 2 b 2

2

Bi 522. i hc Bch KhoaKinh TTng HpS PhmNng NghipYNhaDc 1983


Trong mt phng vi h trc ta Descarter vung gc Oxy, cho h Ellipse (E m ) c phng

x2
vi m l tham s v 0 < m < 1 .
m
1/ Bng php tnh tin trc tung, hy a phng trnh trn v dng chnh tc. T suy ra ta
ca tm v cc nh trn elip. Tm qu tch ca cc nh y.

trnh (Em ) : y2 = 2x

2/ Tm ta cc tiu im ca (Em ) v xc nh qu tch cc tiu im khi m thay i

(0 < m < 1) .

(x m)

S: 1/ (E m ) :

y2

= 1 (E m ) :

X2
+
m2

Y2

= 1 (E m ) c tm I (m; 0) .

( m)
( m)
nh trn trc ln: A (m; m ), A (m; m ) . Qu tch cc nh A , A
m2

l mt

Parabol (P) : y2 = x, (0 < x < 1) .

2/ Tiu im F1,2 m; m m

1
1
Qu tch l ng trn (C) : x + y2 = c

2
4
2

1
1
tm H ; 0, bn knh R = , b i hai im O (0; 0), G (0;1) .
2
2

Bi 523. i hc S Phm Tp. H Ch Minh nm 1993


Chng minh ng thng d : Ax + By + C = 0, tip xc
x2

(E ) : a

y2
= 1 a 2 A2 + b2 B2 = C2 .
b2

TH : B 0 PTHG : a 2A2 + b2B2 x2 + 2a 2 ACx + a C2 b2 B2 = 0 : = 0


HD: 1
.
2 2 2
2 2 2
2 2
2 2 2
2 2
TH2 : B = 0 PTTG : a A .y = a b A b C : T /x a b A b C = 0

Bi 524. i hc Kin Trc nm 1994

"Cn c b thng minh"

DeThiThuDaiHoc.com

Page - 191 -

www.MATHVN.com

Ths. L Vn on

Phn hnh hc

x 2 y2
+
= 1 . Xt mt hnh vung ngoi tip Ellipse (tc l
6
3
cc cnh ca hnh vung u tip xc vi Ellipse. Vit phng trnh cc ng thng cha cc
cnh ca hnh vung .
S: x + y + 3 = 0, x y + 3 = 0, x + y 3 = 0, x y 3 = 0 .

Cho Ellipse c phng trnh (E) :

Bi 525. i hc Xy Dng nm 1996


Trong mt phng vi h trc ta Descarters vung gc Oxy, cho Ellipse
x2
y2
(E) : a2 + b2 = 1, (a > b > 0) .
1/ Gi E l im ty trn Ellipse, chng t rng b OE a .
2/ Gi A, B l hai im thuc Ellipse sao cho OA vung gc vi OB. Hy xc nh v tr ca
A, B trn Ellipse tam gic OAB c din tch ln nht v nh nht. Tnh gi tr ln nht v
gi tr nh nht .
S: 1/ Chng minh theo 2 cch.
2/ (SOAB )

max

(S

OAB min

ab
A, B l cc nh ca (E) .
2
ab
a 2 b2
ab
ab
ab

= 2

A
;
,
B
;

.
2
a + b2
a + b2 a 2 + b2
a 2 + b2
a 2 + b2
=

Bi 526. i hc Ngoi Thng khi D nm 1997

x 2 y2
+
=1
8
4
v ng thng d : x 2y + 2 = 0 . ng thng d ct Ellipse ti hai im B v C. Tm ta
im A trn Ellipse sao cho ABC c din tch ln nht.

Trong mt phng vi h trc ta Descarter vung gc Oxy, cho Ellipse (E) :

S: A 2; 2 .
Bi 527. Hc Vin Quan H Quc T nm 1997

x2
+ y2 = 1 v im A (2; 0) . Gi s M l im di ng trn Ellipse. Gi
4
H l hnh chiu vung gc ca M trn trc Oy. Gi s AH ct OM ti P. Chng minh rng khi
M thay i trn Ellipse th P lun lun chy trn mt ng cong (C) c nh. V th ng

Cho Ellipse (E) :

cong (C) .
S: P chy trn Parabol (P) : y2 = x + 1 b qua im (1; 0) .
Bi 528. i hc Quc Gia Tp. H Ch Minh nm 1997
Cho phng trnh (E) : 4x2 + 9y2 = 36 v im M (1;1) . Lp phng trnh ng thng qua
M v ct (E) ti M1 v M2 sao cho MM1 = MM2 .
S: 4x + 9y 13 = 0 .
Bi 529. i hc M H Ni khi D nm 1997

x 2 y2
x2
+
= 1 v (E2 ) :
+ y2 = 1 . Vit phng trnh ng trn i
6
3
4
qua giao im ca hai Ellipse.

Cho hai Ellipse (E1 ) :

Page - 192 -

DeThiThuDaiHoc.com
"All the flower of tomorrow are in the seeks of today"

cng hc tp mn Ton 10 Tp IIwww.MATHVN.com

Ths. L Vn on

S: Khng c ng trn tha yu cu bi ton do h v nghim.


Bi 530. Hc Vin Bu Chnh Vin Thng nm 1999
x2
y2
+
= 1 vi tiu im F (c; 0) . Tm im M trn Ellipse (E) sao cho
a2
b2
di FM nh nht.

Cho Ellipse (E) :

S: FMmin A1 M (a; 0) (E) .


Bi 531. i hc Nng Nghip I khi A nm 2000

x 2 y2
+
= 1 . Hy vit phng trnh ng trn i qua cc giao im ca
9
4
x2
E
'
:
+ y2 = 1 .
Ellipse cho vi Ellipse ( )
16
Cho Ellipse (E) :

S: (C) : x 2 + y2 =

92
.
11

Bi 532. i hc Nng Nghip I khi B nm 2000

x2
y2
+
= 1 . Hy chng minh rng tch cc khong cch t cc tiu im ca elp
9
4
n mt tip tuyn bt k ca n l mt hng s.
2/ Hy vit phng trnh ng trn i qua cc giao im ca elip cho vi elip
x2
+ y2 = 1 .
16

1/ Cho elp

x.x o y.yo
+
= 1 l tip tuyn bt k ti (x o ; yo ) .
S: d F1, (d) .d F2 ; (d) = 4 = const vi d :

9
4
Bi 533. i hc Thi Nguyn khi A, B t 1 nm 2000

x 2 y2
+
= 1.
Trong mt phng vi h trc ta Descarter vung gc Oxy, cho elip (E) :
25 16
1/ Tm mi lin h gia k v m ng thng d : y = kx + m, (k  ) tip xc vi (E) .
2/ Khi d l tip tuyn ca (E), gi giao im ca d vi cc ng thng x = 5 v x = 5 l
M v N. Tnh din tch FMN theo k, trong F l tiu im ca (E) c honh dng.
3/ Xc nh k tam gic FMN c din tch b nht.

S: 1/ ' = 0 m 2 25k 2 = 16 .
2
2
1  
1
4 + (m + 5k) 64 + (m 5k)
2/3/ S = FM . FN =
2
2

B.C.S

3
16 k = .
5

Bi 534. Hc Vin Ngn Hng Tp. H Ch Minh nm 2001


x2
y2
+
= 1 nhn cc ng thng 3x 2y 20 = 0 v x + 6y = 20 lm
a2
b2
tip tuyn. Hy tnh a 2 v b2 .

1/ Nu elip (E) :

"Cn c b thng minh"

DeThiThuDaiHoc.com

Page - 193 -

www.MATHVN.com

Ths. L Vn on

Phn hnh hc

x2
y2
+
= 1 . Hy tm mi lin h gia a, b, k, m (E) tip xc vi ng
a2
b2
thng d : y = kx + m, (k  ) .

2/ Cho elip (E) :

S: a2 = 40, b2 = 10, k2a2 + b2 = m2 .


Bi 535. i hc Ngoi Thng khi A nm 2001
Cho h ng cong (Cm ) c phng trnh
m 0, m 5 .

x2
y2
+
= 1 , trong m l tham s v
m2 m2 25

1/ Ty theo gi tr ca m, hy xc nh khi no th (Cm ) l elp v khi no th (Cm ) l


hyperbol.
2/ Gi s A l mt im ty trn ng thng x = 1 v A khng thuc trc honh. Chng
minh rng vi mi im A lun lun c bn ng cong ca h (Cm ) i qua A. Hi trong
s bn ng cong (Cm ) , c bao nhiu elp v bao nhiu hyperbol ?
2

2 : Ellipse
(Cm ) Elip m 25 > 0 m > 5

S: 1/
.
2/

ng
A
1;a
c
.

(
)
(Cm ) Hyperbol m2 25 < 0 m < 5
2 : Hyperbol

Bi 536. i hc Y Dc Tp. H Ch Minh nm 2001

x 2 y2
+
= 1 v
9
4
hai ng thng (D) : ax by = 0 v (D ') : bx + ay = 0 vi a 2 + b2 > 0 . Gi M, N l cc

Trong mt phng vi h trc ta Descarter vung gc Oxy, cho elip (E) :


giao im ca (D) vi (E) v P, Q l cc giao im ca (D ') v (E) .
1/ Tnh din tch t gic MNPQ theo a v b.
2/ Tm iu kin i vi a, b din tch t gic MPNQ nh nht.

S: 1/ SMPNQ =

72 a 2 + b2

(9a

)(

+ 4b2 9b2 + 4a 2

2/ (SMPNQ )

min

144
a 2 = b2 .
13

Bi 537. i hc khi D nm 2002

x 2 y2
+
= 1 . Xt im M chuyn ng
16
9
trn tia Ox v im N chuyn ng trn tia Oy sao cho ng thng MN lun tip xc vi (E) .

Trong mt phng vi h ta Oxy , cho elip (E) :

Xc nh ta ca M, N on MN c di nh nht. Tnh gi tr nh nht.

) (

S: M 2 7; 0 , N 0; 21 , MN min = 7 .
Bi 538. D b 1 i hc khi D nm 2002
Trong mt phng vi h ta Oxy , cho elip (E) :

x 2 y2
+
= 1 v ng thng
9
4

dm : mx y 1 = 0 .
1/ Chng minh rng vi mi gi tr ca m, ng thng dm lun ct elip (E) ti hai im phn
bit.
Page - 194 -

DeThiThuDaiHoc.com
"All the flower of tomorrow are in the seeks of today"

cng hc tp mn Ton 10 Tp IIwww.MATHVN.com

Ths. L Vn on

2/ Vit phng trnh tip tuyn ca (E), bit tip tuyn i qua im N (1; 3) .

S: 2/ 5x 4y 17 = 0; x + 2y + 5 = 0 .
Bi 539. D b 2 i hc khi B nm 2003

x 2 y2
+
= 1 v cc im
4
1
M (2; 3), N (5; n) . Vit phng trnh cc ng thng d1, d2 qua M v tip xc vi (E) . Tm

Trong mt phng vi h ta Oxy , cho elip (E) :

n trong s cc tip tuyn ca (E) i qua N c mt tip tuyn song song vi d1 hoc d2.

S: d1 : x = 2; d2 : 2x + 3y 5 = 0; n = 5 .
Bi 540. D b 2 i hc khi B nm 2004

x 2 y2
+
= 1 . Vit phng trnh cc tip
8
4
tuyn ca (E) song song vi ng thng d : x + 2y 1 = 0 .
Trong mt phng vi h ta Oxy , cho elip (E) :

Bi 541. i hc khi D nm 2005

x 2 y2
+
= 1 . Tm to cc
4
1
im A, B thuc (E) , bit rng hai im A, B i xng vi nhau qua trc honh v tam gic ABC
Trong mt phng vi h ta Oxy, cho im C (2; 0) v elip (E) :

l tam gic u.
2 4 3 2

, B ; 4 3 hoc A 2 ; 4 3 , B 2 ; 4 3 .
S: A ;

7 7 7
7
7
7 7 7
Bi 542. D b 1 i hc khi B nm 2005

x 2 y2
+
= 1 . Vit phng trnh tip tuyn
64
9
d ca (E) bit d ct hai hai trc ta Ox, Oy ln lt ti A, B sao cho AO = 2BO .
Trong mt phng vi h ta Oxy, cho elip (E) :

S: Bn tip tuyn tha yu cu b ton: x + 2y 10 = 0, x 2y 10 = 0 .


Bi 543. D b 2 i hc khi D nm 2006
Trong mt phng vi h ta Oxy, lp phng trnh chnh tc ca elip (E) c di trc ln
bng 4 2 , cc nh trn trc nh v cc tiu im ca (E) cng nm trn mt ng trn.
S: (E) :

x 2 y2
+
= 1.
8
4

Bi 544. i hc khi A nm 2008


Trong mt phng vi h ta Oxy, vit phng trnh chnh tc ca elip (E) bit rng (E) c
tm sai bng
S: (E) :

5
v hnh ch nht c s ca (E) c chu vi bng 20.
3

x 2 y2
+
= 1.
9
4

"Cn c b thng minh"

DeThiThuDaiHoc.com

Page - 195 -

www.MATHVN.com

Ths. L Vn on

Phn hnh hc

Bi 545. i hc khi B nm 2010 (Chng trnh nng cao)

x 2 y2
+
= 1 . Gi F1 v F2 l
3
2
cc tiu im ca (E) (F1 c honh m); M l giao im c tung dng ca ng thng

Trong mt phng ta Oxy, cho im A 2; 3 v elip (E) :

AF1 vi (E); N l im i xng ca F2 qua M. Vit phng trnh ng trn ngoi tip tam
gic ABF2.
2

2 3
4

S: (x 1) + y
= .

3
3
2

Bi 546. i hc khi A nm 2011 (Chng trnh nng cao)

x 2 y2
+
= 1 . Tm ta cc im A v B thuc
4
1
(E), c honh dng sao cho tam gic OAB cn ti O v c din tch ln nht.

Trong mt phng ta Oxy, cho elip (E) :

2
2
2
2

S: 1/ M (2; 4) M (3;1) . 2/ A 2; , B 2; A 2; , B 2; .
2
2
2
2

Bi 547. i hc khi A nm 2012 (Chng trnh nng cao)


Trong mt phng ta Oxy, cho ng trn (C) : x2 + y2 = 8 . Vit phng trnh chnh tc
ca elp (E) , bit rng (E) c di trc ln bng 8 v (E) ct (C) ti bn im to thnh
bn nh ca hnh vung.
S: (E) :

Page - 196 -

x2 3y2
+
= 1.
16
16

DeThiThuDaiHoc.com
"All the flower of tomorrow are in the seeks of today"

cng hc tp mn Ton 10 Tp IIwww.MATHVN.com

Ths. L Vn on

E PHNG TRNH NG HYPERBOL



 Kin thc c bn
a/ nh ngha
Cho

c nh vi

. Tp hp im M sao cho

vi a l mt s khng i v

c gi l mt Hyperbol

Vy

. Trong :
gi l hai tiu im ca Hyperbol.

+ Hai im

gi l tiu c ca Hyperbol.

+ Khong cch
+ Trung im I ca
+ Vi im

gi l tm ca Hyperbol.
th cc khong cch

gi l cc bn knh qua tiu ca

im M.
b/ Phng trnh chnh tc v tham s ca Hyperbol
Phng trnh chnh tc ca Hyperbol

vi
.

+ Ta cc tiu im:

+ Vi

. Trong :

l cc bn knh qua tiu im M th

Phng trnh tham s dng lng gic:


.

c/ Hnh dng ca hypebol

nhn cc trc to lm cc trc i xng

v gc to O lm tm i xng.
To cc nh:
di cc trc: trc thc:

Q
.

trc o:

O
R

Tm sai ca

Hnh ch nht c s: to bi cc ng thng:


Phng trnh cc ng tim cn:

"Cn c b thng minh"

DeThiThuDaiHoc.com

Page - 197 -

www.MATHVN.com

Ths. L Vn on

Phn hnh hc

x
MF1

M (H)

d (M, 1 )

MF2

d (M, 2 )

a
=0
e

=e

(H )
( H)

(H ) :

( H)

(H )
2a,
2

x2

a2

y2

2b
2

c =a +b
F1 (c; 0), F2 (c;0)

2c

=1

b2

A1 (a;0), A2 (a;0)
e=

O
R

c
a
y=
x

b
x
a

a
=0
e

(x )

(H) :

OI

(H ) :

(y )

a2

I (; )

X = x
x = X +

Y = y
y = Y +

IXY
(H )

b2

=1

IXY

X2
a2

Y2
b2

=1

(H )

Oxy

(H )
(H )

(H )
a 2 , b2

a, b

(H )
e=

b2 = c2 a 2

A1 (a; 0), A2 (a;0)

c
a

c/ Dng ton 3. V tr tng i ng thng v Hyperbol


Bng cch xt h phng trnh to bi

Page - 198 -

v ng thng

nghim ca phng trnh bng s giao im ca

F1 (c;0), F2 (c; 0)

. Khi , s
.

DeThiThuDaiHoc.com
"All the flower of tomorrow are in the seeks of today"

cng hc tp mn Ton 10 Tp IIwww.MATHVN.com

Ths. L Vn on

(H ) :

(H )
M ( x o ; y o ) (H )

x 2o
a2

y2o
b2

x2
a2

y2
b2

=1

=1

x o , yo

M ( x o ; yo )

(H )

x = a
3
H
:
( ) cos t , t 0;2) \ 2 ; 2
y = b tan t

M (H) M
; b tan t

cos t
M

M (x; y) (H)

cx
+a
a
cx
MF1 = a
a
MF1 =

cx
a
M
x>0
a
cx
MF2 = + a
x<0
M
a

MF2 =

( H)
M (x; y)

( H)

MF1 MF2 = 2a
x2
a2

y2
b2

(H)

=1

d : Ax + By + C = 0

(H ) :

=1

a2A2 b2B2 = C2

a2A2 + b2B2 > 0

(H )

Phng trnh tip tuyn ca Hyperbol


l

"Cn c b thng minh"

ti im
(phng php phn i ta ).

DeThiThuDaiHoc.com

Page - 199 -

www.MATHVN.com

Ths. L Vn on

Phn hnh hc

TM CC THUC TNH V LP PHNG TRNH HYPERBOL (H)


Bi 548. V th cc hypebol (H) sau

x 2 y2

= 1.
9
4
3/ (H) : 25x2 16y2 = 400 .

1/

(H) :

2/
4/

x2

y2
= 1.
9
(H) : x2 4y2 = 1 .

(H) : 16

Bi 549. Xc nh cc ta tiu im, cc nh, di trc thc, di trc o, tiu c, tm sai v vit
phng trnh cc ng tim cn ca hypebol (H) .
1/
3/
5/
7/

x 2 y2

= 1.
16
9
(H) : 4x2 y2 = 4 .

(H ) :

2/
4/

(H) : 25x2 16y2 = 400 .


(H) : 9x2 16y2 = 144 .

6/
8/

x 2 y2

= 1.
9
16
11/ (H) : x2 4y2 = 1 .

9/

x 2 y2

= 1.
16
4
(H) : x2 y2 = 1 .

(H ) :

(H) : 16x2 9y2 = 16 .


(H) : mx2 ny2 = mn, (m, n > 0) .

10/ (H) : 4x2 9y2 = 5 .

(H) :

12/ (H) : 9x2 25y2 = 1 .

Bi 550. Lp phng trnh chnh tc v tham s ca hypebol (H) . Bit rng


1/
2/
3/

di trc thc bng 10, trc o bng 6.


di trc thc bng 8, tiu c bng 10.
C mt tiu im F2 (5; 0) v nh A1 (4; 0) .

4/

di trc thc bng 48, tm sai bng

5/

di trc o bng 6, tm sai bng

6/

C nh A2 (2; 0) v tm sai e =

7/

C di trc thc bng 8 v tm sai e =

13
.
12

5
.
4

3
.
2
5
.
4

5
.
4
3
9/ Tiu im F1 (6; 0) v tm sai e = .
2
4
10/ C tiu c bng 16 v tm sai e = .
3
11/ Mt nh l A (5; 0) , mt tiu im l F (6; 0) .
8/

di trc o bng 12, tm sai bng

12/ Mt tiu im l F (7; 0) v tm sai e = 2 .


13/ Qua A

10; 6 v c tm sai e = 5 .

14/ Qua A (5; 3) v c tm sai e = 2 .


15/ Qua A (2;12) v c tiu im F1 (7; 0) .

Page - 200 -

DeThiThuDaiHoc.com
"All the flower of tomorrow are in the seeks of today"

cng hc tp mn Ton 10 Tp IIwww.MATHVN.com

Ths. L Vn on

( ) ( 6; 1) .
i qua hai im M (2; 6 ), N (3; 4) .
i qua hai im M (6; 1), N (8;2 2 ) .

16/ i qua hai im M 4; 6 , N


17/
18/

19/ C di trc thc bng 8 v mt tim cn l 5x 4y = 0 .

4
20/ C tiu c bng 2 13 v mt tim cn l y = x .
3
21/ Qua A

2;2 v c hai tim cn l y = 2x .

22/ Mt nh l A (3; 0) v mt tim cn l d : 2x + 3y = 0 .


23/ Hai tim cn l d : 2x y = 0 v khong cch gia hai ng chun bng
24/ Tiu c bng 8 v hai tim cn vung gc vi nhau.
25/ Qua A (6; 3) v gc gia hai tim cn l 60 0 .

2 5
.
5

26/ Hai tim cn l d : 3x 4y = 0 v hai ng chun l : 5x 16 = 0 .


27/ C cng tiu im vi elip (E) : 10x2 + 36y2 360 = 0 , tm sai bng

34 9

; v MF1F2 vung ti M.
28/ Qua A
5 5

5
.
3

29/ C nh A2 (3; 0) v ng trn ngoi tip hnh ch nht c s l (C) : x2 + y2 = 16 .


30/ C cng hnh ch nht c s vi elp (E) :

x2
y2
+
= 1.
9
4

5
v din tch ca hnh ch nht c s l 48 (.v.d.t) .
3
32/ C hai tim cn l y = x v c hai tiu im l hai tiu im ca elp
31/ C tm sai e =

x2

y2
= 1.
2
Bi 551. Tm tm sai ca Hyperbol bit gc hp bi tim cn v trc honh bng
1/ 30 0 .
2/ 60 0 .
3/ 45 0 .
Bi 552. Tnh tm sai ca Hyperbol (H) bit:

(E) : 12 +

1/

Hai tim cn vung gc nhau.

2/

Gc gia hai tim cn bng

Bi 553. Tnh gc , 0 < hp bi hai tim cn ca Hyperbol c


2

1/ Tm sai e = 2 .
2/ Tiu c bng bn ln khong cch t mt tiu im n mt tim cn.

Bi 554. Cho Hyperbol (H) :


1/

x2

.
3

y2

= 1.
a 2 b2
Lp cng thc tnh gc to bi hai ng tim cn ca (H) theo a, b . p dng tnh

khi bit tm sai ca (H) bng 3 .


2/

Tm tm sai ca (H) khi bit gc gia hai tim cn ca (H) bng 30 0 .

"Cn c b thng minh"

DeThiThuDaiHoc.com

Page - 201 -

www.MATHVN.com

Ths. L Vn on

Phn hnh hc

TM IM TRN HYPERBOL (H)THA IU KIN CHO TRC S TNG


GIAO

1/
2/
3/
4/
5/

y2
= 1 . Tm nhng im M trn (H) sao cho
9
M nhn hai tiu im di mt gc vung.
M nhn hai tiu im di mt gc 60 0 .
M nhn hai tiu im di mt gc 45 0 .
M c ta nguyn.
M thuc nhnh phi v MF1 nh nht (ngn nht).

6/

M thuc nhnh phi v MF1 ln nht (di nht).

7/

Khong cch t M n ng thng : x y + 3 = 0 t gi tr ln nht, nh nht.

Bi 555. Cho Hyperbol (H) : x 2

Bi 556. Cho Hyperbol (H) : 4x2 y2 = 4 . Tm nhng im M trn (H) sao cho
1/
2/
3/
4/
5/

M nhn hai tiu im di mt gc vung.


M nhn hai tiu im di mt gc 1200 .
M nhn hai tiu im di mt gc 30 0 .
M c ta nguyn.
M thuc nhnh phi v MF1 nh nht (ngn nht).

6/

M thuc nhnh phi v MF1 ln nht (di nht).

7/

Khong cch t M n ng thng : x + y 2 = 0 t gi tr ln nht, nh nht.

1/
2/
3/
4/
5/

x2
y2 = 1 . Tm nhng im M trn (H) sao cho
4
M nhn hai tiu im di mt gc vung.
M nhn hai tiu im di mt gc 1200 .
M nhn hai tiu im di mt gc 60 0 .
M c ta nguyn.
M thuc nhnh phi v MF1 nh nht (ngn nht).

6/
7/
8/

M thuc nhnh phi v MF1 ln nht (di nht).


C bn knh qua tiu im ny bng hai ln bn knh qua tiu im kia.
Khong cch t M n ng thng : y = x + 1 t gi tr ln nht, nh nht.

Bi 557. Cho Hyperbol (H) :

Bi 558. Cho hypebol (H) v ng thng d vung gc vi trc thc ti tiu im bn tri F1 ct (H)
ti hai im M, N.
a/
Tm to cc im M, N.

b/

Tnh MF1, MF2, MN .

2/ (H) : 12x2 4y2 = 48 .


(H) : 16x2 9y2 = 144 .
3/ (H) : 10x2 + 36y2 360 = 0 .
4/ (H) : x2 4y2 = 4 .
Cho hypebol (H) . Tm nhng im M (H) sao cho:
1/

Bi 559.

a/

MF2 = 3MF1 .

b/

MF1 = 3MF2 .

c/

MF1 = 2MF2 .

e/

MF1 = 4MF2 .

1/

(H ) :

x 2 y2

= 1.
9
16

2/

(H ) :

x 2 y2

= 1.
4
12

3/

(H) :

x 2 y2

= 1.
4
5

4/

(H) :

x2
y2 = 1 .
4

Page - 202 -

DeThiThuDaiHoc.com
"All the flower of tomorrow are in the seeks of today"

cng hc tp mn Ton 10 Tp IIwww.MATHVN.com

Ths. L Vn on

Bi 560. Cho hypebol (H) . Tm nhng im M (H) nhn hai tiu im di mt gc vung, vi:
1/

(H ) :

x2
y2 = 1 .
4

2/

(H) :

x 2 y2

= 1.
9
4

x 2 y2
x 2 y2
4/ (H) :

= 1.

= 1.
4
12
9
16
Bi 561. Cho hypebol (H) . Tm nhng im M (H) nhn hai tiu im di mt gc , vi:
3/

(H) :

1/

(H ) :

3/

(H) : 16

x 2 y2

= 1, = 1200 .
4
5
x2

y2
= 1, = 600 .
9

x 2 y2

= 1, = 1200 .
36 13

2/

(H) :

4/

(H) : 16x2 9y2 = 144,

= 450 .

x 2 y2

= 1 . Tm im M trn hyperbol (H) sao cho


16
9
M nhn hai tiu im di mt gc vung.

Bi 562. Cho hyperbol (H) :


1/

24 2
.
5
Bi 563. Cho hyperbol (H) : 3x2 4y2 = 12 c hai tiu im l F1, F2 .
2/

Tng khong cch t M n hai ng tim cn bng

1/

Bit M l im trn (H) c MF1 = 3 . Hy tnh MF2 v tm ta im M.

2/

Tm trn mt nhnh ca (H) hai im P, Q sao cho OPQ l tam gic u.

1/

x 2 y2

= 1 v ng thng d : x 2 y 2 = 0 .
4
8
Chng minh rng d lun ct (H) ti hai im phn bit A, B . Tnh di on AB.

2/

Tm ta im C (H) sao cho

Bi 564. Cho hyperbol (H) :

a/ ABC c din tch bng 5.


b/ ABC cn.
c/ ABC vung.

1/

x 2 y2

= 1 v ng thng d : 2x + 15y 10 = 0 .
25
4
Chng minh rng d lun ct (H) ti 2 im phn bit A, B vi x A > 0 . Tnh di AB.

2/

Tm ta im C (H) sao cho ABC cn ti A.

Bi 565. Cho hyperbol (H) :

TNH CHT CA HYPERBOL (H) V MT S DNG TON KHC

1/

x2
y2 = 1 .
4
Xc nh tiu im v hai tim cn ca (H) .

2/

Ly ty M (x o ; yo ) (H) . Tnh cc khong cch t M n hai tim cn ca (H) .

Bi 566. Cho hyperbol (H) :

Bi 567. Cho hyperbol (H) :


1/

x2

y2

= 1.
a 2 b2
Gi M l im tu trn (H) . Chng minh tch cc khong cch t M n hai ng tim

cn bng mt s khng i v bng

"Cn c b thng minh"

a 2 b2
a 2 + b2

DeThiThuDaiHoc.com

Page - 203 -

www.MATHVN.com

Ths. L Vn on

2/

Phn hnh hc

T mt im N bt k trn (H), dng hai ng thng song song vi hai ng tim cn,
cng vi hai ng tim cn to thnh mt hnh bnh hnh. Chng minh din tch hnh
1
bnh hnh bng ab .
2

Bi 568. Cho hyperbol (H) :


1/

x 2 y2

= 1 c hai tiu im F1, F2 . Hy tnh:


25 16

H = OM2 MF1.MF2 .

Bi 569. Cho hyperbol (H) :

x2

S = (MF1 + MF2 ) 4OM2 .


2

y2

= 1 c hai tiu im F1, F2 v hai nh A1, A2 . im M di ng


a 2 b2
trn (H) v c hnh chiu xung trc honh Ox l P. Chng minh rng

1/

2/

OM2 MF1.MF2 = a 2 b2 .

2/

(MF1 + MF2 )

= 4 OM2 + b2 .

b2

3/

PM2 =

1/

Vit phng trnh cc ng chun ca (H) .

2/

Vit phng trnh cc ng tim cn ca (H) .

3/

Gi M l mt im bt k trn (H) . Chng minh tch cc khong cch t M n hai ng

.PA1.PA2 .
a2
Bi 570. Cho hypebol (H) : 9x2 16y2 144 = 0 .

tim cn bng mt s khng i.


4/ Tm im M trn (H) sao cho bn knh qua tiu im bn tri bng 2 ln bn knh qua tiu
im bn phi ca M.
5/


NF2 = 900 .
Tm im N trn (H) sao cho F
1

6/

Chng minh rng nu mt ng thng d ct (H) ti P, Q v ct hai ng tim cn ti


P ', Q ' th PP ' = QQ ' .

x 2 y2

= 1 . Chng t rng d ct (H)


9
16
ti hai im phn bit v xc nh ta cc giao im. Tnh di on ni hai giao im.

Bi 571. Cho ng thng d : x + y 3 = 0 v hyperbol (H) :

1/

x 2 y2

= 1 v ng thng d : x y + m = 0 .
4
5
Chng minh: d lun ct (H) ti 2 im M, N thuc hai nhnh khc nhau vi x M < x N .

2/

Gi F1, F2 l hai tiu im ca (H) . Tm m NF2 = 2MF1 .

Bi 572. Cho hyperbol (H) :

Bi 573. Cho hyperbol (H) : 8x2 y2 = 8 v ng thng d : 2x y + m = 0 .


1/

Chng minh: d lun ct (H) ti 2 im M, N thuc hai nhnh khc nhau vi x M < x N .

2/

Gi F1, F2 l hai tiu im ca (H) . Tm m NF2 = 2MF1 .

x2
y2 = 1 . Lp phng trnh ng thng d i qua M (0;2) ct (H) ti
4
hai im phn bit A, B sao cho M l trung im ca AB.

Bi 574. Cho hyperbol (H) :

Page - 204 -

DeThiThuDaiHoc.com
"All the flower of tomorrow are in the seeks of today"

cng hc tp mn Ton 10 Tp IIwww.MATHVN.com

Ths. L Vn on

x2
y2 = 1 . Lp phng trnh ng thng d i qua M (0;2) ct (H) ti
4

 
hai im phn bit A, B sao cho 3MA 5MB = 0 .

Bi 575. Cho hyperbol (H) :

x 2 y2
x2
y2

= 1 v elp (E) :
+
= 1 . Chng minh rng cc
16
9
25
9
tip im ny cng nm trn mt ng trn. Vit phng trnh ng trn ny.

Bi 576. Tm ta giao im ca (H) :

x 2 y2

= 1 . Gi d l ng thng qua gc ta O c h s gc k v d '


4
9
l ng thng cng i qua O nhng vung gc vi d.
1/ Tm iu kin ca k d v d ' u ct hyperbol (H) .

Bi 577. Cho hyperbol (H) :

2/

Tnh theo k din tch hnh thoi vi 4 nh l 4 giao im ca d, d ' v (H) .

3/

Xc nh k hnh thoi y c din tch nh nht.

Bi 578. Lp phng trnh ng trn i qua giao im ca (E) :

(x 1)
:

x2
y2
y2
+
= 1 v (H) : x 2
= 1.
9
4
4

Bi 579. Cho elp c phng trnh (E)


thuc vo elp (E) .

1/

+ (y + 2) = 1 v im A c x A =
2

3
v yA > 0
2

Lp phng trnh chnh tc ca Hyperbol (H) c mt tiu im F1 (1; 2), tm I (1; 2)

v i qua im A.
2/ Xc nh ta giao im ca (H) v (E) .

QU TCH IM - TP HP IM (H)
3

Bi 580. Cho M
; cot vi k . Tm tp hp im M.

sin
a

Bi 581. Cho M
; b tan vi (2k + 1) . Tm tp hp im M.

cos
2

3
. Tm tp hp im M tha: 3.PF = 3.d (M, ) .
2
Bi 583. Cho im F (3; 0) v : 3x + 4 = 0 . Tm tp hp im M sao cho 2MF = 3.d (M, ) .
Bi 582. Cho im F (2; 0) v : x =

1
= 0.
4
1/ Tm tp hp im M sao cho MB = 2MH vi H l hnh chiu vung gc ca M trn .
2/ Tm tp hp im N sao cho cc ng thng AN, BN khng song song vi Oy v c tch
cc h s gc bng 4.
Bi 585. Cho hai ng trn (C1 ) c tm l F1 v bn knh R1, (C2 ) c tm l F2 v bn knh R2
Bi 584. Cho hai im A (1; 0), B (1; 0) v ng thng : x

(R1 R 2 ) . Gi M l tm ng trn (C) thay i nhng lun tip xc vi (C1 ) v (C2 ) .


Tm tp hp im M.

Bi 586. Cho im M di ng trn hyperbol (H) :

x 2 y2

= 25 . Tm tp hp trung im ca on
16
9

MF1 v MF2 vi x F < x F .


1

"Cn c b thng minh"

DeThiThuDaiHoc.com

Page - 205 -

Ths. L Vn on

www.MATHVN.com

Phn hnh hc

Bi 587. Cho hyperbol (H) : x2 y2 = 1 v ng thng d : 5x 3y 1 = 0 . Tm im M (H) sao


cho khong cch t M n ng thng d l nh nht.

Bi 588. Cho hai ng trn (C1 ) : (x 3) + y2 = 9 v (C2 ) : (x + 3) + y2 = 4 . Tm tp hp tm M


2

ca ng trn tip xc ng thi vi hai ng trn ni trn.

Bi 589. Cho ng trn (C) : x2 + y2 + 4x = 0 v im F2 (2; 0) .


1/

Tm to tm F1 v bn knh R ca (C) .

2/

Tm tp hp cc tm M ca ng trn (C ') di ng lun i qua F2 v tip xc vi (C) .

3/

Vit phng trnh ca tp hp trn.

1/

Xc nh tm v tnh bn knh ca (C) v (C ') .

2/

Tm tp hp cc tm M ca ng trn (T) tip xc vi (C) v (C ') .

Bi 590. Cho hai ng trn (C) : x2 + y2 + 10x + 9 = 0 v (C ') : x2 + y2 10x + 21 = 0 .

3/ Vit phng trnh ca tp hp trn.


Bi 591. Cho hai ng thng : 5x 2y = 0 v ' : 5x + 2y = 0 .

100
.
29

1/

Tm tp hp (H) cc im M c tch cc khong cch t M n v ' bng

2/

Vit phng trnh cc ng tim cn ca (H) .

3/

Gi N l mt im bt k trn (H) . Chng minh tch cc khong cch t N n cc ng


tim cn ca (H) bng mt s khng i.

Bi 592. Tm tp hp cc im M c t s cc khong cch t n im F v n ng thng


bng e, vi:
1/

F (4; 0), : x 1 = 0, e = 2 .

2/

F (6; 0), : 3x 8 = 0, e =

Page - 206 -

3
.
2

2/
2/

3 2
3 2
,e=
.
2
3
3
F (3; 0), : 3x 4 = 0, e = .
2
F 3 2; 0 , : x

DeThiThuDaiHoc.com
"All the flower of tomorrow are in the seeks of today"

cng hc tp mn Ton 10 Tp IIwww.MATHVN.com

Ths. L Vn on

BI TP QUA CC K THI
Bi 593. Cao ng Marketing Tp. H Ch Minh nm 1997
Cho hyperbol (H) :

x 2 y2

= 1.
16
9

1/ Tm di trc o, trc thc, tm sai, tiu im F1, F2 ca (H) .


2/ Tm trn (H) nhng im sao cho MF1 MF2 .

5
.
4

S: 1/ 2a = 8, 2b = 6, F1,2 (5; 0), e =

4 34

9
9

4 34
; , M3,4
; .
2/ M1,2
5

5
5
5

Bi 594. Cao ng Cng Nghip H Ni nm 2003

x 2 y2

= 1.
16
9
Lp phng trnh elip (E) , bit rng (E) c cc tiu im ca (H) v (E) ngoi tip hnh ch
Trong mt phng vi h trc ta cc vung gc Oxy, cho hypebol (H) :
nht c s ca (H) .
Bi 595. Cao ng Kinh T Ti Chnh nm 2005

x 2 y2

= 1 . Vit phng trnh


Trong mt phng vi h trc ta Oxy, cho hyperbol (H) :
25
9
tip tuyn vi hyperbol (H), bit tip tuyn i qua im A (10;6) .
Bi 596. Cao ng Kinh T K Thut Cng Nghip I nm 2007
Trong mt phng vi h trc ta Descarter vung gc Oxy, cho hypebol (H) c phng

x 2 y2

= 1 v im M (2;1) . Vit phng trnh ng thng d i qua M, bit rng


2
3
ng thng ct (H) ti hai im A, B m M l trung im ca AB.

trnh:

S: d : 3x y 5 = 0 .
Bi 597. i hc Kinh Ti hc S Phmi hc Kin Trci hc Nng Nghip nm 1981
Trong mt phng vi h trc ta Descarter vung gc Oxy, cho h hyperbol c phng trnh
(Cm ) : m2 4 x2 4y2 = 4 m2 4 vi m l tham s.

1/ Ty theo m, hy ch r bn cht (Cm ) v cho bit tm, nh, tiu im, tim cn nu c.

2/ V (C0 ) v C2

) trn cng mt hnh.

S: m = 2 (Cm ) : y2 = 0,
x2
m (2;2) (Cm ) : 2 +
2

m = 0 (Cm ) : x 2 + y2 = 4,

y2

4m

= 1;

m < 2 (C ) : x
m
m > 2
22

y2
4 m2

= 1.

Bi 598. i hc Dn Lp Hng Vng nm 1996

"Cn c b thng minh"

DeThiThuDaiHoc.com

Page - 207 -

www.MATHVN.com

Ths. L Vn on

Phn hnh hc

x2
y2 = 1 .
4
1/ Xc nh tiu im. Vit phng trnh cc tim cn.

Cho hyperbol (H) :

2/ Cho M (x o ; yo ) (H) . Tnh tch s t M n hai tim cn.

) (

S: 1/ F1 5; 0 , F2

1
5; 0 , d1,2 : y = x .
2

2/ d (M, d1 ).d (M, d2 ) =

4
.
5

Bi 599. i hc Dn Lp Thng Long nm 1996


Vi mi t m

< t < , xt M (t) c ta


2
2

1
; y = 3 tan t . Chng minh:
x =

cos t

1/ Khi t thay i, im M (t) vch nn mt nhnh ca hyperbol. Xc nh ta cc tiu im


ca hyperbol .
2/ Chng minh iu kin cn v ng thng ni hai im phn bit M (t1 ), M (t2 ) i
qua mt tiu im ca (H) l tan

S: 1/ Nhnh phi (H) : x2

t1
2

.tan

t2

1
= .
2
3

y
= 1, (x > 0) c F (2; 0) .
3

u = tan t1

2 .
2/ t

t2
v = tan
2

Bi 600. i hc Nng Nghip I nm 1997 h cha phn ban


Cho elip (E) : 4x 2 + 16y2 = 64 .
1/ Xc nh cc tiu im F1, F2, tm sai v v (E) .
2/ M l mt im bt k trn elip. Chng t rng t s khong cch t M n tiu im phi F2
8
v ti ng thng x =
c gi tr khng i.
3
3/ Cho ng trn (C) : x2 + y2 + 4 3x 4 = 0 . Xt ng trn (C ') di ng nhng lun
i qua tiu im phi F2 v tip xc vi ng trn (C) . Chng t rng tm N ca ng
trn (C ') nm trn mt hyperbol c nh. Vit phng trnh ca hyperbol .
S: 1/ F1;2

3
.
2 3; 0 , e =
2

MF
3
2/
.
=
MH
2

x 2 y2

= 1,
3/ (H) :
4
8

2 3

.
x
3

Bi 601. i hc Giao Thng Vn Ti 2 nm 1997


Cho hyperbol (H) : x2 y2 = 8 . Vit phng trnh chnh tc ca Elip (E) i qua im
A (4;6) v c tiu im trng vi tiu im ca hyperbol cho.

S: (E) :

x2
y2
+
= 1.
64 48

Bi 602. i hc Kin Trc H Ni (C S Th c Tp. H Ch Minh) nm 1997

Page - 208 -

DeThiThuDaiHoc.com
"All the flower of tomorrow are in the seeks of today"

cng hc tp mn Ton 10 Tp IIwww.MATHVN.com

Ths. L Vn on

x 2 y2

= 1.
Cho hyperbol (H) :
16
4
1/ Vit phng trnh ng thng d l tip tuyn ca (H) v i qua im A (2; 1) .

M1F2 (F1, F2 l tiu
2/ Gi M1 l tip im ca (H) v d. Chng minh d l phn gic gc F
1

im ca hyperbol).
3/ Tnh th tch vt th trn xoay do quay min phng gii hn bi (H), d, trc Ox quanh Oy.


d (H) = M 5; 3

MF
BF1
21
2
S: 1/ d : 5x 6y 16 = 0 . 2/ CM: 1 1 =
vi
. 3/ V =
.

75
M1F2
BF2
d Ox = B 16 ; 0
5

Bi 603. i hc Dc H Ni nm 1997 h cha phn ban

x 2 y2

= 1.
a 2 b2
1/ Tm tp hp im trong mt phng Oxy sao cho t mi im k c hai tip tuyn vi
(H) v tip tuyn y vung gc vi nhau.
Cho hyperbol (H) :

2/ M l im bt k trn (H) . Gi 1, 2 l hai ng thng i qua M v tng ng song


song vi hai ng tim cn ca (H) . Chng minh rng din tch hnh bnh hnh c gii
hn bi 1, 2 v hai ng tim cn l mt s khng i.

b
S: 1/ x 2 + y2 = a 2 b2 , y .
a

2/ S =

(a

a 2 b2
2

+ b sin
2


vi = (1, 2 ) .

Bi 604. i hc Nng khi B nm 1997 h cha phn ban


Lp phng trnh chnh tc ca hyperbol (H) vi Ox l trc thc, tng hai bn trc a + b = 7,

3
phng trnh hai tim cn: y = x .
4
1/ Tnh di cc bn trc, v hyperbol (H)
2/ Lp phng trnh tip tuyn ca (H) song song vi ng thng d : 5x 4y + 10 = 0 .
S: 1/ (H) :

x 2 y2

= 1.
4 2 32

2/ 5x 4y + 4 = 0,

5x 5y 4 = 0 .

Bi 605. Hc Vin Cng Ngh Bu Chnh Vin Thng nm 1999


Hai ng cong c gi l trc giao nhau ti giao im ca chng khi v ch khi ti hai
tip tuyn tng ng ca hai ng cong vung gc vi nhau. Chng minh hai ng cong sau
b
y trc giao nhau ti giao im ca chng: x2 y2 = a v y = vi a, b = const .
x
HD: Tm ta giao im ca (H) : x2 y2 = a v d : y =

b
.
x

Gi A (x o ; y o ) l mt trong cc giao im .

"Cn c b thng minh"

DeThiThuDaiHoc.com

Page - 209 -

www.MATHVN.com

Ths. L Vn on

Phn hnh hc

Xc nh h s gc k1 ca tip tuyn d1 ca (H) ti A.


Xc nh h s gc k2 ca tip tuyn d2 ca (H) ti A.
Chng t k1k2 = 1 .
Bi 606. i hc Lut H Ni nm 2000
Tm tp hp im M (x; y) trong h ta Oxy sao cho khong cch t M n im F (0; 4)
bng hai ln khong cch t M n ng thng y = 1 . Tp hp l ng g ?
S: Tp hp im M (x; y) l hyperbol (H) :

y2 x 2

= 1.
4 12

Bi 607. i hc Nng Nghip I khi A nm 2000


Trong mt phng vi h trc ta Oxy, cho cc im A (2; 0), B (2; 0), M (x; y) .


1/ Xc nh ta im M, bit rng M nm pha trn trc honh, so o gc AMB = 900 v

s o gc MAB = 300 .

2/ Khi M chuyn ng trong mt phng ta sao cho AMB c s o gc MBA gp hai ln

s o gc MAB . Chng minh rng M chy trn mt nhnh ca ng hyperbol. Xc nh
ta tiu im ca nhnh hyperbol .

S: 1/ M 1; 3 .

2
x + 2 = X, y = Y
9
2
3y2
X2 Y2
3
= 1 . t
2/ (H) : x +
(H ) : 2 2 = 1 .
16
16
3
16
a
b
2 16
= b2
= a ,
3
9

Bi 608. i hc Kin Trc Tp. H Ch Minh khi V nm 2001


Chng minh rng tch cc khong cch t mt im bt k ca hyperbol (H) :
n cc tim cn ca n l mt s khng i.
S: d1.d2 =

x 2 y2

=1
a 2 b2

a 2 b2
= hng s.
a 2 + b2

Bi 609. D b 1 i hc khi A nm 2006

x 2 y2
+
= 1 . Vit phng trnh hypebol
12
2
(H) c hai ng tim cn l y = 2x v c hai tiu im l hai tiu im ca elip (E) .

Trong mt phng vi h ta Oxy, cho elip (E) :

S: (H) :

Page - 210 -

x 2 y2

= 1.
2
8

DeThiThuDaiHoc.com
"All the flower of tomorrow are in the seeks of today"

cng hc tp mn Ton 10 Tp IIwww.MATHVN.com

Ths. L Vn on

F PHNG TRNH NG PARABOL



 Kin thc c bn
a/ nh ngha
Trong mt phng Oxy cho im c nh F v ng thng c nh . Parabol

l tp hp

nhng im M ca mt phng cch u mt ng thng c nh v mt im F c nh


khng thuc .
vi H l hnh chiu ca M ln v
im F c gi l tiu im.
ng thng c gi l ng chun.

S (trung im ca FL) gi l nh ca

ng thng FL c gi l trc i xng ca

gi l tham s tiu ca

b/ Phng trnh chnh tc ca parabol


Cho parabol

c tiu im

v ng chun

vi

Ta tiu im

Phng trnh ng chun


Vi

c phng trnh

th bn knh qua tiu im ca M l

c/ Hnh dng ca parabol

nm v pha bn phi ca trc tung.

nhn trc honh lm trc i xng.

To nh:

Tm sai:

Ngoi dng chnh tc

ngi ta cng xem cc dng phng

trnh sau l phng trnh chnh tc ca parabol:

hay

"Cn c b thng minh"

DeThiThuDaiHoc.com

Page - 211 -

Ths. L Vn on

www.MATHVN.com

Phn hnh hc

TM CC THUC TNH PARABOL LP PHNG TRNH PARABOL


Bi 610. Tm ta tiu im, phng trnh ng chun v bn knh qua tiu im ng vi im
M (P) c x M = 2 ca cc parabol (P) sau
1/
3/
5/
7/
9/

(P) : y2 = x .
(P) : y2 = 4x .
(P) : y2 = 8x .
(P) : y2 + 6x = 0 .
(P) : 3y2 + 12x = 0 .

(P) : y2 = 2x .
4/ (P) : y2 = 6x .
6/ (P) : y2 = 16x .
8/ (P) : 3x2 + 12y = 0 .
10/ (P) : 3x2 12y = 0 .
2/

Bi 611. Lp phng trnh chnh tc ca parabol (P) bit


1/
2/
3/
4/
5/
6/
7/
8/
9/
10/
11/
12/
13/

(P) c tiu im F (2; 0) .


(P) c tiu im F (3; 0) .
(P) c tiu im F (4; 0) .
(P) i qua im M (6; 2) .
(P) i qua im M (1; 4) .
(P) i qua im M (1; 2) .
(P) c ng chun x = 3 .
(P) c ng chun x + 2 = 0 .
(P) c nh A (1; 3) v ng chun d : x 2y = 0 .
(P) c khong cch t tiu im n ng chun bng 2.
(P) c khong cch gia nh v tiu im bng 3.
(P) c khong cch t nh n ng chun bng 2.
(P) c tham s tiu bng 5.

5
.
2
15/ (P) c tiu im F trng vi tiu im bn phi ca elip (E) : 5x2 + 9y2 = 45 .

14/

16/
17/
18/

(P) qua im M vi x M = 2 v khong t M n tiu im l

(P) c tiu im F trng vi tiu im bn phi ca hypebol (H) : 16x2 9y2 = 144 .
(P) c tiu im F trng vi tm ca ng trn (C) : x2 6x + y2 + 5 = 0 .
(P) ct ng thng d : x + 2y = 0 ti hai im M, N v MN = 4 5 .

19/ di dy cung vung gc vi Ox l 8 v khong cch t dy cung n nh l 4.


20/ Mt dy cung ca (P) vung gc vi Ox c di bng 8 v khong cch t nh O ca

21/
22/

(P) n dy cung ny bng 1.


(P) ct ng phn gic ca gc phn t th nht ti hai im A, B v AB = 5 2 .
(P) c nh A (1; 2) v (P) chn trn ng thng d : y = x + 1 mt dy cung
MN = 34 .

Page - 212 -

DeThiThuDaiHoc.com
"All the flower of tomorrow are in the seeks of today"

cng hc tp mn Ton 10 Tp IIwww.MATHVN.com

Ths. L Vn on

TM IM THUC PARABOL THA IU KIN CHO TRC TNG


GIAO
Bi 612. Cho parabol (P) : y2 = 4x v im M (x o ; yo ) (P) .
1/
2/
3/

Tnh khong cch t M n tiu im F.


Tm ta ca im M bit MF = 2 .
ng thng d qua F ct (P) ti hai im A v B. Chng minh: AB = x A + x B + 2 .

Bi 613. Tt nghip THPT nm 2005


Trong mt phng Oxy, cho parabol (P) : y2 = 8x .
1/

Tm to tiu im v vit phng trnh ng chun ca (P) .

2/

Vit phng trnh tip tuyn ca (P) ti im M thuc (P) c tung bng 4.

3/

Gi s ng thng d i qua tiu im ca (P) v ct (P) ti hai im phn bit A, B c


honh tng ng l x1, x2 . Chng minh: AB = x1 + x2 + 4 .

Bi 614. Cho parabol (P) : y2 = 12x .


1/

ng thng d vung gc vi trc i xng ca parabol (P) ti tiu im F v ct (P)

2/

ti hai im M, N. Tnh di on MN.


ng thng d qua tiu im F ca (P) v ct (P) ti hai im A, B. Chng minh rng
tch s y A .y B l hng s.

Bi 615. Cho parabol (P) v ng thng d vung gc vi trc i xng ti tiu im F ct (P) ti hai
im M, N.
a/ Tm to cc im M, N.

(P) : y2 = 6x .
3/ (P) : y2 = 16x .
Cho parabol (P) .

1/

Bi 616.

b/

Tnh MF, MN .

(P) : y2 = 2x .
(P) : y2 = x .

2/
4/

a/

Tm nhng im M (P) cch tiu im F mt on bng k.

b/

Chn M c tung dng. Tm im A (P) sao cho AFM vung ti F.

2/ (P) : y2 = 2x, k = 5 .
(P) : y2 = 8x, k = 10 .
3/ (P) : y2 = 16x, k = 4 .
4/ (P) : y2 = 4x, k = 9 .
Cho parabol (P) v ng thng d c h s gc m quay quanh tiu im F ca (P) ct (P) ti
1/

Bi 617.

Bi 618.

hai im M, N.
a/ Chng minh x M .x N khng i.

b/

Tnh MF, NF, MN theo m.

1/

2/

(P) : y2 = 2x .
(P) : y2 = x .

(P) : y2 = 4x .
3/ (P) : y2 = 16x .
Cho parabol (P) : y2 = 4x

4/

v ng thng d : 2x y 4 = 0 . Tm cc im M d t

1/

Khng k c tip tuyn no n n (P) .

2/

K c mt tip tuyn n (P) .

"Cn c b thng minh"

DeThiThuDaiHoc.com

Page - 213 -

www.MATHVN.com

Ths. L Vn on

3/

Phn hnh hc

K c hai tip tuyn n (P) .

Bi 619. Cho parabol (P) : y2 = x v ng thng d : x y 2 = 0 .


1/

Xc nh ta giao im A, B ca d v (P) .

2/

Tm ta im C (P) sao cho

3/

a/ ABC c din tch bng 6.


b/ ABC u.
Tm im M trn cung AB ca parabol (P) sao cho tng din tch hai phn hnh phng
gii hn bi (P) v hai dy cung MA, MB l nh nht.

Bi 620. Cho parabol (P) : y2 = 4x v ng thng d : 2x y 4 = 0 .


1/

Xc nh ta giao im A, B ca d v (P) .

2/

Tm im X trn (P) sao cho XF = 5 .

3/

Tm ta im C (P) sao cho

4/

a/ ABC c din tch bng 3 10 .


b/ ABC u.
c/ ABC vung.
Tm im M trn cung AB ca parabol (P) sao cho tng din tch hai phn hnh phng
gii hn bi (P) v hai dy cung MA, MB l nh nht.

5/

Tm im N trn (P) c bn knh qua tiu im bng 10 v tung dng.

6/

Tm im Q trn (P) sao cho ONQ vung ti O.

TP HP IM QU TCH
Bi 621. Tm tp hp cc tm M ca ng trn (C) di ng lun i qua im F v tip xc vi ng
thng , vi:
1/ F (2; 0), : x + 2 = 0 .
3/

2/

F (3; 0), : x + 3 = 0 .

F (1; 0), : x + 1 = 0 .

Bi 622. Cho parabol (P) . ng thng d quay quanh O ct (P) ti im th hai l A. Tm tp hp ca



 
a/ Trung im M ca on OA.
b/ im N sao cho NA + 2NO = 0 .
1/
3/

(P) : y2 = 16x .
(P) : y2 = 2x .

2/
4/

(P) : y2 = 4x .
(P) : y2 = x .

BI TP QUA CC K THI
Bi 623. Cao ng Bo Ch Marketting khi A nm 2000
Trong mt phng ta Oxy, cho parabol (P) : y2 = 8x .
1/ Xc nh ta tiu im F v phng trnh ng chun parabol (P) .
2/ Cho im A (0;2) . Vit phng trnh tip tuyn vi parabol (P) , bit rng tip tuyn i qua
im A.
Page - 214 -

DeThiThuDaiHoc.com
"All the flower of tomorrow are in the seeks of today"

cng hc tp mn Ton 10 Tp IIwww.MATHVN.com

Ths. L Vn on

Bi 624. Cao ng Cng ng Tin Giang nm 2003


Trong mt phng vi h trc ta cc Oxy, cho parabol (P) : y = 4x2 . T im M bt k
trn ng chun ca Parabol v hai tip tuyn n (P) , gi T1, T2 l cc tip im. Chng
minh rng T1, T2 v tiu im F ca (P) thng hng.
Bi 625. Cao ng Kinh T K Hoch Nng nm 2004
Trong mt phng vi h trc ta Descarter vung gc Oxy, hy vit phng trnh ca
Parabol c tiu im F (2;2) v ng chun : y = 4 .
S:

(P) : y = 21 x

x +2.

Bi 626. Cao ng S Phm Tr Vinh khi A nm 2005


Trong mt phng vi h trc ta Descarter vung gc Oxy, cho parabol (P) : y2 = x v
im M (1;1) thuc (P) . Vit phng trnh ng trn (C) c tm I thuc ng thng

d : 2x y = 0 v tip xc vi tip tuyn ca (P) ti M.


2
2

3
3
5

S: (C) : x + y =
.
4
2
16

Bi 627. i hc Kinh T i hc Ti Chnh i hc S Phm nm 1980


Trong mt phng vi h trc ta Oxy, cho parabol (P) : y2 4y 8x = 4 .
1/ Xc nh tiu im v v parabol (P) .
2/ Vit phng trnh ng thng cha dy AB ca (P) sao cho im M (0; 4) l trung im
ca on AB.
3/ Tnh din tch hnh phng gii hn bi (P) v dy AB.
S: 1/ (P) : (y 2) = 8 (x + 1) c nh S (1;2) v tiu im F (1;2) .
2

2/ AB : y = 2x + 4 .

3/ S =

2 (y 4) 8 (y 2)
1


8 .dy .

Bi 628. i hc Y Nha Dc nm 1981


Trong mt phng vi h trc ta Descarter vung gc Oxy, cho parabol (P) : y2 = 2x . Gi

t2
F l tiu im ca (P), M l im c ta ; t, (t 0) v N l giao im ca ng MF
2
vi (P), (N M) .
1/ Tnh theo t cc ta trung im ca I ca on MN.
2/ Tm qu tch I khi t thay i (t 0) .

t4 + 1 t2 1

, (t 0 t 1) . 2/ Parabol (P ') : y2 = x 1 b im F 1 ; 0 .
;
S: 1/ I
2
2
2
4t
2
2t
"Cn c b thng minh"

DeThiThuDaiHoc.com

Page - 215 -

www.MATHVN.com

Ths. L Vn on

Phn hnh hc

Bi 629. i hc Bch Khoa i hc Tng Hp Tp. H Ch Minh nm 1982


Trong mt phng vi h trc ta Descarter vung gc Oxy, cho A (2; 0) v im M di
chuyn trn ng trn (C) tm O bn knh bng 2, cn im H l hnh chiu vung gc ca
M ln trc tung.

 
1/ Tnh ta ca giao im P ca cc ng thng OM v AH theo gc = OA, OM .

2/ Xc nh v v qu tch ca P khi M chy trn (C) . Cho bit cc c im ca qu tch ny.

2 cos
2 sin + k2
,
S: 1/ P
;
.
1 + cos 1 + cos k 

2/ (P) : y2 = 4 (x 1) tr nh S (0;1) .

Bi 630. i hc Bch Khoa i hc Tng Hp Tp. H Ch Minh nm 1992


Trong mt phng vi h trc ta Descarter vung gc Oxy, cho im F (3; 0) v ng
thng d c phng trnh: 3x 4y + 16 = 0 .
1/ Tnh khong cch t F n d. T suy ra phng trnh ng trn tm F v tip xc vi d.
2/ Vit phng trnh parabol (P) c tiu im F v nh l gc ta O. Chng minh rng
parabol tip xc vi d. Tm ta tip im.
S: 1/ d (F, d) = 5,

(x 3)

+ y2 = 25 .

16
2/ (P) : y2 = 12x, tip xc ti A ; 8 .
3

Bi 631. i hc Y Dc Tp. H Ch Minh nm 1994


Trong mt phng vi h trc ta Descarter vung gc Oxy, xt hnh b chn pha di bi
parabol (P) : y = x 2, b chn pha trn bi ng thng i qua im M (1; 4) v c h s gc k.
Xc nh k hnh ni trn c din tch nh nht v khi tm tad giao im ca ng
thng v parabol.
S: Smin =

) (

13
12 k = 2 khi A 1 + 3; 4 + 2 3 , B 1 3; 4 2 3
6

Bi 632. i hc Nng Lm nm 1994


Cho parabol (P) : y = x2 v hai im A, B di ng trn (P) sao cho AB = 2a vi a l s
nguyn dng cho trc. Xc nh v tr ca A, B din tch min phng gii hn bi ng
thng AB v (P) t gi tr ln nht.

) (

) (

S: A a; a 2 , B a;a 2 , a + .
Bi 633. i hc Kinh T Tp. H Ch Minh nm 1994 i hc M a Cht nm 1998
Cho M l mt im thuc Parabol (P) : y2 = 64x v N l mt im thuc ng thng
d : 4x + 3y + 46 = 0 .

1/ Xc nh M, N on MN ngn nht.
2/ Vi kt qu tm c cu 1/, chng t rng khi MN vung gc vi tip tuyn ti M
ca Parabol (P) .
37 126
.
S: 1/ M (9; 24), N ;
5
5
Page - 216 -

2/ Hin nhin MN d .

DeThiThuDaiHoc.com
"All the flower of tomorrow are in the seeks of today"

cng hc tp mn Ton 10 Tp IIwww.MATHVN.com

Ths. L Vn on

Bi 634. i hc Hu nm 1995
Trong mt phng vi h trc ta Descarter vung gc Oxy, cho parabol (P) : y =
ng thng d : 2mx 2y + 1 = 0 .

1 2
x v
2

1/ Chng minh rng vi mi gi tr ca m, ng thng d lun i qua tiu im ca (P) v ct

2/

(P) ti hai im phn bit M, N. Tm qu tch trung im I ca on MN khi m thay i.


Tnh gc to bi cc tip tuyn M, N ca parabol (P) .

1
S: 1/ Qu tch l parabol (P ') : y = x2 + .
2

2/ (T1, T2 ) =

(hai tip tuyn vung gc).


2

Bi 635. i hc Nng nm 1995


Cho Parabol (P) : y2 = 2x .
1/ Xc nh ng chun, tiu im ca (P) v v (P) .
2/ Cho ng thng : x 2y + 6 = 0 . Tnh khong cch ngn nht gia v (P) .
3/ Tnh din tch hnh phng gii hn bi (P), trc Ox v tip tuyn ti A (2;2) .
S: 2/ dmin =

4 5
= d A; (P) .
5

2/

(tt) : y = 2 x + 1,

Shp =

4
(vdt) .
3

Bi 636. i hc Kin Trc H Ni khi A nm 1995


Hai ng cong gi l trc giao ti A nu chng ct nhau ti A v ti hai tip tuyn ca hai
x2
y2
ng cong vung gc vi nhau. Tm mi lin h gia a v b elip (E) : 2 + 2 = 1 v
a
b
2
parabol (P) : y = 2x trc giao vi nhau ti giao im ca chng.
S: a 2 = 2b2 .
Bi 637. i hc Lut Tp. H Ch Minh nm 1995
Cho parabol (P) : y2 = 16x . Vit phng trnh tip tuyn ca parabol
1/ i qua im A (1; 4) .
2/ Vung gc vi ng thng : 2x y + 5 = 0 .
S: 1/ 2x + y + 2 = 0 .

2/ x + 2y + 16 = 0 .

Bi 638. i hc Quc Gia H Ni nm 1995


Trong mt phng vi h trc ta Descarter vung gc Oxy, cho hai ng parabol
y = 8 3x 2x2 v y = 2 + 9x 2x2 .
1/ Hy xc nh cc gi tr ca a v b sao cho ng thng y = ax + b ng thi l tip tuyn
ca hai parabol v xc nh ta cc tip im.
2/ Tnh din tch hnh phng gii hn bi hai ng parabol cho v tip tuyn c xc
nh trn.
S: 1/ a = 1; b = 10 M1 (1; 9), M2 (2;12) .

"Cn c b thng minh"

DeThiThuDaiHoc.com

Page - 217 -

www.MATHVN.com

Ths. L Vn on
1
2

2/ Shp =

(x + 10 + 2x
1

Phn hnh hc

+ 3x 8 dx + x + 10 + 2x2 9x 2 dx =
1
2

9
(vdt) .
2

Bi 639. i hc Tng Hp Tp. H Ch Minh khi D nm 1996


Cho phng trnh ng trn (Cm ) : x 2 + y2 4mx 2 (m + 1) y = 1 .
1/ Chng minh rng m, (Cm ) lun i qua hai im c nh.
2/ Tm qu tch tm cc ng trn (Cm ) . Chng minh rng qu tch tip xc vi parabol

(P) : y

= 2x .

1 2
S: 1/ M1 (1;2), M2 ; .
5 5

2/ d : x 2y + 2 = 0 . im chung duy nht H (2;2) .

Bi 640. i hc i Cng Tp. H Ch Minh khi A nm 1996


Cho parabol (P) : y = x 2 2x + 3 v d l ng thng cng phng vi ng thng
y = 2x sao cho d ct (P) ti hai im A v B.

1/ Vit phng trnh ca d khi hai tip tuyn vi (P) ti A v B vung gc vi nhau.
2/ Vit phng trnh ca d khi di ca on thng AB bng 10.
S: 1/ d : 8x 4y + 1 = 0 . 2/ d : 2x y + 4 = 0 .
Bi 641. i hc Quc Gia Tp. H Ch Minh nm 1997
Trong mt phng vi h trc ta Descarter vung gc Oxy, cho im A (0;2) v parabol

(P) : y = x

. Xc nh cc im M trn (P) sao cho AM ngn nht. Chng t rng AM vung

gc vi tip tuyn ca parabol (P) ti M.

6 3

6 3

; hoc M
; . PTTT: d : 2x o x y y o = 0 AM d ti M.
S: M
2 2
2 2
Bi 642. i hc Quc Gia Tp. H Ch Minh khi A, B nm 1997
Cho parabol (P) : y =

15 27
x2
v im A ; .
2
8 8

1
1/ Vit phng trnh ng thng i qua im M1 1; v vung gc vi tip tuyn ca
2

(P) ti M.
2/ Tm tt c cc im M trn (P) sao cho AM vung gc vi tip tuyn ca (P) ti M.
S: 1/ 2x 2y + 3 = 0 .

5 25
3 9
1
2/ M1 1; , M2 ; , M3 ; .

2
2 8
2 8

Bi 643. i hc Nng khi A nm 1997


Cho parabol (P) : y2 = 16x .

Page - 218 -

DeThiThuDaiHoc.com
"All the flower of tomorrow are in the seeks of today"

cng hc tp mn Ton 10 Tp IIwww.MATHVN.com

Ths. L Vn on

1/ Lp phng trnh tip tuyn vi (P) sao cho n vung gc vi ng thng c phng
trnh l : 3x 2y + 6 = 0 .
2/ Lp phng trnh tip tuyn vi (P) qua im M (1; 0) .
S: 1/ (T) : 2x + 3y + 18 = 0 .

2/ 2x y + 2 = 0 .

Bi 644. i hc Nng khi A nm 1998 h cha phn ban


Trong mt phng vi h trc ta Descarter vung gc Oxy, cho parabol (P) : y2 = 2x v
ng thng d : 2x 2my 1 = 0 .
1/ Xc nh tiu im F v vit phng trnh ng chun ca parabol (P) .
2/ Chng minh rng vi mi gi tr ca m, ng thng d lun i qua tiu im F ca (P) v
ct (P) ti hai im phn bit M, N.
3/ Tm qu tch trung im I ca on MN khi m thay i.
1
1
1
S: 1/ F ; 0, x = . 2/ ' = m 2 + 1 > 0 d (P) = {M; N} . 3/ (P ') : y2 = x .

2
2
2
Bi 645. i hc Dc H Ni nm 1998
Lp phng trnh cc tip tuyn chung ca elip (E) :
S: d1 : 3x + 2 3y + 12 = 0,

x 2 y2
+
= 1 v parabol (P) : y2 = 12x .
8
6

d2 : 3x 2 3y + 12 = 0 .

Bi 646. Hc Vin Ngn Hng Tp. H Ch Minh nm 1998


Cho parabol (P) : y2 = x v gi F l tiu im ca (P) . Gi s ng thng d i qua F, ct

(P) ti hai im M1 v M2.


1/ Tnh M1M2 khi d song song vi trc Oy.
2/ Gi s d khng song song vi Oy. Gi k l h s gc ca d. Tnh M1M2 theo k. Xc nh cc
im M1, M2 sao cho M1M2 ngn nht.

1
1
S: 1/ M1M2 = 1 .
2/ d : y = k x , k 0 M1M2 = 2 + 1 > 1 .
4
k

Do , (M1M2 )

min

1 1
1 1
= 1 d // Oy v M1 ; , M2 ; .
4 2
4 2

Bi 647. i hc Ngoi Ng H Ni nm 1998


Cho parabol (P) : y2 = 4x .
1/ Chng minh rng t im N ty thuc ng chun ca parabol c th k c hai tip
tuyn n parabol m hai tip tuyn y vung gc vi nhau.
2/ Gi T1 v T2 l tip im. Chng minh rng ng thng T1T2 lun i qua mt im c nh
khi N chy trn ng chun ca parabol.
3/ Cho M l mt im thuc parabol (M khc nh ca parabol). Tip tuyn ti M ca parabol
ct cc trc Ox, Oy ln lt ti A, B. Tm qu tch trung im I ca AB khi M chy trn
parabol cho.

"Cn c b thng minh"

DeThiThuDaiHoc.com

Page - 219 -

www.MATHVN.com

Ths. L Vn on

S: 1/ Chng minh theo hai cch.

2/ Lun qua F (1; 0) .

Phn hnh hc

1
3/ y2 = x, (x < 0) .
2

Bi 648. i hc Kinh T Quc Dn nm 1999


Cho parabol (P) : y2 = 4x . Mt ng thng bt k i qua tiu im ca parabol cho v ct
parabol ti hai im phn bit A v B. Chng minh rng tch cc khong cch t A v B n
trc ca parabol l mt i lng khng i
HD: Tiu im F (1; 0) . Gi d l ng thng qua F. Tnh tung y A , y B v y A .y B = 4 ???
Bi 649. i hc Anh Ninh H Ni khi D nm 1999 h phn ban
Vit phng trnh tip tuyn chung ca hai parabol: y = x2 + x 2 v y = x2 + 7x 11 .
S: d1 : y = x 2 d2 : y = 7x 11 .
Bi 650. i hc Nng Nghip khi B nm 1999 h cha phn ban
Cho parabol (P) : y2 = x v hai im A (4; 2), B (1;1) thuc parabol. Hy tm im M nm
trn cung ca parabol gii hn bi A v B sao cho din tch tam gic AMB ln nht.

1
AB.d (M, d)
2
1 1
5 2
=
M ; .
4 2
8

S: d AB : x + y 2 = 0 v SAMB =

(SAMB )

max

khi d (M, d)

max

Bi 651. i hc Quc Gia Tp. H Ch Minh t 2 khi A, B nm 1999


Trong mt phng vi h trc ta Descarter vung gc Oxy, cho parabol (P) : y2 = 4x v
hai ng thng: (D) : m 2 x + my + 1 = 0 v (L) : x my + m2 = 0 vi m 0 .
1/ Chng minh (D) (L) v giao im M ca (D) v (L ) di ng trn mt ng thng c
nh khi m thay i.
2/ Chng minh (D) v (L ) lun tip xc vi (P) . Gi A v B ln lt l cc tip im ca

(D) v (L) vi (P) . Chng minh ng thng AB lun i qua mt im c nh khi m


thay i.
 
S: 1/ n(D).n(L) = 0 v qu tch l ng x = 1 .

2/ AB qua im c nh F (1; 0) .

Bi 652. i hc Ngoi Thng Tp. H Ch Minh nm 1999

x2
+ y2 = 1 .
9
1/ Chng minh rng parabol v elip ct nhau ti 4 im phn bit A, B, C, D.
2/ Chng minh rng 4 im A, B, C, D cng nm trn mt ng trn. Xc nh tm v bn
knh ng trn .
Cho parabol (P) : y = x2 2x v elip (E) :

S: 1/ 9x 4 36x 3 + 37x 2 9 = 0 . Xt f (x) = 9x 4 36x 3 + 37x2 9 lin tc trn  v


f (x ) = 0 c 1 nghim x1 trong khong (1; 0), f (x ) = 0 c 1 nghim x2 trong

(0;1), f (x) = 0 c 1 nghim x3 trong (1;2) v f (x) = 0 c 1 nghim x4 trong (2; 3) .

Page - 220 -

DeThiThuDaiHoc.com
"All the flower of tomorrow are in the seeks of today"

cng hc tp mn Ton 10 Tp IIwww.MATHVN.com

2/ (C) : x 2 + y2

16
8
x y 1 = 0 c tm
9
9

Ths. L Vn on

8 4
161
.
I ; v bn knh R =
9
9 9

Bi 653. i hc Thng Long khi A nm 2000


Trong mt phng vi h trc ta Descarter vung gc Oxy, cho parabol (P) : y = 2x2 v
ng thng : y = 2 (x + 2) .
1/ Tm im M (P) sao cho tip tuyn ca (P) ti M song song vi .
2/ Tnh khong cch d t M n .
3/ Gi A, B l hai giao im ca (P) vi . Chng minh rng din tch ca min gii hn bi

(P) v bng

2
tch ca d vi di on AB.
3

1
S: dm : y = 2x .
4
Bi 654. i hc Ngoi Thng H Ni khi D nm 2000
Cho parabol y2 = 8x v im I (2; 4) nm trn parabol. Xt gc vung thay i quanh im I
v hai cnh ca gc vung ct parabol ti M v N (khc im I). Chng minh rng ng thng
MN lun lun i qua mt im c nh.
m2

8 ; m

 

IM.IN = 0
n2

mn + 4 (m + n) + 80 = 0

m, n
A (10; 4) .
S: N ; n
8
A MN
8x o (m + n) yo + mn = 0

A (x o ; y o ) MN

Bi 655. i hc Hng Hi nm 2000


Cho Parabol (P) c tiu im F (2; 0), ng chun Oy, trc i xng Ox.
1/ Lp phng trnh ca parabol (P) .
2/ Chng minh rng t im A bt k trn trc Oy ta lun k c hai tip tuyn n parabol
(P) v hai tip tuyn ny vung gc nhau.
S: 1/ (P) : y2 = 4x 4 .

2/ Phng trnh d i qua A (0; a ) Oy : y = kx + a, (k 0) .

y2 + 4
+ a ky2 4y + 4k + 4a = 0 .
Phng trnh tung giao im y = k.
4
Bi 656. i hc Nng t 1 khi A nm 2000

5
3
Cho parabol (P) c tiu im F 2; v ng chun d : y = .

4
4

1/ Lp phng trnh ca parabol (P) .


2/ Tnh din tch hnh phng gii hn bi parabol (P) v trc Ox.
3/ Vit phng trnh tip tuyn ca parabol (P) song song vi trc Ox.
"Cn c b thng minh"

DeThiThuDaiHoc.com

Page - 221 -

www.MATHVN.com

Ths. L Vn on

S: 1/ (P) : y = x 4x + 3 . 2/ S =
2

(x

Phn hnh hc

4
. 3/ (tt) // Ox : y = 1 .
3

+ 4x 3 dx =

Bi 657. Hc Vin Qun Y H Ni nm 2000


Trong mt phng vi h trc ta Descarter vung gc Oxy, cho parabol (P) : y2 = 64x v
ng thng : 4x 3y + 46 = 0 . Hy vit phng trnh ng trn c tm nm trn ng
thng , tip xc vi parabol (P) v c bn knh nh nht.
2
2

37
126

= 4.
S: (C) : x + y
5
5

Bi 658. i hc Ngoi Thng H Ni khi A, D nm 2000


Cho parabol (P) : y2 = 8x v im I (2; 4) nm trn parabol. Xt gc vung thay i quay
quanh im I v hai cnh ca gc vung ct parabol ti hai im M v N (khc vi im I).
Chng minh rng ng thng MN lun lun i qua mt im c nh.
HD: Nhn xt rng I khong th c hai ng thng song song vi cc trc ta tha YCBT.
Do , phng trnh hai ng d1 v d2 qua I v vung gc vi nhau l
d : y = k (x 2) + 4
M = (P) d
1

(k 0) v N = P d 1
d : y = 1 (x 2) + 4
( ) 2

2
k

MN : (y + 4) k2 + (y + x 6) k + (y + 4) = 0 v MN qua im c nh A (10; 4) .

Bi 659. i hc Ngoi Thng Tp. H Ch Minh khi A nm 2000


Cho parabol (P) : y = x2 v ng thng y = mx + 1 . Chng minh rng khi m thay i,
ng thng lun lun ct parabol ti hai im phn bit A v B. Hy tm qu tch tm vng
trn ngoi tip OAB khi m thay i vi O l gc ta .
S: Tm I nm trn parabol (P ') : y = 2x 2 + 1 .
Bi 660. Trung Tm o To Bi Dng Cn B Y T Tp. H Ch Minh 2001
Trong mt phng vi h trc ta Descarter vung gc Oxy, cho parabol (P) : y2 = 4x v M
l im thay i trn ng thng () : x = 1 .
1/ Tm ta cc tiu im, ng chun ca (P) . Hy v (P) .
2/ Chng minh rng t M lun lun k c hai tip tuyn (d1 ), (d2 ) n (P) v hai tip
tuyn ny vung gc nhau.

3/ Gi M1, M2 ln lt l hai tip im ca hai tip tuyn ca (d1 ), (d2 ) cu 2 vi (P) .


Tm qu tch trung im I ca on M1M2 .

S: 1/ F (1; 0), () : x = 1 .

2/ ' = m 2 + 4 > 0 v kd .k d =
1

3/ Qu tch trung im I ca on M1M2 l parabol (P ') : x =

2 2
4
. =
= 1 .
y1 y 2
4

1 2
y +1.
2

Bi 661. i hc Kin Trc H Ni nm 2001

Page - 222 -

DeThiThuDaiHoc.com
"All the flower of tomorrow are in the seeks of today"

cng hc tp mn Ton 10 Tp IIwww.MATHVN.com

Ths. L Vn on

Trong mt phng vi h trc ta Oxy, cho parabol (P) c nh ti gc ta v i qua im


5
A 2; 2 2 . ng thng d i qua im I ;1 ct (P) ti hai im M, N sao cho MI = IN .
2
Tnh di on MN.

S: MN = 3 5 .
Bi 662. i hc Nng Nghip I khi B nm 2001
Trong mt phng vi h trc ta Oxy, cho parabol vi phng trnh y2 = 8x .
1/ Tm ta tiu im v ng chun ca parabol.
2/ Qua tiu im k ng thng bt k ct parabol ti hai im A v B. Chng minh rng cc
tip tuyn vi parabol ti A v B vung gc nhau.
3/ Tm qu tch cc im M m t c th k c hai tip tuyn vi parabol, sao cho chng
vung gc nhau.
S: 1/ F (2; 0), x = 2 .

2/ k1.k2 =

4 4
. = 1 .
y1 y2

3/ ng thng x + 2 = 0 .

Bi 663. D b 2 i hc khi A nm 2003


Trong mt phng vi h ta Oxy , cho parabol (P) : y2 = x v im I (0;2) . Tm to hai


im M, N thuc (P) sao cho IM = 4IN .
S: M (4; 2), N (1;1) hoc M (36;6), N (9; 3) .
Bi 664. i hc khi D nm 2008
Trong mt phng vi h ta Oxy, cho parabol (P) : y2 = 16x v im A (1; 4) . Hai im


phn bit B, C (B v C khc A) di ng trn (P) sao cho gc BAC = 900 . Chng minh rng
ng thng BC lun i qua mt im c nh.
S: Vit phng trnh ng thng BC BC i qua im c nh I (17; 4) .

"Cn c b thng minh"

DeThiThuDaiHoc.com

Page - 223 -

www.MATHVN.com

Ths. L Vn on

Phn hnh hc

G BA NG CONIC

 nh ngha
ng cnc

l tp hp im c t s khong cch t n mt im c nh v n

mt ng thng c nh khng i qua im c nh y, bng mt hng s dng e.


Hng s dng e chnh l tm sai ca ng cnc
.
+ Nu

l elp.

+ Nu

l parabol.

+ Nu

l hyperbol.

 Cc dng ton thng gp


a/ Dng ton 1. Chng minh ng cong

l mt cnc

 Phng php
Bc 1. Bin i

v dng

Bc 2. Bin lun theo

ta c dng ca ng cong.

b/ Dng ton 2. Lp phng trnh ca mt cnc


 Phng php: Nu bit tm sai e, mt tiu im

v ng chun

ng vi tiu im
.

th ta s dng tnh cht:


c/ Dng ton 3. Tip tuyn ca ng cnc
Trong mt phng ta

cho ng cnc

iu kin cn v d tip xc vi

v ng thng

l
iu kin cn v d tip xc vi

Phng trnh ca
Elp
Hyperbol
Parabol

Trong mt phng ta
vi
trnh

Page - 224 -

ti im

cho ng cnc
thuc

. Phng trnh tip tuyn

tng ng vi cc dng ca phng

nh sau:

DeThiThuDaiHoc.com
"All the flower of tomorrow are in the seeks of today"

cng hc tp mn Ton 10 Tp IIwww.MATHVN.com

Ths. L Vn on

()

()

(E ) :

x2
a

(H) :

y2
2

x2
a2

=1

y2
b2

=1

d:

xox

d:

xox

yo y

yo y

b2
b2

M (x o ; yo ) ()

=1
=1

d : yo y = p (x + x o )

(P) : y2 = 2px

()
d : Ax + By + C = 0

()

M (x o ; yo ) ()
M (x o ; yo ) (),

d : A ( x x o ) + B (y y o ) = 0

d : Ax + By Ax o Byo = 0
: Ax + By + C = 0
d : Ax + By + D = 0
: Ax + By + C = 0
d : Bx Ay + D = 0
d : y = kx + m
d : kx y + m = 0
: Ax + By + C = 0

 
u .u d
cos =  
u . u d

tan =

k k d
1 + k .kd

 
u, u d

k , k d

 Phng php 2. i tm tip im ri s dng phng php phn i


ta gii.
Bc 1. Gi s im
l tip im. Khi phng trnh
tip tuyn c dng
.

) v
"Cn c b thng minh"

(phn i to

DeThiThuDaiHoc.com

Page - 225 -

Ths. L Vn on

Page - 226 -

www.MATHVN.com

Phn hnh hc

DeThiThuDaiHoc.com
"All the flower of tomorrow are in the seeks of today"

cng hc tp mn Ton 10 Tp IIwww.MATHVN.com

Ths. L Vn on

BA
NG
BAI T
TP AAP DU
DUNG
Bi 665. Xc nh ta tiu im, phng trnh ng chun ca cc cnc

x 2 y2
x 2 y2
2/
3/ y2 = 6x .
+
= 1.

= 1.
8
4
15 20
Bi 666. Vit phng trnh cnc trong mi trng hp sau
1/ Tiu im F (3;1), ng chun : x = 0 v tm sai e = 1 .
1/

3/

1
.
2
Tiu im F (2; 5), ng chun : y = x v tm sai e = 2 .

4/

Tiu im F (3; 2), ng chun : x 2y + 1 = 0 v tm sai e = 3 .

5/

Tiu im F (3; 0), ng chun : x + 3 = 0 v tm sai e = 1 .

2/

6/
7/
8/

Tiu im F (1; 4), ng chun : y = 0 v tm sai e =

1
.
2
8
3
Tiu im F (2; 0), ng chun : x = v tm sai e = .
9
2
Cnc l mt elp (E) c khong cch gia hai ng chun l 5 v khong cch gia hai
Tiu im F (2; 0), ng chun : x = 8 v tm sai e =

tiu im l 4.
9/

Cnc l mt elp (E) c tm sai e =

3
v khong cch t tm i xng n ng
4

16
.
3
10/ Cnc l mt hyperbol (H) c hai ng tim cn l 4x 3y = 0 v khong cch gia
chun l

hai ng chun l

18
.
5

11/ Cnc l mt hyperbol (H) c mt ng chun x =

16
5
v tm sai e = .
5
4

1
12/ Cnc c tm sai e = , mt tiu im l F (3;1) v phng trnh ng chun ng
2
vi tiu im l : y + 2 = 0 .
13/ Cnc l mt parabol vi tiu im F (0;2) v ng chun ng vi tiu im l
3x 4y 12 = 0 .

14/ Cnc l mt elp (E) c tm l O, tiu im trn Ox, i qua im M 3;1 v


khong cch gia hai ng chun bng 6.
2
15/ Cnc l mt elp (E) c tm sai e = , mt tiu im F (2;1) v phng trnh ng
3
chun ng vi tiu im l : 3x 4y 12 = 0 .
16/ Cnc l mt hyperbol (H) vi tiu im F (2; 3) ng chun ng vi tiu im l
3x y + 3 = 0 v tm sai e = 5 .

17/ Cnc l mt hyperbol (H) c tm sai e = 2 , mt tiu im F (1;1) v phng trnh


ng chun ng vi tiu im l : x y 2 = 0 .

"Cn c b thng minh"

DeThiThuDaiHoc.com

Page - 227 -

Ths. L Vn on

www.MATHVN.com

Phn hnh hc

18/ Cnc l mt parabol (P) c tiu im l O v ng chun x y 2 = 0 .


Bi 667. Bin lun theo m hnh dng ca cnc c phng trnh
1/
2/

() : (m 1) x2 + my2 = m2 m .
() : m2 x2 + (m2 9) y2 + 18my 9m2 = 0 vi m > 0 .

Bi 668. Chng t rng phng trnh Ax2 + By2 + F = 0 vi AF < 0


1/
2/

3/

L phng trnh ca mt ng trn c tm O (0; 0) nu A = B .

A B
L phng trnh ca mt elp c nh O (0; 0) nu
. Tm ta cc tiu im,

A.B > 0

phng trnh ng chun ca elp .


A B
L phng trnh ca mt hyperbol c nh O (0; 0) nu
. Tm ta cc tiu

A.B < 0

im, phng trnh ng chun ca hyperbol .

Bi 669. Chng t rng phng trnh Ax2 + By2 + Cx + Dy + E = 0 vi A.B > 0


C2

D2

1/ L mt phng trnh ca mt elp nu A.


+
E > 0 . Tm ta cc tiu im,
4A 4C

phng trnh ng chun ca elp .


C2

D2
2/ L mt phng trnh ca mt hyperbol nu A.
+
E < 0 . Tm ta cc tiu
4A 4C

im, phng trnh ng chun ca hyperbol .

C2
D2
+
E = 0.
4A 4C
Bi 670. Chng t rng phng trnh Ax2 + Bx + Cy + D = 0 vi A 0
1/ L mt phng tnh ca mt parabol nu C 0 .
2/ L mt phng trnh ca ng thng nu C = 0 v B2 4AD = 0 .
3/ L phng trnh ca hai ng thng nu C = 0 v B2 4AD > 0 .
3/

L mt im nu

4/ L tp rng nu C = 0 v B2 4AD < 0 .


Bi 671. Chng t rng phng trnh Ay2 + By + Cx + D = 0 vi A 0
1/ L mt phng tnh ca mt parabol nu C 0 .
2/ L mt phng trnh ca ng thng nu C = 0 v B2 4AD = 0 .
3/ L phng trnh ca hai ng thng nu C = 0 v B2 4AD > 0 .
4/

L tp rng nu C = 0 v B2 4AD < 0 .

Bi 672. Chng t rng phng trnh Ax2 + 2Bxy + Cy2 + Dx + Ey + F = 0

()

1/

L phng trnh ca mt elp nu B2 AC < 0 .

2/
3/
4/
5/

L phng trnh ca mt hyperbol nu B2 AC > 0 .


L phhng trnh ca mt parabol nu B2 AC = 0 .
L phng trnh ca mt ng trn nu B = 0 v A = C 0 .
L phng trnh ca mt elp hoc hyperbol c trc cng phng vi trc ta nu
B = 0 v A.C 0 v A C .
L phng trnh ca 2 ng thng nu () c nghim y l phng trnh bc nht theo x.

6/

Bi 673. Cho elp (E) : 4x2 + 16y2 = 64 .


Page - 228 -

DeThiThuDaiHoc.com
"All the flower of tomorrow are in the seeks of today"

cng hc tp mn Ton 10 Tp IIwww.MATHVN.com

1/
2/

Xc nh cc tiu im F1, F2, tm sai v v elp.


M l mt im bt k trn elp. Chng t rng t s khong cch t M n tiu im phi

F2 v ti ng thng x =
3/

Ths. L Vn on

8 3
c gi tr khng i.
3

Cho ng trn (C) : x2 + y2 + 4x 3 4 = 0 . Xt ng trn (C1 ) di ng nhng


lun i qua tiu im phi F2 v tip xc ngoi vi ng trn (C) . Chng t rng cc
tm N ca ng trn (C1 ) nm trn mt hyperbol c nh. Vit phng trnh hyperbol.

Bi 674. Cho hyperbol (H) c phng trnh (H) :


1/
2/
3/

x2

y2

= 1.
a 2 b2
Tnh di phn ng tim cn chn bi hai ng chun.
Tnh khong cch t tiu im n cc tim cn.
Chng minh rng chn ng vung gc h t mt tiu im n cc ng tim cn
nm trn ng chun.

Bi 675. Tnh gc , 0 < 900 gia cc ng tim cn ca hyperbol. Bit khong cch gia cc
tiu im gp 2 ln khong cch gia cc ng chun.
Bi 676. Chng minh rng iu kin cn v ng thng d : Ax + By + C = 0 tip xc vi elp
2

(E ) : x2 + y 2

= 1 l a 2A2 + b2B2 = C2 .

hyperbol (H) :

x2

a
b
Bi 677. Chng minh rng iu kin cn v ng thng d : Ax + By + C = 0 tip xc vi
2

y2
2

= 1 l a 2 A 2 b 2 B2 = C2 .

a
b
Bi 678. Chng minh rng iu kin cn v ng thng d : Ax + By + C = 0 tip xc vi
parabol (P) : y2 = 2px l B2p = 2AC .
Bi 679. Chng minh rng phng trnh tip tuyn vi elp (E) :
M (x o ; yo ) (E) c dng :

xo x
a

yo y
b2

x2
a2

xo x
a2

yo y
b2

= 1 ti im

b2

= 1.

Bi 680. Chng minh rng phng trnh tip tuyn vi hyperbol (H) :
M (x o ; yo ) (H) c dng :

y2

x2
a2

y2
b2

= 1 ti im

= 1.

Bi 681. Chng minh rng phng trnh tip tuyn vi parabol (P) : y2 = 2px ti im
M (x o ; yo ) (P) c dng : y o y = p (x + x o ) .

Bi 682. Cho im M (1;2) . Lp phng trnh tip tuyn ca cnc () i qua M, bit
1/

() :

x 2 y2
+
= 1.
2
8

2/

() : x2

y2
= 1.
2

x2
y2
+
= 1.
9
4
Chng minh rng qua M k c hai tip tuyn n (E) .

Bi 683. Cho im M (3; 4) v elp (E) c phng trnh (E) :


1/

"Cn c b thng minh"

DeThiThuDaiHoc.com

Page - 229 -

Ths. L Vn on

2/

www.MATHVN.com

Phn hnh hc

Xc nh phng trnh hai tip tuyn v lp phng trnh ng thng i qua hai tip
im ca (E) vi hai tip tuyn trn.

3/

Vit phng trnh tip tuyn ca (E) i qua im A (3; 0) .

4/

Vit phng trnh tip tuyn ca (E) i qua im B (2; 3) .

5/

Vit phng trnh tip tuyn ca (E) , bit tip tuyn song song vi d1 : x 2y + 6 = 0 .

6/

Vit phng trnh tip tuyn ca (E), bit tip tuyn vung gc vi d2 : y = x .

7/

Vit phng trnh tip tuyn ca (E), bit tip tuyn to vi d3 : 2x y = 0 mt gc


bng 60 0 .

1/

x 2 y2

= 1 . Vit phng trnh tip tuyn ca (H) . Bit tip tuyn


9
16
i qua im A (3; 0) .

2/

i qua im B (3;2) .

3/

Song song vi ng thng d1 : x y + 1 = 0 .

4/

Vung gc vi ng thng d2 : x 2y + 3 = 0 .

5/

To vi ng thng d3 : x 2y 4 = 0 mt gc bng 45 0 .

Bi 684. Cho hyperbol (H) :

Bi 685. Cho parabol (P) : y2 = 2x . Lp phng trnh tip tuyn ca (E), bit tip tuyn
6/

i qua im A (2;2) .

7/

i qua im B (2; 3) .

8/

Song song vi ng thng d1 : 3x 4y + 1 = 0 .

9/

Vung gc vi ng thng d2 : 4x 3y + 7 = 0 .

10/ To vi ng thng d3 : 2x y = 0 mt gc bng 60 0 .


Bi 686. Cho ng thng : 2x y 1 = 0 v parabol (P) : y2 = 2x .
1/

Lp phng trnh tip tuyn ca parabol (P) vung gc vi ng thng .

2/

Gi M l tip im ca (P) vi tip tuyn d. Hy lp phng trnh ng trn tm M v


tip xc vi ng thng .

Bi 687. Cho parabol (P) : y = x2 2x + 3 v d1 : y = 2x . ng thng d l ng thng cng


phng vi ng thng d1 sao cho d ct (P) ti hai im phn bit A, B.
1/

Lp phng trnh ca d khi hai tip tuyn ca (P) ti A v B vung gc vi nhau.

2/

Lp phng trnh ca d khi di AB = 40 .

(x 1)

Bi 688. Cho im M (2;9) v hyperbol (H) :


ca hyperbol (H) i qua im M.

(y 1)

16

= 1 . Lp phng trnh tip tuyn

Bi 689. Lp phng trnh tip tuyn chung ca


1/
2/

Page - 230 -

x2
y2
+
=1
4
9
x2
y2
E
:
+
( ) 4 9 =1

(E ) :

&

(C) : x2 + y2 = 5 .

&

(H ) :

x 2 y2

= 1.
8
27

DeThiThuDaiHoc.com
"All the flower of tomorrow are in the seeks of today"

cng hc tp mn Ton 10 Tp IIwww.MATHVN.com

x2
y2
+
= 1.
9
4

3/

(E ) :

4/

(E1 ) :

5/
6/

x2
y2
+
= 1.
9
4
x 2 y2
H
:
( ) 9 4 =1
x 2 y2
H
:
( ) 16 4 = 1

7/

(C) : (x + 2)

8/

(H1 ) :

9/

( H) :

10/
11/
12/
13/

+ y2 = 4

x 2 y2

=1
9
4

x 2 y2

=1
9
4
x2
y2
E
:
+
( ) 9 4 =1
x 2 y2
( H) : 9 4 = 1
x2
y2
(E ) : 9 + 4 = 1
(P1 ) : y = x2 + 2x + 2

Bi 690. Cho elp (E) :

x2

Ths. L Vn on

&

(C) : x2 + y2 = 9 .

&

(E 2 ) :

&

(C) : x2 + y2 = 4 .

&

(C) : (x + 2)

&

(P) : y2 = 12x .

&

(H 2 ) :

&

(E ) :

&

(P) : y2 = 12x .

&

(P) : y2 = 2x .

&
&

x2
y2
+
= 1.
4
9

+ y2 = 4 .

x 2 y2

= 1.
4
9

x2
y2
+
= 1.
4
9

(P) : y2 = 2x .
(P2 ) : y = x2 .

y2

= 1 , (a > b) . Chng minh rng tch khong cch t cc tiu im n


a2
b2
mt tip tuyn bt k ca elp (E) bng bnh phng di trc nh ca elp.
+

Bi 691. Cho hyperbol (H) :

x2

y2

= 1 . Mt tip tuyn bt k ca (H) l d tip xc vi (H) ti im


a 2 b2
T. Gi M, N l cc giao im ca tip tuyn d vi cc ng tim cn ca (H) .

1/ Chng minh rng T l trung im ca MN.


2/ Chng minh rng din tch OMN khng ph thuc vo tip tuyn d.
Bi 692. Cho parabol (P) . Chng minh rng tip tuyn ti hai u mt ca dy cung qua tiu vung gc
vi nhau ti mt im trn ng chun.
Bi 693. Cho elp (E) :

x2

Bi 694. Cho elp (E) :

x2

y2

= 1 v hai im M, N trn elp (E), sao cho m tiu im F1, F2 ca


a2
b2
(E) nhn on MN di mt gc vung. Hy xc nh v tr M, N trn tip tuyn y.
+

y2

= 1 vi 0 < b < a c hai tiu im F1, F2 . ng thng di ng d


a2
b2
lun i qua F2 ct (E) ti P, Q.
+

1/

t (Ox, F2P) = , (0 2) . Tnh di F2 P, F2Q theo a, b, .

2/

Chng minh:

3/

1
1
+
khng i.
F2P F2Q
Tm gi tr ln nht v gi tr nh nht ca on PQ.

"Cn c b thng minh"

DeThiThuDaiHoc.com

Page - 231 -

www.MATHVN.com

Ths. L Vn on

4/

ng thng F1P ct elp (E) ti S P . Chng minh

Bi 695. Cho im F (1; 0) v ng thng : x + 4 = 0 .

Phn hnh hc

F2 P
F2Q

F1P
F1S

khng i.

MF
1
= l mt elp (E) .
d (M, ) 2

1/

Chng minh rng qu tch cc im M sao cho

2/

T im N ty trn ng thng k c hai tip tuyn n (E) v tip xc vi (E)


ti P v Q. Chng minh rng ng thng PQ i qua F v xc nh v tr ca N di
on PQ l nh nht.

Bi 696. Cho hai im M (2; m), N (2; n) v elp (E) : x2 + 4y2 = 4 .


1/

Gi A1, A2 l cc nh trn trc ln ca elp. Hy vit phng trnh cc ng thng

A1N v A2M v xc nh ta giao im I ca chng.

2/

Cho MN thay i nhng lun tip xc vi elp (E) . Tm qu tch ca im I.

1/
2/

x2
y2
+
= 1 v A (3; 0), M (3; 0), N (3; 0), N (3; n) .
9
4
Xc nh ta giao im I ca AN v BM.
Chng t rng MN tip xc vi elp (E) th iu kin cn v l m, n tha mn biu

3/

thc m.n = 4 .
Vi m, n thay i nhng MN lun tip xc vi elp (E) . Tm qu tch ca im I.

Bi 697. Cho elp (E) :

Bi 698. Cho elp (E) :

x2

y2

= 1 vi F1, F2 l cc tiu im v A1, A2 l cc nh thuc trc ln


a2
b2
ca n. Ly M (E) v H1, H2 ln lt theo th t l hnh chiu vung gc ca F1, F2 ln
tip tuyn vi (E) ti M. Gi (C) l ng trn ng knh A1A2 .

1/

Chng minh H1, H2 (C) .

2/

Cc ng thng H1F2, H2 F2 ct ng trn (C) theo th t ti cc im K1, K2 .


Chng minh rng H1H2 K2K1 l hnh ch nht.

Bi 699. Cho elp (E) :

x2

Bi 700. Cho elp (E) :

x2

Bi 701. Cho elp (E) :

x2

y2

= 1 vi a > b . Tip tuyn ti M ca elp (E) ct ng chun 2 ti


a2
b2
N. Chng minh rng tiu im F2 nhn MN di mt gc vung.
+

y2

= 1 vi a > b . Tip tuyn ti M ca elp (E) ct hai trc ta ti S1


a2
b2
v S2 . Xc nh ta ca M OS1S2 c din tch l nh nht.
+

y2

+ 2 = 1 vi a > b . T im K ty trn ng chun k c hai tip


a2
b
tuyn n (E) v tip xc vi (E) ti hai im H1, H2 .

1/

Chng minh rng ng thng H1H2 i qua tiu im F ca elp (E) .

2/

Xc nh v tr ca im K a di H1H2 nh nht.

Page - 232 -

DeThiThuDaiHoc.com
"All the flower of tomorrow are in the seeks of today"

cng hc tp mn Ton 10 Tp IIwww.MATHVN.com

Bi 702. Cho elp (E) :

x2

Ths. L Vn on

y2

= 1 vi a > b . Gi A2 (a, 0) l nh trn trc ln ca elp (E) . Gc


a2
b2
vung tA2 z quay quanh A2 ct (E) ti P, Q khng trng vi tm ca elp.

1/
2/

Chng minh rng ng thng PQ lun i qua mt im c nh.


Tip tuyn ca (E) ti P, Q ct nhau ti M. Chng minh rng M chy trn mt ng
thng c nh m ta cn phi tm.

Bi 703. Cho elp (E) :

x2

y2

= 1 vi a > b . Hnh ch nht Q c gi l hnh ch nht ngoi tip


a2
b2
elp (E) nu mi cnh ca Q u tip xc vi (E) . Trong tt c hnh ch nht ngoi tip (E) .
+

Hy xc nh:
1/ Hnh ch nht c din tch nh nht.
2/ Hnh ch nht c din tch ln nht.

x2
y2
+
= 1 . Vit phng trnh cc cnh ca hnh ch
16
9
nht ngoi tip (E) c din tch bng 15.

Bi 704. Cho elp (E) c phng trnh (E) :

x2
y2
x2
y2
+
= 1 v (E2 ) :
+
= 1.
6
3
4
1
Mt tip tuyn bt k ca (E2 ) ct (E1 ) ti hai im P,Q . Chng minh rng cc tip

Bi 705. Cho (E1 ) :


1/

tuyn ca (E1 ) ti P,Q vung gc vi nhau.

2/

T im M (E1 ) k hai tip tuyn Mt1, Mt2 n (E2 ) . Chng minh rng hai gc

t
Mt2, F
MF2 vi F1, F2 l hai tiu im ca (E1 ), (E2 ) c chung ng phn gic.
1
1

3/

Pht biu v chng minh bi ton tng qut.

"Cn c b thng minh"

DeThiThuDaiHoc.com

Page - 233 -

www.MATHVN.com

Ths. L Vn on

Phn hnh hc

H NG DNG TA GII TON I S & GII TCH



 Bt ng thc tam gic
Cho ABC c cc cnh

. Ta lun c:
hay

.
.

Nh vy, ta chn
c ta thch hp, d nhin lin quan n bt ng thc, chng minh
ri s dng mt trong hai bt ng thc trn suy ra kt qu.
 Bt ng thc vct
Cho

Du

. Du

. Du

xy ra

xy ra

xy ra

cng phng

cng phng.
. Do

cng phng

nn

.
Bt ng thc

c gi l bt ng thc Bunhiacpxki.

 Cc ng dng
1/ Chng minh bt ng thc.
2/ Gii phng trnh.
3/ Gii bt phng trnh.
4/ Tm gi tr ln nht gi tr nh nht.

Page - 234 -

DeThiThuDaiHoc.com
"All the flower of tomorrow are in the seeks of today"

cng hc tp mn Ton 10 Tp IIwww.MATHVN.com

Ths. L Vn on

CHNG MINH BT NG THC


Bi 706. Cho a, b, c  . Chng minh: (a + c) + b2 +


HD: t u = (a + c; b), v = (a c; b) .

(a c)

+ b2 2 a 2 + b2 .

Bi 707. Cho a, b, c  . Chng minh: a 2 + 4b2 + 6a + 9 + a 2 + 4b2 2a 12b + 10 5 .




HD: t u = (a + 3;2b); v = (1 a; 3 2b) .
Bi 708. Cho a, b, c  . Chng minh: a 2 + ab + b2 + a 2 + ac + c2 b2 + bc + c2 .

b 3 
c 3

b, v = a + ;
c .
HD: t u = a + ;
2 2
2 2


Bi 709. Chng minh vi mi s thc a ta lun c: a 2 + a + 1 + a 2 a + 1 2 .

1 3  1
3

,
v
a;
=

HD: t u = a + ;

2 2
2
2
Bi 710. Cho a, b, c  . Chng minh:

4 cos2 a cos2 b + sin2 (a b) + 4 sin2 a sin2 b + sin2 (a b) 2 .




HD: t u = 2 cos a cos b; sin (a b) , v = 2 sin a sin b; sin (a b) .

Bi 711. i hc An Ninh Nhn Nhn phn hiu Tp. H Ch Minh nm 1998


Chng minh rng: x + 1 x + 4 x + 4 1 x 2 + 4 8, x 0;1 . Du " = " xy ra

khi no ?



HD: t a = (1;1), b = x; 1 x , c = 4 x; 4 1 x .

Bi 712. B tuyn sinh i hc Cao ng

(a + b)(1 ab)

Chng minh:

(1 + a )(1 + b )
2

1
vi mi a, b .
2

 2a 1 a 2  1 b2
2b

=
HD: t u =
;
,
v
;
.

2
2
1 + a 2 1 + a 2
1 + b 1 + b
Bi 713. Cho ba s dng : a, b, c trong a > c, b > c . Chng minh rng:

c (a c) + c (b c) ab . Du " = " xy ra khi no ?




HD: t u = c; b c , v = a c; c .

Bi 714. Cho a, b, c, d  . Chng minh rng:

)(

1/

(ad + cd)

2/

a 2 + c2 + b2 + d2 (a + b) + (c + d) .


HD: t x = (a, c); y = (b; d) .

a2 + c2 b2 + d2 .
2

Bi 715. i hc khi A nm 2003


Cho ba s dng x, y, z tha iu kin: x + y + z 1 .

"Cn c b thng minh"

DeThiThuDaiHoc.com

Page - 235 -

www.MATHVN.com

Ths. L Vn on

x2 +

Chng minh rng:

1
2

+ y2 +

+ z2 +

x
y
z2
 1  1  1
HD: t a = x, ; b = y, ; c = z, .
x
y
z
Bi 716. D b Cao ng Giao Thng Vn Ti II nm 2003

Phn hnh hc

82 .

Cho ba s x, y, z bt k. Chng minh: x2 + xy + y2 + x2 + xz + z2 y2 + yz + z2 .

y 3
3
3 y z

z, B 0;
y+
z, C ; 0 .
HD: t A x + ;

2 2 2
2 2 2
Bi 717. Cho a, b, c, d l cc s thc bt k.
Chng minh:

a 2 + b2 + c 2 + d2

(a c)

+ ( b d) .
2

HD: t M (a; b), N (c; d), O (0; 0) .

a 2 a + 1 + a2 a 3 + 1 2 vi mi a.


1
1 3 3
HD: t A ;
, B ; , C (a; 0) .
2 2 2
2
Bi 719. Cho a 2b + 2 = 0 .
Bi 718. Chng minh:

Chng minh: a 2 + b2 6a 10b + 34 + a 2 + b2 10a 14b + 74 6 .


HD: Xt d : x 2y + 2 = 0 v A (3;5), B (5;7),C (a; b) d .
Bi 720. Cho x, y, z l cc s thc i mt khc nhau.
Chng minh:

xy
1 + x 2 . 1 + y2

yz
1 + y2 . 1 + z2

>

xz
1 + x2 . 1 + z2

HD: t A (x; yz), B (y; zx),C (z; xy) .


Bi 721. Cho x, y, z l ba s thc bt k tha x + 2y + 3z = 6 .
1 + x2 + 2 1 + y2 + 3 1 + z2 2 10 .

, O (0; 0) v OA + AB + BC OC .
+
2y
+
3z
HD: t A (1; x), B (3; x + 2y),C 6; x
 
2

Chng minh:

Bi 722. Cho a, b  . Chng minh:

a 2 + 4 + a 2 2ab + b2 + 1 + b2 6b + 10 5 .

HD: t A (0; 1), B (3; 3),C (a;1), D (b;2) v AC + CD + BD AB .


Bi 723. Cho x, y  . Chng minh: cos4 x + cos4 y + sin2 x + sin2 y 2 .



HD: t a = cos2 x; cos2 y , b = sin2 x; 0 , c = 0; sin2 y .

Bi 724. Cho x  . Chng minh: cos 4 x + 1 sin 4 x + 1 cos 2x .




HD: t a = cos2 x;1 , b = sin2 x;1 .

Bi 725. Cho ba s thc a, b, c tha a + b + c = 1 .


a 2 + (b c) + b2 + (c a ) + c2 + (a b) 1 .



HD: t u = (a; b c), v = (b; c a ), w = (c;a b) .

Chng minh:

Page - 236 -

DeThiThuDaiHoc.com
"All the flower of tomorrow are in the seeks of today"

cng hc tp mn Ton 10 Tp IIwww.MATHVN.com

Ths. L Vn on

Bi 726. Cho a, b, c > 0 v ab + bc + ca = abc .

b2 + 2a 2
c2 + 2b2
a 2 + 2c2
+
+
3.
ab
bc
ca
 1 2  1 2  1 2

HD: t u = ; , v = ; , w = ; .
a b
b c
c a
Bi 727. Cho a + b + c = 2 v ax + by + cz = 6 .
Chng minh:

16a 2 + a 2 x2 + 16b2 + b2 y2 + 16c2 + c2 z2 10 .





HD: t u = (4a;ax ), v = (4b; by), w = (4c; cz) .

Chng minh:

GIA PHNG TRNH


Bi 728. Gii phng trnh: x2 2x + 5 + x2 + 2x + 10 = 29 .


1
HD: t u = (x 1;2), v = (1 x; 3) v nghim x = .
5
Bi 729. Gii phng trnh: x2 x + 1 + x2 + x + 1 = 2 .

1 3 
1 3

HD: t u = x ;
, v = x ;
v nghim x = 0 .

2 2
2 2
Bi 730. Gii phng trnh: x2 x + 1 + x2 2x + 5 = 9x2 12x + 13 .


1
HD: u = (2x 1;1), v = (x 1;2) v nghim x = .
3
Bi 731. Gii phng trnh: x2 + 4y2 + 6x + 9 + x2 + 4y2 2x 12y + 10 = 5 .


3
HD: t u = (x + 3;2y), v = (1 x; 3 2y) v nghim x = 1, y = .
2
x 2 4x + 5 x 2 10x + 50 = 5 .

Bi 732. Gii phng trnh:

HD: t A (2;1), B (5; 5), C (x; 0) v nghim x =

5
.
4

GIA BT PHNG TRNH


Bi 733. Gii bt phng trnh: 2 (x 3) + 2x 2 x 1 + x 3 .


HD: t u = x 3; x 1 , v = (1;1) v nghim x = 5 .
2

Bi 734. Gii bt phng trnh: x2 + x + 1 x2 x + 1 1 .



1 3 
1 3

HD: t u = x + ;
, v = x ;
v nghim x  .

2 2
2 2

GI TR LN NHT GI TR NH NHT
Bi 735. Tm gi tr nh nht ca hm s: y = x2 8x + 32 + x2 6x + 18 .

"Cn c b thng minh"

DeThiThuDaiHoc.com

Page - 237 -

www.MATHVN.com

Ths. L Vn on

Phn hnh hc

HD: t A (x 4; 4), B (x 3; 3),O (0; 0) v min y = 5 2 khi x =

24
.
7

Bi 736. Tm gi tr nh nht ca hm s: y = x2 2px + 2p2 + x2 2qx + 2q 2 vi p, q 0 .




2pq
HD: t u = (x p; p), v = (q x; q ) v min y = 2 q 2 + p2 khi x =
.
p+q
Bi 737. Tm gi tr nh nht ca biu thc:

P = 2x 2 2x + 1 + 2x2 +

3 + 1 x + 1 + 2x2

3 1 x +1 .

3
1 3
1

; ,C ; , M (x; x) v Pmin = 3 khi x = 0 .


HD: t A (1;1), B
2 2
2
2
Bi 738. Gi (x; y) l nghim ca phng trnh: 2x + 3y = 1 . Tm cp nghim (x; y) sao cho biu thc

P = 3x2 + 2y2 nh nht.


 2 3 
; , v =
HD: t u =
3 2

3x; 2y v Pmin =

4 9
6
khi (x; y) = ; .
35
35 35

Bi 739. Tm gi tr nh nht ca hm s: y = cos2 x 2 cos x + 2 + cos2 x + 6 cos x + 13 .




HD: t u = (1 cos x;1), v = (cos x + 3;2) v min y = 5 .
Bi 740. Cho x2 + y2 = 1 . Tm gi tr ln nht ca biu thc: P = x 1 + y + y 1 + x .


HD: t u = (x; y), v = 1 + y; 1 + x v Pmax = 2 + 2 khi x = y =

2
.
2

Bi 741. Tm gi tr ln nht ca biu thc: P = a + cos x + a + sin x vi x , a 1 .




u = (1;1)

HD: 
v Pmax = 2 2a + 2 khi x = + k2, (k ) .
v = a + cos x; a + sin x
4

Page - 238 -

DeThiThuDaiHoc.com
"All the flower of tomorrow are in the seeks of today"

You might also like